Sie sind auf Seite 1von 339

GRADE 12

MATHEMATICS
MATHS - 12 CHAPTER 6
APPLICATIONS OF DERIVATIVE
CHAPTER 1 A. L'HOSPITAL'S RULE ............................ 181
B. APPLICATIONS OF FIRST
INDUCTION DERIVATIVES ........................................ 187
A. INDUCTION ........................................... 1 C. APPLICATIONS OF SECOND
B. SUMMATION NOTATION ...................... 2 DERIVATIVES ........................................ 207
D. OPTIMIZATION PROBLEMS .............. 217
C. MULTIPLICATION NOTATION ................. 5
E. PLOTTING GRAPHS ........................... 225

CHAPTER 2
FUNCTIONS
A. DEFINITION OF FUNCTIONS .................. 8 CHAPTER 7
B. SPECIAL FUNCTIONS ............................ 31 THE INDEFINITE INTEGRAL
A. ANTI-DERIVATIVE AND THE INDEFINITE
CHAPTER 3 INTEGRAL ............................................. 246
LIMIT OF FUNCTIONS B. METHODS OF INTEGRATION ............. 261
A. DEFINITION OF LIMIT .......................... 38
B. INDETERMINATE FORMS .................... 60

CHAPTER 4 CHAPTER 8
CONTINUITY THE DEFINITE INTEGRAL
A. EVALUATING THE DEFINITE
A. CONTINUOUS FUNCTIONS .................. 81 INTEGRAL ............................................ 300
B. TYPES OF DISCONTINUITY .................. 91

CHAPTER 5
DIFFERENTIATION CHAPTER 9
APPLICATIONS OF DEFINITE INTEGRAL
A. INTRODUCTION TO DERIVATIVES ....... 102
A. INTEGRAL AS THE AREA ..................... 314
B. TECHNIQUES OF DIFFERENTIATION ..... 122
B. OTHER APPLICATIONS OF THE DEFINITE
C. DERIVATIVES OF ELEMENTARY
INTEGRAL .............................................. 324
FUNCTIONS ............................................ 145
CHAPTER 1 induction

1
summation notation

2
3
4
multiplication notation

5
6
CHAPTER 2

FUNCTIONS
A function from a set D to a set R is a rule that corresponds each element of D with a single
element of R.

f:D→R
↓ ↓ y
domain range y = f(x)

For each x in D there exist a single element y in a


R such that f(x) = y. f:D®R
f : x ® y= f(x)
Here x is the variable of f, and y = f(x) is the
image of x. x
O b
f(a) = b
b is the image of a

A. DOMAIN AND RANGE OF A FUNCTION


The domains and ranges of many functions are intervals of real numbers. The domain of the
function is the largest set of real x-values for which f(x) is defined. The range of function is
a set which includes at least all images of the elements in its domain. The largest range of
functions is \.
Study the following charts:

Type of function Its form Domain Example

Polynomial y=2x+5 D: ¡
y=anxn+an–1xn–1+...+a0 ¡
function y=2x2–3x+1 D: ¡

For any polynomial function f the domain is \. The range of the polynomial function depends
on the rule of the function. For example, let us draw the graph of the function y = xn and
find the domain and range with respect to parity of n.

y y n
y = xn y=x

when when
n is n is
even odd x
x O
O
f : ¡ ® ¡+ Ç {0} f:¡®¡
¯ ¯ ¯ ¯
Domain Range Domain Range

8 Limit of Function
Type of function Its form Domain Example

2x – 3
y= D: ¡ – {–1}
Rational f(x) x+1
y= ¡–{x:g(x)=0}
function g(x) x2 + 5
y= D: ¡ – {5, –2}
(x – 1)(x + 2)

In rational expressions, the value of denominator can not be equal to zero, so the numbers
which make the denominator zero must be excluded from the domain of f.
1
Let us recall the graph of function y = x– n = n .
x

y y

1
f(x) y=
when y= when xn
g(x)
n is n is
even odd x
O
x
O

f : ¡ – {0} ® ¡+ f : ¡ – {0} ® ¡ – {0}

Type of function Its form Domain Example

Radical
n-odd ¡ y = 3 x2 + 5x D: ¡
function

If the index of radical is odd, radicand may be y


y = f(x)
negative, positive, or zero, so there is no
restriction for the value of x and the domain is
a
. f:D®R
f : x ® y= f(x)
Here is the graph of y = x where n is odd and
n

n > 1. x
O b
The range of the function is the set of real f(a) = b
numbers. b is the image of a

Functions 9
Type of function Its form Domain Example

Rational y = n f(x) y = x2 – 2 D: ¡ – (–ñ2, ñ2)


¡–{x:f(x)<0}
function n-even 2
y = 4 x – 3x + 2 D: ¡ – (1, 2)

y
If the index of radical is even, radicand can not
be negative, so we must exclude the numbers nx
y:ñ
which make the radicand negative from the
domain.
As seen in the figure, since the radicand is
non-negative both the domain and range are the x
+ O
set ∪ {0}.
f : [0, +¥) ® [0, +¥)

Type of function Its form Domain Example

Exponential y = ax y = 3x D: ¡
¡ 2
function (a Î ¡+ – {1}) y = 5x +2x D: ¡

Logarithmic y = loga f(x) y = log(x2 – 4) D: ¡ – [–2, 2]


function ¡ – {x : f(x) £ 0}
(a Î ¡+ – {1}) 2
y = ln(x – 2x – 3) D: ¡ – [–1, 3]

The exponential function is defined in the set of real numbers but the logarithmic function
is defined only for positive real numbers then we exclude the numbers which make the
function negative or zero from the domain of the logarithmic function.
Study the graph of exponential and logarithmic functions:

y y y y
when a > 1 y = xn y = log ax
y = log ax

1 y = ax
1
x x x x
O O 1 O O 1

f : ¡ ® ¡+ f : ¡+ ® ¡ f : ¡ ® ¡+ f : ¡+ ® ¡

10 Limit of Function
Function Domain Range

y = x2 + 2x + 1 –¥ £ x £ ¥ 0£y

1
y= x ¹ –1 x¹0
x+1

y= 1–x –1 £ x £ 1 0£y£1

y = x2 – 2x – 3 –¥ £ x £ ¥ –4 £ y £ ¥

y = log (x2 + 5x) x Î [–5, 0] ¡

x3 – 1
y= 2 ¡ – {2, 3} ¡
x – 5x + 6

For the function which involves the combinations of different types of functions, the domain
is the intersection of the domains of each function.

EXAMPLE 1 Find the domain and range of the function y = x2 +5 x +6.

Solution First, let us find the sign table of the radicand x2 + 5x + 6.

x –3 –2

x2 + 5x + 6 + – +

The radicand is non-negative in the interval intervals (–∞, –3) and [–2, ∞).
So the domain of the function is D: (–∞, –3] ∪ [–2, +∞).
And the range is [0, +∞).
f(x): (–∞ –3] ∪ [–2, +∞) → [0, +∞).

EXAMPLE 2 Find the domain of the function y =


1
x
+ log( x2 + 2 x − 8).

Functions 11
1
Solution f consists of two functions: g( x) = and h(x) = log(x2 + 2x – 8).
x
1
is defined when 0 < x < +∞.
x
+
The logarithmic function is defined in , so let us find the sign table of x2 + 2x – 8.

x –4 2

x2 + 2x – 8 + – +

log (x2 + 2x – 8) is defined when x < –4 or x > 2.


The domain of the f(x) is the intersection of the domains. So the domain of f is (2, +∞).

EXAMPLE 3 Find the domain of f ( x) = x + 3 +


5
x −42
+ log 2(4 x − 3).

Solution Let us find the domain of each seperate function.


⎧x + 3 ≥ 0 ⎧ x ≥ –3
⎪ 2 ⎪⎪ x ≠ 2 and x ≠ –2
By solving the inequalities ⎨ x – 4 ≠ 0 . We get ⎨
⎪4x – 3 > 0 ⎪ 3
⎩ x>
⎪⎩ 4
⎛3 ⎞
Then the domain of f is the intersection of these three intervals which is ⎜ ,+∞ ⎟ − {2}.
⎝4 ⎠

EXAMPLE 4 Find the image set of the following functions in the indicated intervals.
a. f(x) = 3x + 6, x ∈ [0, ∞) b. f(x) = x2 – 2x + 8, x ∈ [–1, 2]
c. f(x) = x2 – 4x – 5, x ∈ [–1, 1] d. f(x) = 2x – x2, x ∈ [0, 3]

Solution Let us solve this problem by drawing the graph of each function in the given intervals.

a. As it is seen in the graph, f(x) is increasing y


function and in this domain the minimum y = 3x + 6
value of f is f(0) = 6, and the maximum
6
value goes to infinity.
So the image set of f is [6, +∞).

x
–2 O

12 Limit of Function
b. The figure shows the graph of the function.
y 2
y = x – 2x + 8
The minimum value of the function is
b −2
f ( − ) = f ( − ) = f (1) = 7. max 11
2a 2 value
And in this interval the maximum value is
f(–1) = 11. 8 min
7 value
So the image set of f is [7, 11].

x
–1 O 1 2

c. x ∈ [–1, 1] y

min f(x) = f(1) = –8


max f(x) = f(–1) = 0
1 2 3 4 5
The image set is [–8,0]. x
–2 –1 O

–5

–8
–9

d. x ∈ [0, 3]
y
min f(x)
max
The image of the function in the given 1

interval is [–3, 1]. 3


x
O
–1 1 2

–2

–3 min

y = 2x – x2

Functions 13
EXAMPLE 5 Find the range of the following functions in their largest domain.
1
a. y = 2sin x – 3 b. y = − x2 + 4x +5 c. y = x
3 +1
Solution a. The trigonometric function y = sinx is defined from to [–1, 1].
For all x ∈ 1 ≤ sinx ≤ 1
–2 ≤ 2sinx ≤ 2
–2 – 3 ≤ 2sinx – 3 ≤ 1
Hence, the range of y = 2sinx – 3 is [-5,-1].
b. As seen in the graph of the function y
g(x) = –x2 + 4x + 5, 9
when –1 ≤ x ≤ 5, –x2 + 4x + 5 ≥ 0.
Then f ( x) = − x2 + 4x +5 is defined in the
interval [-1,5] and in this interval the 5
minimum value is 0 and the maximum value
is 9, then we can write
0 ≤ –x2 + 4x + 5 ≤ 9
x
0≤ –1 O
− x2 + 4x +5 ≤ 9 2 5

0 ≤ f(x) ≤ 3 then the range of f(x) is [0,3].

c. To find the range of the given function, let us start with the range of 3x.

We know that 0 < 3x < +∞ then 0 < 3x + 1 < +∞, when we take the reciprocal of each
1 1 2
side, we get > > 0 and 2 > > 0.
1 3x +1 3x +1
So the domain is (0,2).

EXAMPLE 6 Find the domain of f(x) = log5(log5x).


y
Solution Logarithmic function is defined in positive real g = log5x
numbers. The function g(x) = log5x is defined
from + to . Now, we must specify in which
interval g(x) gets non-positive values.
x
O 1
As it is seen in the graph, when 0 < x < 1,
g(x) = log5x is non-positive, so [0, 1] must be
excluded from the domain.
+
Therefore, the domain of f(x) is – (0, 1].

14 Limit of Function
EXAMPLE 7 Find the domain of f ( x) = y = | x − 1| − | x + 2|.

Solution The radicand |x – 1| – |x + 2| must be non-negative.


|x – 1| – |x + 2| ≥ 0
|x – 1| ≥ |x + 2| when we take the square of the both side, we get
x – 2x + 1 ≥ x2 + 4x + 4
2

–6x ≥ 3
1 1
x ≤ – . So the domain of the function is (–∞, – ].
2 2

EXAMPLE 8 Find the domain of y = 3 − 12 − x2 .

Solution We have the radicands 12 – x2 and 3 − 12 − x2 , and both of them must be non-negative.

12 – x2 ≥ 0
3 − 12 − x2 ≥ 0
9 ≥ 12 – x2
2
x –3≥0
–ñ3 ≤ ñ3

–2ñ3 2ñ3 –ñ3 ñ3

– + – + – +

The domain of the f is the intersection of these


intervals. –2ñ3 –ñ3 O ñ3 2ñ3

The domain is [–2ñ3, –ñ3] ∪ [ñ3, 2ñ3].

EXAMPLE 9 Find the domain of y = log


5x – 1
x+ 2
.

5x – 1
Solution To find the domain we have to solve the inequalities 1. x + 2 ≠ 0, 2. > 0, and
x+ 2
5x – 1
3. log ≥ 0.
x+ 2
1. x + 2 ≠ 0; x ≠ –2
1
5x – 1 –2 5
2. > 0, 1.
x+ 2 x < –2 or x >
+ – + 5

Functions 15
3. log 5x – 1 ≥ 0; log 5 x – 1 ≥ log1 then 5 x – 1 ≥1 3
x+ 2 x+ 2 x+ 2 –2 4
5x – 1 5x – 1 – x – 2 4x – 3 + – +
– 1 ≥ 0; ≥ 0; ≥ 0.
x+ 2 x+ 2 x+ 2

The domain is the intersection of the solution sets.


3
The domain is (–∞, – 2) ∪ [ , + ∞).
4

Check Yourself 1
1. Find the domain of the following functions:
x x +1
a. y = 2
b. y = 3 2 x +1+ x2 – 1 c. y = 2 x − 4
x +2 x −1
1 x+ 2
log 3 ( )
d. y = 2 x − 1 − 3x +1 e. y = 3 x− 2 f. y = 2 x2 − 4

⎛ x+ 2 ⎞
g. y = 3 − log 2 x h. y = log 2 ⎜ ⎟ i. y = log 1 (log 2(2 x+1))
⎝ 2x + 2 ⎠ 2

⎛ 2 x +1 ⎞ 3x +1
j. y = arc cos(2x – 3) k. y = arcsin ⎜ ⎟ l. y =
⎝ x +1 ⎠ log( x2 + x +1) − 1

m. y = 2 – cos x n. y = tan x, x ∈ [0, 2 π)


2+ sin x

2. Find the image of the following functions in the indicated intervals:


a. y = 2x + 1, x ∈ [1, 5) b. y = x2 – 4x – 5, x ∈ [1, 3)

c. y = cos x + sin x, x ∈ [0, 2π) d. y = log3(x2 – 2x – 8), x ∈ [6, 7]


2
e. y = 2x+1, x ∈ [–1, 3) f. y = x + 2, x ∈ ( 2, 7]

3. Find the range of the following functions in their largest domain:


5
a. y = − x2 +7 x − 12 b. y = –3cos x + 1 c. y = x
2 +3

16 Limit of Function
B. COMPOSITE FUNCTIONS
Function which is formed by composition of elementary functions is called composite function.

f f(x) g
x ⎯→ ⎯→ ⎯→ g( f(x))

EXAMPLE 10 Given f(x) = x2, g(x) = sin x, and h(x) = 2x + 5, find


a. f(g(x)) b. g(f(x)) c. f(h(g(x)))

Solution a. f(g(x)) = f(sin x) = sin2 x


b. g( f(x)) = g(x2) = sin x2
c. f(h(g(x))) = f(h(sin x)) = f(2sin x + 5) = (2sin x + 5)2

EXAMPLE 11 Write the following functions in terms of composite of elementary functions.


2
a. f(x) = (3x – 5)3 b. f(x) = 2x –7
c. f(x) = sin2(log37(x2 – 1))

Solution In fact, there is not only one solution for these questions.
The answer depends on our choices.
a. Let us choose g(x) = 3x – 5 and h(x) = x3 = then f(x) = (3 – 5)3 = h(g)(x)).
2
b. Let us choose g(x)= x2, h(x) = 5x – 7, and t(x) = 2x then f(x) = 25x –7
= t(h(g(x))).
2 7 2
c. Let us choose g(x)= x – 1, h(x) = log3x, t(x) = x , and u(x) = sin x then
f(x) = sin2(log37(x2 – 1)) = u(t(h(g(x)))).

Check Yourself 2
3
2x − 1 ⎞ 7 ⎛ x +5 ⎞
1. f ( x) = ⎛⎜ ⎟ 2. f ( x) = 3+ log 2 x 3. f ( x) = sin ⎜ 2 ⎟
⎝ 7 ⎠ ⎝ x +1 ⎠

Functions 17
C. INVERSE OF A FUNCTION
If the function f : D → R is one-to-one and onto then the function f –1 : R → D is called the
inverse of f.
f(x) = y ⇔ f –1(y) = x

To find the inverse of a given function, it is enough to interchange the places of x and y and
express y in terms of the variable x.
y
For example, let us find the inverse of the f(x) = 2x – 1
polynomial function f (x) : → = 2x – 1, y=x
x+1
f(x) =
y = 2x – 1 2

y +1
x= = f −1( y) 1
2

x +1 x
and the inverse function is f −1( x) = . –1 O 1
2
–1
Recall that the graphs of a function and its
inverse are symmetric with respect to the line
y = x.

+
Remember that the exponential function f (x) : → , f (x) : ax, (a ∈ + – {1}) and the
logarithmic function f (x) : + → , f (x) = logax, (a ∈ +
– {1}) are inverse of each other.

18 Limit of Function
As seen in the figure, their graphs are symmetric with respect to y = x.

y y
y = ax
y=x y=x

y = logax

1 1
y = ax
x x
O 1 O 1

y = logax

EXAMPLE 12 Function Inverse

3x + 1 2x – 1
f –1 : ¡ ® ¡, f(x) = f –1 : ¡ ® ¡, f –1(x) =
3 3
–x + 2
f : ¡ ® ¡, f(x) = 2 – 3x f : ¡ ® ¡, f –1(x) =
3

f : ¡ ® ¡ +, f(x) = 3x f : ¡ ® ¡ +, f –1(x) = log3x

2x + 1 x+1
f : ¡ – {1} ® ¡ – {2}, f(x) = f : ¡ – {1} ® ¡ – {2}, f –1(x) =
x–1 x–2

EXAMPLE 13 Find the inverse of the function f (x) : [2, ∞) → [0, ∞), f (x) : x2 – 4x + 4.

Solution In the given domain and range, f(x) is y

one-to-one and onto, so its inverse is a function y = x2 – 4x + 4


and we can find the inverse function as follows:
x2 – 4x + 4 = y
(x – 2)2 = y
x–2 = ñy
x = ñy + 2 = f –1(y) O 2
x

Then, f –1(x) = ñx + 2, f –1 : f (x)[0, ∞) .

Functions 19
EXAMPLE 14
Solution f(x) = 10x–1 + 2 = y
10x–1 = y–2
x–1 = log10(y – 2)
x = log10(y – 2) + 1 = f –1(y)
Then, f –1(x) = log10(x – 2) + 1, f –1 : (2, ∞) → .

EXAMPLE 15 Find the inverse of the function f: (–4, ∞) → , f(x) = –1 + 3log2(x + 4).

Solution f(x) = –1 + 3log2(x + 4) = y


y +1
log2(x + 4) =
3
y+1
( )
x+4=2 3

y+1
( )
x =2 3
– 4 = f –1( y)
x+1
( )
Then, f –1(x) = 2 3
– 4, f –1: → (–4, ∞).

EXAMPLE 16 For the function f : →


equation f(a) = f –1(a).
, f(x) = x3 – 3x2 + 4x – 1, find the real number a satisfying the

y
Solution We know that the graph of a function and its
inverse are symmetric with respect to the line y y = f(x)
y=x
= x. To solve the problem we have to find the
intersection point of f, f –1, and y = x. a
–
–1 y = f (x)
At the intersection point f(a) = f (a) = a.
x
If f(a) = a then a3 – 3a2 + 4a – 1 = a O a

a3 – 3a2 + 3a – 1 = 0
(a – 1)3 = 0
a = 1.

20 Limit of Function
Check Yourself 3
Find the inverse of the following functions:
x
1. f : → , f ( x) = 3 −
2
⎧5 ⎫ 5x − 2
2. f : − {1} → − ⎨ ⎬ , f ( x) =
⎩2 ⎭ 2x − 2
+
3. f : → , f(x) = 52x–1
+
4. f : → , f(x) = log3(2x + 5)

5. f : → , f(x) = x3 – 3x2 + 3x

6. f : → (1, ∞), f(x) = xx–1 + 1

7. f : → (1, ∞), f(x) = 1 + log(3x + 1)


x +1
8. f : – {1} → (1, ∞) – {11}, f ( x) =10 x −1 +1

9. Find the real value a satisfying the equation for the following functions:
a. f(x) = 3x + 1 b. f(x) = 8x3 – 12x2 + 7x – 1

D. CONSTANT, INCREASING, AND DECREASING


FUNCTIONS
Let f : D → R be a function and x1, x2 ∈ D such that x1 < x2.

1. If f(x1) = f(x2), then f is called a constant


function.
f(x) = c, (c ∈ ) is the constant function.
f(x1) = f(x2) = c.

c y=c

x
x1 O x2

Functions 21
2. If f(x1) = f(x2), then f is called an increasing y
function defined on D.
If f : → , f(x) = x, then
f(x1)
f is called non-decreasing x1 < x2 ⇒ f(x1) < f(x2)
function.
f(x2)

Note O x1 x2
x
The increasing function f : → , f(x) = x, is
called the identity function.

3. If f(x1) = f(x2), then f is called a decreasing y


function.
x1 < x2 ⇔ f(x1) < f(x2)
f(x2)
Note f(x1)
If f(x1) > f(x2), then f is called non-increasing x
O x1 x2
function.

EXAMPLE 17 If f(x) = (5 – a)x2 + (b + 2)x – 3 is a constant function, find (a, b).

Solution Since f is constant function, the coefficient of the terms x2 and x must be zero.
5 – a = 0 and b + 2 = 0. Then (a, b) = (5, –2).

EXAMPLE 18 Show that


a. f : [0, ∞) → , f(x) = ñx + x3 is increasing function
1
b. → – {0} → , f ( x) = is increasing function
x

Solution a. Let x1, x2 ∈ [0, ∞) such that x1 < x2.

For all x1 < x2, x1 < x2 and x13 < x23, x1 + x13 < x2 + x2 3 and f(x1) < f(x2).

Note
Sum of increasing/decresing function is also increasing/decreasing.

1 1
b. Let x1, x2 ∈ [0, ∞) such that x1 < x2. For all x1 < x2 , > , f(x1) > f(x2).
x1 x2
So f is decreasing function.

22 Limit of Function
EXAMPLE 19 Classify the following functions for increasing or decreasing in the indicated intervals.
π π
a. y = sin x, x ∈ [ − , ]
2 2
b. y = 0,5x – 3x, x ∈
c. y = ln x + x2, x ∈ +

Solution a. y = sin x is periodic function and its


graph is as in the figure. y

π π 1
And in the interval [− , ], y = sin
2 2
x is increasing –
p p 2p
2 2 p 3 –2p
function. x
–2p 2p –p
–
x 3
⎛1⎞
b. y = ⎜ ⎟ +( −3x) –1
⎝2⎠
x
⎛1⎞
The exponential function ⎜ ⎟ is
⎝2⎠
1
decreasing function because a = <1.
2
The linear function (–3x) is decreasing function. The sum of decreasing functions is also
decreasing, so f is decreasing function.
c. y = ln x + x2
lnx is increasing function because lnx = logex and e = 2,71>1.
x2 is increasing function in +
, so y = ln x + x2 is increasing function.

EXAMPLE 20
1
b. y = 1 – 2x b. x2 – 3x c. y = b. y = cos x
x2

Solution a. f : → , f(x) = 1 – 2x is decreaasing y


y = 1 – 2x
function.

x
O 1
2

Functions 23
3 y
b. In the interval ( −∞, ] , f is decreasing.
2 y = x2 – 3x
3
In the interval [ ,+∞ ), f is increasing.
2 3
2
x
O 3

9
2

c. In the interval (–∞, 0) f is increasing.


y
In the interval (0, +∞), f is decreasing. 1
y=
x2

x
O

d. f(x) = cosx is the periodic function, so let us y


analyse the function in the interval [–π, π].
In the interval [–π, 0], f is increasing. –p O p 2p
x
–2p
In the interval [0, π], f is decreasing.
y = cos x

EXAMPLE 21 Find the value a + b if f ( x) =


( a − 2)x2 + bx + 4
3x + 2
is a constant function.

Solution ( a − 2)x2 + bx + 4
f ( x) = = k (k ∈ )
3x + 2
( a − 2)x2 + bx + 4 = 3kx + 2k

( a − 2) = 0

b = 3k

a = 2, k = 2, b = 6. a + b = 8.

24 Limit of Function
21
2
EXAMPLE The function f ( x) = ( a − 1)x +( b +5)x is an identity function. Find a and b.
2 x +1

Solution Since f is identity function, f(x) = x. Then


( a − 1)x2 +( b +5)x
=x
2 x +1
( a − 1)x2 +()b +5 x = 2 x2 + x

By the equality of the polynomial a – 1 = 2 and b + 5 = 1, so a = 3 and b = –4.

Check Yourself 4
3mx +1
1. f ( x) = is a constant function. Find m.
6 x +5

2. Decide whether the following functions are increasing or decreasing in the given inter-
vals.
a. f(x) = 2x + 1, ∈
b. f(x) = 1 – x ∈
c. f(x) = –x2 – 8x + 1, x ∈ (–4, ∞)
d. f(x) = –x2 – 2x – 1, x ∈ (–∞, 1)
e. f(x) = x3 + 1, x ∈

E. EVEN AND ODD FUNCTIONS


Given a function f : D → R, for any x ∈ D if
1. f(–x) = f(x) then f is called an even function.
2. f(–x) = –f(x) then f is called an odd function.

Remember that for trigonometric functions:


cos (–x) = cos x, so the function y = cos x is the even function.
And sin (–x) = –sin x
tan (–x) = –tan x
cot (–x) = –cot x, so the functions y = sin x, y = tan x, y = cot x are the odd functions.
The graph of even function is symmetric with respect to y-axis.

Functions 25
y y y = x2
1 y = cos x

3p
– p
2 –2p
x
–2p –p p O p 3p
–
2 2 3

x
–1 O

f(–x) = f(x) f(–x) = - f(x)


cos (–x) = cos (x) (–x)2 = x2

The graph of odd function is symmetric with respect to the origin.

y y y
y = x3
y = sin x
3p p 3p
–
p –2p 2 2 2
x p x x
–2p –p O –
2

f(–x) = –f(x) f(–x) = –f(x) f(–x) = –f(x)


sin (–x) = –sin (x) tan (–x) = –tan (x) (–x)3 = x3

EXAMPLE 22 Classify the following functions for being even or odd..


x2 – 2
a. f(x) = x2 + 3x + 2 b. f(x) = 7 tanx + x3 c. f ( x) = d. f(x) = 3x + 3–x
x6 + x8

e. f ( x) = x2 − 6 x +9 + x2 +6 x +9 f. f(x) = |x – 3| + |x + 4|

26 Limit of Function
Solution Let us find f(–x) and compare with the rule of f(x).

a. f(–x) = x2 + 3x + 2, so f is not even and not odd.

b. f(–x) = –7 tanx – x3 = (7tan x + x3) = –f(x), so f is odd.


x2 – 2
c. f (– x) = = f ( x) , so f is even.
x6 + x8
d. f(–x) = 3–x + 3x = 3x + 3–x = f(x), f is even.

e. f (– x) = x2 +6 x +9 + x2 – 6 x +9 = f ( x) , so f is even.

f. f(–x) = |–x – 3| + |- x + 4|, so f is not even and not odd.

EXAMPLE 21 Complete the graph of the function if it is


a. even b. odd

x
O

Solution a. The graph of even function is symmetric b. The graph of odd function is sym-metric
with respect to y-axis. with respect to the origin.

exen-function y odd-function y

x x
O O

Functions 27
Check Yourself 5
Classify the following functions for being even or odd.

a. f(x) = |x| + cos x

b. f(x) = x3 + sin x

c. f(x) = x4 + x2 + 1

d. f(x) = cos x4 – x3 sin x


x
e. f ( x) =
cos( x3 )
sin x + tan x
f. f ( x) = 2 x3

28 Limit of Function
EXERCISES 1 .1
A. Domain and Range of a Function 3. Find the images of the in dicated interuals for the
following functions:
1. Find the domain and range of the following
functions. a. y = 1 – 3x, x ∈ [–2, 4)

a. y b. y b. y = x2 – 2x – 3, x ∈ (2, 4]
y = g(x) c. y = –x2 + 4x + 5,x ∈ [0, 1)

d. y = log 1 (3 x +1) , x ∈ [1, 5)


x
O 5 2

x e. y = x2x + 5, x ∈ [–1, 1)
O
1
f. y = ( )x+2 , x ∈ [–2, 2)
2
y y
c. d.
3
y = h(x) 2 4. Find the range of the following functions in their
–4 –3 1
3 x largest domain.
–2 –1 O
x –1
O –2 a. y = – x2 + 4

b. y = 1 – 2sin x
2
c. y =
2. Find the domain of the following functions: 1+7 x
x+ 3 2
a. y = 5 b. y = log3(x2 + 5x + 6) d. y = – x – 10 x – 9
x–3
1
c. y = 5 x+1 d. y = 9 – | x2 – 4|
B. Composite Function
x2 – 3 x – 4 2x
e. y = f. y = 3x +1
x2 – 1 x–5 5. Given f(x) = ñx, g(x) = x2, and h(x) = x + 1, find
g. y = ln(x2 – 7x + 10) + x2 – 4 the following:

a. g(h(f(x))) b. f(h(g(x)))
1 – 2 sin x
h. y = , (x ∈ [0, 2 π])
2
x
j. y = log 1 ( ) 6. Write the following funtions in terms of composite
x2 – 4
2
of elementary functions.
k. y = log2x–5(x2 – 3x – 10) 1
a. f ( x) = 5 –
x +1
l. y = log2x–5(log2 x2 – 3logx – 10)
1
b. f ( x) = log 3( 2 )
m. y = log(log 2 – 3 log x – 10), x ∈(0, 2 π) x +5

Functions 29
C. Inverse of a Function E. Even and Odd Functions

7. Find the inverse of the following functions: 9. Classify the following functions for being even or
2−x odd.
a. f: → , f ( x) =
5 e. f(x) = x5 + x3 + x
–2 x – 3
b. f: – {3} → , – {2}, f ( x) = cos x + x2
x+ 3 f. f ( x) =
+
3+ x4
c. f: → , f(x) = 2 ⋅ 33x+4
g. f ( x) = ( x3 ⋅ tan x
d. f: +
→ , f(x) = 2 + ln(5x – 4) x + sin x

e. f: [1, +∞) → , f(x) = 3x2 – 8x + 5

8. Find the real value a stasfying the equation


f(x) = f –1(x) for the following functions:

a. f(x) = 5x – 2

b. f(x) = x3 – 6x2 + 13x – 8

D.Constant, Increasing, and


Decreasing Functions

ax2 + 2 x + b
9. f ( x) = is a constant function. Find
3x2 + bx + 2b
(a, b).

10. Determine whether the following fucntions are


increasing or decreasing in the given intervals:

a. f(x) = x2 – 6x + 1, x ∈ (3, ∞)

b. f(x) = –x2 + 4x – 3, x ∈ (–∞, 2)

c. f(x) = –x3 + 3, x∈
π
d. f(x) = sin x, x ∈ (0, )
2

30 Limit of Function
A. PIECEWISE FUNCTION
The function which is defined by applying different formulas at different parts of its domain
is called piecewise function.

⎧2 x + 1 if x > 2

EXAMPLE 25 ⎪
Given \ → \ f ( x) = ⎨ x2

if 0 ≤ x ≤ 2

⎪⎩ – x2 – 1 if x < 0

a. Draw the graph of f. b. f(–5) + f(2) + f(3) = ?

Solution a. y b. f(–5) = –(–5)2 – 1 = – 26


f(2) = 22 = 4
5
4 f(3) = 2 ⋅ 2 + 1 = 7
3
f(–5) + f(2) + f(3) = –26 + 4 + 7 = –15
2
1
x
O 1 2
–1

EXAMPLE 26 The domain of the function f(x) graphed in the y


figure is [0, 3]. Find a formula for f(x).
2

Solution The graph consists of three line segments. 1


1 0,5
The left segment is valid for 0 ≤ x < 1 and y = .
2 x
O 1 2 3
The second segment is valid for 1 ≤ x < 2 and
y = 1. The third segment is valid for 1 ≤ x < 2
and y = 1. Then the formula of f(x) is found
piecewisely.
⎧1
⎪ 2 if 0 ≤ x ≤ 1

f ( x) = ⎨
1 if 1 ≤ x ≤ 2


⎩2 if 2 < x ≤ 3
Functions 31
EXAMPLE 27 Draw the graph of the piecewise function f : → .

⎧ x2 – 1 if x < –1 or x > 1

⎪ 1 if x = –1 or x = 1 y
f ( x) = ⎨ –
⎪ 2
⎪ x2 if –1 < x < 1

Solution The curve of the parabola y = x2 – 1is drawn for


the interval (– ∞, –1) ∩ (1, ∞). x
O
1 1
Since f ( −1) = − and f (1) = − ,
2 2
1 1
plot the points ( −1, − ) and (1, − ).
2 2
The curve y = –x2 is drawn for the interval (–1,1).

Check Yourself 6 ⎧ x2 if x > 3



1. Given the piecewise function f : → , f ( x) = ⎨3 x+ 4 if 0 ≤ x ≤ 3 , find f(f(f(–1))).
⎪ x3 + 2 if x < 0

2. Find the image of the following functions in the indicated intervals:
⎧ x if x ≥ 1
a. f : → , f ( x) = ⎨
⎩ − x if x < 1
⎧ x2 if x > 1
b. g : → , f ( x) = ⎨ 2
⎩ − x if x ≤ 1
⎧2 x − 1 if x < 0
c. f : ⎪
→ , f ( x) = ⎨ 1 if 0 ≤ x <1
⎪ 1 − x if x ≥ 1

⎧ ln x if 0 < x ≤ e
d. f : → , f ( x) = ⎨ x
⎩ e if x ≤ 0 or x > e

32 Limit of Function
B. ABSOLUTE VALUE FUNCTION
Let f : → . be a function. The absolute value
of f(x) is the function which make the negative
images of the function positive:
⎧ f ( x) if f ( x) ≥ 0
f ( x) = ⎨
⎩− f ( x) if f ( x) < 0

EXAMPLE 28 Draw the graph of y = |x – 3|.

First Solution After drawing the graph of y = x – 3, we can simply draw the graph y = |x – 3| by reflecting
the negative part.

Second Solution When, |x – 3| = x – 3


when x < 0, |x – 3| = –(x – 3) = –x + 3.
⎧ x – 3 if x ≥ 3
That is y = ⎨ . Then we can its graph.
⎩ – x + 3 if x < 3
To solve the absolute value equations or inequalities, we must specify the intervals in which
the function get the negative value, positive value, or zero.

Functions 33
EXAMPLE 29 Draw the graph of |x2 – 1 |.

y
First Solution Let construct the sign table for x2 – 1.

x –1 1 1

x2 – 1 + – +
x
–1 O 1
x2 – 1 gets the positive value if x < –1 or x > 1
x2 – 1 gets zero value if x = –1 or x = 1
And for these values of x, |x2 – 1| = x2 – 1
x2 – 1 gets the negative value if –1 < x < 1. And for these values of x,
⎧ x2 – 1 if x ≤ –1 or x ≥ –1
So x2 ⎨ 2
⎩ x +1 if – 1 < x <1
And now we can draw its graph.

Second Solution There is another method to draw the graph of y = |x2 – 1|. First let us draw the graph of
y = x2 – 1 and then take the symmetry of negative y-valued part with respect to x - axis.
y y
y = x2 – 1 y = |x2 – 1|

1 1

x x
–1 O 1 –1 O 1

EXAMPLE 30 Draw the graph of y = |x – 1| + |x + 2|.


y

Solution First let us define the function in pieces.


3
x –2 1
x–1 – – +
x
x+2 – + + –2 O 1

y –2x – 1 3 2x + 1

⎧ −2 x − 1, if x < −2

y = ⎨ 3, if −2 ≤ x < 1
⎪2 x +1, if x ≥1

34 Limit of Function
EXAMPLE 31 Draw the graphs of the following absolute value functions.
a. f : → f(x) = |sinx| b. f : +
→ f(x) = ln x

Solution a.

b.

Check Yourself 7
Draw the graph of the following absolute value functions:

1. y = |2x – 3| 2. y = |x3| 3. y = x2 + 2x – 3 4. y = x2 + 2x – 3

Functions 35
EXERCISES 1 .2
A. Piecewise Function B. Absolute Value Function
1. Given 3. Express the following functions given in absolute
⎧ x – 1, if x>2 value as a function defined in piece.
⎪ f ( x)+ f (2)
f ( x) = ⎨ x, if 0 < x ≤ 2, find .
⎪ f (3) – f (1) a. f(x) = |x + 3|
⎩ 4x, if x≤0
b. f(x) = |x| + x
c. f(x) = |x2 – x – 2|
d. f(x) = |x – 2| + |x – 3|
e. f(x) = |x + 1| + |x – 1|

4. Draw the graph of the following functions


a. y = |–x| b. y = |2 – 4x|
c. y = |x2 – 1| d. y = |x2 – 4x – 5|
e. y = |log x| f. y = |cos x|
2. Draw the graph of the following piecewise
functions:
⎧⎪ 1, if x > 2
a. f ( x) = ⎨
⎪⎩ –2, if x ≤ 1

⎧⎪ 2 x + 4, if x >1
b. f ( x) = ⎨
⎪⎩ – x, if x ≤1

⎧ x2 +1, if x > 0
c. f ( x) = ⎪⎨
2
⎪⎩ x – 1, if x ≤ 0
⎧⎪ cos x, if 0 < x < π
d. f ( x) = ⎨
⎪⎩ cos x, if π < x < 2 π

Functions 36
CHAPTER 3

LIMIT OF FUNCTIONS
A. CALCULATION OF LIMIT
The brief and the most generalized definition of limit of a function at given point is so
simple. It is the image of the point which is getting closer to that point.
For easier comprehension let us start with the limit of polynomial function
f(x) = anxn + an–1xn–1 + ... + a0.
The limit of polynomial function f(x) when x approaches given point c is f(c) which is the
image of c in this function f. We can denote the limit of f(x) at point c as
lim f ( x) = f ( c).
x→ c
n n–1
If f(x) = anx + an–1x + ... + a0,

lim f ( x) = f(c) ancn + an–1cn–1 + ... + a0.


x→ c

For example, let us calculate the limit of f(x) = 2x when x approaches 5.

lim f ( x) = f(c) ancn + an–1cn–1 + ... + a0.


x→5

EXAMPLE 1 a. lim(4 x – 1) = ?
x→ 2
b. lim( x2 + 3x + 2) = ?
x →–1
c. lim(3t – t2 ) = ?
t →4

Solution a. lim(4 x – 1) = 4 ⋅ 2 – 1= 7
x→ 2

b. lim( x2 + 3 x + 2) = (–1) 2 + 3 ⋅(–1)+ 2 = 0


x→ 2

c. lim(3t – t 2 ) = 3 ⋅ 4 – 4 2 = –4
t→ 4

Check Yourself 1
1. lim 5 x = ? 2. lim x(2 – x) = ?
x→0 x→ 3

3. lim x( x2 – 3x – 15) = ? 4. lim 2 x( x +1) = ?


x→6 x→6

5. lim 3t2 (2t – 1) = ? 6. lim7 = ?


x → –3 x→ 3

Answers
1. 0 2. –3 3. 3 4. 2a2 + 2a 5. –189 6. 7

38 Limit of Function
B. GRAPHICAL ILLUSTRATION OF LIMIT
Now, let us draw the graph of function f : → , f(x) = x + 1 and find lim f ( x).
x→ 3

4.1 y

4.05
y=x+1
4.03
5
4.01

4 4
3.99
3
3.97
2
3.95

3.9 1

x
O 1 2 4 5

2.99 2.85 2.97 2.99 3 3.01 3.03 3.05 3,1

We can approach point 3 from two directions: the right and the left. In both approaches the
limit of f(x) = x + 1 when x approaches 3 is lim( x + 1) = 3+1= 4.
x→ 3

That is to say f(x) is approaching 4 when x approaches 3.

EXAMPLE 2 Given piece-wise function


⎧⎪ – x – 1, x ≠ 2
f: → , f ( x) = ⎨ , find lim f ( x).
x→ 2
⎪⎩ 3, x=2

Limit of Functions 39
Solution Let us draw the graph of f(x), y
x = 2 is a critical point. In this function
3
f(2) = 3 but 3 is not the limit of f at 2. Because
limit is not the value of f at this point, it is the 2
value which f(x) approach when x approach to
1
2. And as seen in the graph we can say that the
–1 1 2
limit of f(x) when x approaches 2 is –3 that is to x
O
say f approaches –3 at that point.
–1

–2

–3

Note
What happens at the given point is not important
for the limit of function at that point.

⎧ 5
⎪ 4, if x<
2

⎪⎪
EXAMPLE 3 Given piece-wise function f : → , f ( x) = ⎨ 2, if

5
x = , find lim f( x).
2 x→
5
2

⎪ 2 x – 1, if 5
x>
⎪⎩ 2
Solution Let us first draw the graph of function, As seen y
5 y = f(x)
in the graph x = is critical point and when we 6
5 2
approach from right hand side, that is to say,
2 5
5
when x is greater than we use the function
2
4
f(x) = 2x – 1 and lim 2 x – 1 = 4. We can say that
x→5
3
f approaches 4 from right-hand side.
2
When we approach x from left-hand side, that
5 5
is to say, x is less than but very close to we 1
2 2
use the function f(x) = 4 which is constant x
O 1 2 5 3 4
function and its limit is 4 when x approaches 2
from left hand side. So lim f ( x) = lim 4 = 4.
x→5 x→5

As a result, f approaches 4 from both sides.

40 Limit of Function
⎧ x2 – 3, if x < –2
⎪⎪
EXAMPLE 4 Piece-wise function f(x) is given as f : → , f ( x) = ⎨ 2,

if x = –2, find lim f ( x).
x →−2

⎪⎩ x + 5, if x > –2
y
y = f(x)
Solution We can draw the graph of function by 6
considering the critical point x = –2.
5
In this graph the critical point is x = –2.
4
When x is greater than –2, that is to say, when
we approach –2 from right hand side, the 3
function f(x) = x + 5 and lim x +5 = 3,
x →–2
2
f approaches 3.
On the other hand, when x is less than –2, it 1

means, approaching from left hand side the x


2 –2 –1 O
function f(x) = (x2 – 3) and lim( x – 3) =1,
x →–2

Notice that in this function we get different


results with respect to direction of the limit we
took. We will explain and interpret the limit of functions at this kind of points later.

Check Yourself 2
1. Given f : → , f(x) = 3, lim f ( x) = ?
x→ 2

2. Given f : → , f(x) = –x – 2, lim f ( x) = ?


x→ 3

⎧⎪ – x2 , if x ≠ –2
3. Given f : → , f ( x) = ⎨ , lim f ( x) = ?
x → –2
⎪⎩ 1, if x = –2
⎧ x2 – 1, if x >1


3. Given f : → , f ( x) = ⎨ 1, if x =1, lim f ( x) = ?
x →1

⎪⎩ x2 +1 if x < 1

Limit of Functions 41
C. DEFINITION OF LIMIT
In the previous section we followed the instruction to help student to get the general concept
of limit but mathematicians like definitions and now we need to define what the limit is.
Before explaining the definition of limit, let us study on one more example to get better
comprehension.
x2 – 4
Let us find the limit of function f : – {2} → , f ( x) = when x approaches 2 that is
x–2
x2 – 4
lim .
x→ 2 x – 2

Examine that we need to use somewhat different y


method to take the limit of this function because
x2 – 4
f ( x) = is undefined at x = 2. 4
x–2
But it must be emphasized that for limit it 3

doesn't matter what happens at that point, the 2


important point is what happens when we
1
approach that point, so by applying simple
x
factorization methods we can get O 1 2
x2 – 4 ( x – 2)2 ( x + 2)
lim = lim
x→ 2 x – 2 x→ 2 ( x – 2)
= lim( x + 2) = 2+ 2 = 4.
x→ 2

x–2
We have been able to simplify that =1, because x is very close to 2 but not equal to 2,
x–2
x–2
so is not zero over zero and can be simplified. And here the graph of function
x–2
x2 – 4
f : – {2} → , f ( x) = .
x–2

Neighborhood
Let x0 be a real number and a be a positive real number less than 1. Now let us think about
the real number x between x0 – a and x0 + a such that x0 – a < x < x0 + a.
x is an element of the interval (x0 – a, x0 + a) and this interval is called the
a-neighborhood of x0.

x0 – a x0 x0 + a

a-neighborhood of x0

x is a real number between x0 – a and x0 + a.

42 Limit of Function
For example let us take x0= 5 and a = 0.1 then 0.1-neighborhood of 5 is the interval
(5 – 0.1, 5 + 0.1) = (4.9, 5.1)

x0 – a x0 + a
4 5 6

0,1-neighborhood of 5.

Check Yourself 3
1. Find 0.01-neighborhood of 7 and show the interval on number line.
5
2. Find 0.2-neighborhood of and show the interval on number line.
2

ε - neighborhood
Now let x be an element of ε-neighborhood of x0.
x ∈ (x0 – ε, x0 + ε)

x0 – e x0 x0 + e

e-neighborhood of x0

x0 – ε < x < x0 + ε
Let us add x0 to all sides of inequality –ε < x – ε < ε
This inequality can be written simply in absolute value |x – x0| < ε
For example let us say x ε (3.8, 4.2)
x ε (4 – 0.2, 4 + 0.2)
4 – 0.2 < x < 4 + 0.2
–0.2 < x – 4 < 0.2
|x – 4| < 0.2
x is real number in 0.2-neighborhood of 4.
x0 – a x0 + a
3 4 6

x is here

Limit of Functions 43
Definition
Let f : → be a function, x0 and L be real numbers.

Given any ε about L if there exist δ about x0 such that for all x
|x – x0| < δ ⇒ |f(x) – L| < δ then
the limit of f is L as x approaches the point x0 and it is denoted by lim f ( x) = L.
x→0

|x – x0| < δ
–δ < x – x0 <δ
x ∈ (x0 – δ, δ + x0) δ-neighborhood of x0.

x0 – d x0 x0 + d

L+d
And |f(x) – L| < ε
– < f(x) – L < ε L

f(x) ∈ (L – ε, L + ε) ε - neighborhood of L. L–d

y
y = f(x)

L+e

L–e

x
O x0 – d x0 x0 + d

⎧⎪ 2 x – 1, if x ≠ 2
EXAMPLE 5 Given f : → , f ( x) = ⎨
⎪⎩ 1, if x = 2
show that lim f( x) = 3 by using definition of limit.
x→ 2

44 Limit of Function
Solution To prove that lim f (2 x – 1) = 3 we need to show that
x→ 2

∀ ε > 0, ∃ δ > 0 such that |x – 2| < δ ⇒ |f(x) – 3| < ε.


Now let us start with the second part of the above expression which is |f(x) – 3| < ε.
And let us try to show that for all ε > 0 we can find a number δ.
|f(x) – 3| < ε ⇔ |(2x – 1) – 3| < δ
y
ε
Now notice that we have got x which y = f(x)
2 4
is similar with the first part of the above 3+e
3
expression that is |x – 2| < δ. 3–e

Therefore, we found that there is a real number 2


ε ε
δ which is equal to , so choose δ = . 1
2 2
ε x
And we can say that ∀ ε > 0, ∃ δ = > 0 and O
2 –1 1 2 3 4
ε 2– e 2+e
|x – 2| < ⇒ |f(x) – 3| < ε. 2 2
2 ε ε
For x ∈ (2 – δ, 2 + δ) = (2 – , 2 + ),
2 2
f(x) ∈ (3 – ε, 3 + ε)
As ε → 0 (goes to 0), the point x approaches
2 and the value of f(x) goes to 3.
Thus, lim f (2 x – 1) = 3 .
x→ 2

EXAMPLE 6 Show that lim f (3x – 5) = 4 by using the definition of limit.


x→ 3

Solution We will use the definition of limit with x0 = 3, f(x) = 3x – 5 = 4 and L = 4.


To satisfy the definition, we need to show that for any ε > 0, there exist δ > 0 such that for
all x. 0 < |x – 3| < δ ⇒ |(3x – 5) – 4| < ε.
Now let us change the second part of the above expression like that
|(3x – 5) – 4| < ε
|3x – 9| < ε
ε
3 ⋅ |x – 3| <
3

Limit of Functions 45
All these inequalities are equivalent, y y = f(x)
ε
so our definition is hold when |x – 3| < .
3 4+e
ε
Therefore, we take δ = . 4
3 4–e
Here, we have to emphasize that the value 3
ε
δ = is not only value of δ that will satisfy the 2
3
definition.
1
Any smaller positive δ will work as well.
x
Definition asks for us to find just one δ that O 1 2 3 4
satisfies the inequality not the best one. 3– e 3+e
2 2

Check Yourself 4
By using the definition of limit prove the following and find the value of δ.

1. lim(7 x – 2) = 5
x →1

2. lim(4 x + 3) = –5
x →–2

Answers
ε ε
1. 2.
7 4

D. ONE-SIDED LIMITS: RIGHT-HAND LIMITS AND


LEFT-HAND LIMITS
As we have confronted before sometimes the limit of function gets different values as x
approaches a number x0 from different sides. When this happen, we call the limit of f as x
approaches x0 from the right the right-hand limit of f at x0 and denote as
lim f ( x),
x→ x0+

and, the limit of f as x approaches x0 from the left the left-hand limit of f at x0 and denote as
lim f ( x) .
x→ x0 –

46 Limit of Function
Let us study on the function y = f(x) graphed in
the figure, and find the one sided limits at the y

points 2, 5, and 6. 5
a. lim– f ( x) = (3), but lim+ f( x) = 5
x→2 x→2 4
b. lim– f ( x) = (3), but lim+ f( x) = 2
x→5 x→5 3
c. lim– f ( x) = 3 and lim+ f ( x) is not exist.
x→6 x→6 2
As a result, at point 2 one-sided limits tend to
get different values: 3 and 5; at point 5 get the 1

same value: 2; at point 6 only left-hand limit


x
exists. O 1 2 4 5 6 7

Definition
a. The limit of the function f(x) as x approaches y
x0 from the right equals L if: for any ε > 0
L+e
there exists δ > 0 such that for all x,
L
x0 < x < x0 + δ ⇒ |f(x) – L| < δ
for all x in the interval (x0, x0 + δ) L–e

x
O x0 x0 + d

y
b. The limit of the function f(x) as x approaches L+e
x from the left equals L if: for any ε > 0 there
L
exists δ > 0 such that for all x,
x0 – δ < x < x0 ⇒ |f(x) – L| < ε L–e

for all x in the interval


(x0 + δ, x0), f(x) ∈ (L – ε, L + ε). O
x
x0 – d x0

Limit of Functions 47
EXAMPLE 7 5 5
Find the one-sided limits at all integer values of f : [– , ] →
2 2
which is graphed in the
figure.
y
Solution a. lim+ f ( x) = 0, lim– f ( x) = 0
x→ 2 x→ 2
3

b. lim+ f ( x) =1, lim– f ( x) = 0


x →1 x →1
2

c. lim+ f ( x) = –1, lim f ( x) = –1 1


x→0 x →0–

x
d. lim f ( x) = –1, lim f ( x) =1 O
x → –1+ x → –1– –2 –1 1 2

–1
e. lim f ( x) = 0, lim f ( x) = 0
x → –2+ x → –2 –

Definition existence of limit


A function f(x) has a limit at point x0 if and only if the right-hand and left-hand limits at x0
exist and are equal. y

a
lim f ( x) = L ⇔ lim+ f ( x) = L and b
x → x0 x→ x

lim f ( x) = L
x → x0 – x2
x
x1 O
The function f has limit at point x2 but has no
a
limit at point x1.

⎧⎪ x – 1, if x > 2
EXAMPLE 8 Given f : – {2} → , f ( x) = ⎨
⎪⎩ – x + 3, if x < 2
, find the limit of f(x) at the point x0 = 2

y
Solution The point x0 = 2 is the critical point of f(x).
Therefore, let us examine the one-sided limits
at the point 2.
3
lim+ f ( x) = lim(
+
x – 1) = 2 – 1=1
x→ 2 x→ 2
2
lim– f ( x) = lim(–

x+ 3) = –2+ 3 =1
x→ 2 x→ 2
1
Since lim+ f ( x) = lim– f ( x) =1, they exist
x→ 2 x→ 2
x
O 1 2 3
and are equal, then lim f ( x) =1.
x→ 2

48 Limit of Function
Check Yourself 5 y

1. The graph of f : [1, 7] → , f(x) is given in the figure. Find one-sided 7


6
limits at all integer values of the domain and find at which point f 5
has a limit. 4
3
⎧⎪ x2 +1, if x >1 2
2. Given a function f : – {1} → , f ( x) = ⎨ , find the 1
limit of f(x) at the point x0 = 1. ⎪⎩ x – 1, if x < 1 x
O 1 2 3 4 5 6 7

⎧ x, if –1 ≤ x < 0 or 0 < x ≤ 1
⎪⎪
3. Given a function f : → , f ( x) = ⎨ 1, if x=0

⎪⎩ 0, if x < –1 or x < – 1 or x >1
a. Find one-sided limits for the points –1, 0, and 1.
b. At which point does limit of f(x) exist?

E. LIMIT OF SPECIAL FUNCTIONS


We have learnt the definition and basic concepts of the limit. Now, we will learn about the
limit of some special functions: Absolute value function, Sign function, and Greatest integer
function.
We know that at a given point, limit exists if the right-hand limit and left-hand limit exist and
are equal.
Like the limit of piecewise function, to find the limit of functions involving absolute value,
sign, or greatest integer functions we have to check for one-sided limits: the right-hand limit
and left-hand limit. If they exist and are equal then we can say limit exist at the given point.

EXAMPLE 9 Given f : – {2} → , f ( x) =


| x – 2|
x–2
+ x + 3 , find lim+ f ( x) and lim– f ( x).
x→ 2 x→ 2

Solution Since the function f involves the absolute value expression |x – 2|, x0 = 2 is the critical point
for f.
Then we need to approach 2 from both right and left sides.
If x > 2, x – 2 > 0 and |x – 2| = x – 2 then
| x – 2| x– 2
f ( x) = + x+ 3 = + x + 3 =1+ x + 3 = x + 4.
x–2 x–2

Limit of Functions 49
If x < 2, x – 2 < 0 and |x – 2| = –(x – 2) then y
| x – 2| – ( x – 2)
f ( x) = + x+ 3 = + x+ 3 6
x–2 x–2
5
= –1+ x + 3 = x + 2. 4
3
⎧⎪ x + 4, if x > 2 2
So f ( x) = ⎨ 1
⎪⎩ x + 2, if x < 2 x
–3 –2 –1 O 1 2 3
lim f ( x) = lim+ f ( x + 4) = 2+ 4 = 6
x → 2+ x→ 2

lim f ( x) = lim– f ( x+ 2) = 2+ 2 = 4
x → 2– x→ 2

Remark
In the limit of special function when we need to find the right or left hand limits, sometimes
it is very useful to define a positive real number h which goes to zero.
+
For h ∈ and h → 0, the following limits are
x (x ® 3+)
equal: 3

lim f ( x) = lim f ( a + h) and 3 + x (h ® 0)


x → a+ x→0

(x ® 3–) h
lim– f ( x) = lim f ( a – h) 3
x→ a x→0

+ (h ® 0) 3–h
For exampe, when h ∈ and h → 0,
lim f ( x) = lim f (3+ h) and
x → 3+ x→0

lim f ( x) = lim f (3 – h)
x → 3– x→0

EXAMPLE 10 Given f(x) = sgn(4 – x), find lim+ f ( x) and lim– f ( x) . Decide whether lim f ( x) exist or not.
x→ 4 x→ 4 x→ 4

First Solution x0 = 4 is the critical point.


y
When x is greater than four (x → 4+), 4 – x is y = sgn(4 – x)
negative and sgn(4 – x) = –1 then
x
lim f ( x) = lim+ sgn(4 – x) = –1. O 1 2 3 4
x → 4+ x→ 4 –1
When x is smaller than four(x → 4–), 4 – x is
positive and sgn(4 – x) = 1 then
lim f ( x) = lim– sgn(4 – x) =1.
x → 4– x→ 4

50 Limit of Function
Check Yourself 6
Evaluate the following limits:
x +| x | | x2 – 4|
1. lim( x2 | x +1|) 2. lim 3. lim
x →–1 x – 2
x →–3 x →–1 x

Answers
1. 18 2. 0 3. –4

F. INFINITY AS A LIMIT

In this section we will use the word "infinity". In fact, there is no real number infinity, infin-
ity is a concept which explain us the stuation in which domain or range of some functions
exceed all the bounds.
Let us study on the limits of the function
graphed in the figure. y

f: – {0} → – {c} graphed in the figure.


What about the following limits at the point x0 = 0:
lim f ( x) and lim+ f ( x). c
x →0– x→0

As seen in the figure when we approach to zero x ® 0– x ® 0+

from the left-hand side the value of f(x) gets x


O
larger and larger. For any chosen number M, if
x approachs zero close enough from the
left-hand side, the value of f(x) exceeds M. We
say f(x) tends to positive infinity:
And also, when we approach to zero from the right-hand side the value of f(x) gets smaller
and smaller. For any chosen number –M, if x approachs zero close enough from the
right-hand side, the value of f(x) be less than –M. We say f(x) tends to negative infinity:
lim f ( x) and lim f ( x).
x →+∞ x→ – ∞

Limit of Functions 51
In both cases, f(x) gets the infinite limits as x y
approaches to a real number.
Now, what about the following limits of f(x):
lim f ( x) and lim f ( x).
x →+∞ x→ – ∞

c
As it is seen in the graph of the function, when
x ® –¥ x ® +¥
the value of x increases, the value of f(x)
x
approaches a number c. For chosen any ε > 0, O
we can find a number M, such that for all
y = f(x) ® +¥
x > M, the value of f(x) will be in the
ε-neighborhood of c, that is f(x) approach the
number c closer and closer. Then we can write
lim f ( x) = c.
x →+∞

Similarly, if x tents to negative infinity by getting smaller and smaller values, the value of f(x)
approaches the number c as seen in the figure. For chosen any ε > 0, we can find a number
N such that for all x < N, the value of f(x) will be in the ε-neighborhood of c, that is f(x)
approaches the number c, while x goes to negative infinity. Then we can write
lim f ( x) = c.
x →+∞

In both limits, when x approaches negative or positive infinity, f(x) approaches real numbers.

EXAMPLE 16 Given f : – {0} →


1
, f ( x) = , and study on the graph to find the limits:
x
a. lim+ f ( x) b. lim– f ( x) c. lim f ( x) d. lim f ( x)
x→ 0 x→ 0 x →+∞ x→ – ∞

Solution a. When x gets closer to zero from the right-hand y


side, f(x) approaches +∞.
1 y=1
If x = 0.00001, =10000 x
x
b. When x gets closer to zero from the left-hand
side, f(x) approaches –∞. x ® 0– x ® 0+
1 x
If x = –0.00001, = –10000 O
x
1
lim = – ∞.
x→ 0– x

52 Limit of Function
c. When x approaches –∞, f(x) approaches zero. y
1
If x =1.000.000, = 0.000001
x y=1
x
1
lim = 0.
x →+∞ x
x ® –¥
d. When x approaches –∞, f(x) approaches
x
zero. O
x ® –¥
1
If x = –10.000.000, = –0.0000001
x
1
lim = 0.
x→ – ∞ x

EXAMPLE 17 Find the limit of the function f : – {0} → , f ( x) =


2 x2 – x
x2 – 4
, as x approaches to +∞.

1
x2 (2 – )
2 x2 – x x
Solution lim f ( x) = lim 2 = lim
x →+∞ x→+∞ x – 4 x→+∞ 4
(1+ 2 )
x
0
1
(2 – )
4 x = 2.
Since x → +∞, and 2 → 0 so, lim
x x →+∞ 4
(1+ 2 )
x 0

EXAMPLE 18 Find the limit of the function f ( x) =


1
x –1
at x0 = 1.

Solution The function f is not defined at x0 = 1 and y


x0 = 1 is the critical point. y= 1
x–1
So let us check for the right and left-hand
limits.
Let h > 0 be a very small number, then
x ® –¥
1
1 1 1 x
lim f ( x) = lim+ = lim = lim =+ ∞. O –
x®1 x ® 1+
x →1+ x →1 x – 4 h → 0 1+ h – 1 h→0 h

1 1 1
lim f ( x) = lim– = lim = lim = – ∞.
x →1– x →1 x–1 h → 0 1– h – 1 h → 0 –h
The right and left-hand limits are not equal, so
lim f ( x) does not exist.
x →1

Limit of Functions 53
Remark
For some functions f : → , the following infinite limits are possible:

a. lim f ( x) =+∞ b. lim f ( x) = – ∞


x →+∞ x →+∞

c. lim f ( x) =+∞ d. lim f ( x) = – ∞


x→ – ∞ x→ – ∞

For example, let us consider the limit of the following polynomial functions:
lim x3 =+∞ lim x3 = – ∞ lim x2 =+ ∞ lim (– x2 ) = – ∞
x →+∞ x→+∞ x→+∞ x→+∞

y y y

+
x®1 x ® 1+
x ® –¥ x ® –¥ x ® +¥
x x x
O O O
x ® +¥

Check Yourself 7 y

1. For the function graphed in the figure, find the following


limits of:
2
a. lim f ( x) b. lim f ( x)
x →+∞ x→ – ∞

x
c. lim f ( x) d. lim f ( x) O 1 3
x →1 x →3

2. Perform the following limits:


2x + 3 x3 +1
a. lim b. lim
x →+∞ 5 x + 4 x →+∞ x4
x2 – 2 x + 3 | x|
c. lim d. lim
x →+∞ 2 x2 + 3x – 1 x→ – ∞ | x |+1
1 – x2 x2 – 2
e. lim f. lim
x →+∞ 1+ 3 x2 x→ – ∞ 4x – 2

54 Limit of Function
3. Perform the following limits:
1 1 1 1
a. lim 2 b. lim– c. lim+ d. lim
x →0 x x →1 x – 1 x→ 2 x2 – 4 x→ e 1 – ln x

e. lim sin x f. lim 5x – 1


x →π+ sin x x→ 2 2 – x

4. Perform the following limits:

a. b. lim(1 – x – x2 ) c. 3
lim( x5 + x4 + x3 ) lim(2 x – )
x→ – ∞ x →∞ x→ – ∞ x
Answers
1. a. 2 b. 2 c. +∞ d. not exist
2 1 1
2. a. b. 0 c. d. 1 e. – 1 f. –
5 2 3 4
3. a. +∞ b. –∞ c. +∞ d. not exist e. +∞ f. +∞
4. a. –∞ b. –∞ c. –∞

Theorem Limit Combination Theorem


a. lim[ f ( x)+ g( x)]= a + b
x→ x0

b. lim[ f ( x) – g( x)]= a – b
x→ x0

c. lim[ f ( x) ⋅ g( x)]= a ⋅ b
x→ x0

f ( x) a
d. lim[ ]= , (b ≠ 0)
x→ x0 g( x) b
e. lim k ⋅ f ( x) = k ⋅ a (k ∈ )
x→ x0

EXAMPLE 19 Given f(x) = 5 and g( x) =


1
x
(x ≠ 0), evaluate the following limits:

f ( x)
a. lim[ f ( x) + g( x)] b. lim[ f ( x) ⋅ g( x)] c. lim[ ]
x→ –2 x→+∞ x→–3 g( x)

1 1 9
Solution a. lim[ f ( x)+ g( x)]= lim f ( x)+ lim g( x) = lim 5+ lim = 5+ (– ) = .
x → –2 x → –2 x → –2 x → –2 x → –2 x 2 4
1
b. lim [ f ( x) ⋅ g( x)]= lim f ( x) ⋅ lim g( x) = lim 5 ⋅ lim = 5 ⋅0 = 0.
x →+∞ x →+∞ x→+∞ x→+∞ x→+∞ x
f ( x) 5
c. lim[ ]= lim = ⋅ lim 5 ⋅ x = 5 ⋅ lim x = 5(–3) = –15.
x → –3 g( x) x → –3 1 x → –3 x →–3

Limit of Functions 55
EXAMPLE 20 Given f : – {0} → , f ( x) =
1
1+ 22 / x
, find the following limits:

f ( x)
a. lim[ f ( x) + g( x)] b. lim[ f ( x) ⋅ g( x)] c. lim[ ]
x→ –2 x→+∞ x→–3 g( x)

1 1 9
Solution a. lim[ f ( x)+ g( x)]= lim f ( x)+ lim g( x) = lim 5+ lim = 5+ (– ) = .
x → –2 x → –2 x → –2 x → –2 x → –2 x 2 4

1
b. lim [ f ( x) ⋅ g( x)]= lim f ( x) ⋅ lim g( x) = lim 5 ⋅ lim = 5 ⋅0 = 0.
x →+∞ x →+∞ x→+∞ x→+∞ x→+∞ x

f ( x) 5
c. lim[ ]= lim = ⋅ lim 5 ⋅ x = 5 ⋅ lim x = 5(–3) = –15.
x → –3 g( x) x → –3 1 x → –3 x →–3

56 Limit of Function
EXERCISES 2 .1
A. Calculation of Limit ⎧ 2, if x > –3

Find the limits in questions 1-7. 10. f : → f(x) = ⎨ 1, if x = –3

⎩ x + 3, if x < –3
1. lim(3 x + 2)
x→5
a. lim f ( x) b. lim f ( x) c. lim f ( x)
x →–5 x →–3 x →–1

2. lim( x2 – 2 x)
x →–4

C. Definition of Limit

3. lim 3(2t – 1)( t +1) 11. Find 0,3-neighborhood of 12.


x→ 2

4. lim17 12. Find 0,1-neighhborhood of –5 and show the


x→ 4
interval on number line.

5. lim(2 k +1)
x →–2

13. Find 0,01-neighborhood of 4.

6. lim(2
x→ a
x +1) ⋅ x2

14. Find 0,05-neighborhood of 5.

7. lim(
x→ b
x2 – 2 b +1)

By using the definition of limit prove the


B. Graphical Illustration of Limit following limits in questions 15-17.

Draw the graph of functions and examine the


given limits in questions 8-10.
15. lim
x→5
2 x = 10
8. f : → f(x) = 4x – 7, lim f ( x).
x→ 3

⎧ x2 , if x > 0 16. lim(2


1
x + 3) = 2
x →–
⎪⎪ 2
9. f : → f(x) = ⎨ 3, if x = 0
⎪ 2
⎪⎩ – x , if x < 0
a. lim f ( x)
x →–5
b. lim f ( x)
x →–3
17. lim
x→ 3
5=5

Limit of Functions 57
D. One-side Limits: Right and Left- 20. Given f : →
Hand Limits
⎧ x, if x >1

18. y f(x) = ⎪ 2, if x =1 or x = –1
⎨ 2
⎪ – x +1, if –1 < x <1

⎩ – x, if x < –1
a. Find one-sided limits at the points –1, 0 and 1.
–3 b. Find at which point (s) does the limit exists.
x
–2 –1 O
1 2 3
–1

–2

–3 21. Given f : →
⎧ –3, if x >1

a. lim+ f ( x) = ? b. lim+ f ( x) = ? f(x) = ⎨ 2, if x =1 or x ≤ –2
x → –1 x→ 2 ⎪ 2
⎩ x – 4, if –2 < x <1
c. lim– f ( x) = ? d. lim+ f ( x) = ?
x→0 x → –2
Examine the limit at the points –2, 0, and 1 then
e. lim+ f ( x) = ? f. lim f ( x) = ? find at which point (s) does he limit exists.
x → –2 x → –2–

22. y

19. y 3

2
2

1
1
x
–3 –2 –1 O 1 2 3
x
–3 –2 –1 O 1 2 3 4
–1

–1
–2

The graph of f is shown in the figure. At which The graph of the function f : (–3, 3) → is given at
7 9
integer values of x in the interval (– , ) does the figure. Find at which integer value (s) of the
2 2
the limit exist. domain does the limit exist.

58 Limit of Function
26. Perform the following limits:
⎧ x – 2, if x> 3 2 x2 +1 1
⎪ a. lim b. lim ( +1)
x →+∞ 3x2 + 4
⎪ 3, if x= 3 x →+∞ x
23. Given f : → f(x) = ⎨
⎪ – x +5, if 1 ≤ x<3 x2 – 1 2| x|
⎪ c. lim( ) d. lim( )
⎩ 1, if x <1 x→ – ∞ x2 x→ – ∞ x

a. lim f ( x) = ? b. lim f ( x) 4 x2 + x – 5 1
e. lim f. lim(1+ cos )
x →1 x→ 3 x→ – ∞ 3 3
x +1 x→∞ x

E. Limit of Special Functions 27. Perform the following limits:


1 x
24. Find the following Limits: a. lim b. lim–
x →0 | x| x→ 3
2
x –9
| x – 2|
a. lim+ b. lim|9 – x2
x→2 | 2 – x| x→ 3 x3 + 2
c. lim d. lim( x – tan x)
x →3 x | x – 1| – 6 x→

π
| x|
d. lim | x – 5|
2
c. lim+
x→0 x2 + x x → 5– x – 5

| cos x| | x| 28. Evaluate the following limits where


e. lim+ f. lim–
x→
π cos x x→0 x x+ 2
2 f ( x) = .
x( x +1)( x – 2)

a. lim f ( x) b. lim– f ( x)
x→ – ∞ x → –1

c. lim+ f ( x) d. lim– f ( x)
x → –1 x→ 0

e. lim+ f ( x) f. lim– f ( x)
25. The graph of the function y x→ 0 x→ 2

f is given in the figure. g. lim+ f ( x) h. lim f ( x)


x→ 2 x →+∞
Eualuate the folloving
limits:
1
a. lim f ( x)
x →+∞
x
–2 –1 O 1 29. Perform the following limits:
b. lim f ( x) –1 y = f(x)
x→ – ∞
x3
a. lim f (– x2 +1) b. lim
c. lim f ( x) –2 x →+∞ x → – ∞ x2 +5
x→1

d. lim f ( x) 2 x3 – x2 +1 1
x→1
c. lim d. lim
x → – ∞ x2 + 2x+1 x →+∞ 1
1– 5x
e. lim f ( x)
x→–2

59
In the previous section, we have discussed the existence of a limit of a function at any point,
and we found that it is not necessary for the function to be defined at that point to have a
limit.
Even the given function f is not defined at x0, in some cases the limit of the function may exist,
as the point x approach to the point x0 or infinity. Such situations are called indeterminate
forms.
x2 – 9
For example, the function f ( x) = is not defined at x0 = 3.
x–3
0
When x0 = 3, we get the indeterminate of the form . But we know that lim f ( x) is
0 x→ 3

completely different than f(3). It doesn't matter what happens at the given point, the matter
is what happens when x is getting closer to that point. So limit may exist even at the
undefined points.
x2 + x +1 ∞
Similarly, the function f ( x) = 2
gets the indeterminate of the form as x
2x – 3 ∞
approaches infinity.
0
In the following section we will investigate the methods for indeterminate of the forms ,
∞ 0
, ∞ – ∞, 0 ⋅ ∞, and 1∞.

0
A. THE INDETERMINATE OF THE FORM
0
Let f and g be two functions and x0 ∈ \, and assume that f(x0) = 0 and g(x0) = 0, then
f ( x) 0
lim has the indeterminate of the form . In this case there exist a function h(x) which
x → x0 g( x )
0
is the common factor of the functions f and g where h(x0) = 0 such that f(x) = f1 ⋅ h(x) and

g(x) = g1(x) ⋅ h(x).


Then we can write that
f ( x) f ( x) ⋅ h( x)
lim = lim 1 ,
x → x0 g( x) x → x0 g1( x) ⋅ h( x)
since in limit process x ≠ x0 we can simplify the factor h(x) then
f1( x) ⋅ h( x) f ( x) f1( x0 )
lim = lim 1 =
x → x0 g1( x) ⋅ h( x) x→ x0 g1( x) g1( x0 )
which is the result of the limit.

Limit of Functions 60
EXAMPLE 21 Find lim
x→ 3
x2 − 9
x−3
.

0
Solution As x approaches 3, we get the indeterminate of the form . When we factor the expression
0
in the numerator, we get
x2 − 9 ( x − 3)( x + 3)
lim = lim = lim( x+ 3) = 6.
x→ 3 x − 3 x→ 3 ( x − 3) x→ 3

EXAMPLE 22 Find lim


x→ 2
x2 − 5x +9
x2 − x − 2
.

0
Solution This limit has the indeterminate of the form
as x approaches 2, so we need to factor the
0
numerator and denominator and find common factor:
x2 – 5x +9 (x – 2 )(x – 3 ) (x – 3 ) 1
lim = lim = lim =– .
x→ 2 2
x – x–2 x → 2 (x – 2 )(x +1) x → 2 (x +1) 3

EXAMPLE 23 Find lim


x →1
x −1
x3 − 1
.

0
Solution Indeterminate of the form exists.
0
First let us multiply both numerator and the denominator with the conjugate of ñx – 1.

x –1 ( x – 1)( x +1) ( x − 1)
lim 3
= lim 3 = lim 3 .
x →1 x –1 x →1
( x – 1)( x +1) x →1
( x − 1)( x +1)

We know that x3 – 1 = (x – 1(x2 + 2x + 1), then

( x – 1) (x − 1)
lim 3
= lim 2
x →1
( x – 1)( x +1) x →1
( x – 1)( x + 2 x+1)( x +1)

1 1
= lim = .
x→1
( x2 + 2 x +1)( x +1) 8

61 Limit of Function
2
EXAMPLE 24 Find lim
x →∞

x
2⎞
2
.
⎜ 2+ ⎟ – 4
⎝ x⎠

0
Solution The indeterminate of the form . Let us factor the denominator:
0
2 2 ⎛2⎞
⎜ ⎟
lim x = lim x = lim ⎝ x⎠
x →∞
⎛ 2⎞
2 x →∞ ⎛ 2 ⎞⎛ 2 ⎞ x →∞ ⎛ 2 ⎞⎛ 2⎞
⎜ 2+ ⎟ – 4 ⎜ 2+ – 2 ⎟⎜ 2+ + 2 ⎟ ⎜ ⎟⎜4+ ⎟
⎝ x⎠ ⎝ x ⎠⎝ x ⎠ ⎝ x ⎠⎝ x⎠

1 1
= lim = .
x →∞ ⎛ 2⎞ 4
⎜ 4+ ⎟
⎝ x⎠

Check Yourself 9
Perform the following limits:
x2 – 2 x – 8 x2 – 9 x3 – x2 – x +1
1. lim 2. lim 3. lim
x→ 4 x2 – 3 x – 4 x→ 3 x3 – 27 x→ 3 3x2 – 6 x + 3

x + 3x – 2 x–4 1 – x4 – 1+ x4
4. lim 5. xlim 6. lim
x→ 3 x–2 →16 4
x–2 x→0 x4

4+ 3x – 2 4 – 2x
7. lim 8. lim
x→ 2 x x→ 2 4 x – 16

Answers
6 2 2 1 3 1
1. 2. 3. 4. 5. 4 6. –1 7. 8. –
5 9 5 4 4 8

0
The Indeterminate of the Form in the Trigonometric Functions
0
As x approaches point x0, the limit of trigonometric functions is the image of x0.
2
For example, lim cos x =1, lim sin x = 0, and lim tan x= .
x→0 x→0
x→
π 2
4

sin x
Now, what about the limit lim .
x→0 x
0
This limit has the indeterminate of the form , and we need to apply something different
0
methods as follows:

Limit of Functions 62
In the figure, PR = sin x, OR = cos x,
and QA = tan x. y

Now let us notice the areas of, ∆OPR, ∆OQA,


Q
and the area of the part of the circle OPA. B(0, 1)

We can say that A(∆OPR) < A(OPA) < A(∆OQA). P

By using the area formulas we get


tan x
1 x 1
cos x ⋅ sin x < π ⋅ 12 ⋅ < ⋅ 1 ⋅ tan x and 1
2 2π 2 sin x
cos x ⋅ sin x < x < tan x.
For x → 0+ sin x > 0 and when we divide each O
x
cos x R A(1, 0)
side of the inequality by sin x we get
x 1
cos x < < .
sin x cos x
x 1
Now let us take the limit of each side as lim+ cos x ≤ lim+ ≤ lim+ .
x→0 x→0 sin x x → 0 cos x
x x
Since we get 1 ≤ lim+ ≤ 1, we can say that lim+ =1.
sin x
x→0 x → 0 sin x
x 1 x x
Similarly, lim− cos x ≤ lim− ≤ lim− then 1 ≤ lim− ≤ 1 and lim− =1.
x→0 x → 0 sin x x → 0 cos x x → 0 sin x x → 0 sin x

x
So we found that lim =1.
x→0 sin x
sin x
What about lim ?
x→0 x

0
For the limit of trigonometric functions concerning the indeterminate of the form , we
0
sin x
can use the following as a rule: lim =1
x→0 x

Conclusion
tan x
1. lim =1
x→0 x
sin ax a tan ax a
2. lim = and lim =
x→0 bx b x → 0 bx b
sin ax a tan ax a
3. lim = and lim =
x→0 sin bx b x → 0 tan bx b
sin ax a tan ax a
4. lim = and lim =
x→0 tan bx b x → 0 sin bx b

63 Limit of Function
Now let us prove two of the conclusions:
tan x sin x 1 sin x 1
1. lim = lim ⋅ = lim ⋅ lim =1 ⋅1=1
x→0 x x→0 x cos x x→0 x x→0 cos x
tan ax tan ax
⋅ ax
tan ax a 1 a a
2. lim = lim ax = lim ax ⋅ = ⋅ =
x → 0 tan bx x → 0 tan bx x → 0 tan bx b 1 b b
⋅ bx
bx bx

EXAMPLE 25 Find lim


4 − x2
x → 2 sin(2 − x)
.

4 – x2 (2 – x)(2+ x) (2 – x)
Solution lim = lim = lim ⋅ lim(2+ x) =1 ⋅4 = 4.
x →1 sin(2 – x) x→ 2 sin(2 – x) x → 2 sin(2 – x) x→ 2

EXAMPLE 26 Find lim


x →π
tan x
3( x − π)
.

Solution Since tan x = –tan(π – x).


tan x − tan( π − x) 1 tan( π − x) 1 1
lim = lim = lim = ⋅1= .
x →π 3( x − π) x→π −3( π − x) 3 x→π ( π − x) 3 3

x
sin 2
EXAMPLE 27 Find lim
x→0 3x2
2.

2
x x x ⎛ x⎞
sin 2 sin 2 sin 2 ⎜ sin ⎟
1
2 = lim 1 1
2 = ⋅ lim 2= 1 2 = 1 ⋅12 = 1
Solution lim lim ⎜ ⎟
x → 0 3 x2 3 x → 0 4x 2
3 4 x → 0
⎛ x⎞
2
12 x → 0
⎜ x 12 12
⎜ ⎟ ⎜ ⎟⎟
4 ⎝2⎠ ⎝ 2 ⎠

1 − cos 2 x
EXAMPLE 28 Evaulate the indeterminate form lim
x→0 8 x2
.

Solution Since cos 2x = 1 – 2sin2 x.


1 − cos 2 x 1 − (1 − 2 sin 2 x) 1 −1+ 2 sin 2 x sin 2 x 1 sin x 2
lim 2
= lim 2
= lim 2
= lim 2
= lim( )
x→0 8x x → 0 8x x → 0 8x x → 0 4x 4 x → 0 x
1 2
= ⋅ 1 = 4.
4
Limit of Functions 64
EXAMPLE 29 Evaulate the indeterminate form limπ
x→
2
cos x
π − 2x
.

π
Solution Since cos x = sin( − x) , we can write
2
π 1
sin( − x) sin ( π − 2 x)
cos x 2 2 1
lim = lim = lim = .
π ( π − 2 x) π ( π − 2 x) π ( π − 2 x) 2
x→ x→ x→
2 2 2

Check Yourself 10
Perform the following limits:
x
sin
tan 2 x sin x 2
1. lim 2. lim 3. lim
x → 0 tan5 x x → 0 tan 3 x x → 0 3x

sin(2 x − 2) sin 2 2 x sin5 x


4. lim 5. lim 6. lim
x →1 4x − 4 x→0 x2 x→0 5x ⋅ cos5 x

tan πx 3x2 cos(3 x + )
7. lim 8. lim 9. lim 2
x →1 1 − x x → 0 1 − cos 2 x x → 0 sin( π − 2 x)

Answers
1 1 1 1 3 3
1. 2. 3. 4. 5. 4 6. 1 7. –π 8. 9.
2 3 6 2 2 2


B. THE INDETERMINATE OF THE FORM

Let f and g be two functions and x0 ∈ \, and assume that as x approaches x0, both f(x) and

g(x) approach positive or negative infinity. That is, lim f ( x) = ±∞ and lim g( x) = ±∞.
x→ x0 x→ x0

f ( x) ∞
Then lim has the indeterminate of the form . We can remove the indeterminate of
g( x)
x→ x0 ∞
∞ 0
the form by transforming into the indeterminate of the form .
∞ 0

65 Limit of Function
Let P(x) = anxn + an–1xn–1 + ... + a0 and Q(x) = bmxm + bm–1xm–1 + ... + b0 be two polynomi-

al function, then
an −1 a
xn ( an + +...+ no )
P( x) a xn + an −1xn −1 +...+ ao x x = lim an x .
n
lim = lim n m = lim
x →+∞ Q( x) x→+∞ b x + b
m −1x
m −1
+...+ bo x→+∞ b b x→+∞ b x m
m xm ( bm + m −1 +...+ mo ) m
x x
As x approaches ∞, limit of the rational expression goes to zero, so we have eliminated them.
To conclude we can write that
⎧ an
⎪ b , if n = m
P( x) a x n
⎪ m
lim = lim n m = ⎨
⎪0, if n < m
x →+∞ Q( x) x→+∞ b x
m

⎩ ±∞ if n > m
a. If the degree of the polynomials are equal then the limit is the ratio of the leading coeffi-
cients.
b. If the degree of the polynomial in denominator is greater than the degree of the polyno-
mial in numerator, then the limit is zero.
c. If the degree of the polynomial in denominator is less than the degree of the polynomial
in numerator, then the limit goes to infinity.

EXAMPLE 30 Evaluate the following limits:


5 x 3 – 2 x2 + x – 1 –2 x3 – 1 x2 + 2
a. lim b. lim c. lim
x →∞ 4x3 – x2 +12 x →∞ x5 + x3 +1 x →−∞ x + 4

Solution ∞
The indeterminate of the form exists.

5 x 3 − 2 x2 + x − 1 5
a. lim = , because the polynomials have the same degree. And its solution
x →∞ 4x3 − x2 +12 4
is as follows:
2 1 1
x3 (5 − + − )
5x3 − 2 x2 + x − 1 3
x x2 x3 = lim 5 x = 5 .
lim = lim
x →∞ 4x3 − x2 +12 x→∞ 1 12 x→∞ 4 x3
4
x3 (4 − + 3 )
x x

Limit of Functions 66
−2 x3 − 1
b. lim = 0, because the degree of the polynomial in the denominator is bigger. We
x →∞ x5 + x3 +1

can solve it like:


1
x3 ( −2 − 3 )
−2 x3 − 1 x −2 x3 −2
lim 5 = lim = lim 5 = lim 2 = 0.
3
x →∞ x + x +1 x →∞ 5 1 1 x →∞ x x →∞ x
x (1+ 2 + 5 )
x x
x2 + 2
c. lim = −∞, because the degree of the polynomial in the numerator is bigger. And
x →−∞ x + 4

here is the solution:


⎛ 2 ⎞
x2 ⎜ 1+ 2 ⎟
x2 + 2 ⎝ x ⎠ x2
lim = lim = lim = lim x = −∞.
x →−∞ x + 4 x→−∞ ⎛ 4⎞ x→−∞ x x→−∞
x ⎜ 1+ ⎟
⎝ x⎠

EXAMPLE 31 Evaluate the following indeterminate forms:


2x + 4 x − 4x2 − 1
a. lim b. lim
x →∞
9 x2 + 2 x →−∞
x2 + 2 x + 2

Solution ∞
The indeterminate of the form exists.

⎛ 4⎞ ⎛ 4⎞ ⎛ 4⎞
x ⎜ 2+ ⎟ x ⎜ 2+ ⎟ x ⎜2+ ⎟
2x + 4 ⎝ x⎠ ⎝ x⎠ ⎝ x⎠ 2
a. lim = lim = lim = lim = .
x →∞ 2
9x + 2 x→∞
⎛ 2 ⎞ x→∞
2 x→∞
2 3
9x2 ⎜ 1+ 2 ⎟ 3 x 1+ 2 3 x 1+ 2
⎝ 9 x ⎠ 9x 9x

b. The indeterminate of the form exists.

⎛ 1 ⎞ ⎛ 1 ⎞
2
x − x ⎜4 − 2 ⎟ x − (−x ) ⎜ 4 − 2 ⎟
x − 4x − 1 ⎝ x ⎠ ⎝ x ⎠
lim = lim = lim
x →−∞ 2
x + 2x + 2 x→−∞
2 2 x→−∞
2 2
x 1+ + 2 − x 1+ + 2
x x x x

1 ⎛ 1 ⎞
x+ x 4 − x ⎜⎜ 1+ 4 − 2 ⎟⎟
x2 = lim ⎝ x ⎠ 1+ 4
= lim = = −3.
x →−∞
2 2 x→−∞
2 2 − 1
− x 1+ + 2 −x 1+ + 2
x x x x

67 Limit of Function
EXAMPLE 32 Find lim
x→0
cot 2 x
cot 3 x

Solution ∞ 0
Indeterminate of the form exists. We need to transform it to indeterminate of the form .
∞ 0
1
cot 2 x tan 3 x 3
lim = lim tan 2 x = lim = .
x → 0 cot 3 x x→0 1 x → 0 tan 2 x 2
tan 3x

EXAMPLE 33 Find limπ


x→
tan x
cot( x −
π
)
.
2
2

Solution π 0
As x approaches exists. We need to transform it to indeterminate of the form . We know
2 0
π tan x tan x tan x
that cot( x − ) = − tan x then lim = lim = −lim = −1
2 x→
π
⎛ π ⎞ x→ π − tan x x → tan x
π
2 cot ⎜ x − ⎟ 2 2
⎝ 2⎠

EXAMPLE 34 Find lim(


x →∞
2x
cot
1
).

x
∞ 1 1
Solution Indeterminate of the form exists. Instead of , we can write .
∞ x 1
tan
x
1
tan
2x x x) = 2 ⋅1= 2.
Then lim( ) = 2 lim( ) = 2 lim(
x →∞ 1 x→∞ 1 x→∞ 1
cot
x 1 x
tan
x

Limit of Functions 68
Check Yourself 11
Perform the following limits:
2 3
x5 + 2 x +7 x2 + 2 x − 12
1. lim 1 – x +7 x3 2. lim 3. lim
x →∞ 5+ x – x x →−∞ x2 +12 x – 1 x →−∞ 1 − x3

3x2 +1 x − 9 x2 − 1 cot 5 x
4. lim 5. lim 6. lim
x →−∞
4x + 4 2 x →∞ 2
x +1+ 3x x→0 cot 6 x
π
tan x2 + cos x x2 x
7. lim 2 8. lim 9. lim( − )
x →π cot( x − π) x →∞ 3 x2 x →∞ 3x + 4 3
Answers
1 6 1 4
1. –7 2. –∞ 3. 0 4. ∞ 5. – 6. 7. –2 8. 9. –
2 5 3 9

C. THE INDETERMINATE OF THE FORM 0 ⋅ ∞


Let f and g be two functions and x0 ∈ \, and let us assume that as x approaches x0, f(x)
approach zero but g(x) approach positive or negative infinity.
That is, lim f ( x) = 0 and lim g( x) = ±∞.
x → x0 x → x0

Then lim [ f ( x)g( x)] has the indeterminate of the form 0 ⋅ ∞.


x→ x0

We can remove the indeterminate of the form 0⋅ ∞ by transforming into the indeterminate of the
0 ∞
form or as follows:
0 ∞
f ( x) 0
lim [ f ( x)g( x)]= lim
x→ x0 x→ x0 1 is the indeterminate of the form 0 .
g( x)

g( x) ∞
lim [ f ( x)g( x)]= lim
x→ x0 x→ x0 1 is the indeterminate of the form ∞ .
f ( x)

1
EXAMPLE 35 Find lim( x ⋅ sin ).
x →∞ x

Solution 0
The indeterminate of the form 0 ⋅ ∞. We can transform into the form .
0

69 Limit of Function
1
sin
1 x ).
lim( x ⋅ sin ) = lim(
x →∞ x x →∞ 1
x
1
1 sin
1
Let us take = t and as x → ∞, → 0 so t → 0, then lim( x ) = lim( sin t ) =1.
x x x →∞ 1 t→0 t
x

EXAMPLE 36 Find lim(2 x ⋅ cot x).


x→0

Solution Indeterminate of the form 0 ⋅ ∞.


1 x
lim(2 x ⋅ cot x) = lim 2 x ⋅ = 2 ⋅lim = 2 ⋅1= 2
x→0 x→0 tan x x → 0 tan x

EXAMPLE 37 Find lim


x →−∞
1
x +5
⋅ (3x − 2).

Solution Indeterminate of the form 0 ⋅ ∞.


1 3x − 2
lim ⋅ (3x − 2) = lim = 3.
x →−∞ x +5 x →−∞ x +5

EXAMPLE 38 Find limπ (π − 2 x ) ⋅ tan x


x→
2

Solution Indeterminate of the form 0 ⋅ ∞.


π
2( − x)
π ( π − 2 x) 2
lim( π − 2 x) ⋅ tan x = lim( π − 2 x) ⋅ cot( − x) = lim = lim = 2.
π π 2 π π π π
2 tan( − x) 2 tan( − x)
x→ x→ x→ x→
2 2
2 2

Limit of Functions 70
EXAMPLE 39 2
Find lim x (cos
x →∞
1
x
− 1).

Solution Indeterminate of the form ∞ ⋅ 0. We know that cos 2x = 1 – sin2 x, then


⎛ 1 ⎞
sin2 ⎜ ⎟
⎛ 1 ⎞ ⎛ ⎛ 1 ⎞ ⎞ ⎛1 ⎞ ⎝ 2x ⎠
lim x ⎜ cos − 1 ⎟ = lim x2 ⎜1 − 2 sin 2
2
⎜ ⎟ −1 ⎟= −2 lim x2 sin 2 ⎜ ⎟= −2 lim
x →∞
⎝ x ⎠ x →∞
⎝ ⎝ 2 x ⎠ ⎠ x →∞
⎝ 2x ⎠ x →∞ 1
x2
2
⎛ ⎛ 1 ⎞⎞
⎜ sin ⎜ 2 x ⎟ ⎟ 2

= −2 lim ⎜ ⎝ ⎠ ⎟ = −2 ⎛ 1 ⎞ = − 1.
1 ⎜ ⎟
x →∞ ⎜ ⎟ ⎝2⎠ 2
⎜ x ⎟
⎝ ⎠

Check Yourself 12
Perform the following limits:
x 9 2π 1
1. lim( ⋅ sin ) 2. lim( x ⋅ tan ) 3. lim( x + 2)
x →∞ 3 x x →∞ x x →∞ ( x2 − 1)

1 2 2 1
4. lim x2 +5 x +1 ⋅ 5. lim(2 x ⋅ cot x ) 6. lim( ⋅ sin 2 x ⋅ tan x)
x →∞ x+ 4 x→0 x→0 x2

7. lim(( π − x)cot 2 x)
x →π

Answers
1
1. 3 2. 2π 3. 0 4. 1 5. 2 6. 2 7. –
2

D. THE INDETERMINATE OF THE FORM ∞ – ∞


Let f and g be two functions and x0 ∈ \, and let us assume that as x approaches x0, both f(x)
and g(x) approach infinity. That is,
lim f ( x) = ∞ and lim g( x) = ∞
x→ x0 x→ x0

Then lim [ f ( x) − g( x)] has the indeterminate of the form ∞ – ∞. We can remove the
x→ x0
0 ∞
indeterminate of the form ∞ – ∞ by transforming into the indeterminate of the form or .
0 ∞

71 Limit of Function
EXAMPLE 40 ⎛ 2
Find lim ⎜ 2
x →1 ⎝ x − 1

1 ⎞
⎟.
x −1⎠

Solution The indeterminate of the form exists as x approaches 1.


⎛ 2 1 ⎞ ⎛ 2 1 ⎞ 2 – x–1
lim ⎜ 2 – ⎟ = lim ⎜ – ⎟ = lim
x →1 x – 1
⎝ x – 1⎠ x →1
⎝ ( x – 1)( x +1) x – 1 ⎠ x →1 ( x – 1)( x+1)

1– x –1 1
= lim = lim =– .
x→1 ( x – 1)( x +1) x →1 ( x +1) 2

EXAMPLE 41 Find lim ⎜


⎛ 1
x → 0 sin x


1 ⎞
tan
⎟.
x⎠

1 1
Solution Both lim and lim have the limit ∞, so the indeterminate of the form ∞ – ∞.
x→0 sin x x → 0 tan x

⎛ 1 1 ⎞ ⎛ 1 cos x ⎞ 1 – cos x (1 – cos x)(1+ cos x)


lim ⎜ – ⎟ = lim ⎜ – ⎟= lim = lim
x → 0 sin x tan x ⎠ x→0 ⎝ sin x sin x ⎠ x→0 sin x sin x(1+ cos x)
⎝ x→0

1 – cos 2 x sin 2 x sin x 0


= lim = lim = lim = = 0.
x → 0 sin x(1+ cos x) x → 0 sin x(1+ cos x) x → 0 1+ cos x 2

EXAMPLE 42 2 2
Find lim( 4 x – 6 x + 3 – 4 x + 3 x+1).
x →∞

Solution The indeterminate of the form ∞ – ∞. Let us multiply and divide the expression with its
conjugate. Then we get.

( 4 x2 – 6 x + 3 − 4 x2 + 3 x +1)( 4 x2 – 6 x+ 3 + 4 x2 + 3 x+1)
lim
x →∞
( 4 x2 – 6 x + 3 + 4 x2 + 3 x +1)

4x2 – 6 x + 3 – (4 x2 + 3 x +1) –9 x + 2
= lim = lim
x →∞ ⎛ 2 ⎛ 6 3 ⎞ ⎛ 3 1 ⎞⎞
x →∞ 2x + 2x
⎜⎜ 4x ⎜ 1 − + 2 ⎟ + 4 x2 ⎜1+ + 2 ⎟ ⎟⎟
⎝ ⎝ 4 x 4x ⎠ ⎝ 4 x 4x ⎠⎠

⎛ 2 ⎞
–9 x ⎜ 1 – ⎟
⎝ 9x ⎠ –9 x 9
= lim = lim =– .
x →∞ 2x+ 2x x →∞ 4x 4

Limit of Functions 72
Theorem
2
Let a>0 be a real number and f be a function defined by f ( x) = ax + bx + c , then
b
lim f ( x) = lim ax2 + bx + c = lim a x +
x →±∞ x→±∞ x→±∞ a

⎛ b c⎞
Proof lim f ( x) = lim ax2 + bx + c = lim a ⎜ x 2 + x + ⎟
x →±∞ x→±∞ x→±∞
⎝ a a⎠

b c
= lim a x2 + x+ ( a > 0)
x →±∞ a a

⎛ b ⎞
2
4ac − b 2 as x → ±∞, we may omit the
= lim a ⎜ x2 + ⎟ + 2
⎝ 2a ⎠ 4a 2 constant expression 4ac –2 b .
x →±∞

4a
2
⎛ b ⎞
= lim a ⎜ x2 + ⎟
x →±∞
⎝ 2 a⎠

b
= lim a x + .
x →±∞ a

EXAMPLE 43 2 2
Find lim( 4 x +8 x +1 − 4 x – 3 x – 1).
x →∞

Solution The indeterminate of the form ∞ – ∞.


We can use the theorem:

⎛ 8 –3 ⎞
lim( 4 x2 +8 x +1 – 4 x2 – 3 x – 1) = lim ⎜ 4 x+ – 4 x+ ⎟
x →∞ x →∞
⎝ 2 ⋅ 4 2 ⋅4 ⎠

⎛ ⎛ 3 ⎞⎞
= lim ⎜ 2( x +1) – 2 ⎜ x – ⎟ ⎟
x →∞
⎝ ⎝ 8 ⎠⎠

⎛ 6⎞
= lim ⎜ 2 x + 2 – 2 x+ ⎟
x →∞
⎝ 8⎠

6 22 11
= 2+ = = .
8 8 4

73 Limit of Function
EXAMPLE 44 2
Find lim( x – 2 x – 3 – x – 4).
x →∞

Solution The indeterminate of the form ∞ – ∞.


We can use the theorem:
⎛ –2 ⎞
lim( x2 – 2 x – 3 – x – 4) = lim ⎜ 1 ⋅ x+ − x– 4 ⎟
x →∞ x →∞
⎝ 2 ⋅1 ⎠
= lim (x – 1 – x – 4 )
x →∞

= –5.

EXAMPLE 45 Find lim


x →∞
4x2 +1 – x2
9 x2 + 2 – x2 + 2
.

Solution ∞–∞
The indeterminate of the form . Let us use the theorem:
∞–∞
0 0
4 x+
– 1 x+
2
4x +1 – x 2
2⋅4 2 ⋅1 2x– x 2x – x
lim = lim = lim = lim
x →∞ 2 2
9x + 2 – x + 2 x→∞ 0 0 x→∞ 3 x – x x→∞ 3 x – x
9 x+ – 1 x+
2⋅9 2 ⋅1
x 1
= lim = .
x →∞ 2x 2

Check Yourself 13
Perform the following limits:
1 2 ⎞ ⎛ 1 x ⎞ ⎛ 1 1 ⎞
1. lim ⎛⎜ – ⎟ 2. lim ⎜ 2 – ⎟ 3. lim ⎜ − ⎟
x→1 x – 1 1– x2 ⎠ x→ 2 x – 5 x +6 x–2 ⎠ x→π sin x tan x ⎠
⎝ ⎝ ⎝

4. lim(tan x − sec x) 5. lim( x2 + 3x +1 – x) 6. lim( x2 + 4 x +1 – x2 + 3 x + 3)


π x→∞ x →∞
x→
2

7. lim( x + 2 – x – 2 ) 8. lim(2 x3 + 3 x3 +1) 9. lim( x2 – 8 x +1+ x+ 4)


x →∞ x →−∞ x →−∞

10. lim(log(9 x+1) – log 3( x+ 4))


x →∞

Answers
1 3 1
1. 2. –∞ 3. 0 4. 0 5. 6. 7. 0 8. ∞ 9. 8 7. 2
2 2 2

Limit of Functions 74
E. THE INDETERMINATE OF THE FORM ∞ – ∞
Let f and g be two functions and x0 ∈ \, and let us assume that as x approaches x0, f(x)
approach 0 and g(x) approach infinity. That is,
lim f ( x) = 0 and lim g( x) = ∞
x → x0 x → x0

g( x )
Then lim(1+ f ( x)) has the indeterminate of the form 1∞. We can remove the indeterminate
x → x0

of the form 1∞ by using the following rule:

Let lim f ( x) = 0 and lim g( x) = ∞ , and lim f ( x) ⋅ g( x) = k ( k ∈ \) then


x → x0 x → x0 x → x0

g( x )
lim(1+ f ( x)) = e k , where e ≅ 2.718
x → x0

1
EXAMPLE 46 For f ( x) =
x
and g(x) = x find lim(1+ f ( x)) g( x ).
x →∞

Solution 1
Since lim f ( x) = lim = 0,
x →∞ x→∞ x
lim g( x) = lim x = ∞, and
x →∞ x →∞

⎛1 ⎞
lim f ( x) ⋅ g( x) = lim ⎜ ⋅ x ⎟ =1 then k = 1 and by the given rule
x→∞ x
x →∞
⎝ ⎠
x
⎛ 1⎞
lim (1+ f ( x) )
g( x )
= lim ⎜1+ ⎟ = e1 = e.
x →∞ x→∞
⎝ x⎠

3
EXAMPLE 47 For f ( x) =
x
and g(x) = 5x, find lim(1+ f ( x)) g( x ).
x →∞

Solution 3
Since lim f ( x) = lim = 0,
x →∞ x→∞ x
lim g( x) = lim 5 x = ∞, and
x →∞ x →∞

⎛3 ⎞
lim f ( x) ⋅ g( x) = lim ⎜ ⋅ 5 x ⎟ =15 then k = 1 and by the given rule
x→∞ x
x →∞
⎝ ⎠
5x
⎛ 3⎞
lim (1+ f ( x) )
g( x )
= lim ⎜1+ ⎟ = e15.
x →∞ x→∞
⎝ x⎠

75 Limit of Function
3 x −1

EXAMPLE 48 x →∞

For lim ⎜1+

7 ⎞

2 x +1 ⎠
and g(x) = 5x, find lim(1+ f ( x)) g( x ).
x →∞

Solution 2 2
It is seen that f ( x) = and lim f ( x) = lim =0
2 x +1 x →∞ x →∞ 2 x +1
g( x) = 3x +1 and lim g( x) = lim 3 x+1= ∞
x →∞ x→∞

2 6x+ 2 6
Since lim f ( x) ⋅ g( x) = lim ⋅(3 x+1) = lim = = 3 then k = 3 and
x →∞ x→∞ 2x + 1 x →∞ 2 x +1 2
3 x −1
⎛ 7 ⎞
by the given rule lim ⎜1+ ⎟ = e3.
x →∞
⎝ 2 x +1 ⎠

EXAMPLE 49 For lim ⎜


x →∞
⎛ x − 1⎞
⎝ x ⎠
⎟ .

x −1 1
Solution The expression can be written as 1 − .
x x
1
Then we can take f ( x) = − , and g(x) = x, as x → ∞, f(x) → 0 and g(x) → ∞.
x
⎛ 1⎞
Since lim f ( x) ⋅ g( x) = lim ⎜ − ⎟ ⋅( x) = –1, k = –1.
x →∞ x →∞
⎝ x⎠
x x
⎛ x − 1⎞ ⎛ 1⎞ −1 1
So lim ⎜ ⎟ = lim ⎜1 − ⎟ = e = .
x →∞
⎝ x ⎠ x →∞
⎝ x⎠ e

4x−2

EXAMPLE 50 ⎛ 3x +1 ⎞
Find lim ⎜
x →∞ 3x – 1



.

3x +1 3x – 1+ 2 2
Solution We can write = =1+ .
3x – 1 3x – 1 3x – 1
4 x− 2 4 x−2
⎛ 3x +1 ⎞ ⎛ 2 ⎞ 2
Then lim ⎜ ⎟ = lim ⎜1+ ⎟ and we can take f ( x) = and g(x) = 4x – 2.
x →∞ 3x – 1
⎝ ⎠ x→∞
⎝ 3 x –1⎠ 3x – 1

⎛ 2 ⎞ 8x – 4 8 8
Since lim f ( x) ⋅ g( x) = lim ⎜ ⎟ ⋅(4 x – 2) = lim = , k= .
x →∞ 3x – 1 x →∞ 3 x – 1 3 3
x →∞
⎝ ⎠
4 x− 2 4 x− 2 8
⎛ 3x +1 ⎞ ⎛ 2 ⎞
So lim ⎜ ⎟ = lim ⎜1+ ⎟ = e 3.
x →∞ 3x – 1 3x – 1 ⎠
⎝ ⎠ x→∞

Limit of Functions 76
Check Yourself 14
Perform the following limits:
x x x
5 x−2⎞ 2 x +1 ⎞
1. lim ⎛⎜ 1+ ⎞⎟ 2. lim ⎛⎜ ⎟ 3. lim ⎛⎜ ⎟
x →∞
⎝ x⎠ x →∞
⎝ x ⎠ x →∞
⎝ 2x ⎠
4x 2 x+5 x+4
2x – 5 ⎞
4. lim ⎛⎜ 3x – 2 ⎞⎟ 5. lim ⎛⎜ 1 – 3 ⎞⎟ 6. lim ⎛⎜ ⎟
x →∞ 2 x + 3
x →∞
⎝ 3x ⎠ x →∞
⎝ 2 x +7 ⎠ ⎝ ⎠
3 x +1
7. lim ⎛⎜ x + 2 ⎞⎟
x →∞ x +5
⎝ ⎠
Answers
1 8
1
1. e5 2. 3. e 2 4. e 3 5. e–3 6. e–4 7. e–9
e2

77 Limit of Function
EXERCISES 2 .2
0 2005 x + 2010 x4 + ax + 2
A. The Indeterminate of the Form e. lim f. lim
0 x →∞ x2 +1 x →∞ ( x2 – 1)2 – a

1. Perform the following limits:


| x |+2 x – 1 x2 + x – 2
x–5 x2 – x – 6 g. lim h. lim
a. lim 2 b. lim 2 x → – ∞ 3x + 4| x |+5 x→ – ∞ 5
x5 – 1
x → 5 x – 25 x → 5 x + 3x – 2

2 2

c. lim
x2 – 5 x – 6
d. lim
5 x3 – 5 i. lim log( x + 2 x +1) j. lim( x – x )
x →–1 x +1 x →1 x –1 x →∞ x2 – 7 x →∞ x + 4 3
x2 – 9 x–2 2 x 3 – x2 2x
e. lim f. lim k. lim( – x) l. lim
x→ 3 x2 – 5x +6 x→ 2
2 x – 3x + 4 2
x→ – ∞ 1+ 2 x2 x →∞ 2
cot
2 – | x| x
g. lim 8 x +9 – 7 h. lim
x→5
1– x – 4 x →–2 2+ x tan 3 x sin x
m. lim n. lim
3
x –m 3 2 x+4 π sec5 x x →∞ π– x
j. lim 2 – 16 x→
i. lim 2
x→ m 2 x – mx – m2
2 x→ 0 2 x+1 – 2
2x
o. lim
x →∞
x3
3x – 1
2. Perform the following limits:
sin5 x x–3
a. lim b. lim tan
x→0 7x x→ 3 tan(2 x – 6) C. The Indeterminate of the form 0 – ∞
sin(3 x – 6) 1 – cos 2 x 4. Perform the following limits:
c. lim d. lim
x→ 2 tan( x – 2) x→0 sin 2 x
6 5
sin x cos x +1 a. lim( x ⋅ sin ) b. lim x2 – x + 3 ⋅
e. lim f. lim x →∞ 7x x →∞ x+ 2
x →π ( π – x)2
x→0
1 – cos 2 x
3
π – 2x c. lim( 3x ⋅ tan x ) d. lim(sin 2 x ⋅ cot 5 x)
g. lim cos 2x – 1 h. lim x →∞ x x→ 0
π sin 2 x
x→0 x x→
2 x4 + ax + 2
e. lim(tan x ⋅ cot 3 x ) f. lim
i. lim sin2 x 2
j. lim x + x + 2
x→0 π 2 2
x → ( x – 1) – a
2
x→0
cos x – cos 2 x x→ – ∞
3+ 5 x5 +1 πx
g. lim(tan πx ⋅ tan )
x →1 2

B. The Indeterminate of the Form



D. The Indeterminate of the form ∞ – ∞

3. Perform the following limits: 5. Perform the following limits:
3x3 + 2 x2 +1 x2 + x +1 x+ 2 4 x 4
a. lim b. lim a. lim( – ) b. lim( – )
x →∞ x3 – 1 x →∞ 3x3 + 4 x→0 x 1 – x2 x →1 x – 1 x2 – 2 x +1
1 1
x2 +1 x2 + x + 2 c. lim( – ) d. lim(cot x – cosec x)
c. lim d. lim x→
π cos x cot x x →0
x→ – ∞ 5x – 4 x→ – ∞ 5 5
3+ x +1 2

Limit of Functions 78
6. Perform the following limits: x 8
d. lim( ⋅ sin )
2
a. lim(3 x – x +5 x +1) x →∞ 2 x
x →∞

e. lim( x( x + 2 – x ))
b. lim( x2 + x +1 – x2 + 2 x – 3) x→0
x →∞

f. lim(log x2 + x – log x2 – 3 x)
2 2
c. lim( x2 + 2 x + 2 – x+1) x →∞
x →∞

g. lim( 1 2
– )
d. lim( x x2 +1 – x2 ) x→
π 1 – sin x cos 2 x
x →∞ 2

e. lim( x x2 – 2 + x2 ) h. lim(log 2 8 x+1+ log 1/ 2 2 x+5)


x→ – ∞ x →∞

f. lim( x2 – 1 – x2 +1)
x→ – ∞

E. The Indeterminate of the Form 1∞

7. Perform the following limits:


2 3
a. lim(1+ )x b. lim(1 – )3 x
x →∞ x x →∞ x
x+ 3 x 6 x +1 2 x
c. lim( ) d. lim( )
x →∞ x x →∞ 6x
2 2 x+1 5x +1 3 x+1
e. lim(1 – ) f. lim( )
x →∞ 3x +1 x →∞ 5x – 1

7 x + 2 9 x –1
g. lim(1+ )2 x h. lim( )
x→ – ∞ 5x + 3 x →∞ x+ 4

3x +1 3 x+1
i. lim( )
x →∞ 3x + 2

Mixed Problems

8. Perform the following Limits


4
a. lim(2+ 2 x sin )
x →∞ x

b. lim((1
π
– sin x) ⋅ tan x)
x→
2

c. lim(( x2 + sin 2 x) ⋅ cot 2 x)


x→0

79 Limit of Function
CHAPTER 4

CONTINUITY
CHAPTER 5

DIFFERANTIATION

Differentiation 1
A. TANGENTS
The word ‘tangent’ comes from the Latin word tangens, which means ‘touching’. Thus, a
y
tangent line to a curve is a line that “just touches” the curve. In other words, a tangent
line should be parallel to the curve at the point of contact. How can we explain this idea
clearly? Look at the figures below.
A

A A A
x
a tangent line to a curve B

original curve zoomed in once zoomed in twice


y
B

As we zoom in to the curve near the point A, the curve becomes almost indistinguishable from
A
the tangent line. So, the tangent line is parallel to the curve at the point A.
How can we find the equation of a tangent to a curve at a given point? The graphs below
x show one approach.
a secant line to a curve t
y y
B(x, f(x)) B

f(x) – f(a) B

A(a, f(a)) A B
The slope of the line is
x–a
the tangent of the angle
between the line and the
positive x-axis.
a x x x

a a The first graph shows the curve y = f(x). The points A(a, f(a)) and B(x, f(x)) are two points on
this curve. The secant line AB has slope mAB, where
positive negative f (x) − f (a)
slope slope mAB = .
x−a
Now suppose that we want to find the slope of the tangent to the curve at point A. The
second graph above shows what happens when we move point B closer and closer to
zero no point A on the curve. We can see that the slope of the secant line AB gets closer and
slope slope closer to the slope of the tangent at A (line t). In other words, if m is the slope of the
tangent line, then as B approches A, mAB approaches m.

102 Derivatives
Definition tangent line
The tangent line to the curve y = f(x) at the point A(a, f(a)) is the line through A with the
slope
f (x) − f (a )
m = lim ,
x→ a x−a
provided that this limit exists.

Example 1 Find the equation of the tangent line to the curve y = x2 at the point A(1, 1).

Solution We can begin by calculating the slope of the tangent.


Here we have a = 1 and f(x) = x2, so the slope is

f (x) − f (1) x2 − 1
m = lim = lim
x →1 x −1 x →1 x − 1

The equation of a line (x − 1)(x +1)


through the point (x1, y1) m = lim = lim( x+1) =1+1= 2.
x →1 (x − 1) x →1
with slope m:
y – y1 = m(x – x1).
Now we can write the equation of the tangent at point (1, 1):
y – y1 = m(x – x1)
y – 1 = 2(x – 1)
y = 2x – 1.

Example 2 Find the equation of the tangent line to the curve y = x3 – 1 at the point (–1, –2).

Solution Here we have a = –1 and f(x) = x3 – 1, so the slope is


f ( x) – f ( −1) ( x3 − 1) − (( −1)3 −1) x3 +1
m = lim = lim = lim
x→ –1 x – (–1) x→ –1 x +1 x→ –1 x +1

( x +1)( x2 – x + 1)
x3+ y3=(x+ y)(x2–xy+y2) m = lim
x→ –1 ( x +1)
m = lim( x2 – x +1) = (–1) 2 – (–1)+1
x→ –1

m = 3.

So the equation of the tangent line at (–1, –2) with slope m = 3 is


y – y1 = m(x – x1)
y – (–2) = 3(x – (–1))
y + 2 = 3x + 3
y = 3x + 1.

Differentiation 103
We can also write the expression for the slope of a tangent line in a different way. Look at the
graphs below.
y y y=f(x)
B B
y=f(x)

A A

a a+h x h x
h
h
h

From the first graph, writing x = a + h gives us the slope of the secant line
f (a+ h) − f (a)
mAB = .
h
We can see in the second graph that as x approaches a, h approaches zero. So the
expression for the slope of the tangent line becomes:
f (a + h) − f (a )
m = lim .
h→0 h

THE SLOPE OF A TANGENT LINE TO A CURVE

The slope of a tangent line to a curve y = f(x) at x = a is


f (a + h) − f (a )
m = lim .
h→0 h

Example 3 Find the equation of the tangent line to the curve y = x3 at the point (–1, –1).

Solution Let f(x) = x3. Then the slope of the tangent at (–1, –1) is
f ( − 1+ h) − f (–1) ( −1+ h) 3 −( −1) 3
m = lim = lim
h→0 h h→0 h
(–1)3 + 3(–1) 2 h+ 3(–1) h2 + h3 −(–1) 3
3 3 2 2 3 m = lim
(x+y) =x +3x y+3xy +y h→0 h
h(3 − 3h + h2 )
m = lim = lim(3 − 3 h+ h2 ) = 3.
h→0 h h→0

So, the equation of the tangent at point (–1, –1) is


y – (–1) = 3(x – (–1))
y + 1 = 3x + 3
y = 3x + 2.

104 Derivatives
Example 4 Find the equation of the normal line to the curve y =
2
x
at the point (2, 1).

Solution Recall that a normal line is a line which is perpendicular to a tangent. The product of the

slopes mt of a tangent and mn of a normal is –1.


Let us begin by finding the slope of the tangent.
2 2 2
– –1
f (2+ h) – f (2) 2+ h 2 2+ h
mt = lim = lim = lim
h→ 0 h h→ 0 h h→ 0 h
2 – (2+ h) –h –1
mt = lim = lim = lim
h→ 0 h( h + 2) h → 0 h( h + 2) h → 0 2+ h

1
mt = – .
2
We have mt ⋅ mn = –1.

mn ⋅ mt = –1 –1 –1
The product of slopes of So, mn = = = 2.
mt 1
the tangent line and the –
normal line at a point 2
equals –1. The equation of the normal line passing through the

point (2, 1) with the slope mn = 2 is


y – y1 = mn(x – x1)
y – 1 = 2(x – 2)
y = 2x – 3.

Check Yourself 1
1. Find the equation of the tangent line to each curve at the given point P.
a. f(x) = x2 – 1 P(–1, 0)

b. f(x) = x3 + 1 P(0, 1)
1 1
c. f(x) = P( , 2)
x 2
2. Find the equation of the normal line at point P for each curve in the previous question.
Answers
1. a. y = –2x – 2 b. y = 1 c. y = –4x + 4

1 1
2. a. y = x+ b. x = 0 c. y = 1 x + 15
2 2 4 8

Differentiation 105
B. VELOCITIES
position at position at
Imagine you are in a car driving time t = 2 time t = 2+h

across a city. The velocity of the car


will not be constant. Sometimes
the car will travel faster, and
sometimes it will travel slower.

However, the car has a definite x


0
velocity at each moment. This is S(2 + h) – S(2)
called the instantaneous velocity of
S(2)
the car. How can we calculate the
instantaneous velocity?
S(2+h)
To answer this question, let us look at a simpler example: the motion of an object falling
through the air. Let g = 9.8 m/s2 be the acceleration of the object due to gravity. We know
from physics that after t seconds, the distance that the object will have fallen is
1 2
s(t) =gt meters or s(t) = 4.9 t2 meters.
2
Suppose we wish to calculate the velocity of the object after two seconds. We can begin by
calculating the average velocity over the time interval [2, 2 + h]:
distance travelled
average velocity =
elapsed time

s(2 + h) − s(2) 4.9(4 + 4h+ h2 − 4)


= = = 19.6 + 4 .9 h
h h

If we shorten the time period, the average velocity is becoming closer to 19.6 m/s, the value
of instantaneous velocity.
More generally, we can calculate the instantaneous velocity V(a)
of an object at time t = a by the limit of the average velocities:
s(a + h) − s( a)
V(a ) = lim
h→ 0 h
This is not the first time we see the above formula. It is the
same formula that we use for the slope of the tangent line to a
curve. Remember that
f (a + h) − f (a )
m = lim .
h→ 0 h
This means that the velocity at time t = a is equal to the slope
of the tangent line at A(a, s(a)).

106 Derivatives
Example 5 A stone is dropped from the top of the Eiffel Tower. What is the velocity of the stone after five
seconds?

Solution We use the equation of motion s(t) = 4.9t2 to find the


velocity V after five seconds:
s(5+ h) − s(5) 4.9(5+ h) 2 − 4.9(5) 2
V(5) = lim = lim
h →0 h h →0 h
4.9(25 +10 h+ h2 − 25) 4.9(10 h+ h2)
V(5) = lim = lim
h →0 h h →0 h
4.9h(10+ h)
V(5) = lim = lim(49+ 4.9 h) = 49 m/s.
h →0 h h →0

Example 6 A particle moves along a straight line with the equation of motion s(t) = t2 + 3t + 1, where
s(t) is measured in meters and t is in seconds.
a. Find the average velocity over the interval [1, 2].
b. Find the instantaneous velocity at t = 2.
Solution a. Average velocity is the ratio of distance travelled to elapsed time. So, we have
s(2) – s(1) (2 2 + 3 ⋅ 2+1) – (1 2 + 3 ⋅1+1)
average velocity = = = 6 m/s.
2 –1 1
b. Let V(2) be the velocity after two seconds.
s(2+ h) – s(2) ( h+ 2 )2 + 3 ⋅( h + 2) +1 −[2 2 + 3 ⋅2 +1]
V(2) = lim = lim
h→ 0 h h→ 0 h
h2 + 4h + 4+ 3h +6+1 − 11 h2 +7 h
V(2) = lim = lim = lim( h+7) = 7 m/s.
h→ 0 h h→ 0 h h→ 0

Check Yourself 2
1. A basketball player throws a ball upward at a speed of 20 m/s. This means that after t
seconds, the ball’s height will be s(t) = 20t – 4.9t2.
a. Find the average velocity of the ball over the interval [1, 2].
b. Find the instantaneous velocity of the ball after two seconds.
2. The displacement of a particle moving in a straight line is given by the equation of motion
s(t) = 2t3 + 3t – 2, where t is measured in seconds and s(t) is in meters.
a. Find the average velocity of the particle over the following intervals.
i. [1, 3] ii. [1, 4] iii. [2, 4]
b. Find the instantaneous velocity of the particle at each time.
i. t = 2 ii. t = 3 iii. t = 4
Answers
1. a. 5.3 m/s b. 0.4 m/s 2. a. i. 29 m/s ii. 45 m/s iii. 59 m/s b. i. 27 m/s ii. 57 m/s iii. 99 m/s

Differentiation 107
C. RATES OF CHANGE
In section A we learned how to find the slope of a tangent line and in section B we learned
how to calculate the instantaneous velocity of an object from a given acceleration. We can
say that acceleration is a rate of change: it shows how fast or slowly a quantity (the velocity)
changes from one moment to the next. Other examples of rates of change are how fast a
population grows, or how fast the temperature of a room changes over time.

The problem of finding a rate of change is mathematically equivalent to finding the slope of
a tangent line to a curve. To understand why, suppose y is a quantity that depends on
another quantity x. Thus, y is the function of x and we write y = f(x).

Look at the graph of f(x). If x increases by an amount h, then y increases by f(x + h) – f(x).
y y=f(x)

B(x+h, f(x+h))
f(x+h)
f(x+h) – f(x) is the
f(x+h) – f(x) change in y that corresponds
A(x, f(x)) to a change h in x.
f(x)
h
x x+h x

f (x+ h) − f (x)
The difference quotient is called the average rate of change of y with
h
respect to x over the interval [x, x + h] and can be interpreted as the slope of the secant line
AB. If we take the limit of the average rate of change, then we obtain the instantaneous rate
of change of y with respect to x, which is interpreted as the slope of the tangent line to the
curve y = f(x) at A(x, f(x)).

The following summarizes this part:

RATES OF CHANGE

1. The average rate of change of f over an interval [x, x + h] is


f (x+ h) − f (x)
.
h
2. The instantaneous rate of change of f(x) at a point x is
f (x + h) − f (x)
lim .
h→0 h

108 Derivatives
Example 7 A student begins measuring the air temperature in a room at eight o’clock in the morning.
2
She finds that the temperature is given by the function f (t ) =16+ t 2 °C, where t is in
3
hours. How fast was the temperature rising at 11:00?

Solution We are being asked to find the instantenous rate of change of the temperature at
t = 3, so we need to find the following limit:
f (3+ h) – f (3)
rate of change = lim
h →0 h
2 2
16 + (3 + h)2 – (16 + (3) 2 )
= lim 3 3
h →0 h
2 2
16+ ⋅ (9+6 h + h2 ) – 16 − ⋅ 9
= lim 3 3
h →0 h
2
(9 + 6 h + h2 – 9)
= lim 3
h →0 h
2h(6+ h)
= lim
h →0 3h
2
= lim(6+ h)
3 h →0
2
= ⋅ 6 = 4 °C per hour
3

Example 8 A manufacturer estimates that when he produces x units of a certain commodity, he earns
R(x) = x2 – 3x – 1 thousand dollars. At what rate is the revenue changing when the
manufacturer produces 3 units?

Solution We need to find the instantaneous rate of change of the revenue at x = 3, so

R(3+ h) – R(3) (3+ h) 2 – 3(3+ h) – 1 – (3 2 – 3 ⋅3 –1)


rate of change = lim =
h→ 0 h h
9+6 h + h2 – 9 – 3 h 3 h + h2
= lim = lim = lim(3+ h) = 3.
h→ 0 h h→ 0 h h→ 0

It follows that revenue is changing at the rate of $3000 per unit when 3 units are produced.

In conclusion, rates of change can be interpreted as the slope of a tangent. Whenever we


solve a problem involving tangent lines, we are not only solving a problem in geometry but
also solving a great variety of problems in science.

Differentiation 109
D. DERIVATIVE OF A FUNCTION
f (x+ h) − f (x)
Up to now we have treated the expression as a ‘difference quotient’ of the
h
function f(x). We have calculated the limit of a difference quotient as h approaches zero.

Since this type of limit occurs so widely, it is given a special name and notation.

Definition derivative of a function


The derivative of the function f(x) with respect to x is the function f ′(x) (read as “f prime of x”)
defined by
f (x + h) − f (x)
f ′(x) = lim .
h→ 0 h
The process of calculating the derivative is called differentiation. We say that f(x) is
differentiable at c if f ′(c) exists.

Thus, the derivative of a function f(x) is the function f ′(x), which gives
1. the slope of the tangent line to the graph of f(x) at any point (x, f(x)),
2. the rate of change of f (x) at x.

FOUR-STEP PROCESS FOR FINDING f′′ (x)

1. Compute f(x + h).


2. Form the difference f(x + h) – f(x).
f (x+ h) − f (x)
3. Form the quotient .
h
f (x + h) − f (x)
4. Compute f ′(x) = lim .
h→ 0 h

Example 9 Find the derivative of the function f(x) = x2.

Solution To find f ′(x), we use the four-step process:


1. f(x + h) = (x + h)2 = x2 +2xh + h2
2. f(x + h) – f(x) = x2 + 2xh + h2 – x2 = 2xh + h2
2
3. f (x+ h) − f (x) = 2 xh+ h = h(2x+ h ) = 2x+ h
h h h
4. lim(2 x + h) = 2 x
h→ 0

Thus, f ′(x) = 2x.

110 Derivatives
Example 10 Find the derivative of the function f(x) = x2 – 8x + 9 at x = 1.

Solution We apply the four-step process:


1. f(x + h) = (x + h)2 – 8(x + h) + 9 = x2 + 2xh + h2 – 8x – 8h + 9
2. f(x + h) – f(x) = x2 + 2xh + h2 – 8x – 8h + 9 – (x2 – 8x + 9) = 2xh + h2 – 8h
2
3. f (x+ h) − f (x) = h + 2 xh − 8h = h + 2x − 8
h h
f (x + h) − f (x)
4. lim = lim( h + 2 x − 8) = 2 x − 8
h→0 h h→0

So, f ′(x) = 2x – 8 and f ′(1) = 2 ⋅ 1 – 8 = –6.

This result tells us that the slope of the tangent line to the graph of f(x) at the point x = 1 is
–6. It also tells us that the function f(x) is changing at the rate of –6 units per unit change
in x at x = 1.

Example 11 Let f(x) =


1
x
.

a. Find f ′(x).
b. Find the equation of the tangent line to the graph of f(x) at the point (1, 1).

1 1 h
− −
f (x + h) − f (x) x(x + h) 1 1
Solution a. f ′(x) = lim = lim x + h x = lim = lim( − )= − 2 .
h→0 h h→0 h h→0 h h→0 x(x + h) x

b. In order to find the equation of a tangent line, we y


y = –x+2
have to find its slope and one point on the
tangent line. We know that the derivative gives us
the slope of the tangent. Let m be the slope of the 2
1
tangent line, then 1 y=
x

1 1 2 x
m = f ′(1) = −
= −1. 1
12 y=
x
So, the equation of the tangent line to the graph
of f(x) at the point (1, 1) with the slope m = –1
is
y – 1 = –1(x – 1)
y = –x + 2.

Differentiation 111
Example 12 The function f(x) = ñx is given. Find the derivative of f(x) and the equation of the normal
line to f(x) at the point x = 1.

f (x + h) − f (x) x+ h − x ⎛ x + h – x x+ h + x ⎞
Solution f ′(x) = lim = lim = lim ⎜ ⋅ ⎟
h→ 0 h h → 0 h h → 0
⎝ h x+ h + x ⎠
x+ h − x h 1
f ′(x)=lim = lim = lim
h→ 0
h( x + h + x ) h→ 0
h( x + h + x ) h→ 0
x+ h + x
1
f ′(x)=
2 x

Remember that if mt is the slope of a tangent and mn is the slope of a normal at the same
point, then mt ⋅ mn = –1. So, we can find the slope of the normal from the slope of the
tangent. Then we can write the equation of normal line to f(x) at the point x = 1.
The slope of the tangent is

1
1
mt = f ′(1) = = .
2⋅ 1 2
The slope of the normal is
1 1
mn = − = − = −2.
mt 1
2
The equation of the normal line is
y – y0 = mn ⋅ (x – x0)
y – 1 = –2(x – 1) (Note that y0 = f(x0), that is y0 = f(1) = 1)
y = –2x + 3.

Check Yourself 3
1. Find the derivative of the function f(x) = 2x + 7.
2. Let f(x) = 2x2 – 3x.
a. Find f ′(x).
b. Find the equation of the tangent line to the graph of f(x) at the point x = 2.
3. Find the derivative of the function f(x) = x3 – x.
1
4. If f ( x) = , find the derivative of f(x).
x+ 2
Answers
1
1. 2 2. a. 4x – 3 b. y = 5x – 8 3. 3x2 – 1 4. −
2 ( x + 2)3

112 Derivatives
E. LEFT-HAND AND RIGHT-HAND DERIVATIVES
When we were studying limits we learned that the limit of a function exists if and only if the
left-hand and the right-hand limits exist and are equal. Otherwise the function has no limit.
From this point, we may conclude that the derivative of a function f(x) exists if and only if
f ( x + h) − f ( x ) f ( x + h) − f ( x )
f ′( x− ) = lim− and f ′( x+ ) = lim+ exist and are equal.
h→ 0 h h → 0 h
These expressions are respectively called the left-h hand derivative and the right-h hand
derivative of the function.

Example 13 Show that the function f(x)= ñx does not have a derivative at the point x = 0.

Solution Here we should find the left-hand derivative and the right-hand derivative. If they exist, then
we will check whether they are equal or not.
Let us find the left-hand derivative:
f (0 + h) − f (0) 0 +h − 0 h
f ′(0 − ) = lim− = lim− = lim− .
h→ 0 h h → 0 h h → 0 h
Since h < 0, ñh is undefined and this limit does not exist. So the left-hand derivative does
not exist either.
Thus, the function f(x)= ñx has no derivative at the point x = 0.

⎧⎪ x2 − 1, x ≥1
Example 14 f(x) is given as f ( x) = ⎨
⎪⎩2 x − 2, x <1
.

Does this function have a derivative at the point x = 1?

Solution We will find the left-hand and the right-hand derivatives.


f (1 + h) − f (1) 2(1 + h) − 2 − 0 2h
f ′(1− ) = lim− = lim− = lim− =2
h→ 0 h h→ 0 h h→ 0 h
f (1 + h) − f (1) (1 + h)2 − 1 − 0 h2 + 2 h
f ′(1+ ) = lim+ = lim+ = lim+ = lim( h + 2 ) = 2.
h→0 h h→ 0 h h→ 0 h h→ 0 +

The left-hand and the right-hand derivatives are equal to each other. Thus, the derivative of

the function at the point x = 1 exists and

f ′(1) = f ′(1–) = f ′(1+) = 2.

Differentiation 113
F. DIFFERENTIABILITY AND CONTINUITY
Recall that if f ′(c) exists, then the function f(x) is differentiable at point c. Similarly, if f(x)
is differentiable on an open interval (a, b), then it is differentiable at every number in the
interval (a, b).

Example 15 Where is the function f(x) = |x| differentiable?

Solution We can approach this problem by testing the differentiability on three intervals:
x > 0, x < 0 and x = 0.
1. If x > 0, then x + h > 0 and |x + h| = x + h.
Therefore, for x > 0 we have
| x + h | – | x| x+ h – x h
f ′(x) = lim = lim = lim = lim1=1.
h→ 0 h h → 0 h h → 0 h h→ 0

So, f ′(x) exists and f (x) is differentiable for any x > 0.


2. If x < 0, then |x| = –x and |x + h| = –(x + h) if we choose h small enough such that
it is nearly equal to zero.
Therefore, for x < 0 we have

| x + h | – | x| –( x + h) – (– x) –h
f ′(x) = lim = lim = lim = lim(–1) = –1
x→0 h x → 0 h h → 0 h h→ 0

So, f ′(x) exists and f(x) is differentiable for any x < 0.


3. For x = 0 we have to investigate the left-hand and the right-hand derivatives separately:
|0+ h| – |0| | h| h
lim+ = lim+ = lim+ = lim+ 1=1
h→ 0 h h→0 h h→ 0 h h→ 0

|0+ h| – |0| | h| –h
lim = lim– = lim– = lim(–1) = –1
h→ 0– h h→0 h h→ 0 h h→ 0–

Since these limits are different, f ′(x) does not exist. So, f(x) is not differentiable for
x = 0.
In conclusion, f(x) is differentiable for all the values of x except 0.
Alternatively, from the graph of f(x), we can see that f(x) does not y
y = f(x)
have a tangent line at the point x = 0. So, the derivative does not
exist.
Note that the function does not have a derivative at the point where x
the graph has a ‘corner’.

114 Derivatives
If a function f(x) is differentiable at a point, then its graph has a non-vertical tangent line at
A function f is this point. It means that the graph of the function cannot have a ‘hole’ or ‘gap’ at this point.
continuous at x = a
Thus, the function must be continuous at this point where it is differentiable.
if and only if
lim f ( x) = f ( a).
x→ a

Note
If f(x) is differentiable at a, then f(x) is continuous at a.

The converse, however, is not true: a continuous function may not be differentiable at every
point.
For example, the function f(x) = |x| is continuous at 0, because lim f (x) = 0 = f (0).
x →0

But it is not differentiable at the point x = 0.

⎧2 x2 − x, x>2
⎪⎪
Example 16 The piecewise function f(x) is given as f ( x) = ⎨6,
⎪ 3
x = 2.
⎪⎩ x − 2, x<2
a. Is f(x) continuous at x = 2?
b. Is f(x) differentiable at x = 2?

Solution a. Since lim f ( x) = f (2), f(x) is continuous at x = 2.


x→ 2

b. Let us find the left-hand and the right-hand derivatives of the function f(x) at the point x = 2.

f (2+ h) − f (2) (2 + h)3 − 2 − 6


f (2 − ) = lim− = lim−
h→ 0 h h→ 0 h

2 3 + 3 ⋅ 2 2 h + 3 ⋅ 2 h2 + h3 – 8 h(12+6 h+ h2 )
= lim− = lim−
h→ 0 h h→ 0 h

= lim(12+6

h+ h2 ) =12
h→ 0

f (2+ h) − f (2) 2 ⋅(2+ h)2 −(2 + h) − 6


f (2 + ) = lim+ = lim+
h→ 0 h h→ 0 h

2 ⋅ (4+ 4h + h2 ) − 2 − h − 6 8+8 h + 2 h2 − 2 − h − 6
= lim+ = li m+
h→ 0 h h→ 0 h

h(7+ 2 h)
= lim+ = lim(7+ 2 h) = 7.
h→ 0 h h→ 0 +

Since f ′(2+) ≠ f ′(2–), the derivative of the function f(x) does not exist at the point x = 2.
So, the function is continuous at x = 2, but it is not differentiable at the same point.

Differentiation 115
We have seen that a function f(x) is not differentiable at a point if its graph is not
continuous at x = a. The figures below show two more cases in which f(x) is not
differentiable at x = a:
y y y

a x a x a x

a discontinuity a corner a vertical tangent

CRITERIA FOR DIFFERENTIABILITY

For the following cases the function is not differentiable at a given point:
1. the graph has a discontinuity at the point,
2. the graph has a ‘corner’ at the point,
3. the graph has a vertical tangent line at the point.

Example 17 Explain why the function shown in the graph


on the right is not differentiable at each of the
y
y = f(x)

points x = a, b, c, d, e, f, g.

Solution The function f(x) is not differentiable at the


points x = a, b, c because it is discontinuous at
each of these points. The derivative of the
function f(x) does not exist at x = d, e, f a b c d e f g x
because it has a corner at each of these points.
Finally, the function is not differentiable at x = g because the tangent line is vertical at that point.

116 Derivatives
Check Yourself 4
⎧ x − 1, x <1
1. Given that f ( x) = ⎨ 2 , show that the derivative of f(x) does not exist at the
⎩ x − 1, x ≥1
point x = 1.

2. f(x) = |x2 – 4x + 3| is given. Find the derivative of f(x) at the point x = 3.

3. The graph of a function f is given below. State, with reasons, the values at which f is not
differentiable.
y
y = f(x)

-1
1 2 3 4 5 6 7 8 9 10 11 x

Answers
1. compare f ′(1–) and f ′(1+).
2. does not exist.
3. x = –1, corner; x = 4, discontinuity; x = 8, corner; x = 11, vertical tangent.

Differentiation 117
At the beginning of our study of derivatives we have learned that a curve lies very close to its
tangent line near the point of contact. This means that for the same value of x near the point
of tangency, the values of y on the curve and tangent line are approximately equal to each other.
This fact gives us a useful method for finding approximate values of functions.
We can use the tangent line at (a, f(a)) as an approximation to the function f(x) when x is near
a. The equation of this tangent line is
y = f(a) + f ′(a)(x – a).
So, our approximation becomes
f(x) ≈ f(a) + f ′(a)(x – a).
This type of approximation is called the linear approximation or tangent line approximation of
f(x) at a. The linear function whose graph is the tangent line
L(x) = f(a) + f ′(a)(x – a)
is called the linearization of f(x) at a. The geometric interpretation of linear approximation is
show in the figure.
y y = f(x)

y = f(x)

f(a)
L(a)

a x

The linear approximation f(x) ≈ L(x) is a good approximation when x is near a. It is very
useful in physics for simplifying a calculation or a theory. You might think that a calculator can
give us better approximation than the linear approximation. But a linear approximation gives an
approximation over an entire interval, which can be more useful. For this reason, scientists
frequently use linear approximation in their work. The following example illustrate the use of
linear approximation method to simplify calculation.
For example, let us find the linearization of the function f ( x)= x + 2 at a = 2, and use it to
approximate the numbers 3,99 and 4,01 .

First, we have to find f ′(2), the slope of the tangent line to the curve f ( x)= x + 2 when x = 2.
1
The derivative of f(x) is f ′( x)=( x + 2 )′ = .
2 x+ 2
1 1
So, f ′(2)= = .
2 2+2 4
The linearization is given by
L(x) = f(a) + f ′(a) ⋅ (x – a)
1 x 3
L(x) = f(2) + f ′(2) ⋅ (x – 2) = 2 + (x – 2) = + .
4 4 2
x 3
The linear approximation is therefore x + 2 ≈ L( x)= + .
4 2
In particular we have

1.99 3
3.99 = 1.99+ 2 ≈ L(1.99) = + =1.9975
4 2
2.01 3
4.01 ≈ L(2.01) = + = 2.0025.
4 2
x 3
The graphs of f ( x)= x + 2 and its linear approximation L( x)= + are shown below. We
4 2
see that our approximations are overestimates because the tangent line lies above the curve.
y
x 3
L(x) = +
4 2

2 f(x) = x + 2

–2 2 x

The following table shows estimates from the linear approximation with the actual values.

approximation actual value

3.99 1.9975 1.99749...

4.01 2.0025 1.00249...


EXERCISES 1 .1
A. Tangents C. Rates of Change
1. Find the slope of the tangent line to the graph of
each function at the given point. 5. The volume of a spherical cancer tumor is given
4 3
a. f(x) = 5x – 1 ; x = 3 by the function V( r ) = π r , where r is the
3
b. f(x) = 4 – 7x ; x = 2 radius of the tumor in centimeters. Find the rate

c. f(x) = x2 – 1 ; x = –1 of change in the volume of the tumor when


2
d. f(x) = 3x2 – 2x – 5 ; x = 0 r = cm.
3
› e. f(x) = x3 – 3x + 5 ; x = 1
6. A certain species of eagle faces extinction. After a
› f. f(x) = x + ñx ; x = 4 conservation project begins, it is hoped that the
1 eagle population will grow according to the rule
› g. f ( x) = ; x= 2
x2 N(t) = 2t2 + t + 100 (0 ≤ t ≤ 10), where N(t)
4x denotes the population at the end of the year t.
› h. f ( x) = ; x= 2
x +1 Find the rate of growth of the eagle population
when t = 2 and the average rate of growth over
the interval [2, 3].
2. Find the equation of the tangent line to each
function at the given point.
a. f(x) = 2x + 5 at (2, 9)
b. f(x) = x2 + x + 1 at (1, 3)
› c. f(x) = x3 – x at (2, 6)

› d. f(x) = 2ñx at (4, 4)

B. Velocities
7. The fuel consumption (measured in litres per
3. A particle moves along a straight line with
hour) of a car travelling at a speed of v kilometers
the equation of motion s(t) = t2 – 6t – 5, where s
per hour is c = f(v).
is measured in meters and t is in seconds. Find
the velocity of the particle when t = 2. a. What is the meaning of f ′(v)?

b. What does the statement f ′(20) = – 0.05 mean?


4. If a stone is dropped from a height of 100 m, its
height in meters after t seconds is given by
s(t) = 100 – 5t2. Find the stone’s average velocity
over the period [2, 4] and its instantaneous
velocity at time t = 4.

120 Derivative
D. Derivative of a Function ⎧ x +6, x> 3
8. Each limit below represents the derivative of a 14. Given that f ( x) = ⎪⎨x2 , x = 3 find f ′(3).
function f(x) at x = a. Find the function f and the ⎪ 3
⎩ x – 6 x, x> 3
number a in each case.
(1+ h)10 − 1 3
a. lim b. lim 8+ h − 2 Mixed Problems
h→ 0 h h→ 0 h
3x − 81 cos(π + h)+1 15. Consider the slopes y
c. lim d. lim
x→ 4 x − 4 h→0 h of the tangent lines A E
to the given curve at
9. Find the derivative of each function. each of the five B D
2 x +1
a. f(x) = 3 – 2x + x › b. f ( x) =
2
points shown. List
x −1 C
these five slopes in
3 x
› c. f ( x) = › d. f ( x) = 3x +1 decreasing order.
x
16. If the tangent to the graph of f(x) = x2 – 2ax + 3
E. Left-Hand and Right-Hand ›
at x = –1 is parallel to the line 2x – y = 1, find a.
Derivatives
6x, 0≤x≤8
10. Let f(x) = . 17. At which point of the curve y = x2 + 4 does its
9x – 24, 8 < x ››
tangent line pass through the origin?
Does the function have derivative at x = 8? Why
or why not?
18. An arrow is shot upward on a planet. Its height (in
›
11. Given that f(x) = |x – 1|, find f ′(1). meters) after t seconds is given by h(t) = 60t – 0.6t2.
a. At what time will the arrow reach the top?
F. Differentiability and Continuity
12. b. With what velocity will the arrow hit the
y
ground?

19. Given the continuous function


y = f(x) ››
⎧ x2 +10 x +8, x ≤ –2

f ( x) = ⎨ax2 + bx + c, –2 < x < 0,
–2 –1 1 3 6 x
⎪ 2
⎩ x + 2 x, x≥0
find a, b and c such that its graph has a tangent
The graph of f(x) is given. At what numbers is f(x)
touching it at three points..
not differentiable? Why?

20. Given that f(x) = |x2 – 2x|, find f ′(1).


x2 + 7x, x ≤ 1 ›
13. Let f(x) = .
9x – 24, x > 1
Does the function have derivative at x = 1? Why 21. Using linear approximation calculate ò99.
or why not? ››

Differentiation 121
A. BASIC DIFFERENTIATION RULES
Up to now, we have calculated the derivatives of functions by using the definition of the
derivative as the limit of a difference quotient. This method works, but it is slow even for
quite simple functions. Clearly we need a simpler, quicker method. In this section, we begin
to develop methods that greatly simplify the process of differentiation. From now on, we will
use the notation f′′(x) (f prime of x) to mean the derivative of f with respect to x. Other books
and mathematicians sometimes use different notation for the derivative, such as
d dy
f (x) = y′ = = Dx( f (x)).
dx dx
All of these different types of notation have essentially the same meaning: the derivative of a
function with respect to x. Finding this derivative is called differentiating the function with
respect to x.
In stating the following rules, we assume that the functions f and g are differentiable.
Our first rule states that the derivative of a constant function is equal to zero.

THE DERIVATIVE OF A CONSTANT FUNCTION

If c is any real number, then c′ = 0.

We can see this by considering the graph of the constant y


function f(x) = c, which is a horizontal line. The tangent line to y=c
a straight line at any point on the line coincides with the straight
slope = 0
line itself. So, the slope of the tangent line is zero, and therefore
the derivative is zero.
We can also use the definition of the derivative to prove this x
result: The slope of the tangent to
f (x + h) − f (x) c −c the graph of f(x) = c, where c is
f ′(x) = lim = lim = lim 0 = 0. constant, is zero.
h→ 0 h h→ 0 h h→ 0

Example 14 a. If f(x) = 13, then f ′(x) = (13)′ = 0.

1 ⎛ 1 ⎞′
b. If f (x) = − , then f ′( x) = ⎜ − ⎟ = 0 .
2 ⎝ 2⎠

122 Derivatives
Next we consider how to find the derivative of any power function f(x) = xn.
Note that the rule applies not only to functions like f(x) = x3, but also to those such as
1
g(x) = 4 x3 and h( x) = = x −5 .
x5

THE DERIVATIVE OF A POWER FUNCTION (POWER RULE)

If n is any real number, then (xn)′ = nxn – 1.

Example 15 a. If f (x) = x, then f ′( x) = x ′ = 1 ⋅ x1−1 = 1 .

b. If f (x) = x2 , then f ′( x) = ( x2 ) ′ = 2 ⋅ x 2 −1 = 2 x.

c. If f (x) = x3 , then f ′( x) = ( x 3 ) ′ = 3 ⋅ x 3 −1 = 3 x 2 .

Note
To differentiate a function containing a radical expression, we first convert the radical
expression into exponential form, and then differentiate the exponential form using the Power
Rule.

Example 16 a. If f (x) = 2 x3 , then f ( x) = x 3 / 2 in exponential form

3 3 / 2 −1 3 1 / 2
f ′(x) = (x3 / 2 )′ = x = x .
2 2
1
b. If f (x) = , then f ( x) = x −1 in exponential form
x
1
f ′(x) = (x−1 )′ = −1 ⋅ x −1−1 = −x −2 = − .
x2

Differentiation 23
The proof of the Power Rule for the general case (n ∈ ) is not easy to prove and will no be
given here. However, we can prove the Power Rule for the case where n is a positive integer.

Proof (Power Rule) If f(x) = xn, then f (x + h) − f (x) (x + h)n − x n


f ′(x) = lim = lim .
h→ 0 h h→ 0 h
Here we need to expand (x + h)n and we use the Binomial Theorem to do so:
⎡ n n −1 n ⋅ (n − 1) n − 2 2 ⎤
⎢⎣ x + nx h + x h + ⋅ ⋅ ⋅ + nxh n −1 + h n ⎥ − x n
2 ⎦
f ′(x) = lim
h→ 0 h
n ⋅ (n − 1) n − 2 2
n ⋅ xn −1h + x h + ⋅ ⋅ ⋅+ nxh n −1 + h n (every term includes h as a factor,
f ′(x) = lim 2
so h’s can be simplified)
h→ 0 h
⎡ n ⋅ (n − 1) n − 2 ⎤
f ′(x) = lim ⎢ nxn −1 + x h + ⋅ ⋅ ⋅+ nxh n − 2 + h n –1 ⎥ = n ⋅ x n −1 (if h = 0, then every term including
h→ 0
⎣ 2 ⎦ h as a factor will be zero)

Check Yourself 5
Differentiate each function by using either the Constant Rule or the Power Rule.

1. f(x) = 2 2. f(x) = 0.5 3. f (x) = – 1 4. f (x) = 3


3 2
1
7. f (x) = 1
3
5. f(x) = x3 6. f (x) = x7 8. f (x) =
x2 x3
Answers
73 4 2 3
1. 0 2. 0 3. 0 4. 0 5. 3x2 6. x 7. – 8. −
3 x3 2 x5

The next rule states that the derivative of a constant multiplied by a differentiable function
is equal to the constant times the derivative of the function.

THE CONSTANT MULTIPLE RULE

[c ⋅ f (x)]′= c ⋅ f ′(x) , c∈

Example 17 a. If f (x) = 3x, then f ′( x) = (3 x) ′ = 3 ⋅ ( x) ′ = 3 ⋅ 1 = 3 .

b. If f (x) = 3x4 , then f ′( x) = (3 x 4 ) ′ = 3( x 4 ) ′ = 3 ⋅ (4 x 3) =12 x 3 .

124 Derivatives
Proof (Constant Multiple Rule) If g(x) = c ⋅ f(x), then
g(x + h) − g(x) c ⋅ f (x + h) − c ⋅ f (x)
g′(x) = lim = lim
h→ 0 h h→ 0 h
f (x + h) − f (x)
g′(x) = c ⋅ lim
h→ 0 h
g′(x) = c ⋅ f ′(x).

Example 18 a. If f (x) = −
2
x3
6
, then f ′( x) = (–2 x −3 ) ′ = −2( x −3) ′ = –2(–3 x −4) = 6 x −4 = 4 .
x

b. ⎛1 ⎞ 5 5
If f (x) = 5 x, then f ′( x) = (5 x1 / 2 ) ′ = 5( x1 / 2 ) ′ = 5 ⎜ x −1 / 2 ⎟ = x −1 / 2 = .
⎝2 ⎠ 2 2 x

Next we consider the derivative of the sum or the difference of two differentiable functions.
The derivative of the sum or the difference of two functions is equal to the sum or the
difference of their derivatives. Note that the difference is also the sum since it deals with
addition of a negative expression.

THE SUM RULE

[ f (x) ∓ g(x)]′ = f ′(x) ∓ g′(x)

Note
We can generalize this rule for the sum of any finite number of differentiable functions.

[ f (x) ∓ g(x) ∓ h(x) ∓ ...]′ = f ′(x) ∓ g ′(x) ∓ h ′(x) ∓ ...

Differentiation 125
Now, let’s verify the rule for a sum of two functions.

Proof (Sum Rule) If S(x) = f(x) + g(x), then

S′(x) = lim
S(x + h) − S(x)
= lim
[ f (x + h)+ g(x + h)] − [ f (x)+ g(x) ]
h→ 0 h h → 0 h

S′(x) = lim
[ f (x + h) − f (x)]+ [g(x + h) − g(x) ]
h→ 0 h
f (x + h) − f (x) g(x + h) − g(x)
S′(x) = lim + lim
h→ 0 h h→ 0 h
S′(x) = f ′(x)+ g ′(x).

Example 19 a. If f (x) = x−2 +7, then f ′( x) = ( x−2 +7) ′= ( x−2) ′+(7) ′= −2 x−3+0 = −2 x−3.

2
b. If g(t) = t + 5 , then g′( t) = ⎛ t
2
−2 ⎞
′ ⎛ t2 ⎞′ −2 1 2 −2
⎜ +5t ⎟=⎜ ⎟ +(5 t ) ′= ( t ) ′+5( t ) .′
5 t2 ⎝5 ⎠ ⎝5 ⎠ 5

1
g(t )′ = (2t 2–1 )+5(–2 t–2–1 )
5
2 2t 10
g(t )′ = t − 10t −3 = − 3 .
5 5 t
Notice that in this example, the independent variable is t instead of x. So, we differentiate
the function g(t) with respect to t.

By combining the Power Rule, the Constant Multiple Rule and the Sum Rule we can
differentiate any polynomial. Let us look at some examples.

Example 20 Differentiate the polynomial function f(x) = 3x5 + 4x4 – 7x2 + 3x + 6.

Solution f ′(x) = (3 x5 + 4 x4 − 7 x2 + 3 x +6)′

f ′(x) = (3 x5 )′+(4 x4 ) ′+(–7 x2 ) ′+(3 x) ′+(6) ′

f ′(x) = 3(x5 )′+ 4( x4 ) ′ − 7( x2 ) ′+ 3( x) ′+(6) ′

f ′(x) = 3 ⋅ 5 x4 + 4 ⋅ 4 x3 − 7 ⋅ 2 x+ 3 ⋅1+0

f ′(x) =15 x4 +16 x3 − 14 x+ 3

126 Derivatives
Example 21 It is estimated that x months from now, the population of a certain community will be
P(x) = x2 + 20x + 8000.
a. At what rate will the population be changing with respect to time fifteen months from
now?
b. How much will the population actually change during the sixteenth month?

Solution a. The rate of change of the population with respect to time is the derivative of the
population function, i.e.
rate of change = P′(x) = 2x + 20.
Fifteen months from now the rate of change of the population will be:
P′(15) = 2 ⋅ 15 + 20 = 50 people per month.
b. The actual change in the population during
the sixteenth month is the difference between
the population at the end of sixteen months
and the population at the end of fifteen
months. Therefore,
the change in population = P(16) – P(15)
= 8576 – 8525
= 51 people.

Check Yourself 6
1. Find the derivative of each function with respect to the variable.
3 4
a. f (x) = b. f (r ) = π r 3 c. f(x) = 0.2ñx
2x 3

d. f(x) = 3x2 + 5x – 1 e. 4 t2 3 2
f. f (x) = x − 4x + 3
f (t ) = − +t
t3 3 x

x3 − 3x2 + 3x − 1
2. Find the derivative of f (x) = .
x −1
x2 x − x
3. Differentiate f (x) = .
x x+ x
Answers
3 0.1 12 2t 3
1. a. − 2 b. 4πr2 c. d. 6x + 5 e. – – +1 f. 2 x − 4 −
2x x t4 3 x2
2. 2x – 2 3. 1

Differentiation 127
B. THE PRODUCT AND THE QUOTIENT RULES
Now we learn how to differentiate a function formed by multiplication or division of
functions. Based on your experience with the Constant Multiple and Sum Rules we learned
in the preceding part, you may think that the derivative of the product of functions is the
product of separate derivatives, but this guess is wrong. The correct formula was discovered
by Leibniz and is called the Product Rule.
The Product Rule states that the derivative of the product of two functions is the derivative
of the first function times the second function plus the first function times the derivative of
the second function.

THE PRODUCT RULE

[ f (x)g(x)]′ = f ′(x)g(x) + f (x)g ′(x)


Be careful! The derivative of the product of two functions is not equal to the product of the
derivatives:
We can easily see this by looking at a particular example.
Let f(x) = x and g(x) = x2. Then

f (x)g(x) = x ⋅ x2 = x3 f ′(x) =1 and g′( x) = 2 x


[ f (x)g(x)]′ ≠ f ′(x)g′(x)
[ f (x)g(x)] ′ = 3 x2 f ′(x)g′(x) =1 ⋅ 2 x = 2 x

[ f (x)g(x)]′ ≠ f ′(x)g′(x).

Example 22 Find the derivative of the function f(x) = x(x + 1).

Solution By the Product Rule,


f ′(x) = x ⋅ (x+ 1)′+ (x) ′ ⋅ (x+1) = x ⋅ 1+1 ⋅ ( x+1) = 2 x+1 .
We can check this result by using direct computation:
f(x) = x(x + 1) = x2 + x so, f ′(x) = 2x + 1, which is the same result.
Note that preferring direct differentiation when it is easy to expand the brackets is always
simpler than applying the Product Rule.

Example 23 Differentiate the function f(x) = (2x2 + 1)(x2 – x).

Solution f ′(x) = (2 x2 +1)′ ⋅ (x2 − x)+( 2x 2 + 1) ⋅ (x2 − x)′


f ′(x) = (4 x)(x2 − 1) + (2 x2 + 1)(2 x − 1)
f ′(x) = 4x3 − 4x+ 4x3 − 2x2 + 2x − 1
f ′(x) = 8 x3 − 2 x2 − 2 x − 1

128 Derivatives
Example 24 Differentiate the function f(x) = (x3 + x –2)(2ñx + 1).

Solution First, we convert the radical part into exponential form:


f (x) = ( x3 + x – 2)(2 x +1) = ( x3 + x – 2) ⋅(2 x1/ 2 +1).
Now, by the Product Rule,
f ′(x) = (x3 + x – 2) ′ ⋅(2 x1/ 2 +1)+( x3 + x – 2) ⋅(2 x1/ 2 +1) ′
f ′(x) = (3 x2 +1)(2 x1/ 2 +1)+( x3 + x – 2) ⋅ x −1/ 2 = 6 x5 / 2 + 3 x2 + 2 x1/ 2 +1+ x5 / 2 + x1/ 2 – 2 x −1/ 2
f ′(x) = 7 x5 / 2 + 3x2 + 3x1 / 2 – 2 x–1/ 2 +1.

Let us look at the proof of the Product Rule.


Proof (Product Rule) If P(x) = f(x)g(x), then
P(x + h) − P(x) f (x + h)g(x + h) − f (x)g(x)
P′(x) = lim = lim
h→ 0 h h → 0 h

By adding –f(x + h)g(x) + f(x + h)g(x) (which is zero) to the numerator and factoring, we have:
f (x + h)g(x + h) − f (x + h)g(x ) + f (x + h)g(x ) − f (x)g(x)
P′(x) = lim
h→ 0 h
f (x + h) [g(x + h) − g(x) ]+ g(x) [f (x + h) − f (x) ]
P′(x) = lim
h→ 0 h
⎛ ⎡ g(x + h) − g(x) ⎤ ⎡ f (x + h) − f (x) ⎤ ⎞
P′(x) = lim ⎜ f (x + h) ⎢ ⎥ + g(x) ⎢ ⎥⎟
h→ 0
⎝ ⎣ h ⎦ ⎣ h ⎦⎠
g(x + h) − g(x) f (x + h) − f (x )
P′(x) = lim f (x + h) ⋅ l im + lim g(x) ⋅ lim
h→ 0 h→ 0 h h→ 0 h→ 0 h
P′(x) = f (x) ⋅ g ′(x)+ g(x) ⋅ f ′(x) = f ′(x)g(x)+ f (x)g ′(x).

Example 25 Differentiate the function f(x) = (x2 + 1)(3x4 – 5x)(x3 + 2x2 + 4).

Solution In this example we have a product of three functions, but we are only able to apply the rule
for the product of two functions. So, before we proceed we must imagine the function as
a product of two functions as follows:
f ( x) = ( x2 +1)(3 x4 − 5 x) ( x3 + 2 x2 + 4)

f ′( x) = [( x2 +1)(3 x4 − 5 x)]′ ( x3 + 2 x2 + 4) +( x2 +1)(3x 4 − 5 x)( x3 + 2 x2 + 4)′


requires product rule once more

f ′( x) = [2 x(3x4 − 5x)+( x2 +1)(12 x3 − 5)] ( x3 + 2 x2 + 4) +( x2 +1)(3 x4 − 5 x)(3x2 + 4x ).


Our aim is to introduce this method and because any further simplification is time
consuming, we will stop at this point.

Differentiation 129
The derivative of a quotient is the denominator times the derivative of the numerator minus
the numerator times the derivative of the denominator, all divided by the square of the
denominator. Or,

⎛ numerator ⎞′ derivative of the numerator ×denominator – numerator × derivative of the denominator


⎜ ⎟ = .
⎝ denominator ⎠ the square of the denominator

THE QUOTIENT RULE

⎛ f (x) ⎞′ f ′(x)g(x) − f (x)g ′(x)


⎜ g(x) ⎟ = , g( x) ≠ 0
⎝ ⎠ ( g(x))2

The quotient rule is probably the most complicated formula you will have to learn in this text.
It may help if you remember that the quotient rule resembles the Product Rule.
⎛ f (x) ⎞′ f ′(x) Also note that like in the Product Rule, the derivative of a quotient is not equal to the
⎜ g(x) ⎟ ≠ g′(x)
⎝ ⎠ quotient of derivatives.

Example 26 Find the derivative of the function f (x) =


3x+ 1
2x − 1
.

Solution Using the Quotient Rule:


(3x+ 1)′ (2 x − 1) − (3 x+1) (2 x − 1)′
f ′(x) =
(2 x − 1)2
3 ⋅ (2 x − 1) − (3 x+1) ⋅ 2 6 x − 3 − 6 x − 2
f ′(x) = =
(2 x − 1)2 (2 x − 1)2
5
f ′(x) = − .
(2 x − 1)2

27
2
Example Differentiate the rational function f (x) = x + x − 21 .
x −1

Solution According to the Quotient Rule,


(2 x+ 1) ⋅ (x − 1) − (x2 + x − 21) ⋅ 1
f ′(x) =
(x − 1)2

2 x2 − x − 1 − x2 − x+ 21 x2 − 2 x+ 20
f ′(x) = =
(x − 1)2 (x − 1)2

x2 − 2 x+ 20
f ′(x) = .
x2 − 2 x+ 1

130 Derivatives
Example 28 Differentiate the function f (x) =
2 x2 + 3x+ 1
2x
.

Solution Before trying to use the Quotient Rule let us simplify the formula of the function:
2 x2 + 3x+ 1 2 x2 3x 1 3 1
f (x) = = + + = x+ + x −1.
2x 2x 2x 2x 2 2
In this example, finding the derivative will be easier and quicker without using the Quotient Rule.
1 −2 1 x2 − 1
f ′(x) = 1+ 0 − x =1− 2 =
2 x x2

Note
We do not need to use the Quotient Rule every time we differentiate a quotient. Sometimes
performing division gives us an expression which is easier to differentiate than the
quotient.

Let us verify the Quotient Rule.

f (x)
Proof (Quotient Rule) Let Q(x) = and Q(x) be differentiable.
g(x)
We can write f(x) = Q(x)g(x).
If we apply the Product Rule: f ′(x) = Q′(x)g(x) + Q(x)g′(x)
Solving this equation for Q′(x), we get
f (x)
f ′(x) − ⋅ g ′(x)
f ′(x) − Q(x)g ′(x) g(x)
Q′(x) = =
g(x) g(x)

f ′(x)g(x) − f (x)g ′(x)


Q′(x) = .
( g(x))2

Example 29 f(x) = ñx ⋅ g(x), where g(4) = 2 and g′(4) = 3. Find f ′(4).

Solution f ′(x) = ( x ⋅ g( x)) ′ = ( x ) ′ ⋅ g( x)+ x ⋅ g ′( x)

g(x)
f ′(x) = + x ⋅ g ′(x)
2 x
g(4) 2 13
So f ′(4) = + 4 ⋅ g ′(4) = +2 ⋅3 = .
2 4 2⋅2 2

Differentiation 131
Check Yourself 7
1. Find the derivative of each function using the Product or the Quotient Rule.

a. f(x) = 2x(x2 + x + 1) b. f(x) = (x3 – 1)(x2 – 2)

⎛ 1 ⎞⎛ 1 ⎞ ⎛ 1 ⎞
c. f (x) = ⎜ 2 + x ⎟⎜ + 1⎟ d. f (x) = ( x + 1) ⎜ x 2 + ⎟
⎝x ⎠⎝ x ⎠ ⎝ x⎠
2 x+ 4 x −1
e. f (x) = f. f (x) =
3x − 1 x +1
x2 − x+ 10 x3 + 3x2 − 5x+ 6
g. f (x) = h. f (x) =
x+ 1 2x

2. If f(x) is a differentiable function, find an expression for the derivative of each function.
f (x) 2
1+ xf (x)
a. y = x2f(x) b. y = c. y = x d. y =
x2 f (x) x

3. Suppose that f and g are two functions such that f(5) = 1, f ′(5) = 6, g(5) = –3 and

g′(5) = 2. Find each value.

⎛ f ⎞′ ⎛ g ⎞′
a. (fg)′(5) b. ⎜ ⎟ (5) c. ⎜ ⎟ (5)
⎝ g⎠ ⎝f⎠
Answers
2 3 5 1
1. a. 6x2 + 4x + 2 b. 5x4 – 6x2 – 2x c. 1 – – 4 d. x x + 2x −
x 3
x 2 2x x
14 1 2
3 3
e. – f. g. x + 2 x −2 11 h. x − +
(3x – 1)2 x( x +1) 2
( x +1) x2 2

2 2 2
′ ′ ′
2. a. 2xf(x) + x2f ′(x) b. f ( x) ⋅ x 4– 2 xf ( x) c. 2 xf ( x) – f (2x) ⋅ x d. x f ( x2 ) – 1
x ( f ( x)) x
20
3. a. –16 b. – c. 20
9

132 Derivatives
C. THE CHAIN RULE
We have learned how to find the derivatives of expressions that involve the sum, difference,
product or quotient of different powers of x. Now consider the function given below.
h(x) = (x2 + x – 1)50
In order to differentiate h(x) using the rules we know, we need to expand h(x), then find the
derivative of each separate term. This method is, however, tedious!

Consider also the function m( x) = x2 + x − 1. This function is also difficult to differentiate


using the rules we have learned. For each of the two functions h(x) and m(x), the
differentiation formula we learned in the previous sections cannot be applied easily to
calculate the derivatives h′(x) and m′(x).
We know that both h and m are composite functions because both are built up from simpler
functions. For example, the function h(x) = (x2 + x – 1)50 is built up from the two simpler
functions f(x) = x50 and g(x) = x2 + x – 1 like this:
h(x) = f(g(x)) = [g(x)]50 = (x2 + x – 1)50
Here we know how to differentiate both f and g, so it would be useful to have a rule that tells
us how to find the derivative of h = f(g(x)) in terms of the derivatives of f and g.

THE CHAIN RULE

[ f ( g( x))]′ = f ′( g( x)) ⋅ g′( x )

outer argument derivative of derivative of


function of the the outer the argument
outer function of the outer
function function

For example, if h(x) = f(g(x)) = (x2 + x – 1)50, then


h′(x) = f ′(g(x)) ⋅ g′(x) = 50(x2 + x – 1)49 ⋅ (2x + 1).

1. [(x2 + x – 1)50]′ ≠ 50(x2 + x – 1)49


2. [(x2 + x – 1)50]′ ≠ 50(2x + 1)49
Example 30 Differentiate the function f(x) = (3x2 + 5x)2005.

Solution 2
By the Chain Rule, f ′( x) = 2005(3 x +5 x)
2004
⋅ (3 x2 +5 x) ′= 2005(3 x2 +5 x) 2004 ⋅(6 x+5).

Example 31 Suppose m(x) = f(g(x)) and g(1) = 5, g ′(1) = 2, f(5) = 3 and f ′(5) = 4 are given.
Find m ′(1).

Solution By the Chain Rule, m′(x) = f′(g(x)) ⋅ g ′(x). So m ′(1) = f ′(g(1)) ⋅ g′(1) = f ′(5) ⋅ 2 = 4 ⋅ 2 = 8.

Note
The Chain Rule can be generalized for the composition of more than two functions as follows:
[ f1( f2( f3(...fn(x)...)))]′ = [ f1′( f2( f3(...fn(x)...)))] ⋅ ( f2′( f3(...fn(x)...))) ⋅ ( f3′(...fn(x)...)) ⋅ ... ⋅ fn′(x)

Using the Chain Rule we can generalize the Power Rule as follows:

GENERAL POWER RULE

[(f(x))n]′ = n(f(x))n – 1 ⋅ f ′(x)

By using this rule we can more easily differentiate the functions that can be written as the
power of any other functions.

Example 32 Differentiate the function m( x) = x2 + x − 1.

1
Solution We can rewrite the function as m(x) = (x2 + x – 1)2 and apply the General Power Rule:
1
1 –
m′( x) = ( x2 + x − 1) 2 ⋅ ( x2 + x − 1) ′
2
1
1 –
m′( x) = ( x2 + x − 1) 2 ⋅ (2 x +1)
2
2 x +1
m′( x) =
2 x2 + x − 1

1
Example 33 Differentiate the function f ( x) = 2
x + 3x
.

Solution f ′( x) =[( x2 + 3x)−1]′ = −1( x2 + 3 x) −2 ⋅( x2 + 3 x) ′

f ′( x) = −1( x2 + 3 x) −2 ⋅ (2 x + 3)

2x + 3
f ′( x) = −
( x2 + 3x)2

134 Derivatives
34
1

Example Differentiate the function f(x) = (2x3 + x2 – 15) 3 .

4
1 –
Solution f ′( x) = − (2 x3 + x2 − 15) 3 ⋅(2 x3 + x2 −15) ′
3
1 1
f ′( x) = − ⋅ ⋅ (6 x2 + 2 x)
3 3 (2 x + x2 − 15)4
3

6 x2 + 2 x
f ′( x) = −
3 3 (2 x3 + x2 − 15)4

35

−3
Example Differentiate the function f ( x) = (( x +1) 3 +5 x) .

2 2
− −
Solution f ′( x) = −3(( x +1) 3 +5 x) −4 ⋅ (( x+1) 3 +5 x) ′
2 5
− 2 −
f ′( x) = −3(( x +1) 3 +5 x) −4 ⋅ ( − ( x+1) 3 ⋅ ( x+1) ′+5)
3
2 5
− 2 −
f ′( x) = −3(( x +1) 3 +5 x) −4 ⋅ (5 − ( x+1) 3 )
3

Example 36 Differentiate the function f ( x) = (2 x − 3)5 ⋅ x2 − 2 x.

Solution The function is the product of two expressions, so we can use the Product Rule:

f ′( x) = ((2 x – 3) 5 )′ ⋅ x2 − 2 x +(2 x − 3) 5 ⋅ ( x2 − 2 x) ′
1
1 –
f ′( x) = 5 ⋅ (2 x − 3)4 ⋅ 2 ⋅ x2 − 2 x +(2 x − 3) 5 ⋅ ⋅( x2 − 2 x) 2 ⋅(2 x – 2)
2
(2 x − 3)5 ⋅ (2 x − 2)
f ′( x) =10(2 x − 3) 4 x2 − 2 x +
2 x2 − 2 x
7

Example 37 ⎛ 2t +1 ⎞
Differentiate the function g(t ) = ⎜
⎝ t−3 ⎠
⎟ .

Solution
⎛ 2t +1 ⎞ ⎛ 2t +1 ⎞′
6

g′(t ) = 7 ⎜ ⎟ ⋅⎜ ⎟ (by the Power Rule)


⎝ t−3 ⎠ ⎝ t −3 ⎠
6
⎛ 2t +1 ⎞ 2 ⋅ ( t − 3) − 1 ⋅ (2 t +1)
g′(t ) = 7 ⎜ ⎟ ⋅ (by the Quotient Rule)
⎝ t−3 ⎠ (t − 3)2
6
⎛ 2t +1 ⎞ 2t − 6 − 2t − 1
g′(t ) = 7 ⎜ ⎟ ⋅ (simplify)
⎝ t−3 ⎠ (t − 3)2

−49(2t +1)6
g′(t ) = .
(t − 3)8

Notation
dy
Remember that if y = f(x), then we can denote its derivative by y′ or .
dx
If y = f(g(x)) such that y = f(u) and u = g(x), then we can denote the derivative of f(g(x))
dy dy du
by y′ = f ′(g(x)) ⋅ g′(x) or y′ = f ′(u) ⋅ u′(x) or = ⋅ .
dx du dx
dy dy du
The notation = ⋅ is called Leibniz notation for the Chain Rule.
dx du dx

Example 38 Given that y = u2 – 1 and u = 3x2 + 1, find


dy
dx
by using the Chain Rule.

Solution By the Chain Rule,

dy dy du
= ⋅
dx du dx
dy d d
= ( u2 – 1) ⋅ (3 x+1) (find the derivative of the first function with respect to u
dx du dx and the second function with respect to x)
dy
= (2 u – 1) ⋅ 3
dx
dy
= (2 ⋅ (3 x+1) – 1) ⋅ 3
dx
dy
=18 x + 3.
dx

136 Derivatives
Check Yourself 8
1. Find the derivative of f(x) = (2x + 1)3.
2. Differentiate y = (x3 – 1)100.
1
3. Find f ′(x) given f ( x) = 3
.
2
x + x +1
x3 − 1
4. Find the derivative of g( x) = 4 .
x3 +1
1 dy
5. y = and u = 3x – 1 are given. Find .
u dx
Answers
2 x +1
1. 6(2x + 1)2 2. 300x2(x3 – 1)99 3.
3 ( x2 + x +1)4
3

1 x3 +1 43 6 x2 3
4. ( ) 5. –
4 x3 – 1 ( x3 +1)2 (3x – 1)2

Differentiation 137
D. HIGHER ORDER DERIVATIVES
If f is a differentiable function, then its derivative f ′ is also a function, so f ′ may have a
derivative of its own, denoted by (f ′)′ = f ′′. This new function f ′′ is called the second
derivative of f because it is the derivative of the derivative of f. Look at three different ways
of writing the second derivative of a function:
d2 y
f ′′( x) = y′′ = 2
dx

2x
Example 39 Find the second derivative of the function f ( x) =
x –1
.

Solution By the Quotient Rule,


2 ⋅ ( x – 1) – 2 x ⋅ 1 2 x – 2 – 2 x 2
f ′( x) = 2
= 2
=– .
( x – 1) ( x – 1) ( x – 1) 2

Now differentiate f ′(x) to get f ′′(x):

⎛ 2 ⎞′ 4
f ′′( x) = ⎜ – 2
–2 –3
⎟ = (–2( x – 1) )′ = 4( x – 1) ⋅1=
⎝ ( x – 1) ⎠ ( x – 1)3

Note
Before computing the second derivative of a function, always try to simplify the first derivative
as much as possible. Otherwise the computation of the second derivative will be more tedious.

Notation
If we differentiate the second derivative f ′′(x) of a function f(x) one more time, we get the
third derivative f ′′′(x). Differentiate again and we get the fourth derivative, which we write
as f (4)(x) since the prime notation f ′′′′(x) begins to get difficult to read. In general, the
derivative obtained from f(x) after n successive differentiations is called the nth derivative or
dn y
the derivative of order n and written by f (n)(x) or .
dxn

Example 40 Find the derivatives of all orders of the polynomial function


f(x) = x5 + 4x4 + 2x3 – 5x2 – 6x + 7.

Solution f ′(x) = 5x4 + 16x3 + 6x2 – 10x – 6


f ′′(x) = 20x3 + 48x2 + 12x – 10
f ′′′(x) = 60x2 + 96x + 12
f (4)(x) = 120x + 96
f (5)(x) = 120
f (n)(x) = 0 (for n > 5)

138 Derivatives
Example 41 1
Find a general expression for the nth derivative of the function f ( x) = .
x
dy 1
Solution = ( x–1 )′ = – x–2 = – 2
dx x
d2 y 2
2
= (– x–2 )′ = 2 x–3 = 3
dx x
d3 y 6
3
= (2 x–3 )′ = –6 x–4 = – 4
dx x
d4 y 24
= (–6 x–4 )′ = 24 x–5 = 5
dx4 x
d5 y 120
5
= (24 x–5 )′ = –120 x–6 = – 6
dx x
n!=n⋅(n–1)⋅(n–2)⋅...⋅3⋅2⋅1
dn y n –( n+1) (–1)n n!
for any natural number n = (...)′ = (–1) ⋅ n! x =
dxn xn+1

Check Yourself 9
1. Find the second derivative of each function.
x –1
a. f(x) = x3 – 3x2 + 4x + 5 b. f ( x) =
x+ 2
2. Find the third derivative of each function.
2/3 1
a. f(x) = x b. f (t ) = ( t 2 – 1)5
2
Answers
6 7 2 2 2
2. a. 8 x 3 b. 15t( t − 2) (3t − 2)

1. a. 6x – 6 b. –
( x + 2)3 27 2

Differentiation 139
Motion is one of the key subjects in physics. We define
many concepts and quantities to explain the motion in
one dimension. We use some formulas to state the
relations between the quantities. Derivative plays an
important role in defining the quantities and producing
the formulas from other derivatives. Here we will give the
definition of important concepts and formulas that
includes the uses of derivative.
DISPLACEMENT
The displacement is the change in the position of an object. If we denote the position at time t1
by x1, and the position at time t2 by x2, then the displacement is the difference between these two
points; this is defined by
∆x = x2 – x1.
The time interval is, similarly,
∆t = t2 – t1.
We use the capital Greek letter ∆(delta) to show a change in a variable from one value to
another.
VELOCITY
The velocity describes how fast the position of an object
changes. It is measured over a certain time interval. If a
car has a displacement ∆x in a particular time interval ∆t,
then the car's average velocity, Vav, over that time interval
is defined by
displacement x2 − x1 ∆x
Vav = = = .
time interval t2 − t1 ∆t
The definition of the average velocity includes a time interval. We learn more about the motion
when smaller time intervals are used. Because of this, we define the instantaneous velocity as
follows.
The instantaneous velocity at a time t is the velocity of an
object at that given instant of time. In other words, it is the
limit of the average velocity as ∆t approaches zero:
x(t + ∆t ) − x(t )
V(t )= lim
∆t → 0 ∆t
From the definition given above, we may conclude that the instantaneous velocity is the
derivative of the displacement with respect to time t.
∆x dx
V (t )= lim =
∆t → 0 ∆t dt
ACCELERATION
The term acceleration refers to the rate of change in velocity of an object with respect to time.
We define the average acceleration, aav, in terms of velocity v1 at time t1 and v2 at time t2:
v2 − v1 ∆v
aav = =
t2 − t1 ∆t
Now we will define the instantaneous acceleration as follows.
∆v
The instantaneous acceleration is the limit of the expression as the time interval goes to
zero. ∆t

∆v dv
a(t ) lim ==
∆t dt
∆t → 0

This means that the instantaneous acceleration is the derivative of velocity with respect to time.
Also it is the second derivative of the displacement.

For example, the position function x(t) of a car moving along a straight line is given as
x(t) = 4t2 + 6t – 20 m where t is in seconds.
The derivative of the position function gives the velocity function.
dx
V (t )= =(4t 2 +6t − 20)′ = 8t +6 m/s
dt
The acceleration is
dv
a(t )= =(8t +6)′ = 8 m/s 2.
dt
So, the car moves with constant acceleration.
EXERCISES 1 .2
A. Basic Differentiation Rules B. The Product and The Quotient Rules

1. Find the derivative of each function by using the 2. Find the derivative of each function by using the
rules of differentiation. Product or the Quotient Rule.

a. f(x) = 5x(x2 – 1)
a. f(x) = ñ2
b. f(x) = (2x + 3)(3x – 4)
1
b. f (x ) = − c. f(x) = 10(3x + 1)(1 – 5x)
151
d. f(x) = (x3 – 1)(x + 1)
c. f(x) = eπ
e. f(x) = (x3 – x2 + x – 1)(x2 + 2)
1 8 f. f(x) = (1 + ñt )(2t2 – 3)
d. f ( x) = x
12 3
g. f ( x) =
2x + 4
e. f(x) = 2x0.8
x −1
h. f ( x) =
5 4/5 2 x +1
f. f ( x) = x
4
1 − 2x
i. f ( x) =
2 1+ 3x
g. f ( x) =
6
411
j. x
f ( x) = 2
h. f(x) = 0.3x 0.7
x +1

x2 + 2
–12
k. f ( x) =
i. f(x) = 7x 2
x + x +1

l. x + 3x
j. f(x) = 5x2 – 3x + 7 f ( x) =
3x − 1

x3 + 2 x2 + x − 1 3. Given that f(1) = 2, f ′(1) = –1, g(1) = –2 and


k. f ( x) =
x g′(1) = 3, find the value of h′(1).
4 3 2
l. f ( x) = − +
t4 t3 t a. h(x) = f(x) ⋅ g(x)

m. f ( x) = x + 3 x + 5 x b. h(x) = (x2 + 1) ⋅ g(x)

2 1 xf ( x)
n. f ( x) = x + + c. h( x) =
x x x + g( x)

1 3 f ( x) ⋅ g( x)
o. f ( x) =1 − + d. h( x) =
x x f ( x) − g( x )

142 Derivatives
x − 3x x 7. h(x) = g(f(x)) and f(2) = 3, f ′(2) = –3, g(3) = 5
4. Differentiate the function f ( x) = by and g′(3) = 4 are given. Find h′(2).
x
simplifying and by the Quotient Rule. Show that

both of your answers are equivalent. Which

method do you prefer? Why? dy


8. By using the Chain Rule, find for each function.
dx

5. f(3) = 4, g(3) = 2, f ′(3) = –6 and g′(3) = 5 are a. y = u2 – 1, u = 2x + 1


given. Find the value of the following expressions.
b. y = u2 + 2u + 2, u = x – 1
a. (f + g)′(3) b. (fg)′(3)
1
c. y = , u = x3
⎛ f ⎞′ ⎛ f ⎞ u –1
c. ⎜ ⎟ (3) d. ⎜ ⎟ (3)
⎝ g⎠ ⎝ f − g⎠
1
d. y = u + , u = x2 – x
u
C. The Chain Rule
6. Find the derivative of each function.
D. Higher Order Derivatives
a. f(x) = (3x – 1)2

b. f(x) = (x2 + 2)5 9. Find the second derivative of each function.

c. f(x) = (x5 – 3x2 + 6)7 a. f(x) = 3x2 – 7x + 2

d. f(x) = (x – 2)–3 b. f(x) = (x2 + 1)7


e. f ( x) = 2
x2
(5x2 + 3x – 1)2 c. f ( x) =
x –1
1
f. f ( x) =
4x2 +1 d. f ( x) = 2 x – 1

g. f ( x) = ( x +1+ x ) 3

› h. f(x) = (x – 1)5 ⋅ (3x + 1)1/3


10. Find the third derivative of each function.
(1 – 3 x)7
› i. f ( x) =
(2 x +1)4 a. f(x) = 5x4 – 3x3
3x – 9 3 2
› j. f ( x) = ( ) b. f(x) =
2x + 4 x

2x – 1 c. f ( x) = 3x – 2
› k. f ( x) =
3x +1
› l. f(x) = 3x + [2x2 + (x3 + 1)2]3/4 d. f(x) = (2x – 3)4

Differentiation 143
Mixed Problems 16. The concentration of a certain drug in a patient’s
bloodstream t hours after injection is given by
11. Find the equation of the tangent line to the graph
0.2t
of the function f(x) = (x3 + 1)(3x2 – 4x + 2) at C(t ) = .
t2 +1
the point (1, 2). a. Find the rate at which the concentration of
the drug is changing with respect to time.
x
12. The curve y = 2 is called a serpentine b. How fast is the concentration changing
x +1
curve. Find the equation of the tangent line to the 1 hour after the injection? What about after 2
hours?
curve at the point x = 3.

13. f is a differentiable function. Find an expression


for the derivative of each of the following functions.

a. y = x2ñxf(x)

b. y = x3(f(x))2
x3
c. y =
f ( x)
x + xf ( x ) 17. g(x) = f(x2 + 1) is given. Find g ′(1) if f ′(2) = 3.
d. y = ›
x

14. Prove that (fgh)′ = f ′gh + fg′h + fgh′ if f, g and h 18. Find an expression for the derivative of
› ›
are differentiable functions. ⎛ g( x)h( x) ⎞
f⎜ ⎟ if f, g, h, m and n are differentiable
⎝ m( n( x)) ⎠
15. A scientist adds a toxin to a colony of bacteria. He functions.

estimates that the population of the colony after t 19. If the tangent to the graph of f at point (2, 3) has an
24t +10 ›
hours will be P(t ) = 2 thousand bacteria. angle of 60° with x-axis, find the slope of tangent to
t +1 the graph of g(x) = f 2(x) – x ⋅ f(x) at x = 2.
Find the estimated rate of change of the

population after three hours.


20. Given that f ( x) = x x x and f ′′( a) = − 7 ,
› 64
find a.

21. Given that f(4 ⋅ g(x) + 7) = x3 – 2x2 + 3 and


›
g(x) = 1 – x, find f ′(–1).

22. Find an expression for the nth derivative of the


› 1
function f ( x) = .
2x

144 Derivatives
A. DERIVATIVES OF EXPONENTIAL AND LOGARITHMIC
FUNCTIONS
In this section, we will study the derivative formulas for exponential and logarithmic
function. Let us begin with the derivative formulas for exponential functions.

1. Exponential Functions
DERIVATIVE OF NATURAL EXPONENTIAL FUNCTION

(ex)′ = ex

Proof (Derivative of Natural Exponential Function)


Let f(x) = ex, then by the definition of derivatives
f ( x + h) − f ( x ) e x+ h − e x e x (e h − 1) eh − 1
f ′( x) = lim = lim = lim = e x ⋅ lim .
h→0 h h →0 h h →0 h h→0 h
We are unable to evaluate this limit using the techniques we have learned before. But the
calculations in the following table helps us to guess it correctly.
→h←
h – 0.1 – 0.01 – 0.001 0 0.001 0.01 0.1

eh − 1
0.9516 0.9950 0.9995 1 1.0005 1.0050 1.0517
h

eh − 1
The table tells us that, lim =1.
h→ 0 h
x x
So, f ′(x) = e ⋅ 1 = e .

Example 42 Find the derivative of the function f(x) = x2ex.

Solution By the Product Rule,


f ′( x) = ( x2 e x )′ = ( x2 ) ′e x + x 2( e x ) ′ = 2 xe x + x 2e x = xe x( x + 2).

Differentiation 145
43
5
Example Find the derivative of the function f(x) = (ex + 1)2 .

Solution By the General Power Rule,


3 3
5 5
f ′( x) = ( e x + 1) 2 ⋅ ( e x +1) ′ = e x( e x +1) 2.
2 2

CHAIN RULE FOR NATURAL EXPONENTIAL FUNCTION

(e f(x))′ = e f(x) ⋅ f ′(x)

Example 44 Find the derivative of the function f(x) = e3x+1.

Solution By the Chain Rule,


f ′( x) = e 3x +1 ⋅ (3x +1) ′ = 3 e3 x +1.

Example 45 Find
dy
dx
2
if f ( x) = xe x −1.

Solution By the Product Rule,


dy 2 2 2 2 2
=1 ⋅ e x −1 + x ⋅ e x −1 ⋅ ( x2 − 1)′ = e x −1 + 2 x2 e x −1 = e x −1(2 x2 +1).
dx

DERIVATIVE OF EXPONENTIAL FUNCTION

( ax )′ = ax ln a , a ∈ \+

Proof (Derivative of Exponential Function)


By using the identity a = eln a we can rewrite the expression as
x
f(x) = ax = eln a = ex ln a (by the property of logarithm)
x
f ′( x) = e x ln a ⋅ ln a = e ln a ⋅ ln a

f ′( x) = a x ln a.

Example 46 Given that f(x) = 3x, find f ′(x).

Solution f ′(x) = 3x ⋅ ln 3

146 Derivatives
Example 47 Find the derivative of the function f(x) = ex ⋅ 2x.

Solution By the Product Rule,


f ′(x) = (ex )′ ⋅ 2x + ex ⋅ (2x)′ = ex 2x + ex 2x ln 2 = (2e)x (ln 2 + 1).

CHAIN RULE FOR EXPONENTIAL FUNCTION

(a f(x))′ = a f(x) ⋅ ln a ⋅ f ′(x) , a ∈ \+

Example 48 Find the derivative of the function f(x) = 5x +1.


2

Solution By the Chain Rule,


2 2
f ′( x) = 5 x +1 ⋅ ln 5 ⋅ ( x2 +1) ′ = (ln 5) ⋅2 x ⋅5 x +1.

32 x+1
Example 49 If f ( x) =
x2 +1
, find f ′( x).

(32 x +1 )′ ⋅ ( x2 +1) – 3 2 x+1 ⋅ ( x2 +1) ′ (by the Quotient Rule)


Solution f ′( x) =
( x2 +1)2

32 x+1 ⋅ ln 3 ⋅ 2 ⋅ ( x2 +1) − 2 x ⋅ 3 2 x+1 (by the Chain Rule)


f ′( x) =
( x2 +1)2

2 ⋅ 32 x+1[( x2 +1) ⋅ ln 3 − x]
f ′( x) =
( x2 +1)2

Check Yourself 10
Find the derivative of each function.
3 2 3 – ex
1. f(x) = 2xex 2. g(x) = ex + 3x + 1 3. f(x) = ex
2
4. f(x) = e2x – 1 5. g(x) = (x + 1)ex –1
6. f(x) = ex ⋅ 3x
2 34 x+1
7. g(x) = 3x +x+1
8. h( x) =
x2 – 1
Answers
3 + 3x2 + 1 3 – ex
1. 2ex(x + 1) 2. ex ⋅ (3x2 + 6x) 3. ex ⋅ (3x2 – ex)
2–1
4. 2e2x – 1 5. ex ⋅ (2x2 + 2x + 1) 6. ex ⋅ 3x(1 + ln 3)
2+x+1 2 ⋅ 34 x+1 ⋅ (2 ln 3 ⋅ ( x2 − 1) − x)
7. 3x ⋅ ln 3 ⋅ (2x + 1) 8.
( x2 − 1)2

Differentiation 147
2. Logarithmic Functions

DERIVATIVE OF NATURAL LOGARITHMIC FUNCTION

1
(ln x)′ = , x> 0
x

Example 50 Find the derivative of the function f(x) = x ln x.

Solution By the Product Rule,


1
f ′( x) = ( x)′ ⋅ ln x + x ⋅(ln x) ′ =1 ⋅ ln x+ x ⋅ = ln x+1.
x

Example 51 Find the derivative of the function f ( x) = ln 5 x3 .

3
5
Solution We can write x3 = x 5 . Now,

3
3 3 1 3
f ′( x) = (ln x 5 ) ′ = ( ln x) ′= ⋅ = .
5 5 x 5x

Example 52 Find the derivative of the function f(x) = (ln x + x)2.

Solution Applying the General Power Rule,


f ′( x) = 2(ln x + x) ⋅ (ln x+ x) ′ (by the General Power Rule)

⎛1 ⎞ (by the Sum Rule and the derivative of ln (x))


f ′( x) = 2(ln x + x) ⎜ +1 ⎟.
⎝ x ⎠

CHAIN RULE FOR NATURAL LOGARITHMIC FUNCTION

f ′( x)
(ln f ( x))′ =
f ( x)

Example 53 Find the derivative of the function f(x) = ln(x2 + 3x + 1).

( x2 + 3x +1)′ 2 x+ 3
Solution f ′( x) = 2
= 2 .
x + 3x +1 x + 3x +1

148 Derivatives
DERIVATIVE OF LOGARITHMIC FUNCTION

1
(log a x)′ = , x > 0, a > 0, a ≠ 1
x ln a

Proof (Derivative of Logarithmic Function)


Let f(x) = loga x, then af(x) = x.
Differentiating both sides of the equation af(x) = x with respect to x, we get
af(x)(ln a) f ′(x) = 1.
Note that we are looking for f(x).
1 1
f ′( x) = =
a f ( x ) ln a x ln a

Example 54 Find the derivative of the function f(x) = log2 x.

1
Solution f ′( x) =
x ln 2

CHAIN RULE FOR LOGARITHMIC FUNCTION

f ′( x)
(log a f ( x))′ =
f ( x) ⋅ ln a

Example 55 Find the derivative of the function f(x) = log2(x3 + 2x).

( x3 + 2 x)′ 3x2 + 2
Solution f ′( x) = =
( x3 + 2 x) ⋅ ln 2 ( x3 + 2 x)ln 2

Example 56 2
Find the derivative of the function f(x) = ex +3x ⋅ log3(2x – 4).

Solution By the Product Rule,


2 2
f ′(x) = (ex +3x)′ ⋅ log3(2x – 4) + ex +3x ⋅ (log3(2x – 4))′
2 2 2
f ′(x)= ex +3x ⋅ (2x + 3) ⋅ log3(2x – 4) + ex +3x ⋅ .
(2 x – 4)ln 3

Differentiation 149
Check Yourself 11
Find the derivative of each function.
7
1. f(x) = x2 ln x 2. g( x) = ln x5 3. h(x) = (ex + ln x)2

4. f(x) = ln(x2 – 5x + 1) 5. h(x) = [ln(x2 + x + 1)]2 6. f(x) = ln(ex + 2)

7. f(x) = log2(x2 + 5x – 1) 8. g(x) = log3[ln(x2 + 1)] 9. g(x) = log2(ex + x – 1)

Answers
5 1
1. x(2ln x + 1) 2. 3. 2(ex + ln x) ( e x + )
7x x

2x – 5 2 x +1 ex
4. 5. 2ln(x2 + x + 1) ⋅ ( 2
) 6. x
2
x – 5x+ 1 x + x +1 e +2

2 x +5 2x 2( e x +1)log( e x + x – 1)
7. 8. 9.
2
( x +5 x – 1)ln 2 ( x +1)[ln ( x2 +1)]ln 3
2
( e x + x – 1)ln10

3. Logarithmic Differentiation
Sometimes the task of finding the derivative of a complicated function involving products,
quotients, or powers can be made easier by first applying the laws of logarithms to simplify
it. This technique is called logarithmic differentiation. Let us look at some examples.

Example 57 f(x) = x(3x – 1)(x2 + 3) is given. Find the first derivative of f by using logarithmic
differentiation.

Solution f(x) = x(3x – 1)(x2 + 3)


ln f(x) = ln [x(3x – 1)(x2 + 3)] (take the logarithms is of both sides)
2
ln f(x) = In(x) + ln(3x – 1) + ln(x + 3) (write logarithms as sums)
ln (ab) = ln a + ln b f ′( x) 1 3 2x (take derivatives of both sides)
= + + 2
a, b > 0 f ( x) x 3x – 1 x + 3
1 3 2x
f ′( x) = f ( x)( + + ) (isolate f ′(x))
x 3x – 1 x2 + 3
1 3 2x (substitute for f(x))
f ′( x) = x(3x – 1)( x2 + 3)( + + )
x 3x – 1 x2 + 3

150 Derivatives
3

Example 58 Find the derivative of the function f ( x) =


x5 ⋅ 3 x2 +1
(2 x +1)7
.

Solution First find the logarithms of both sides of the expression:


⎛ 53 3 2 ⎞ 3 1
ln f ( x) = ln ⎜ x ⋅ x +1 ⎟ = ln x + ln( x2 +1) − 7 ln(2 x+1).
⎜ (2 x +1)7 ⎟ 5 3
⎝ ⎠
a
ln( ) = ln a – ln b Now differentiate both sides of the equation with respect to x:
b
ln ap = p ln a f ′( x) 3 1 1 1 7 ⋅2
a, b > 0 = ⋅ + ⋅ 2 ⋅ 2x −
f ( x) 5 x 3 x +1 (2 x +1)
3
⎛ 3 2x 14 ⎞ ⎜⎛ x5 ⋅ 3 x2 +1 ⎟⎞ ⎛ 3 2x 14 ⎞
f ′( x) = f ( x) ⎜ + 2 − ⎟= ⎜ + 2 − ⎟.
⎝ 5x 3x + 3 2 x +1 ⎠ ⎜ (2 x +1)7 ⎟ ⎝ 5x 3x + 3 2 x +1 ⎠
⎝ ⎠

If we had not used logarithmic differentiation here, finding the derivative would have been a
long and complicated process.

Example 59 Given that x > 0, find the derivative of f(x) = xx.

Solution First find the logarithms of both sides of the expression:


ln f(x) = ln xx = x ⋅ ln x.
Differentiate both sides of the equation with respect to x:
f ′( x)
= x′ ⋅ ln x + x ⋅ (ln x) ′ =1+ ln x
f ( x)

f ′( x) = f ( x)(1+ ln x) = x x(1+ ln x).

Check Yourself 12
Find the derivative of each function by using logarithmic differentiation.
2
e x +1 ⋅ ( x2 – 1)10 x2 +1
1. f(x) = (3x – 1)5(x3 + 1)6 2. f ( x) = 3. f ( x) = 4
x x2 – 1
1 1
4. g( x) = x x
5. h(x) = xln x 6. f ( x) = (ln x) ln x

Answers
2
⎛ 15 18 x2 ⎞ 20 x 1 ⎞ e x +1 ⋅ ( x2 – 1)10
1. ⎜ + 3
5 3
⎟(3x – 1) ( x +1)
6
2. ⎛⎜ 2 x + 2 – ⎟⋅
⎝ 3x – 1 x +1 ⎠ ⎝ x – 1 2x ⎠ x

x ⎞ 4 x2 +1 1 – ln x 1x 2 ln x ln x 1 – ln(ln x) 1
3. ⎛ – 4. ⋅x 5. ⋅x 6. ⋅ (ln x) ln x
⎜ ( x2 – 1)( x2 +1) ⎟ ⋅ 2 x2 x 2
x ln x
⎝ ⎠ x –1
Differentiation 151
B. DERIVATIVES OF TRIGONOMETRIC FUNCTIONS
Let us begin by looking at the derivatives of the sine and cosine functions.

DERIVATIVES OF SINE AND COSINE

(sin x)′ = cos x


(cos x)′ = –sin x

Example 60 Find the derivative of the function f(x) = (sin x + cos x)2.

Solution f ′( x) = 2(sin x + cos x)(sin x+ cos x) ′ (by the General Power Rule)

f ′( x) = 2(sin x + cos x)(cos x – sin x) (by the sum, derivative of the sine and cosine)
2 2
f ′( x) = 2(cos x – sin x) (simplify)
cos 2x = cos2 x – sin2 x
f ′( x) = 2 cos 2 x (by the trigonometric identity)

Now let us derive the formula for the derivative of the function f(x) = sin x.

Proof (Derivative of Sine Function)


By the definition of the derivative, we have
f ( x + h) − f ( x ) sin( x + h) − sin x
f ′( x) = lim = lim
h→ 0 h h → 0 h
sin x cos h + cos x sin h − sin x sin x cos h − sin x cos x sin h ⎞
f ′( x) = lim = lim ⎛⎜ + ⎟
h→ 0 h h→ 0 ⎝ h h ⎠
sin h
lim =1
h→0 h ⎛ ⎛ cos h − 1 ⎞ ⎛ sin h ⎞ ⎞ cos h − 1 sin h
f ′( x) = lim ⎜ sin x ⎜ ⎟ + cos x ⎜ ⎟ ⎟ = lim sin x ⋅ lim + l im cos x ⋅ lim
lim
cos h − 1
=0
h→ 0
⎝ ⎝ h ⎠ ⎝ h ⎠⎠ h→ 0 h→ 0 h h →0 h→ 0 h
h→0 h
f ′( x) = sin x ⋅ 0 + cos x ⋅ 1 = c os x.

Example 61 Find the derivative of the function f(x) = x ⋅ sin x.

Solution By the Product Rule


f ′( x) = ( x ⋅ sin x) ′ = ( x) ′ ⋅ sin x+ x ⋅(sin x) ′= sin x+ xcos x.

152 Derivatives
Example 62 Find the derivative of the function f ( x) =
ex
cos x
.

⎛ e x ⎞′ ( e x )′ ⋅ cos x − e x ⋅ (cos x) ′
Solution f ′( x) = ⎜ ⎟ = (by the Quotient Rule)
⎝ cos x ⎠ cos 2 x
e x cos x + e x sin x
f ′( x) = (differentiate)
cos 2 x
e x (cos x + sin x)
f ′( x) =
cos 2 x (simplify)

CHAIN RULE FOR SINE AND COSINE

(sin f(x))′ = cos f(x) ⋅ f ′(x)


(cos f(x))′ = – sin f(x) ⋅ f ′(x)

Example 63 Find the derivative of the function f(x) = cos(x3 – x).

Solution f ′( x) = (cos( x3 − x)) ′ = − sin( x3 − x) ⋅ ( x3 − x) ′= −sin( x3 − x) ⋅(3 x2 −1)

Example 64 Find the derivative of the function f(x) = sin3x2.

Solution In this example we have the composition of three functions.

f ( x) = sin 5 x2 = (sin( x2 ))5


We apply the Chain Rule beginning from the outermost function:

f ′( x) = ((sin( x2 ))5 ) ′ = 5(sin( x2 ))4 ⋅ ( sin( x2 )) ′

f ′( x) = 5(sin( x2 ) )4 ⋅ cos( x2 ) ⋅ ( x2 ) ′
sin 2x = 2sin x cos x
f ′( x) = 5(sin( x2 ) )4 ⋅ cos( x2 ) ⋅ 2 x

f ′( x) =10 x si n( x2 )cos( x2 )

Example 65 Find the derivative of the function f(x) = (sin (ex) – cos (lnx))100.

Solution f ′(x) = [(sin (ex) – cos (lnx))100]′

f ′(x) = 100 ⋅ (sin (ex) – cos (lnx))99 ⋅ (sin (ex) – cos (lnx))′ (by the General Power Rule)

sin (ln x) (by the Chain Rule


f ′(x) = 100 ⋅ (sin (ex) – cos (lnx))99 ⋅ (excos (ex) + )
x for sine and cosine)

Differentiation 153
Check Yourself 13
Find the derivative of each function.
sin x
1. f(x) = x – 3 sin x 2. f(x) = xcos x 3. f ( x) =
1+ cos x
ex
4. f ( x) = 5. f(x) = cos2(x2 + x – 1) 6. f(x) = sin(ex + x2)
sin x + cos x
7. f(x) = cos2(ln x + 1) 8. f(x) = ex sin(ex)
Answers
1
1. 1 – 3cos x 2. cos x – xsin x 3.
1+ cos x
2 ⋅ e x ⋅ sin x
4. 5. –sin(2x2 + 2x – 2) ⋅ (2x + 1) 6. cos(ex + x2) ⋅ (ex + 2x)
(sin x + cos x) 2
– sin(2 ln x+ 2)
7. 8. ex(sin(ex) + ex cos(ex))
x

DERIVATIVES OF OTHER TRIGONOMETRIC FUNCTIONS

(tan x)′ = sec2 x = 1 + tan2 x (tan f(x))′ = sec2 f(x) ⋅ f ′(x)

(cot x)′ = –csc2 x = –(1 + cot2 x) (cot f(x))′ = –csc2 f(x) ⋅ f ′(x)
sin x
(sec x)′ = sec x ⋅ tan x = (sec f(x))′ = sec f(x) ⋅ tan f (x) ⋅ f ′(x)
cos2 x
cos x
(csc x)′ = –csc x ⋅ cot x = − (csc f(x))′ = –csc f(x) ⋅ cot f(x) ⋅ f ′(x)
sin2 x

Example 66 Find the derivative of the function f(x) = tan(x2 – 3x + 1).

Solution f ′(x) = sec2(x2 – 3x + 1) ⋅ (x2 – 3x + 1)′ = sec2(x2 – 3x + 1) ⋅ (2x – 3)


or = (1 + tan2(x2 – 3x + 1))(2x – 3).

Example 67 Find the derivative of the function f(x) = csc(ln x).

1 csc(ln x) ⋅ cot(ln x)
Solution f ′(x) = –csc (ln x) ⋅ cot (ln x) ⋅ (ln x)′ = –csc (ln x) ⋅ cot (ln x) ⋅ =–
x x

Example 68 Find the derivative of the function f(x) = e–cot x.


2

Solution 2 2 2
f ′(x) = e–cot x ⋅ (–cot2 x)′ = e–cot x ⋅ (–2cot x)′ ⋅ (cot x)′ = e–cot x ⋅ (–2cot x) ⋅ (–csc2 x)
2
f ′(x) = 2e–cot x ⋅ cot x ⋅ csc2 x

154 Derivatives
sec x
Example 69 Find the derivative of the function f ( x) =
1+ tan x
.

Solution By the Quotient Rule,


(sec x)′ ⋅ (1+ tan x) − sec x ⋅(1+ tan x) ′ (by the Quotient Rule)
f ′( x) =
(1+ tan x)2
sec x tan x ⋅ (1+ tan x) − sec x ⋅ sec 2 x
f ′( x) = (differentiate)
(1+ tan x)2
sec x(tan x + tan 2 x − sec 2 x)
f ′( x) = (factorize)
(1+ tan x)2
sec x(tan x − 1)
f ′( x) = (simplify using tan2 x + 1 = sec2 x)
(1+ tan x)2

Check Yourself 14
Find the derivative of each function.
tan x
1. f ( x) = 2. f(x) = 4 sec x – cot x
x
3. f(x) = excsc x 4. f(x) = ln(tan x)

5. f(x) = tan2(ln x) 6. f(x) = cot(x2 – x + 1)


Answers
x sec 2 x – tan x
1. 2. 4sec xtan x + csc2 x
x2
1
3. excsc x(1 – cot x) 4.
sin x cos x
2 tan(ln x) ⋅ sec 2(ln x)
5. 6. csc2(x2 – x + 1) ⋅ (1 – 2x)
x

Differentiation 155
C. DERIVATIVES OF SPECIAL FUNCTIONS
1. Absolute Value Functions
y
y = |x|
⎧⎪ g′( x), g( x) > 0
If f(x) = |g(x)|, then f ′( x) = ⎨ .
⎪⎩ − g′( x), g( x) < 0
In general, a derivative does not exist when the function has 0 as
x
value because of a ‘corner’ as demonstrated in the graph on the right. corner

When g(x) = 0, the derivative exists only when the right-hand side and the left-hand side
derivatives are equal to each other.
We can also formulate the derivative expression as follows:
| g( x)|
f ′( x) = ⋅ g ′( x) = sgn[ g( x)] ⋅ g ′( x), g( x) ≠ 0.
g( x)

Example 70 Find the derivative of the function f(x) = |1 – x2| at the points x = 2, x = 1 and x = 0.

Solution Let us begin by trying to rewrite the function as a piecewise function.


The roots of the function are x = –1 and x = 2.
2
⎪⎧ x − 1, x < −1 and x ≥ 1
Then the function f will be f ( x) = ⎨ .
2
⎪⎩1 − x , –1 ≤ x < 1 y
Let us find f ′(2): y = f(x)
A piecewise function is a
function that is defined Note that f(2) ≠ 0.
by different formulae in
1
different parts of its For x = 2, f ′(x) = (x2 – 1)′ = 2x.
domain.
So, f ′(2) = 2 ⋅ 2 = 4.
–1 1 x
Let us find f ′(1):
Since f(1) = 0, we will check the left-hand and the right-hand derivatives.
For x < 1, f ′(x) = (1 – x2)′ = –2x.
So, f ′(1–) = –2 ⋅ (1) = –2.
For x > 1, f ′(x) = (1 – x2)′ = 2x.
So, f ′(1+) = 2 ⋅ 1 = 2.
Since f ′(1–) ≠ f ′(1+), f ′(1) does not exist.
Let us find f ′(0):
Note that f(0) ≠ 0.
For x = 0, f ′(x) = (1 – x2)′ = –2x.
So, f ′(0) = –2 ⋅ 0 = 0.

156 Derivatives
Example 71 Given that f(x) = |x3 – 4x2 + 4x|, find the derivative of f(x) at the point x = 2.

3 2
⎧−
⎪ x + 4x − 4x,x < 0
Solution The piecewise form of the function is f ( x) = ⎨ .
3 2
⎪⎩ x − 4x + 4x, x ≥ 0
Since f(2) = 0, we will check the left-hand and the right-hand derivatives.
For 0 ≤ x < 2, f ′(x) = (x3 – 4x2 + 4x)′ = 3x2 – 8x + 4
f ′(2–) = 3 ⋅ (2)2 – 8 ⋅ 2 + 4 = 0.
For x > 2, f ′(x) = (x3 – 4x2 + 4x)′ = 3x2 – 8x + 4
f ′(2+) = 3 ⋅ (2)2 – 8 ⋅ 2 + 4 = 0.
Since the left-hand and the right-hand derivatives are equal to each other, the derivative of
the function exists at the point x = 2 and f ′(2) = 0.

Example 72 Given that f(x) = |x – x2|, find f ′(2) and f ′(3).

| x − x2 |
Solution Since f(2) ≠ 0 and f(3) ≠ 0, we can use the formula f ′( x) = ⋅ (1 − 2 x).
x − x2
| 2 − 22 |
f ′(2) = ⋅ (1 − 2 ⋅ 2) = −1 ⋅( −3) = 3
2 − 22
| 3 − 32 |
f ′(3) = ⋅ (1 − 2 ⋅ 3) = −1 ⋅( −5) = 5
3 − 32

Example 73 Given that f(x) = |cos x|, find the derivative of f(x) at the points x =
π
3
and x = π.

π
Solution For x = , cos x > 0. So, f(x) = cos x and f ′(x) = – sin x.
3
π π 3
f ′( ) = − sin( ) = −
3 3 2
For x = π, cos x < 0. So, f(x) = –cos x and f ′(x) = sin x.
f ′(π) = sin π = 0

Example 74 Given that f(x) = |x3 – 9| + x2, find f ′′(2).

Solution For x = 2, x3 – 9 < 0 and so f(x) = –x3 + 9 + x2.


If we take the derivative twice,
f ′(x) = –3x2 + 2x
f ′′(x) = –6x + 2.
Therefore, f ′′(2) = –6 ⋅ 2 + 2 = –10.

Differentiation 157
Check Yourself 15
1. Given that f(x) = |x2 – 3x|, find f ′(3) and f ′(5).
2. Given that f(x) = |x4 – 2x2 + 1|, find the derivative of f(x) at the point x = 1.
π π
3. Given that f(x) = |sin x|, find f ′( ) and f ′( ) .
6 2
Answers
1. does not exist; 7 2. 0 3. 3 ; 0
2

2. Sign Functions
Note that a sign function has the range {–1, 0, 1}. When y
slope = 0
it takes –1 and 1 as its value, the graph is a horizontal line. 1
y = sgn x
Since the slope of a horizontal line is 0, we have 0 as the
derivative. When the function takes 0 as its value, the
graph has a discontinuity. So, the derivative does not exist. x
no slope
Look at the graph on the right: –1
In conclusion, if f(x) = sgn (g(x)), then
slope = 0
⎧⎪0, g( x) ≠ 0
f ′( x) = ⎨ .
⎪⎩does not exist, g( x) = 0

Example 75 Given that f(x) = sgn(x2 – x),


find the derivative of f(x) at the points
x = –2 and x = 1.

Solution We begin by finding the value of


f(–2) and f(1):
f(–2) = sgn((–2)2 – (–2)) = sgn(6) = 1
f(1) = sgn(12 – 1) = sgn(0) = 0.
Since f(–2) ≠ 0, f ′(–2) = 0.
Since f(1) = 0, f ′(1) does not exist
(f(x) is not continuous at x = 1).

158 Derivatives
Example 76 Find the largest interval on which the function f(x) = sgn(x2 – x – 6) is differentiable.

Solution We can rewrite the function as a piecewise function:


⎧ 1, x < −2 and x > 3

f ( x) = ⎨ −1, −2 < x < 3 .

⎩ 0, x = −2 and x = 3

Since f(x) is not continuous at the points y

x = –2 and x = 3, it is not differentiable. 1 y = f(x)


It has a derivative at all other points, and
this is equal to zero. –2 3
x
So, the largest interval on which f is
differentiable is \ \ {–2, 3}.
–1

3. Floor Functions
⎧⎪0, g( x) ∉ ]
If f(x) = ag(x)b, then f ′( x) = ⎨ .
⎪⎩ may not exist, g( x) ∈ ]
When g(x) ∈ ], f(x) is certainly continuous and differentiable. However, when g(x) ∉ ], we
cannot be certain. It may be differentiable or not. In order to determine whether a floor
function is differentiable or not at a given value, we check the left-hand and the right-hand
derivatives.

Example 77 Given that f ( x) = a 2 x + 1b


3
, find the derivative of f(x) at the points x = 2 and x = 4.

2 x +1 5 2x + 1
Solution For x = 2, = ∉ ]. Since the expression is not an integer for x = 2, f ′(2) = 0.
3 3 3
2 x +1
For x = 4, = 3 ∈ ]. Here we have to find the left-hand and the right-hand derivatives,
3
because the expression 2 x +1 is an integer for x = 4.
3
f ( x ) − f (4) 2 −3 −1
f ′(4− ) = lim− = – = − =+∞
x→ 4 x−4 4 −4 0
f ( x) − f (4) 3 −3 0
f ′(4+ ) = lim+ = + = =0
x→ 4 x−4 4 − 4 0+

Since f ′(4–) ≠ f ′(4+), f ′(4) does not exist.

Differentiation 159
Example 78 π
Given that f(x) = asin xb, find f ′( ) and f ′(π).
6

y
π 1 π
Solution sin( ) = ∉ ]. So, f ′( ) = 0.
6 2 6 1
At x = π, f(x) is not continuous (see the graph). So, it is –p/2 p 2p
p/2 x
not differentiable. Thus the derivative of f(x) does not
–1
exist at this point. y = f(x)

Example 79 Given that f(x) = ax2b, find f ′(0).

Solution For x = 0, f(x) = 0 ∈ ]. So, we have to find the left-hand and the right-hand derivatives of
the function f(x) = ax2b at the point x = 0.
f ( x) − f (0) a x2 b − 0 0
f ′(0 − ) = lim− = lim− = − =0
x→0 x−0 x→0 x −0 0

f ( x) − f (0) ax2 b − 0 0
f ′(0 + ) = lim+ = lim+ = + =0
x→0 x−0 x→0 x −4 0
Notice that if –1 < x < 1, ax2b = 0.
Since f ′(0–) = f ′(0+), the derivative of f(x) = ax2b exists at the point x = 0 and f ′(0) = 0.

Example 80 3
Given that f(x) = x|x| + axbsgn(x), find f ′( ).
2

⎛ 3⎞
Solution For x = ⎜ ⎟ , we can rewrite the function as f(x) = x ⋅ x + 1 ⋅ 1 = x2 + 1.
⎝2⎠

3
For x = ⎛⎜ ⎞⎟ , we have the same function f(x) = x ⋅ x + 1 ⋅ 1 = x2 + 1.
⎝2⎠
3 3 3
So, for x = we have f ′(x) = 2x. That gives f ′( ) = 2 ⋅ = 3.
2 2 2

Check Yourself 16
1. Given that f(x) = sgn (x2 + x), find f ′(–1) and f ′(2).
π π
2. Given that f(x) = acos xb, find f ′( ) and f ′( ).
3 2
3. Given that f(x) = |x + 3x – 4| + ax b + sgn (x2 – 1), find f ′(0).
2 2

Answers
1. does not exist; 0 2. 0; does not exist 3. –3

160 Derivatives
D. IMPLICIT DIFFERENTIATION
Up to now we have worked with the functions expressed in the form y = f(x). In this form,
the variable y is expressed easily in terms of the variable x. A function in this form is said to
be in the explicit form. However, some functions cannot be written in explicit form. Consider
the following equation:
y5 + y + x = 0
If we are given a value of x, we can calculate y in this equation. However, we cannot write
the equation in the form y = f(x). We say that x determines y implicitly, and that y is an
implicit function of x. Look at the same more implicit functions:
x5 + 2xy2 – 3y4 = 7
y – 2y2 = x
x2 – y2 + 4y = 0
How can we differentiate an implicit function? Recall the Chain Rule for differentiation. In
an implicit function, y is still a function of x, even if we cannot write this explicitly. So, we
can use the Chain Rule to differentiate terms containing y as functions of x. For example, if
we are differentiating in terms of x,
dy
(y4)′ = [(f(x))4]′ = 4( f(x))3f ′(x) = 4y3y′ or (y4)′ = y3 ,
dx
dy
(7y)′ = 7y′ or (7y)′ = 7 .
dx

The procedure we use for differentiating implicit functions is called implicit differentiation.
Let us summarize the important steps involved in implicit differentiation.

IMPLICIT DIFFERENTIATION

1. Differentiate both sides of the equation with respect to x. Remember that y is really
a function of x and use the Chain Rule when differentiating terms containing y.
dy
2. Solve the resulting equation for y′ or in terms of x and y.
dx

Example 81 Find y′ given the equation y5 + y + x = 0.

Solution ( y5 + y + x)′ = (0)′ (differentiate both sides)

( y5 )′ +( y)′+( x) ′ = 0 (by the Sum Rule)

5y4 y′ + y′ +1= 0 (by the Chain Rule)

y′(5y4 +1) = −1 (factorize)


1
y′ = − (isolate y′ )
5y4 +1

Differentiation 161
Example 82 Find
dy
dx
given the equation y3 – y2x + x2 – 1 = 0.

Solution ( y3 − y2 x + x2 − 1)′ = (0)′ (differentiate both sides)

( y3 )′ − ( y2 x)′ + ( x2 )′ − (1)′ = 0 (by the Sum Rule)

dy dy
3y2 − (2 y x + y2 ) + 2 x − 0 = 0 (by the Chain Rule and the Product Rule)
dx dx
dy
(3y2 − 2 yx) = y2 − 2 x (factorize)
dx
dy y 2 – 2x dy
= 2 (isolate )
dx 3y − 2 yx dx

Example 83 The equation x2 + y2 = 4 is given.


dy
a. Find by implicit differentiation.
dx
b. Find the slope of the tangent line to the curve at the point (ñ3, 1).
c. Find the equation of the tangent line at this point.

Solution a. Differentiating both sides of the equation with respect to x, we obtain


( x2 + y2 )′ = (4) ′

( x2 )′ +(y2 )′ = 0

dy
2x + 2y =0
dx
dy x
=– ( y ≠ 0).
dx y

b. The slope of the tangent line to the curve at the point (ñ3, 1) is given by
dy dy x 3
dx ( a, b)
m= =– =– = – 3.
dx ( 3, 1) y( 3 , 1)
1
is used for slope of the
curve at the point (a, b). c. We can find the equation of the tangent line by using the point-slope form of the
equation of a line. The slope is m = –ñ3 and the point is (ñ3, 1). Thus,
y – y1 = m(x – x1) y

y – 1 = –ñ3(x – ñ3) y + 3x – 4 = 0

ñ3x + y – 4 = 0. 2

A sketch of this tangent line is given on the right.


–2 2 4 x
The line x + ñ3y – 4 = 0 is tangent to the graph
of the equation x2 + y2 = 4 at the point (ñ3, 1). x2 + y2 = 4
–2

162 Derivatives
Example 84 Find the derivative with respect to x of the implicit function x2 + y2 + x2 = 2.

Solution Differentiating both sides of the given equation with respect to x, we obtain

d 2 d d
( x + y2 )1/ 2 + ( x2 ) = (2)
dx dx dx
1 2 d 2
( x + y2 )1/ 2 ( x + y2 ) + 2 x = 0
2 dx
1 2 dy
( x + y2 )–1/ 2 (2 x + 2 y )+ 2 x = 0
2 dx
dy
2x + 2y = –4 x( x2 + y2 )1/ 2
dx
dy
2y = –4 x( x2 + y2 )1/ 2 – 2 x
dx
2 2
dy –2 x x + y – x
= .
dx y

Check Yourself 17
dy
1. Find by implicit differentiation.
dx
a. x3 + x2y + y2 = 5 b. x2y + xy2 = 3x
c. exey = 1 d. exln y = 1

2. Find the equation of the tangent line to each curve at the given point.
2 2
a. x2y3 – y2 + xy – 1 = 0; (1, 1) b. x – y =1; (–5, 9 )
16 9 4
2 2
c. x 3 + y 3 = 4; (1, 3 3) d. ln y = xy; (0, 1)

Answers
–3x2 – 2 xy 3 – 2 xy – y2
1. a. b. c. –1 d. –yln y
x2 + 2 y x2 + 2 xy
3 5 5
2. a. y = – x+ b. y = – x–4 c. y = –ñ3x + 4ñ3 d. y = x + 1
2 2 4

Differentiation 163
E. DERIVATIVES OF PARAMETRIC FUNCTIONS
Sometimes we express the variables x and y as functions of a third variable t by a pair of
functions.
x = f(t), y = g(t)
Functions like these are called parametric functions, and the variable t is called the parameter.

PARAMETRIC DIFFERENTIATION

dy
dy dx
= dt , ≠0
dx dx dt
dt
dy
This enables us to find the derivative of a parametric function ( ) without having to
dx
eliminate the parameter t.

Example 85 Find the derivative with respect to x of the parametric curve x = t + 2 and y = 2t2 – 1.
dy
dy 4t
Solution = dt = = 4t
dx dx 1
dt

Example 86 If a is a positive constant and x = a cos t, y = a sin t, then find


dy
dx
.

dy
dy a cos t
Solution = dt = = – cot t
dx dx – a sin t
dt

Example 87 The parametric curve is given by the equations x = t +1 and y = t2 + 3t. Find the slope of
its tangent at x = 2.
dy
Solution Let us begin by finding in terms of t.
dx
dy d 2
(t + 3t )
dy 2t + 3
= dt = dt = = 2( t +1)1/ 2 (2 t + 3)
dx dx d 1
( t +1)
dt dt 2 t +1

For x = 2, x = t +1 = 2. So, t = 3.
dy
m= t=3 = 2 ⋅ ( t + 1)1/ 2 ⋅ (2t + 3) t=3 = 2 ⋅(3+1)1/ 2 ⋅(2 ⋅3+ 3) = 2 ⋅2 ⋅9 = 36.
dx

164 Derivatives
Example 88 The parametric equations x = 1+et – 1, y = t2 + ln t describe a curve in the plane. Find the
tangent line to the curve at t = 1.
dy
Solution First we need to find in terms of t:
dx
dy 1
2t +
dy 2t 2 +1 1 2t 2 +1
= dt = t –1 t = ⋅ t –1 =
dx dx e t e t ⋅ et –1
dt
dy 2t 2 +1 2 ⋅ 12 +1 3
m= = = = = 3.
dx t=1 te t –1
t=1 1 ⋅ e1–1 1

If t = 1, then x = 1 + et–1 = 1 + e1–1 = 2 and y = t2 + lnt = 12 + ln 1 = 1 + 0 = 1.


Therefore, the line is tangent to the given curve at the point (2, 1).
The equation of the line passing through (2, 1) with the slope m = 3 is
y – 1 = 3(x – 2)
y = 3x – 5.

PARAMETRIC DIFFERENTIATION OF SECOND ORDER

dy′
d2 y dy′
= = dt
dx2 dx dx
dt
dy
1. Express y′ = in terms of t.
dx
2. Differentiate y′ with respect to t.
dx
3. Divide the result by .
dt

Example 89 A parametric curve is given by the equations x = 1 + et, y = t2et.


2
Find the second derivative d y .
dx2
dy d 2 t
dy (t e )
dy 2te t + t 2 e t
Solution First, we find in terms of t: = dt = dt = = t2 + 2t.
dx dx dx d t e t
(1+ e )
dt dt
dy d dy d
Then, we differentiate with respect to t: ( ) = (t 2 + 2t ) = 2t + 2.
dx dt dx dt
dx 2
Finally, we divide the result by = et to obtain d y = 2t + 2 .
dt dx2 et

Differentiation 165
Example 90 Find
d2 y
dx2
, if x = 2t – t2 and y = 1 – t3.

dy d
(1 – t3 )
dy dy –3t 2
Solution First, find in terms of t: = dt = dt = .
dx dx dx d 2
(2t – t ) 2 – 2 t
dt dt

dy d dy d 3t 2 –6t ⋅ (2 – 2t) – (–3t 2 ) ⋅(–2)


Then, differentiate with respect to t: ( ) = (– )= .
dx dt dx dt 2 – 2t (2 – 2t)2

dx dx d 2 y –12t +12t 2 – 6t 2 6t 2 – 12t 3t


Finally, divide the result by : : 2= 2
= 3
=– .
dt dt dx (2 – 2t) ⋅ (2 – 2 t) 8 ⋅(1 – t) 4( t – 1) 2

Example 91 Find the second derivative of the paremetric curve


given by x = 1+ et and y = 1 – sin t.

Solution First, dy = – cost t .


dx e

d dy sin t ⋅ e t – ( – cos t)e t sin t + cos t


Second, ( )= = .
dt dx et et

d2 y sin t + cos t sin t + cos t


Finally, = = .
dx et ⋅ et e 2t

Check Yourself 18
dy
1. Find for each parametric curve.
dx
t –1 t +1
a. x = 2t + 3, y = t2 – 1 b. x = 5cos t, y = 5sin t c. x = , y=
t +1 t –1
d2 y
2. Find for each parametric curve.
dx2
dy
a. x = ln t, y = 1 + sin t b. x = 3t2 + 2, y = 2t2 – 1 c. = 4+ sin 2 t , x = cos 2 t
dx
Answers
2
t +1 ⎞
1. a. t b. –cot t c. − ⎛⎜ ⎟
⎝ t −1⎠
1
2. a. tcos t – t2sin t b. 0 c. –
4 4+ sin 2 t

166 Derivatives
F. DERIVATIVES OF INVERSE TRIGONOMETRIC
FUNCTIONS
In order to find the derivatives of the inverse trigonometric functions, we can use implicit
differentiation. For example, what is the derivative of Arcsin x?
π π
Let y = Arcsinx, then sin y = x and − ≤ y ≤ .
2 2

Now, if we differentiate sin y = x implicitly with respect to x,

dy dy 1
we get cos y ⋅ =1 or = .
dx dx cos y
π π 1
x
cos y ≥ 0, since − ≤ y ≤ . So, cos y = 1 − sin 2 y = 1 − x2 .
2 2 y
d 1 1 1 – x2
Therefore, (Arcsin x) = = .
dx cos y 1 − x2
We can use a similar method to find the derivative of the other inverse trigonometric
functions.

DERIVATIVE OF INVERSE TRIGONOMETRIC FUNCTIONS

1 f ′( x)
(Arcsin x)′ = (Arcsin f ( x)) ′=
1 − x2 1 − ( f ( x))2

1 f ′( x)
(Arccos x)′ = – (Arccos f( x)) ′= –
2
1− x 1 − ( f ( x))2

1 f ′( x)
(Arctan x)′ = (Arctan f( x)) ′=
1+ x2 ( f ( x))2 +1

1 f ′( x)
(Arccot x)′ = – (Arccot f( x)) ′= –
1+ x2 ( f ( x))2 +1

Example 92 Find the derivative of the function f(x) = Arcsin (x2).

( x2 )′ 2x
Solution f ′( x) = (Arcsin( x2 )) ′ = =
2 2
1 − (x ) 1 − x4

Example 93 Find the derivative of the function f(x) = Arccot (e3x).

Solution ( e3 x )′ 3e 3 x
f ′( x) = (Arccot( e 3 x ))′ = − = −
1+( e3 x )2 1+ e6 x

Differentiation 167
Example 94 Find the derivative of the function f(x) = x2 – Arccos (ex).

Solution By the Sum Rule,

ex ex
f ′( x) = ( x2 – Arccos( e x )) ′= ( x2 ) ′– (Arccos( e x)) ′= 2 x– (– ) = 2 x+ .
1 – e2 x 1 – e2 x

Example 95 Find the derivative of the function f(x) = x⋅ Arctan ñx.

Solution By the Product Rule,

1 1 x
f ′( x) = ( x ⋅ Arctan x ) ′ =1 ⋅ Arctan x + x ⋅ ⋅( x –1/ 2 ) = Arctan x + .
1+ ( x ) 2
2
2(1 + x)

Example 96 Find the equation of the tangent line to the curve f(x) = 2 Arccos
x
2
at x = ñ3.

⎛ x ⎞′ 1
⎜ ⎟ 2
f ′( x) = –2 ⋅ ⎝2⎠ = –2 ⋅ 2 =–
Solution 2 2
x x 4 – x2
1 – ⎛⎜ ⎞⎟ 1 – ⎛⎜ ⎞⎟
⎝2⎠ ⎝2⎠

The slope of the tangent line is f ′(ñ3 ) = –2.

2π 2π
The curve passes through the point ( 3, ) since f ′(ñ3 ) = .
3 3
The equation of the tangent line is: y – 2π = –2( x – 3) or y = –2 x+ 2 3 + 2 π .
3 3

Check Yourself 19
1. Differentiate the functions.
a. f(x) = (ex – Arcsin x)2 b. f(x) = eArccos x
c. f(x) = (Arctan x)3 d. f(x) = ln Arccot(x2 + 5x)
2. Find the equation of the tangent line to the curve y = Arccos 2x at the point where it
crosses the y-axis.
Answers
⎛ 1 ⎞ e Arccos x 3(Arctan x)2
1. a. 2( e x − Arcsin x) ⎜ e x − ⎟ b. − c.
⎝ 1 − x2 ⎠ 1 − x2 1+ x2

2 x +5 π
d. − 2. y = –2 x +
Arccot( x2 +5 x) ⋅ (1+( x2 +5 x) 2 ) 2

168 Derivatives
EXERCISES 1 .3
A. Derivatives of Exponential and x +1
l. f ( x) = ln m. f ( x) = ln x – 1
Logarithmic Functions x –1 x +1

1. Differentiate the functions. n. f(x) = x2ln x o. f ( x) = ln x2


a. f(x) = 3ex b. f(x) = e3x – 1 p. f ( x) = ln x + x q. f(x) = ln(ñx – 1)–2
2–1
c. f(x) = ex d. f(x) = e–2x r. f(x) = ln(x2 – x) s. f(x) = ex ln x
1
e. f(x) = 2x f. f ( x) = ( )x 2
3 t. f ( x) = ln( x x − x) u. ln ( x +1)( x – 2)
e x –1
g. f(x) = ex ⋅ 3x h. f(x) = xex
x x +1
e x +1 v. f ( x) = log w. f ( x) = log 3
i. f(x) = x + 2e2 x
j. f ( x) = x x +1 x –1
e –1
x. f ( x) = ( e x – log 2 x2 )3
k. f ( x) = e x – 1 l. f(x) = (ex + x)100

1 y. f ( x) = x2 – log 3 e x
m. f ( x) = n. f(x) = (ex + x)(2ex – 1)
x
e +1
z. f ( x) = (log(1+ e x )) 3
e + e– x
x 1
o. f ( x) = p. f ( x) = e 2x
3
x –1
3. Find the derivative of each function by using
q. f ( x) = e x+1
r. f ( x) = e x+1 logarithmic differentiation.

6x – 1 a. f(x) = (2x – 1)7(x4 – 3)11


–2t 2 x2–1
s. f(x) = e + x e t. f ( x) = x
3 +1 b. f(x) = x(x + 1)(x2 + 1)
53 x+1 c. f ( x) = 3 x2 e x
2
–1
⋅ ( x3 – x)–2
2 + 4x
u. f(x) = 3x v. f ( x) =
x2 + e x
2

x2 – x – 1 ex + 2 x d. f ( x) = 4 + 3x
w. f ( x) = 2 x. f(x) = 2 ⋅3 3
x2 +1
e. f(x) = xñx
f. f(x) = (ln x)x+1
2. Differentiate the functions.
4. Find the equation of the tangent line to the graph
a. f(x) = 3ln x b. f(x) = ln 4x 2
of y = ex –1 at the point P(1, 1).
c. f(x) = 3ln 4x d. f(x) = 3ln (2x + 1)
e. f(x) = ln x 7
f. f(x) = ln ñx 5. Find the equation of the tangent line to the curve
y = ex + e–x at the point (0, 2).
g. f(x) = log3 x h. f(x) = log1/2 x
i. f(x) = xlog x j. f(x) = log2 (x2 + 1) 6. Find the equation of the tangent line to the graph
k. f(x) = ln (4x – 6x + 3) 2
of y = x2ln x at the point (1, 0).

Differentiation 169
B. Derivatives of Trigonometric 8. Find the equation of the tangent line to the curve
Functions at the given point.
a. y = ex – cos x + 1; x=0
7. Differentiate the functions.
b. y = x cos x; x=π
a. f(x) = sin (3x – 5)
9. For what values of x does the graph of
b. f(x) = cos (x2 – 1) f(x) = x + 2 sin x have a horizontal tangent line?
c. f(x) = sin x – cos x C. Derivatives of Special Functions
d. f(x) = 2 tan x + sec x
10. Find the required values using the given data:
e. f(x) = sin x ⋅ tan x
⎛ −

f. f(x) = 2x tan x – x cos x a. f ( x) =| 2 x − 3x2 |, f ′(0 − )+ f ′ ⎜ ⎜⎛ 2 ⎟⎞ ⎟+ f ′(1) = ?
⎜⎝ 3 ⎠ ⎟
⎝ ⎠
g. f(x) = cos2(2x3 – 3x)
⎛ 1⎞
h. f(x) = sin3(ln cos 2x) b. f ( x) = x2 + a xb + sgn( x – 2), f ′ ⎜ ⎟ = ?
3
⎝ 3⎠
i. f ( x) = sin x + e
tan x c. f(x) = (2x + 3) ⋅ sgn(x2 + 1), f ′(ñ2) = ?
3x +1
1 – cos x 10 d. f ( x) = sgn(4 x +5) ⋅a b, f ′(2) = ?
j. f ( x) = ( ) 2
1+ cos x
4x
k. f ( x) =
cot x 11. Given that f ( x) =| x2 − 4| − sgn( x3 + x)+ ,
2 x2 − 1
1+ sec x
find the number of different x-values for which the
l. f(x) = (1 + sec x) ⋅ (1 – cos x)
function is not differentiable.
2
m. f(x) = tan x – x – 1 D. Implicit Differentiation
3
n. f ( x) = cot( x – 1) dy
12. Find for each equation below.
1 – sec 2 ( x3 – 1) dx
a. 5x – 4y = 3 b. xy – y – 1 = 0
tan(2 x – 1) 3
o. f ( x) = ( ) y
2 + ln x c. x3 + x2 – xy = 1 d. – 3x2 = 5
x
p. f(x) = (cos ex + x cos ex)2 e. 2x2 + 3y2 = 12 f. x2 + 5xy + y3 = 11
q. f(x) = (esin x + cos x + x cos ex)2 g. x2y3 – xy = 8 h. òxy – 3x – y2 = 0
1+ sin 2 x x
r. f ( x) = ln( ) i. ex+y – ex–y = 1 j. ln = xy – 1
1 – cos 2 x y

s. f(x) = [x2sin(x – 1)]5 13. Find the equation of the tangent line to the given
3 curve at the indicated point.
t. f ( x) = sec 2( 2x )
x –1 a. 4x2 + 2y2 = 12; (1, –2)

u. f(x) = tan2[ln(2x + 1)] b. 2x2 + xy = 3y2; (–1, –1)


x c. exy + x2 = 2x; (1, 0)
v. f ( x) = csc 3 e 2– ln x
x –1 d. ln(x – y) + 1 = 3x2; (0, –e–1)

170 Derivatives
E. Derivatives of Parametric Functions Mixed Problems
dy
14 . Find for each pair of parametric equations. 19. Find the given order derivative by finding the first few
dx
a. x = 3t – 1 and y = t2 – 2t derivatives and observing the pattern that occurs.
1 27 d99
b. x = t(t + 1) and y = t –
t a. d 27 (cos x) b. (sin 2 x)
3 2 2 dx dx99
c. x = t – t – 1 and y = t + 3t + 1
35 51
d. x = t +1 and y = t2 + 3t c. d 35 ( x sin x) d. d 51 ( e3 x+1 )
dx dx
e. x = 3 t and y = 4 – t 2
2
f. x = 4cos t and y = 5sin t
20. Find the second derivative d y2 of each implicit
g. x = t + ln t and y = 1 – et › function. dx

15. Find the equation of the tangent line at the given a. x2y – 1 = 0 b. x3 + y4 = 20
point P. c. y2 + xy = 8 d. 3
x + 3 y =1
1
a. x = + t 2 , y = t2 – t + 1, P(2, 1)
t 21. Write the equation of the line which is tangent to
b. x = 3t2 + 2, y = 2t4 – 1, P(5, 1) ›
the curve y = x2 – 2|x – 1| at exactly two points.

d2 y 22. Find the required values using the given data:


16. Find .
dx2
a. f ( x) = Arctan 2 x+ ln 1+ 4 x2 , f ′(1) = ?
a. x = t2 – t b. x = ñt + 1
1 ⎛ 2⎞
y = t3 + 3t + 1 y= +1 b. f ( x) = sin x , f ′ ⎜ π ⎟ = ?
t
⎝ 16 ⎠
c. x = et – 1 d. x = 2sin2 t
y = In t y = 3cos2 t c. f(x) = (5x + 1)5x+1, f ′(0) = ?

F. Derivatives of Inverse π
Trigonometric Functions 23. Given that < x<π ,
2
17 . Differentiate the functions.
|1 − tan x | ⋅ sgn(tan x)
x differentiate f ( x) = .
a. f(x) = Arcsin cos x
2 a b
2
x2 – 1
b. f(x) = Arccos
x 24. Given that f(x) = x3|x2 – 2|, find f ′(–2) + f ′′(1).
x – Arcsin x
c. f ( x) =
ex x2 +1
25. Given that f ( x) = , solve f ′(x) > 0.
d. f(x) = ln Arccos ex › x2 − 1
2
e. f(x) = Arctan (x + x – 1)
f. f(x) = Arctan x – 1 – x2 26. Given that f ( x) = (2 − x + 2 )2 , solve f ′(x) = 0.
›
g. f(x) = Arcsin (tan x)

18. Find the equation of the tangent line and the 27. Given the parametric equations x = z2 + 2z – 2,
› dy
x y = sin(x + 2), z = ln t, find
normal line to y = Arcsin when x = 1.
2 dt t =1

Differentiation 171
CHAPTER SUMMARY
1. Introduction to Derivatives • The derivative of the first derivative of a function is
the second derivative of the function. The derivative of
• A tangent line to a curve is a line that touches the curve.
the second derivative is the third derivative of the
• The slope of a tangent line to the curve y = f(x) at the function.
point A(a, f(a)) is
• The nth derivative of a function is denoted by f (n)(x).
f ( x) – f ( a ) f ( a + h) – f ( a )
m = lim or m = lim .
x→a x–a h→ o h 3. Derivatives of Elementary Functions
• The problem of finding the slope of the tangent line to • Derivatives of exponential, logarithmic and trigonometric
the graph of a function f(x) at the point A(x, f(x)) is functions:
mathematically equivalent to the problem of finding the
rate of change of f(x). Function Derivative Chain Rule
• The average rate of change of f(x) over the interval
ex ex ef(x) ⋅ f ′(x)
f ( x + h) – f (x)
[x, x + h] is .
h ax ax ⋅ ln a af(x) ⋅ ln a ⋅ f ′(x)
• The derivative of f(x) with respect to x is the instantaneous rate
f ( x + h) – f (x) 1 f ′( x)
of change of f(x) and f ′( x) = lim . ln x
h→ o h x f ( x)
• The process of calculating the derivative of a function is 1 f ′( x)
loga x
called differentiation. x ln a f ( x) ln a
• If f ′(a) exists, then the function f(x) is differentiable at a.
sin x cos x cos f(x) ⋅ f ′(x)
• If a function f(x) is differentiable on the interval (a, b),
then it is differentiable for every value in that interval. cos x –sin x –sin f(x) ⋅ f ′(x)
• If a function f(x) is differentiable at a, then
tan x sec2 x sec2 f(x) ⋅ f ′(x)
± its graph has a non-vertical tangent line at a,
± it is continuous at a. cot x –csc2 x –csc2 f(x) ⋅ f ′(x)
• In the following cases a function is not differentiable at a
given point: sec x sec x ⋅ tan x sec f(x) ⋅ tan f(x) ⋅ f ′(x)
± if its graph has a corner,
csc x –csc x ⋅ cot x –csc f(x) ⋅ cot f(x) ⋅ f ′(x)
± if it is not continuous,
± if its graph has a vertical tangent line. • Logarithmic differentiation is a technique that can be
used to find easily the derivatives of complicated
2. Techniques of Differentiation functions involving products, quotients and powers.
• Constant rule: (c)′ = 0 • If f(x) = |g(x)|, then
• Power rule: (xn)′ = n ⋅ xn–1 | g( x)|
f ′( x) = ⋅ g′( x) = sgn( g( x)) ⋅ g′( x), g( x) ≠ 0.
• Constant Multiple rule: [c ⋅ f(x)]′ = c ⋅ f ′(x) g( x)
• Sum rule: [f(x) + g(x)]′ = f ′(x) + g′(x) • If f(x) = sgn(g(x)), then
• Product rule: [f(x) ⋅ g(x)]′ = f ′(x) ⋅ g(x) + f(x) ⋅ g′(x) ⎧⎪0, g( x) ≠ 0
f ′( x) = ⎨ .
f ( x) ⎞′ f ′( x) ⋅ g( x) – f ( x) ⋅ g ′( x) ⎪⎩does not exist, g( x) = 0
• Quotient rule: ⎛⎜ ⎟ =
⎝ g( x) ⎠ ( g( x))2 • If f(x) = ag(x)b, then
• Chain rule: [ f(g(x))]′ = f ′(g(x)) ⋅ g′(x) ⎧⎪0, g( x) ∉ ]
f ′( x) = ⎨ .
⎪⎩ may not exist, g( x) ∈ ]
• General Power rule: [(f(x))n]′ = n ⋅ [ f(x)]n–1 ⋅ f ′(x)

172 Derivatives
• Method of implicit differentiation:
Concept Check
1. Differentiate both sides of the equation with respect to
x. (Remember that y is really a function of x and the • What is a tangent line to a curve?
Chain Rule should be used to differentiate the terms
containing y.) • What is the expression for the slope of the tangent line to
2. Solve the resulting equation for y′ in terms of x and y. the curve y = f(x) at the point (a, f(a))?

• Derivatives of parametric functions: • Explain the geometrical meaning of the derivative.


If x = f(t) and y = g(t), then
• Explain the physical meaning of the derivative.
dy
dy
± = dt , • State the limit definition of f ′(x).
dx dx
dt • What do we mean when we say “when x = 3, the
value of the function is 1”?
dy′
d 2 y dy′
± = = dt . • What do we mean when we say “when x = 3, the
dx2 dx dx
dt derivative of the function is 1”?

• Derivatives of inverse trigonometric functions: • What do we mean when we say “f is differentiable at a”?

Function Derivative Chain Rule • What is the relation between differentiability and
continuity?

1 f ′( x) • State the Power Rule.


Arcsin x 2
1– x 2 1 – ( f ( x))
• State the Constant Multiple Rule.

• State the Sum Rule.


–1 – f ′( x)
Arccos x • State the Product Rule.
1– x2 1 – ( f ( x))2

• State the Quotient Rule.

1 f ′( x) • State the Chain Rule.


Arctan x
1+ x2 1+( f ( x))2
• Which rules must be applied in order to find the following
derivatives?
–1 – f ′( x)
Arccot x [(f(x) ⋅ g(x))2005]′; ⎡⎢
f ( x) ⎤′
1+ x2 1+( f ( x))2 + h( x) ⋅ m( x) ⎥ ; (f(g(h(x)))]′
⎣ g ( x ) ⎦

• Is it possible that the derivative of a function is equal to


itself? Give an example.

• Is it possible that the derivative of a function is equal to


negative of it? Give an example.

• Explain how logarithmic differentiation works.

• Explain how implicit differentiation works.

Chapter Summary 173


CHAPTER REVIEW TEST 1A
1. f(x) = x3 – 2x2 + 1 is given. Find the value of the 5. The graph of the y

f ( x) – f (2) parabola y = ax2 + bx + c


limit lim .
x→ 2 x–2 y = ax2 + bx + c is
t
2 shown on the right such
A) 2 B) 3 C) 4 D) 5x + 1 E) 3x – 4 1/2 45°
that t is its tangent at x
the point (1/2, –1).
By using the information –1

given in the figure, find


the value of a + b + c.
1
A) –2 B) − C) 0 D) 10 E) 15
2
2. f(x) = e – lnx is given. Find the value of the limit
x

f (1+ h) – f (1)
lim .
h→ 0 h

A) 0 B) 1 C) e D) e – 1 E) e + 1 6. f(x) = ax2 – 3x3 + 4 and f ′(2) = 8 are given. Find


the value of a.

A) 3 B) 4 C) 7 D) 11 E) 29

3. Find the slope of tangent line to the curve 7. f(x) = x2ex is given. Find f ′(x).
y = x4 – (lnx)2 at the point x = 1.
A) ex(2x + 1) B) x(2ex + x2)
A) 4 B) 5 C) 6 D) 10 E) 15
C) xex(x + 2) D) 2xex

E) xex

8. f(x) = sin x and g(x) = x2 + 3 are given.


4. Find the equation of the tangent line to the
Find the derivative of f(g(x)).
π
function f(x) = 3cos 2x – 1 at the point M( , − 1).
2 A) 2x cos (x2 + 3) B) 2x sin x

A) y = 1 B) y = 2 C) y = –4 C) x + sin x D) x sin (x2 + 3)

D) y = x – 2 E) y=–1 E) –x cos (x2 + 3)

174 Derivatives
⎧⎪ x3 , x ≤1 d
13. Find (ln(cos x)).
9. f ( x) = ⎨ is given. Find f ′(1). dx
⎪⎩3x, x >1
A) –tan x B) –sec x C) –cot x
A) 0 B) 1 C) 2 D) 3 E) does not exist
1 1
D) − E)
sin x cos x

14. Find the derivative of the function f(x) = (sin x)x.

10. f(x) = 2x2 – 3x + 1 is given. Find f ′′(1). A) (sin x)xcos x


B) sin x[ln (sin x) + cos x]
A) 8 B) 6 C) 4 D) 3 E) 1
C) ln (sin x) + xcot x
D) (sin x)x[ln(sin x) + xcot x]
E) (sin x)x(sin x + xcos x)

11. f(x) = tan x – cot x is given. Find f ′(x). 15. If the parametric function is given by the equations
dy
4 3 x = sin2 θ, y = sin2 θ, find .
A) B) C) 2tan x 2 dx
sin2 2x sin 2x
A) 0 B) 1 C) –1 D) sin 2θ E) –tan 2θ
D) tan2 x + cot2 x E) sin 2x

16. The implicit function excos y + eysin x = 0 is


dy
given. Find .
dx
A) tan x B) tan y
x
e x y
C) ⋅ (cos y + sin x) D) e x cos y + ey cos x
12. f ( x) = 2 x − 1 is given. Find f ′(5). ey e sin y − e sin x
x y

A)
1
B)
1
C)
1
D)
1
E)
1 E) e y cos y + ex cos x
2 3 4 5 6 e sin x − e sin y

Chapter Review Test 1A 175


CHAPTER REVIEW TEST 1B
dy x2
1. y = 3xt – x2t2 is given. Find . 5. If the curve y = is tangent to the straight line
dt a
with the equation x – y = 1, then find the value
A) 3 – 2t B) 3t – 2t2x C) 3x – 2x2t
of a.
D) 3x + 2x2 E) 3t + 2x2t
A) 5 B) 4 C) 3 D) 2 E) 1

6. If f(x) = |2 – x| + 2, then find the value of the


x
e −1 expression f ′(1) + f ′(3).
2. f ( x) = is given. Find f ′(x).
e x +1
A) 0 B) 1 C) 2 D) 3 E) 4
ex
A) 2ex B) 2ex(ex + 1) C)
e x +1
2e x ex
D) E)
x
( e +1)2
x
( e − 1)2

7. The graph given on y

the right belongs to 1


the function f(x).
f ( x) 1 2 4 x
If g( x) = , then
x
find the slope of the y = f(x)
3. Given that f(1) = 3, lim f ( x) − 3 = 6 and
x →1 x −1 tangent line to the
3
h(x) = x ⋅ f(x), find h′(1). graph of g(x) at the point x = 2.

A) 3 B) 6 C) 15 D) 18 E) 20 1
A) − 1 B) − C) 2 D) 1 E) 0
4 2

8. Find the shortest distance between the curve


4. f(3x – 5) = 2x2 + x – 1 is given. Find the value of 4
y= and the origin (0, 0).
the expression f ′(1) + f(1). x

A) 10 B) 12 C) 14 D) 16 E) 18 A) 8 B) 4 C) 2 D) 4ñ2 E) 2ñ2

176 Derivatives
1 d2
9. f(x) = |x – 3| + sgn(x – 1) + a x + b is given. 13. Find (sin x + cos x)2.
2 dx2
Find the derivative of the function at the point A) 2(cos x – sin x) B) 2(sin x – cos x)
1 2
C) sin x – cos x 2
D) 2 cos 2x
x= .
2
E) –4 sin 2x
A) does not exist B) 0 C) 1

D) 2 E) 18

2
10. Find d 2 (sin e x ). 14. If the parametric curve is given by the equations
dx
d2 y
2x x x x x x = t3 – 2t, y = t3 – 3t, find .
A) e sin e B) e (cos e – sin e ) dx2 t =1

C) ex(cos ex – sin ex) D) ex(sin ex – cos ex) A) 20 B) 12 C) 6 D) –20 E) –30


E) ex(cos ex – exsin ex)

15. The implicit function e2xy – 4x2 + y3 + 7 = 0 is


x 3
11. f ( x) = − is given. Find f ′(9). dy
3 x given. Find .
dx t=0

A) 9 B) 3 C) 1 D) 1 E) 1
3 6 9 A) 1 B) 2 C) – 1 D) – 1 E) – 2
3 3 4 3 3

x2 − 3x + 3 df (2)
12. f ( x) = ln( 2
) is given. Find . 16. f(x) = (x + 2)x is given. Find the derivative of the
x − x+ 4 dx
function at the point x = –1.
1 3 5
A) B) e C) ln3 D) E)
2 2 2 A) –2 B) –1 C) 0 D) 1 E) 18

Chapter Review Test 1B 177


CHAPTER REVIEW TEST 1C
4
16+ x − 2
5. y
1. Find lim .
x→0 x l
M
2
1 1 1
A) B) 1 C) 1 D) E) y = f(x)
2 4 8 16 32
–3 3 x

A is tangent to the curve y = f(x) at the point M(3, 2).


f ( x)
If h( x) = , find h′(3).
x
⎧ x2 + 2, x ≤ 1
2. f ( x) = ⎪⎨ is given. 2 5 1 1
⎪⎩2 x +1, x > 1 A) B) − C) − D) E) 4
9 9 9 3 3
Find the derivative of the function at x = 1.

A) 0 B) 1 C) 2 D) 3 E) does not exist

6. f(x + 2) = ex ⋅ g(x2 + 1) and g(1) = 5 are given.


Find the value of f ′(2).

A) 2 B) 3 C) 4 D) 5 E) 7

x+ 2 2 df (3)
3. f ( x) = ( ) is given. Find .
x−2 dx
7. Find the slope of the normal line to the function
A) –300 B) –200 C) –150 D) –90 E) –40
π
f(x) = sin(cos 5x) at the point x = .
10

1 1 2
A) − 4 B) − C) D) E) 4
5 5 5 5 5

4. f(x) = |x2 + 3x – 4| + ax2b + sgn(x2 – 1) is given.


At which one of the following points does the 8. f(x) = (x – 1)2 ⋅ (2x – t) and f ′′(0) = 0 are given.
derivative of the function exist? Find the value of t.

A) 1 B) 0 C) –1 D) –2 E) –3 A) 4 B) 2 C) 0 D) –2 E) –4

178 Derivatives
d 2 ( x3 ⋅ e x ) 13. f(x) = ln(1 – x) is given. Find the derivative of
9. Find e − x ⋅ .
dx2 order n.
A) x3+3x2+3x B) x3+3x2+6x C) x3+3x2+9x
( −1)n n! ( −1)n +1( n – 1)!
3 2 3 2 A) B)
D) x +6x +6x E) x +9x +3x ( x − 1)n ( x − 1)n

( −1)n +1( n − 1) ( −1)2 n +1( n − 1)!


C) D)
( x − 1)n ( x − 1)2 n +1
2 n −1
E) ( −1) ( n2 n+1)!
( x − 1) −1
10. f ( x) = 2+ x is given. Find f ′(4).
1 1
A) 1 B) 4 C) D) 1 E)
2 4 16

14. The tangent line to the curve y = x3 at the point


A(2, 8) intersects the curve at another point
B(x0, y0). Find x0.
3 5
A) − B) C) –3 D) –4 E) –5
2 2
11. Which one of the following is correct for the
x3
tangent lines to the curve y = at the points
| x|
x = a and x = –a?

A) They are perpendicular to each other. 15. If the parametric curve is given by the equations
B) They are parallel to each other. dy
x = sin(ln t), y = etln t, find .
C) The angle between them is 30°. dx t =1

D) They are parallel to x-axis. e 3 1


A) e B) 2e C) D) E)
E) They are parallel to y-axis. 2 2 2

16. The implicit function sin (xy) + cos (xy) = 0 is


1 dy
df ( ) given. Find .
4 . dx
12. f ( x) = (arcsin 2 x ) is given. Find
2

dx y
A) 1 B) − C) 2xy
π 2
π 2 x
A) B) C) π2 D) π E) 2π
4 2 D) ycos (xy) E) xcos x – ycos y

Chapter Review Test 1C 179


CHAPTER 6

APPLICATION OF
DERIVATIVE
0
A. THE INDETERMINATE FORM –
0
Guillaume de Let us consider the following limit where both f(x) and g(x) approach to zero as x → a:
L'Hospital f ( x)
lim
(1661-1704) x → a g( x)
0
If we substitute x = a in this limit, we obtain a fraction of the form , which is a
0
meaningless algebraic expression. This limit may or may not exist and is called an
0
indeterminate form .
0
French mathematician solved a From earlier studies you have learned to calculate such limits by using the limit theorems.
difficult problem posed by Pascal
at age 15. He published the first In this section, we will discuss a very powerful method known as L’Hospital’s Rule. This rule
book ever on differential calculus
0
gives a connection between derivatives and limits of the indeterminate form .
“L'Analyse des Infiniment Petits 0
pour l'Intelligence des Lignes
Courbes” (1696). In this book,
L'Hospital introduced L'Hospital's
rule. Within the book, L'Hospital L’HOSPITAL’S RULE
thanks Bernoulli for his help. An
earlier letter by John Bernoulli Let the functions f and g be differentiable on an open interval that contains the point a.
gives both the rule and its proof,
so it seems likely that it was f ′( x)
Bernoulli who discovered the
Suppose that lim f(x) = lim g(x) = 0 and lim exists. Then,
x→a x→a x→ a g′( x)
rule. L'Hospital’s name is spelled
both “L'Hospital” and “L'Hôpital”,
the two being equivalent in f ( x) f ′( x)
lim = lim = L.
French spelling. x→ a g( x) x→ a g′( x)

Example 1 Find lim


x →0
sin x
x
.

Solution Since lim sin x = 0 and lim x = 0, we can apply L’Hospital’s Rule.
x→0 x→0

sin x (sin x)′ cos x


lim = lim = lim = cos 0 =1
x→0 x x → 0 ( x)′ x → 0 1

Note
Using L’Hospital’s Rule, differentiate both the numerator and the denominator seperately. Do
not apply the Quotient Rule.

181 Derivatives
Example 2 Find lim
x→ 4
x–4
x2 – 4
.

Solution If we attempted to use L’Hospital’s Rule, we would get


x–4 ( x – 4)′ 1 1
lim 2 = lim 2 = lim = .
x→ 4 x – 4 x → 4 ( x – 4)′ x→ 4 2 x 8
This is wrong!
x–4 0
Since lim 2 does not give the indeterminate form , we cannot apply L’Hospital’s Rule
x→ 4 x – 4
0
here. Let us find the correct answer:
x–4 4–4 0
lim 2
= 2 = =0
x→ 4 x – 4 4 – 4 12

Note
0
Before applying L’Hospital’s Rule, verify that we have the indeterminate form .
0

Example 3 Find lim


x → –1
x3 + x + 2
x +1
.

x3 + x + 2 (–1)3 +(–1)+ 2 0 0
Solution lim = = ( form; appl y the r ule)
x → –1 x +1 –1+1 0 0
x3 + x + 2 ( x3 + x + 2)′ 3 x2 +1
lim = lim = lim = 3 ⋅ (–1) 2 +1= 4
x → –1 x +1 x → –1 ( x +1)′ x → –1 1

Example 4 Find lim


x →1
ln x
x2 – 1
.

ln x 0 0
Solution lim = ( form; apply the rule )
x →1 x2 – 1 0 0
1
ln x (ln x)′ 1
lim 2 = lim 2 = l im x =
x →1 x – 1 x →1 ( x – 1)′ x →1 2 x 2
Example 5 Find lim
x →–1
2x + 3 – 1
x +5 – 2
.

0
Solution We have the indeterminate form . So, we can use L’Hospital’s Rule:
0
2
2x + 3 – 1 ( 2 x+ 3 – 1) ′ 2 x+5 2 ⋅ 4
lim = lim = lim 2 2 x + 3 = lim = = 4.
x → –1
x +5 – 2 x → –1
( x +5 – 2)′ x → –1 1 x → –1
2 x+ 3 1
2 x +5
Note
f ′( x) 0
If lim is still indeterminate form , we use L’Hospital’s Rule again.
x→ a ′
g ( x) 0

f ( x) ′′
f (x)
That gives lim = lim .
x → a g′( x) x → a g ′′( x )
0
In fact, whenever L’Hospital’s Rule gives , we can apply it again until we get a different result.
0

Example 6 Find lim


x→0
x – sin x
x2
.

x – sin x 0 0
Solution lim = ( form; apply the rule)
x→0 x2 0 0
x – sin x ( x – sin x)′ 1 – cos x 1 – cos 0 0 0
lim = lim = lim = = ( form; apply the rule agai n)
x→0 x 2 x → 0 ( x )′
2 x → 0 2x 2 ⋅0 0 0

1 – cos x (1 – cos x) ′ sin x 0


lim = lim = li m = =0
x→0 2x x→0 (2 x)′ x→0 2 2

7
x
Example Find lim e – ex2 .
x →1 ( x – 1)

e x – ex 0 0
Solution lim = ( form; apply the rule)
x →1 ( x – 1)2 0 0
e x – ex ( e x – ex)′ ex – e 0
lim = lim = lim ( form; apply the rule again)
x →1 ( x – 1)2 x →1 [( x – 1)2 ]′ x →1 2( x – 1) 0
ex – e ( e x – e )′ ex e
lim = lim = lim =
x →1 2( x – 1) x →1 [2( x – 1)]′ x →1 2 2

Check Yourself 1
Find the following limits:
x3 – 8 x2 – 8 x ex – 1 sin x – 1
1. lim 2. lim 3. lim 4. lim
x → 2 x2 – 4 x–4 cos 2 x +1
x→ 4 x →0
ln x +1 x →π/2

Answers
1. 3 2. 6 3. 2 4. –1/4

183 Derivatives

B. THE INDETERMINATE FORM –


L’Hospital’s Rule is also valid for the indeterminate form . It is expressed as follows:

f ′( x)
Suppose that lim f(x) = ±∞, lim g(x) = ±∞ and lim exists.
x→a x→a x → a g′( x)
f ( x) f ′( x)
Then, lim = lim .
x→ a g( x) x → a g′( x)

Example 8 Find lim


x2 – 3x +5
x →∞ 7+ 2 x – 3 x2
.

x2 – 3x +5 ∞ ∞
Solution lim = ( form; apply the rule)
x →∞ 7+ 2 x – 3 x2 –∞ ∞

x2 – 3x +5 ( x2 – 3 x+5) ′ 2x– 3 ∞
lim 2
= lim = lim ( still f orm; apply the ru le)
x →∞ 7+ 2 x – 3 x x→∞ (7+ 2 x – 3 x2 )′ x→∞ 2 – 6 x ∞

2x – 3 (2 x – 3)′ 2 1
lim = lim = lim =–
x →∞ 2 – 6x x →∞ (2 – 6 x) ′ x→∞ –6 3

Example 9 Find lim


ex + 2x
x →∞ e 2 x – 3
.

ex + 2x ∞ ∞
Solution lim = ( form; apply the rule)
x →∞ e 2 x – 3 ∞ ∞
ex + 2x ( e x + 2 x)′ ex + 2 ∞
lim = lim = lim ( stil l form; apply the rule)
x →∞ e 2 x – 3 x →∞ ( e 2 x – 3)′ x →∞ 2 ⋅ e 2 x ∞

ex + 2 ( e x + 2)′ ex 1 1
lim = lim = lim = lim = =0
x →∞ 2 ⋅ e 2 x x →∞ ( 2 ⋅ e 2 x )′ x →∞ 4 ⋅ e2 x x →∞ 4 ⋅ e x ∞

1
Example 10 Find lim+ x .
x →0 ln x

1
Solution When x → 0+, → ∞ and ln x → −∞. So, we can apply L' Hôs pital's Rule.
x
1 1 1
( )′ – 2
1
lim x = lim+ x = lim+ x = lim+ − = – ∞.
x → 0 + ln x x → 0 (ln x)′ x→0 1 x→0 x
x

Applications of the Derivative 184


Note
L’Hospital’s Rule cannot be applied directly to the indeterminate forms ∞ ⋅ 0 and ∞ – ∞.
0 ∞
But it may be possible to convert them into the form or into the form .
0 ∞

Example 11 x →∞
1
Find lim x sin .
x
This limit leads to the form ∞ ⋅ 0, but we can change it to
Solution
0 1
the form by writing x = .
0 1
x

f g 1
f ⋅g= or f ⋅ g = sin
1 1 1 x=0
lim x ⋅ sin = lim
g f x →∞ x x→∞ 1 0
x
0
Since we have the form , we can apply L’Hospital’s Rule.
0
1 1 1 1
sin (sin )′ ( − 2 ) ⋅ cos
lim x = lim x = lim x x = lim cos 1 =1
x →∞ 1 x →∞ 1 x→∞ 1 x→∞ x
( )′ − 2
x x x

Example 12 1
Find lim( − x
x→0 x
1
e −1
).

Solution We have the indeterminate form ∞ – ∞, but we write


1 1 ex − 1 − x 0 0
lim( − x ) = lim = ( form; apply the rule)
x→0 x e −1 x → 0 x ⋅ ( e x − 1) 0 0

ex − 1 − x ( e x − 1 − x)′ e x −1 0 0
lim = lim = lim x = ( form; apply the rul e again )
x→0 x ⋅ ( e − 1) x→0 [ x ⋅ ( e − 1)]′ x→0 e − 1 + x ⋅ ex 0
x x
0

ex − 1 ( e x − 1)′ ex 1
lim x x
= lim x x
= lim x x x
= .
x→0 e −1+ x ⋅ e x → 0 ( e − 1 + x ⋅ e )′ x → 0 e + e + x ⋅e 2

Check Yourself 2
Find the following limits:

2 x2 – 5 x +7 x3 – 3x +5 x 1 1
1. lim 2. lim 3. lim 4. lim xe − x 5. lim( − )
x →∞ 3x + 4 x →∞ ex x →∞
ln x
x →∞ x→0 x sin x
Answers
1. ∞ 2. 0 3. ∞ 4. 0 5. 0

185 Derivatives
EXERCISES 2 .1
0 ∞
A. The Indeterminate Form – B. The Indeterminate Form ––
0 ∞

1. Find the following limits: 2. Find the following limits:

x x2 + x +1 9 – x2
2 sin a. lim b. lim
x – 4x + 4 3 x →∞ 3x3 + 4
a. lim 2 b. lim 2
x→ – ∞ x – 2 x – 3
x→ 2 x + x – 6 x →0 2 x

ex – 1 x2 + 4
3 c. lim d. lim
1+ 3x – 1 4x + 4 x →∞ e x +1 x →∞ x
c. lim d. lim
x →0 x x → –1 5 x2 +6 x +1
ln x ex
e. lim f. lim
1 – cos x x →∞ x5 x →∞ x + x2
e. lim f. lim 1 – x
x→0 x2 x →1 ln x

ln(1+ e x )
g. lim h. lim ln x
x2 – 8 x
x →∞ 2x x →∞
x
g. lim h. lim sin(2 x + 4)
x→ 2 x–2 x → –2 x2 – x – 6
cot 3 x 2
i. lim j. lim x + cos x
x →0 cot 5 x x →∞ 3 x2
2 – 3 x+ 6 3 3
i. lim j. lim 3 – x
x→ 2 x2 – 4 x→ 3
x– 3

2 x2 + 3 x – 2
k. lim l. lim sin x – sin a
x →1/ 2 6x – 3 x→ a x–a Mixed Problems
3. Find the following limits:
e2 x – 1 2
m. lim n. lim tan( x – 4) 3
tan x
a. lim x 3 + x − 2
2
x → 0 ln( x + x +1) x→ 2 4 – x2 b. lim
x→1 2 x − 3 x + 1 x→
π 1 + tan x
2

5x x
x x cos + sin
o. lim 3 – 7 p. lim 6 3 › c.
1
lim x ln(1 + ) › d. lim(tan x − sec x)
x →0 x x →π x–π x →∞ x x→
π
2

arctan 3 x ecos x – 1 6
x −1 πx
q. lim r. lim e. lim sin x › f. lim
x →0 arctan 4 x x →π / 2 cos x tan
x→0 3x x →1 2 2

x π
arctan( ) –
x2 / 3 – 4 2 4 1
s. lim t. lim › g. lim( − csc x) › h. lim( x2 + 3 x − x )
x→8 x–8 x→ 2 x–2 x→0 x x→∞

Applications of the Derivative 186


A. INTERVALS OF INCREASE AND DECREASE
In this section, we will first briefly review the increasing
y
and decreasing functions and then discuss the y=x2
relationship between the sign of the derivative of a
function and the increasing and decreasing behavior of
the function.
Recall the graph of the function f(x) = x2. As we move
x
from left to right along its graph, we see that the graph of
f falls for x < 0 and rises for x > 0. The function f is
2
the graph of f(x) = x

said to be decreasing on (–∞, 0) and increasing on (0, ∞).

Definition increasing and decreasing functions

A function f is increasing on an interval I if f(x) increases as x increases on I. That is, for any
x1 < x2 on I, f(x1) < f(x2).
Similarly, f is decreasing on an interval I if f(x) decreases as x increases on I. That is, for any
x1 < x2 in I, f(x1) > f(x2).

a decreasing function an increasing function

Note
We refer to a function as increasing or decreasing only on intervals, not at particular points.

187 Derivatives
We now learn how the first derivative can be used to determine where the function is
increasing or decreasing. Let us look at the following graphs.
y y

y = f(x) y = f(x)
a
x

0° < α < 90°


a b x a b x
positive slope
f is increasing f is decreasing
positive slopes (f′(x) > 0) negative slopes (f′(x) < 0)

Observe that the function f is increasing on the interval (a, b) and the tangent lines to the
graph of f have positive slope on that interval. We know that the slope of each tangent line is
given by the derivative f ′(x). Thus, f ′(x) must be positive on (a, b).
Similarly, we expect to see a decreasing function when f ′(x) is negative. These observations
lead to the following important theorem.

Theorem
Let f(x) be a differentiable function on the interval I.
a. If f ′(x) > 0 for all the values of x on I, then f(x) is increasing on the interval I.
b. If f ′(x) < 0 for all the values of x on I, then f(x) is decreasing on the interval I.

Note
According to the theorem above, when we are asked to determine the intervals of increase
and decrease for a given function, we must examine the sign of the derivative of the
function. To do this, we shall construct the sign chart of the first derivative. We assume that
you are familiar with constructing the sign chart of a function from your earlier studies.

Example 13 Find the intervals where the function f(x) = x2 – 4x + 3 is increasing and where it is
decreasing.

Solution Let us construct the sign chart of f ′(x). y

f ′(x) = 2x – 4 and x = 2 is a root of f ′(x) = 0 3


x –¥ 2 +¥

f ¢(x) – +
decreasing increasing 2
f (x)
( ) ( ) 1 3 x

the graph of
From the chart, f ′(x) > 0 when x > 2 and f ′(x) < 0 when x < 2.
f(x) = x2 – 4x + 3
So, f is increasing on (2, ∞) and decreasing on (–∞, 2).
It would also be true to say that f is increasing on [2, ∞) and decreasing on (–∞, 2].

Applications of the Derivative 188


Note
As in the previous example, we use an “up arrow” ( ) for the intervals where the function
is increasing and a “down arrow” ( ) for the intervals where the function is decreasing.

Example 14 For what values of x is the function f(x) = (x – 1)3 either increasing or decreasing?

Solution f ′(x) = 3(x – 1)2. y


2
f ′(x) ≥ 0 for any of the values of x because (x – 1) > 0.
We conclude that f is always increasing for all the values of x.
The graph of f is shown in the figure. 1 x
-1
Note that when x = 1 we have f ′(x) = 0.
But that does not affect the increase since it is just a point.
the graph of (fx) = (x – 1)3

Example 15 3
Determine where f(x) = ex – 3x is increasing.

3
Solution f ′(x) = ex – 3x ⋅ (3x2 – 3)
3
Since ex – 3x > 0 for all the values of x, it is enough to check the sign of 3x2 – 3.
3x2 – 3 = 0 ⇒ 3(x – 1)(x + 1) = 0 ⇒ x = –1 and x = 1.

x –¥ –1 1 +¥

f ¢(x) + – +

f(x)

The chart suggests that f(x) is increasing for x < –1 and x > 1.

Example 16 For what values of a is the function


f(x) = ax3 – 2x2 + 2x – 3 increasing for all real numbers?

Solution Since f is increasing for all real numbers, f ′(x) > 0.


f ′(x) = 3ax2 – 4x + 2 > 0
This is possible only if 3a > 0 and ∆ < 0
2
Given ax + bx + c = 0, 3a > 0. So, a > 0. (1)
∆ = b2 – 4ac. 2
∆ = (– 4)2 – 4 ⋅ 3a ⋅ 2 = 16 – 24a < 0. So, a > . (2)
3
2
By (1) and (2), we have a > .
3

189 Derivatives
Example 17 For what values of m is f ( x) =
mx – 2
x+ 3
always decreasing in its
domain?

Solution Since f is always decreasing, f ′(x) < 0 for all the values of x except
x = –3.
m( x + 3) – ( mx – 2) 3 m + 2
f ′( x) = = < 0.
( x + 3)2 ( x + 3)2

Then, we have 3m + 2 < 0 because (x + 3)2 is always positive.


2
Thus, m < – .
3

18
y
EXAMPLE The graph of the function f is given on the interval (a, b). State
whether each of the following functions is increasing or decreasing y = f (x)
on (a, b).

a. g(x) = x2 – f(x) b. h(x) = f 2(x) c. f ( x)


m( x) =
x
a b x

Solution From the graph, we conclude that x < 0 and f(x) > 0 on (a, b). Also, we have f ′(x) > 0
because f is increasing on (a, b). Now let us find the derivative of each function.

a. g′(x) = 2x – f ′(x) < 0. So g is decreasing on (a, b).


b. h′(x) = 2 ⋅ f (x) ⋅ f ′(x) > 0. So h is increasing on (a, b).

c. m′(x) = f ( x) ⋅ x2− f ( x) < 0. So m is decreasing on (a, b).
x

Check Yourself 3
1. Find the intervals where each function is increasing or decreasing.
1
a. f(x) = x3 – 3x2 + 6 b. f ( x) = c. f(x) = ln x
3x + 4
kx +1
2. The function f ( x) = is always increasing in its domain. Find k.
x +1
Answers
1. a. increasing on (–∞, 0) and (2, ∞), decreasing on (0, 2)
4
b. decreasing on ⎛⎜ −∞, − 4 ⎞⎟ and ⎛⎜ − , ∞ ⎞⎟
⎝ 3 ⎠ ⎝ 3 ⎠
c. increasing on (0, ∞)
2. k > 1.

Applications of the Derivative 190


B. MAXIMUM AND MINIMUM VALUES
1. Absolute and Local Maximum and Minimum
In many applications we need to find the largest or the smallest value of a specified quantity.
Here are a few examples:
• What is the shape of a container that minimizes the manufacturing costs?
• At what temperature does a certain chemical reaction proceed most rapidly?
• Which path requires the least time to travel?
These problems can be reduced to finding the maximum or minimum value of a function.
Let us first explain what we mean by maximum and minimum values.

Definition absolute maximum and minimum


A function f has an absolute maximum at c if f(c) ≥ f(x) for all the values of x in its domain.
Similarly, f has an absolute minimum at c if f(c) ≤ f(x) for all the values of x in its domain.

Note
Here is some terminology: If a function f has a maximum (or minimum) at x = c, then we
say that f(c) is a maximum (or minimum) value of f and (c, f(c)) is a maximum (or
minimum) point of f.
An extremum of a function is either a maximum or minimum value of that function.

The figure on the right shows the graph of a y

function f with absolute maximum at x = a f(a)

and absolute minimum at x = d. Note that


(a, f(a)) is the highest point on the graph y = f(x)

and (d, f(d)) is the lowest point.


In the same graph, if we consider only f(d)
I1 I2
the values of x sufficiently near b (for ( ) ( )
a b c d x
example, in the interval I1), then f(b) is the
smallest of those values of f(x). In other words, no nearby points on the graph of f are lower
than the point (b, f(b)). To define such points, we use the word “local”. So, we say that the
function f has a local minimum at the point x = b.
Similarly, f has a local maximum at x = c because f(c) is the largest value of f(x) in the
interval I2. We see that no nearby points on the graph are higher than the point (c, f(c)). We
now state the formal definition:

191 Derivatives
Definition local maximum and minimum
A function f has a local maximum at c if f(c) ≥ f(x) for all the values of x in an interval I
containing c.
Similarly, f has a local minimum at c if f(c) ≤ f(x) for all the values of x in an interval I
containing c.
The figure on the right illustrates some y absolute and
local max
local and absolute extrema of a function
local max
f with the domain [a, e]. We see that f has not absolute
a local maximum at x = c, and a local
minimum at x = b and x = d. Also, f has
local min
an absolute minimum at x = b and an not absolute

absolute maximum at x = e. Observe b


that the absolute minimum is also local, a c d e x
but the absolute maximum is not local y = f(x)
because it occurs at the endpoint x = e.
absolute and
local min

Note
1. A function has at most one absolute maximum and one absolute minimum. But it may
have more than one local maximum or minimum.
2. An absolute extremum of a function is either a local extremum or an endpoint.

Check Yourself 4
1. Explain the difference between an absolute maximum y
and a local maximum. y = g(x)

2. The graph of a function with the domain [a, f] is given


on the right. For each of the points from a to f, state
whether the function has a local maximum or
minimum, or an absolute maximum or minimum.

a b c d e f x
Answers
1. absolute max.: f(c) ≥ f(x) for all the values of x in the domain of f.
local max.: f(c) ≥ f(x) for all the values of x in an interval I containing c.
2. local max. at x = b, x = d
local min. at x = c, x = e
absolute max. at x = d
absolute min. at x = a.

Applications of the Derivative 192


2. Finding the Local Extrema
We now learn how the first derivative can be used to locate y y = f(x)
the local extrema. We first consider the functions that have
derivatives at the local extremum points. The figure on the
right shows the graph of a function f with a local
maximum at x = a and a local minimum at x = b. Observe
that the tangent lines to the graph at these points are a b x
horizontal (parallel to the x-axis) and therefore each has
slope 0. Remember that the slope of the tangent line is given by the derivative. So, we say that

f ′(a) = 0, f ′(b) = 0.
This analysis reveals an important characteristic of the local extrema of a differentiable
function.

Theorem
If f has a local extremum at c, and f ′(c) exists, then f ′(c) = 0.

Note y y = f(x)
The converse of this theorem is not true in general.
That is, when f ′(c) = 0, f does not necessarily have a
maximum or minimum at x = c. For example, consider
x
the function f(x) = x3. Here, f′(x) = 3x2, so f′(0) = 0. But,
f has neither a local maximum nor a local minimum at
x = 0.

Example 19 The function f(x) = 2x3 – mx + 5 has a local minimum


at x = 1. Find m.

Solution Since f(x) is a polynomial function, it is differentiable


everywhere. By the theorem above, we have f ′(1) = 0.
f ′(x) = 6x2 – m
f ′(1) = 0
6 ⋅ 12 – m = 0
m=6

193 Derivatives
Example 20 Find the local extrema of the function f(x) = |x|.

Solution Let us plot the graph of f.


y
⎧⎪ x, x ≥ 0 y = f(x)
f ( x) =| x |= ⎨
In some books, an ⎪⎩− x, x < 0
extremum that occurs
at a point where the We see that f has a local minimum at x = 0.
x
derivative does not But there is no horizontal tangent there.
exist is called a singular
point. In fact, there is no tangent line at all since f ′(x) is not defined at x = 0.

Note
The function f(x) = |x| shows that a local extremum of a function may exist at which the
derivative does not exist. As a consequence, we say that the local extrema of any function f
occurs at the points c where f ′(c) = 0 or f ′(c) does not exist. Such points are given a special
name.

Definition critical point


The value c in the domain of f is called a critical point if either
1. f ′(c) = 0, or
2. f ′(c) does not exist.

Example 21 Find the critical points of f(x) = 2x3 – 9x2 + 12x – 7.

Solution The derivative of f is f ′(x) = 6x2 – 18x + 12 = 6(x – 1)(x – 2).


Since f ′(x) is defined for all the values of x, the only critical points are the roots of f ′(x) = 0.
Therefore, x = 1 and x = 2.

x2
Example 22 Find the critical points of f ( x) =
x –1
.

Solution The Quotient Rule gives

2 x( x – 1) – x2 ⋅ 1 x( x – 2)
f ′( x) = = .
( x – 1)2 ( x – 1)2
Since x = 0 and x = 2 are the roots of f ′(x) = 0, they are critical points. Next, observe that
f ′(x) does not exist when x = 1. However, since f is not defined at that point, the point
x = 1 is not a critical point.

Applications of the Derivative 194


23
y
Example In the figure on the right the graph of a y = f(x)
function f with critical points at x = a, b, c, d,
and e are shown.
a. State why these points are critical.
b. Classify each of them as a local maximum,
a local minimum, or neither.
a b c d e x

Solution a. Observe that there are horizontal tangents at the points x = c, d, and e, so f ′(x) = 0 at
these points. Next, f ′(x) does not exist at x = a because the tangent line at this point is
vertical. Finally, since there is a corner at x = b, f ′(x) does not exist there.

b. From the graph of f, we say that f has a local maximum at x = d, and a local minimum at
x = b and x = e. Note that f ′(c) = 0 and f ′(a) does not exist, and f has no local extrema
at these points. We conclude that not every critical point gives rise to a local extrema.

Check Yourself 5
Find the critical points of the following functions.

1. f(x) = x3 – 3x + 4 2. f(x) = 1 – ñx 3. f(x) = x2ln x


Answers
1
1. –1, 1 2. 0 3.
e

195 Derivatives
3. The First Derivative Test
So far we have learned that any extremum of a function f must occur at any critical point of
f. In the previous example we have seen that not every critical point is a maximum or a
minimum.Therefore, we need a test that helps us classify critical points as local maximum,
local minimum, or neither.
Suppose that the function f is continuous at c and that f is defined on some open interval
containing c.

f¢>0 + – f¢<0
If f is increasing on the left of c and
+ –
decreasing on the right, then f should inc. dec. + –
have a local maximum at x = c. + –

c x

y
If f is decreasing on the left of c and + –
increasing on the right, then f should + –
dec. inc.
have a local minimum at x = c.
+ –
+ –
f¢<0 f¢>0

c x

y y
+ –
If f is increasing on both sides or + f¢>0 –
decreasing on both sides, then f + f¢<0 –
+ –
should have neither a local maximum f¢>0 f¢<0
+ –
nor a local minimum at x = c. + –
c x c x

Moreover, we know that f(x) is increasing where f ′(x) > 0 and decreasing where f ′(x) < 0.
These observations are the basis of the following test.

THE FIRST DERIVATIVE TEST

Let c be a critical point of a function f(x).


1. If f ′(x) changes from positive to negative at c, then f(x) has a local maximum at c.
2. If f ′(x) changes from negative to positive at c, then f(x) has a local minimum at c.
3. If f ′(x) does not change sign at c, then f(x) has no local maximum or minimum at c.

Applications of the Derivative 196


Example 24 Find the critical points of the function f(x) = x3 – 3x2 – 9x + 6, and classify each critical
point as a local maximum, a local minimum, or neither.

Solution f ′(x) = 3x2 – 6x – 9 = 3(x – 3)(x + 1)


x = 3 and x = –1 are the critical points (where f ′(x) = 0). Since f ′(x) is a polynomial
function, it is differentiable everywhere. Thus, we have no points c such that f ′(c) is not
defined.
x –¥ –1 3 +¥

f ¢(x) + – +

f(x)

(max) (min)
From the sign chart, f increases for x < –1 and decreases for –1 < x < 3. So, f has a local
maximum at x = –1.
Similarly, f decreases for –1 < x < 3 and increases for x > 3. So, f has a local minimum at x = 3.

Example 25 Find the local extrema of the function f(x) = x2/3 + 2.

2
2 –1 2
Solution f ′( x) = ⋅ x3 = 1
3
3 ⋅ x3
There is no root of f ′(x) = 0.
Now we will look for the values of x such
that f ′(x) is not defined but f(x) is
defined. We see that f is defined for all
the values of x but f ′ is not defined at
x = 0. So, 0 is a critical point.

x –¥ 0 +¥

f ¢(x) – +

f(x)

(min)

Thus, the first derivative test tells us that


x = 0 is a local minimum of f.

197 Derivatives
Example 26 Find the local extrema of the function f(x) = |x – 1|.

Solution If x > 1, then x – 1 > 0. So, f(x) = x – 1.


If x < 1, then x – 1 < 0. So, f(x) = 1 – x.
1, x>1
f ′(x) =
–1, x<1

Since f ′(1– ) ≠ f ′(1+), f ′(x) is not defined at x = 1.


So, x = 1 is a critical point. Furthermore, f ′(x) is not equal to zero anywhere.
For x > 1, we have f ′(x) = 1 > 0. So, f(x) is increasing on this interval.
For x < 1, we have f ′(x) = –1 < 0. So, f(x) is decreasing on this interval.

x –¥ 1 +¥

f ¢(x) – +

f(x)

(min)
Thus, f has a local minimum at x = 1.

Example 27 Find the local extrema of the function f(x) = 5x3 + 4x.

Solution f ′(x) = 15x2 + 4 is always positive. There is no real solution of f ′(x) = 0.


x –¥ +¥

f ¢(x) +

f(x)

f is increasing for all the values of x.


Since f(x) is a polynomial function, f(x) is continuous and differentiable everywhere.
Thus, f(x) has no local extrema.

Check Yourself 6
Find the local extrema of the following functions.
x2 +1
1. f(x) = 2x2 – 2x + 5 2. f(x) = 1 – x4 3. f(x) = 4. f(x) = |x2 – x|
x
Answers
1 1
1. min.: x = 2. max.: x = 0 3. max.: x = –1, min.: x = 1 4. max.: x = , min.: x = 0, x = 1
2 2

Applications of the Derivative 198


Example 28 If the function f(x) = x3 + ax2 + 15x + b has a local maximum at the point (1, 10), then
find a and b.

Solution We know that an extremum of a function must occur at a point where f ′(x) = 0 or f ′(x) does
not exist. Since f is a polynomial function, f is differentiable everywhere.
So, we have f ′(1) = 0.
f ′(x) = 3x2 + 2ax + 15
f ′(1) = 0
3 ⋅ 12 + 2a ⋅ 1 + 15 = 0
a = –9
Since the point (1, 10) is on the graph of f, we say that f(1) = 10.
f(1) = 10
13 + a ⋅ 12 + 15 ⋅ 1 + b = 10
a + b = –6
b = 3 (since a = –9)

Example 29 The graph of the derivative of the function f(x) is shown in y


y = f ¢(x)
the figure. Find the intervals where f(x) is increasing or
decreasing and find the local extrema of f. -2 -1
1 x

Solution We know that f(x) is increasing when f ′(x) > 0. f ′(x) > 0 means that the graph of f (x) must
be above the x-axis. In the figure f ′(x) > 0 for x > 1. So, f(x) is increasing for x > 1. Similarly,
f(x) is decreasing when the graph of f ′(x) is below the x-axis. So, f(x) is decreasing for x < 1.

x –¥ 1 +¥

f ¢(x) – +

f(x)

(min)

From the chart, f has a local minimum at x = 1.

199 Derivatives
Example 30 For what values of m does the function f(x) = x3 – (m – 1)x2 + 3x – 2 have no local extrema?

Solution Since f has no extrema, there must be no root of f ′(x) = 0.

f ′(x) = 3x2 – 2(m – 1)x + 3

The equation 3x2 – 2(m – 1)x + 3 = 0 must have no root.


A quadratic equation has
1. two solutions when We need ∆ < 0:
∆ > 0.
2. one solution when [–2(m – 1)]2 – 4 ⋅ 3 ⋅ 3 < 0
∆ = 0.
3. no solution when 4(m2 – 2m + 1) – 36 < 0
∆ < 0.
4(m2 – 2m – 8) < 0

4(m – 4)(m + 2) < 0

Let us construct a chart to solve the above inequality:

–2 4
+ – +

So –2 < m < 4.

Check Yourself 7
1. Find the local minimum value of y = e2x – 4ex – 6x.

2. The function f(x) = x3 – 3x2 – 9x + a has a local maximum value of 10. Find a.

3. The graph of the derivative of the function f(x) is given. Find the local extrema of the
function f.
y

–2 1 2 4
–1 3 x

y = f ¢(x)

Answers
1. –3 – 6ln 3 2. 5 3. max.: x = 0, x = 4, min.: x = –2, x = 2

Applications of the Derivative 200


4. Finding the Absolute Extrema
In most types of maximum-minimum problems, we are more interested in the absolute
extrema rather than the local extrema. Recall that the absolute extrema of a function are the
largest and the smallest values of that function in its whole domain. The following figures
show the graphs of several functions and give the maximum and minimum values of the
functions if they exist.

y y y

x
x
–1
x

minimum value is – 1 minimum value is 0 no minimum value


maximum value is 1 no maximum value no maximum value

We have seen that some functions have absolute extrema, whereas other do not. In what
conditions does a function have both the absolute maximum and the absolute minimum?
The following theorem answers this question.

Theorem
If a function f is continuous on a closed interval [a, b], then f has both an absolute maximum
and an absolute minimum on [a, b].

The above theorem guarantees the existence of the absolute extrema of a continuous
function on a closed interval [a, b]. Moreover, we know that each absolute extremum can
occur either at a critical point in the interior of [a, b] or at an endpoint of the interval. The
following steps give a useful method for finding the absolute extrema of a continuous
function on [a, b].

CLOSED INTERVAL METHOD

1. Find the critical points of f on the interval [a, b].


2. Evaluate f(x) at each critical point.
3. Evaluate f(a) and f(b).
4. The largest of the values of f found in Steps 2 and 3 is the absolute maximum, the
smallest of these values is the absolute minimum.

201 Derivatives
Example 31 Find the absolute extrema of the function f(x) = x2 – 4x + 3 on [0, 3].

Solution Let us apply the Closed Interval Method step by step:


1st Step: To find the critical points of f, we must solve f ′(x) = 0 and also find where f ′(x)
does not exist.
f ′(x) = 2x – 4 = 0 gives x = 2.
The domain of the function is [0, 3]. So, x = 2 is in the domain.
And there is no point where f ′(x) is not defined.
Thus, the only critical point on [0, 3] is x = 2. y
max
3
nd 2
2 Step: f(2) = 2 – 4 ⋅ 2 + 3 = –1
3rd Step: f(a) = f(0) = 02 – 4 ⋅ 0 + 3 = 3
f(b) = f(3) = 32 – 4 ⋅ 3 + 3 = 0 1 2 3
rd x
4 Step: fmax[0, 3] = 3, fmin[0, 3] = – 1. -1 y = f (x)
The graph of f confirms our results. min

Example 32 Find the maximum and minimum values of the function f(x) = 2x3 + 12x2 + 18x + 6 on
the closed interval [–2, 0].

Solution f ′(x) = 6x2 + 24x + 18 = 6(x + 1)(x + 3)


f ′(x) = 0 when x = –1 and x = –3.
But x = –3 is outside the interval [–2, 0]. So, we do not take it.
The only critical point is x = –1.
Additionally, we should consider the endpoints of the interval [–2, 0].
Now, we evaluate f(x) at x = –1, –2, and 0:
f(–1) = –2
f(–2) = 2
f(0) = 6
fmax[–2, 0] = 6, fmin[–2, 0] = –2.

Notation
fmax[a, b] denotes the maximum value of the function f on the interval [a, b].
fmin[a, b] denotes the minimum value of the function f on the interval [a, b].

Applications of the Derivative 202


Example 33 Find the absolute extrema of the function f(x) = 2 – |x – 1| on [–1, 3].

Solution If x > 1, then x – 1 > 0. So, f(x) = 2 – (x – 1) = 3 – x.


If x < 1, then x – 1 < 0. So, f(x) = 2 – (1 – x) = x + 1.

⎧ 3 – x, x>1

We have f(x) = ⎨ 2, x=1.

⎩ x + 1, x<1
We say that f ′(1) does not exist because f ′(1–) = 1 and f ′(1+) = –1 are not equal.
So x = 1 is a critical point.
Next we consider the endpoints of the interval [–1, 3]. So, we should find the values of f for the
points x = 1, –1, and 3.
f(1) = 2
f(–1) = 0
f(3) = 0
fmax[–1, 3] = 2, fmin[–1, 3] = 0.

Example 34 x 2
Find the maximum and minimum values of the function f ( x) = + on [–3, 3].
2 x

1 2 x2 – 4
Solution f ′( x) =– 2=
2 x 2 x2
x = –2 and x = 2 are the roots of f ′(x) = 0. So, they are critical points. Next, f ′(x) does not
exist when x = 0, but f is not defined at this point; so 0 is not a critical point.
Now let us find the values of f for critical points and endpoints.
13 13
f (–3) = – , f ( –2) = –2, f (2) = 2, f(3) =
6 6
13 13
f max [–3,3]= fmin =[–3, 3]= –
6 6

Check Yourself 8
Find the absolute extrema of each function on the given interval.

1. f(x) = x2 – x + 2, [0, 1] 2. g(x) = x3 – 2x2 + x + 1, [–1, 2]


3. h(x) = x – ñx, [1, 4]
Answers
7
1. fmax = 2, fmin = 2. fmax = 3, fmin = –3 3. fmax = 2, fmin = 0
4

203 Derivatives
EXERCISES 2 .2
A. Intervals of Increase and Decrease 3. Find the intervals where the function
f(x) = sin x + cos x is decreasing on [0, 2π].
1. You are given the graphs of two functions.
Determine where the functions are increasing 4. Show that the function f(x) = Arctanx is increasing
and where they are decreasing. for all the values of x.
a. y b. y
5. For what values of m is the function
y = f (x) 1
f ( x) = − x3 + mx2 − 4x +1 decreasing
3
–2 2 for all real numbers?
x –3 –1 x
–1 y = f (x)
1
6. The function f(x) = ax3 – (a – 2)x2 + x
3
2. Find the intervals where each of the following
is always increasing for all the values of x. Find a.
functions is increasing or decreasing.
a. f(x) = 3 – 8x x2 − ax
7. Find the values of a, so that f ( x) = 2 is
b. f(x) = x2 + 1 x − 4x + 3
always decreasing in its domain.
c. f(x) = –x2 + 4x + 3
8. Find a, so that the function
d. f(x) = x3 + 6x
x3 f(x) = x3 – 3x2 + 3ax + 15 is increasing on
e. f(x) = – 2x2 + 2 (–∞, –2) and (4, ∞), and decreasing on (–2, 4).
3
f. f(x) = 3x4 + 4x3 – 12x2 9. Let f be an increasing function on (0, ∞). State
1
g. f(x) = whether each of the following functions are
2–x
increasing or decreasing on the same interval.
h. f(x) = x2/3
5−x a. –f(x) b. x + f(x)
i. f(x) = 1
x2 c. d. f(x2)
3
f ( x)
j. f(x) = x –1
1 10. The graph of the function f is y
k. f ( x) = 2 x − 5 given on the closed interval
a b
l. f(x) = e x2 – 4x + 3 [a, b]. State whether each of
x
ln x the following functions are
› m. f ( x) = 2 increasing or decreasing on
x
π⎞ [a, b].
› n. f ( x) = sin ⎜ x +

⎟ a. x ⋅ f(x) y = f(x)
⎝ 3⎠
2x
4 ⋅ 0.5 x b. f 2(x) + x
› o. f ( x) = 0.5 – + 3x – 2
2 ln0.5 ln0.5 c. x2 – f(x)

Applications of the Derivative 204


B. Maximum and Minimum Values 3 − 6 x − x2
⎛1⎞
› h. f(x) = ⎜ ⎟
11. For each of the points from x1 to x9, state whether ⎝2⎠
f has a local maximum or minimum, and an › i. f(x) = x2 – 3x + xln x
absolute maximum or minimum.
›› j. f(x) = sin2x + sin x
y
y = f (x)
15. The function f(x) = x3 – 9x2 + 15x + 7 is given.
Find the sum of the x-coordinates of its local
extrema.

16. The graph of y = ax2 + bx has an extremum at


x1 x2 x3 x4 x5 x6 x7 x8 x9 x (1, –2). Find the values of a and b.
12. Find the critical points of the following functions.
17. Find k, if f(x) = x3 – 2x2 – 7x + k has a local
3 2 x
a. f(x) = x + x b. f(x) = x e maximum value of 8.
1
c. f(x) = 2x – 2 d. f(x) = |x + 1|
x 18. Given that f(x) = x3 + ax2 + bx + 1 has a local
maximum at x = –1 and a local minimum at
13. y x = 2, find a and b.

19. Find the local minimum value of the function


y = f (x) f(x) = x2ex – 3ex.

20. Find the value of m, if the curve y = x3 + 2mx2 + 30


a b c d e x
is tangent to the line y = –2.
In the figure above the graph of a function f with
the critical points at a, b, c, d, and e are shown. 21. Find the relation between a and b, if the function
f(x) = ax3 + bx + c has one local maximum and
a. State why these points are critical.
one local minimum.
b. Classify each of them as a local maximum, a
local minimum, or neither.
22. For what values of m does the function
14. Find the local extrema of the following functions. 2
f(x) = mx + 1 have no local extrema?
a. f(x) = 8x + x2 x −1

b. f(x) = –x3 + 3x + 2 23. y


x3
c. f(x) = + 2x2 + 4x + 5 2
3
1 3 7
d. f(x) = (x – 1)2(x + 3)2
–2 6 x
x
e. f(x) =
x +1 y = f ¢ (x)
–3
16
f. f(x) = x2 –
x The graph of the derivative of a function f is
g. f(x) = |4 – x2| given. Find the local extrema of f.

205 Derivatives
24. Find the absolute extrema of each function on Mixed Problems
the given interval.
29. Given a parametric function y = f(x) with
a. f(x) = 2x2 – 4x + 3, [0, 2] ››
y = 2t2 + 4t + 5
b. f(x) = –x2 + 2x – 1, [–2, 2] x = t3 + t.
a. Find the intervals of increase and decrease of f.
c. f(x) = x3 – 6x, [1, 4]
b. Find the local extrema of f.
d. f(x) = 2x3 – 15x2 + 24x + 19, [0, 2]
30. y
e. f(x) = x2 – 4ñx, [0, 3] ›
f. f(x) = x5 – 5x4 + 1, [0, 5] –3 –1 2 4 6
–4 5
g. f(x) = 9x2 – x4, [–3, 3]
y = f ¢(x)
h. f(x) = 9 – x2 , [–1, 2]
1
i. f(x) = x – , [1, 3]
x The graph of the derivative of a function f is
j. f(x) = 3x , 2/3
[–1, 1] given.
a. Find the intervals of increase and decrease of f.
x–1
k. f(x) = , [0, 4] b. Find the local extrema of f.
x +1
› l. f(x) = |x2 – 3x|, [1, 4]
31. At what point does the tangent to the curve
› x3
1
› m. f ( x) = log 32 x − 3 log 22 x+8 log 2 x+1, [1, 8] y= – 2 x2 + x – 5 have the smallest slope?
3 3
› n. f(x) = 9sin x – sin 3x + 3, [–π, 0] 32. In the figure, graph of f ′(x) y
› y = f ¢(x)
is given. Given that the
25. Find the sum of the smallest value and the greatest equation f(x) = 0 has only
one root and that root is a b
x
value of f(x) = x2 – 4x + 8 on [–2, 3].
positive, plot a rough graph
of f(x).

26. If the point (1, 4) is the highest point of the graph 33. Find the range of the function
of f(x) = ax2 + 2x + b, find a + b. ›
⎧⎪3x4 − 4 x3 − 24 x2 + 48 x, x ≥ 0.5
f ( x) = ⎨ .
⎪⎩8 x3 +12 x2 + 2, x < 0.5
27. Let f(x) = ax3 – bx. Find a and b, if f(2) = 4 is the
maximum value of f on [0 , 4]. 34. For which values of a does the interval ⎡⎢0, 1 ⎤⎥
›› ⎣ 3⎦
completely include the range of the function
28. Find the maximum value of f(x) = sinx + cosx. 1
f ( x) = 4 ?
› 3x − 8ax3 +12 a2 x2 + a

Applications of the Derivative 206


A. CONCAVITY
In this section we discuss the concept of concavity. As illustrated in the following figures, two
increasing graphs on an interval may have different shapes. This depends on how the graphs
bend or turn. As we scan the graphs from left to right, we see that the graph of f turns to the
left (upward), while the graph of g turns to the right (downward). We say that the function f
is concave up on the interval (a, b) and the function g is concave down on
the interval (a, b). We now define concavity geometrically.

y y

y=f(x)
y=g(x)

a b x a b x

increasing, concave up increasing, concave down

Definition concavity
A function f is concave up on an interval I if the graph of f lies above all of its tangent lines
on the interval I.
Similarly, f is concave down on I if the graph of f lies below all of its tangent lines on I.

207 Derivatives
concave up (slopes increasing) concave down (slopes decreasing)
The graphs above illustrates the definition of concavity. Now, we shall see that the
second derivative f ′′ tells us where f is concave up and where f is concave down. If f is
concave up on (a, b), then the slopes of the tangent lines increase from left to right as shown
f ′ > 0 ⇔ f is increasing in the left figure above. This means that the first derivative f′ is increasing on (a, b). We know
f ′ < 0 ⇔ f is decreasing that if f ′ is an increasing function, then its derivative f ′′ must be positive on (a, b). In a
similar way, it can be shown that if f is concave down on (a, b), then f ′′(x) < 0 on (a, b).
These observations suggest the following theorem.

Theorem
Let the function f be twice differentiable on the interval I.
1. If f ′′(x) > 0 for all the values of x on the interval I, then f is concave up on I.
2. If f ′′(x) < 0 for all the values of x on the interval I, then f is concave down on I.

EXAMPLE 35 Determine where the following functions are concave up and where they are concave down.

a. f(x) = 9 – x2 b. f(x) = ex c. f(x) = x3


y
Solution a. f(x) = 9 – x2 y = 9 – x2

f ′(x) = –2x and f ′′(x) = –2


Since f ′′(x) < 0 for all the values of x, f is concave down
As a conclusion of the
above theorem, we must everywhere. x
examine the sign of the
second derivative. y
b. f(x) = ex y = ex
f ′(x) = ex and f ′′(x) = ex
As shown on the right, f is concave up on (–∞, ∞) because
f ′′(x) > 0 for all the values of x. x

c. f(x) = x3 x –¥ 0 +¥ y
y = x3
2
f ′(x) =3x and f ′′(x) = 6x f ¢¢(x) – +
Setting f ′′(x) = 0 gives x = 0.
f(x) concave concave x
down up

From the sign chart, f′′(x) changes sign from negative to positive at the point x = 0. Observe
that the point (0, 0) on the graph of f(x) = x3 is where f changes from concave down to
concave up. We call it the inflection point of f.

Applications of the Derivative 208


Definition Inflection point
An inflection point is a point where a graph changes its direction of concavity.

y y y y

a x a x a x a x

f ¢¢(a) = 0 f ¢¢(a) = 0 f ¢¢(a) does not exist f ¢¢(a) does not exist

Note
At each inflection point, either
1. f ′′(a) = 0 or
2. f ′′(a) does not exist.

FINDING THE INFLECTION POINTS

To find the inflection points of a function, follow the steps.


1. Find the points where f ′′(x) = 0 and f ′′(x) does not exist. These points are the
possible inflection points of the function f.
2. Construct the sign chart of f ′′(x). If the sign of f ′′(x) changes across the point
x = a, then (a, f(a)) is an inflection point of f.

Example 36 Investigate f(x) = (x + 1)4 for concavity and find the inflection points.

Solution f ′(x) = 4(x + 1)3 y

f ′′(x) = 12(x + 1)2


4
x = –1 is a double root of f ′′(x) = 0.
x –¥ –1 +¥

f ¢¢(x) + +

f(x) concave concave –1 x


up up
the graph of f(x) = (x + 1)4
From the sign chart, f(x) is concave up for all the values of x.
Also, we have f ′′(x) = 0 when x = –1.
But (–1, 0) is not the inflection point of f because f ′′ does not change sign across x = –1.

209 Derivatives
Example 37 Find the intervals of concavity and the inflection points for f ( x) =
1 4
2
x – 4 x3 +9 x2 – 7 x+5.

Solution f ′(x) = 2x3 – 12x2 + 18x – 7


f ′′(x) = 6x2 – 24x + 18 = 6(x – 3)(x – 1)
Because f ′′(x) exists everywhere, the possible inflection points are the solutions of the
equation f ′′(x) = 0; that is, x = 1 and x = 3.
x –¥ 1 3 +¥

f ¢¢(x) + – +

f (x) concave up concave down concave up

(inf) (inf)

From the sign chart for f ′′, we see that f is concave up on (–∞, 1) and (3, ∞) and concave
down on (1, 3). It would also be true to say that f is concave up on (–∞, 1] and [3, ∞) and
concave down on [1, 3].
Also, observe that f ′′(x) changes sign at x = 1 and x = 3. Therefore, the points (1, f(1)) and
(3, f(3)) are the inflection points of f.

Example 38 Find the intervals of concavity and the inflection points for f(x) = cos x – sin x on [0, 2π].

Solution f ′(x) = –sin x – cos x and f ′′(x) = –cos x + sin x


Since f ′′ is differentiable on [0, 2π], we must find the solutions of f ′′(x) = 0 on [0, 2π].
π 5π
f ′′(x) = 0 ⇒ –cos x + sin x = 0 ⇒ tan x = 1 ⇒ x = and x = .
4 4
sin
p 5p
f ¢¢ = 0 x 0 2p
4 4
+
+
– f ¢¢(x) – + –

+ cos concave concave concave


– f(x)
down up down
–
(inf) (inf)
f ¢¢ = 0
π 5π
The sign chart of f ′′ shows that f is concave down on (0, ) and ( , 2π) and concave up on
4 4
π 5π
( , ).
4 4
π π π 2 2 5π 5π 5π 2 2
f ( ) = cos − sin = − = 0 and f ( ) = cos −sin =− + =0
4 4 4 2 2 4 4 4 2 2
π 5π
So, ( , 0) and ( , 0) are the inflection points of f.
4 4

Applications of the Derivative 210


Example 39 The function f(x) = x3 + ax2 + bx + 3 has an inflection point at (1, 3). Find the values of a
and b.

Solution f ′(x) = 3x2 + 2ax + b and f ′′(x) = 6x + 2a


We know that the inflection point of f occurs at a point where f ′′(x) = 0 or f ′′(x) does not
exist.
Since (1, 3) is an inflection point, f ′′(1) = 0.
Also we have f(1) = 3 because the point (1, 3) is on the graph of f.
f ′′(1) = 0 ⇒ 6 ⋅ 1 + 2a = 0 ⇒ a = –3
f(1) = 3 ⇒ 13 + a ⋅ 12 + b ⋅ 1 + 3 = 3 ⇒ a + b = 0 ⇒ b = 3

Example 40 Find the equation of the tangent line to the curve f ( x) = x2 +


1
x
at its inflection point.

Solution We first need to find the inflection point of f.


1 2
f ′( x) = 2 x − and f ′′( x) = 2+ 3
x2 x
Setting f ′′(x) = 0 gives x = –1 and (–1, 0) is the inflection point.
f ′′ does not exist when x = 0 but this point is not the inflection point of f. (Why?)
Now we can find the equation of the tangent line at the point (–1, 0). The slope of the
equation is
1
m = f ′( −1) = 2 ⋅ ( −1) − = −2 − 1 = −3.
( −1)2
Using the point-slope form of a line,
y – y1 = m ⋅ (x – x1)
y – 0 = –3 ⋅ (x + 1) or y = –3x – 3.

211 Derivatives
Note
In the beginning of this section we have
seen that an increasing graph can be either
concave up or concave down. This shows
that the increase and decrease of a function
is independent of the concavity of the
function.
Remember that the sign of the first derivative determines where f is increasing and decreasing,
whereas the sign of the second derivative determines where f is concave up and concave down.

RELATIONSHIP BETWEEN A GRAPH AND ITS DERIVATIVES

y y y y
y = f(x)
y = f(x)

y = f(x)
y = f(x)

x x x x
f ¢ > 0, f increasing f ¢ > 0, f increasing f ¢ < 0, f decreasing f ¢ < 0, f decreasing
f ¢¢ > 0, f concave up f ¢¢ < 0, f concave down f ¢¢ > 0, f concave up f ¢¢ < 0, f concave down

Check Yourself 9
1. Find the intervals of concavity and the inflection points for each function.
x +1
a. f(x) = x3 – 2x2 – 7x + 3 b. f ( x) = c. h(x) = x + ex
x–1
2. The function f(x) = x3 + ax2 + bx + 2 has an inflection point at (1, –1). Find a and b.
y
3. The graph of a function y = f(x) is shown in the figure.
y = f(x)
a. Find the intervals of increase and decrease of f.
b. Find the intervals of concavity of f.

1 2 3 4 5 6 7 8 9 x
Answers
2 2 2
1. a. concave up: ( , ∞ ) , concave down: (– ∞, ) , inflection point x = .
3 3 3
b. concave up: (1, ∞), concave down: (–∞, 1), no inflection point.
c. concave up: (–∞, ∞), no inflection point.

2. a = –3, b = –1 3. a. increasing: (1, 2), (4, 6), (8, 9) b. concave up: (3, 6), (6, 9)
decreasing: (2, 4), (6, 8) concave down: (1, 3)

Applications of the Derivative 212


B. THE SECOND DERIVATIVE TEST
We have seen that the first derivative test helps us classify the critical points of a function f.
Here we learn an alternative test for determining whether a critical point of f is a local
maximum or a local minimum.

THE SECOND DERIVATIVE TEST

Let f be twice differentiable on an interval I and c be a critical point of f in I such that


f ′(c) = 0.
1. If f ′′(c) > 0, then f has a local minimum at x = c.
2. If f ′′(c) < 0, then f has a local maximum at x = c.

y y
y = f(x)

y = f(x)

c x c x

f ¢(c) = 0, f ¢¢(c) > 0 f ¢(c) = 0, f ¢¢(c) < 0


local minimum at c. local maximum at c.

The graphs above illustrates the second derivative test. We know that f is concave up near c
if f ′′(c) > 0. This means that the graph of f lies above its horizontal tangent at c and so
f has a local minimum at c.

Example 41 Apply the second derivative test to find the local extrema of the function
f(x) = x3 – 3x2 – 9x + 6.

Solution f ′(x) = 3x2 – 6x – 9 and f ′′(x) = 6x – 6.


So, f ′(x) = 0 gives x = –1 and x = 3, the critical points of f.
To apply the second derivative test, we compute f ′′ at these points.
f ′′(–1) = –12 and f ′′(3) = 12
Since f ′′(–1) < 0, the second derivative test implies that f(–1) = 11 is a local maximum value
of f. And since f ′′(3) > 0, it follows that f(3) = –21 is a local minimum value.
Remember that we had found the same results by using the first derivative test in Example 24.

213 Derivatives
Note
The second derivative test can be used only when f ′′ exists. Moreover, this test fails when
f ′′(c) = 0. In other words, if f ′(c) = 0 = f ′′(c), then there might be a local
maximum, a local minimum, or neither at the point x = c. In such cases we must use the
first derivative test.

Example 42 Find the local extrema of the function f(x) = 3x5 – 5x3 + 3.

Solution f ′(x) = 15x4 – 15x2 = 15x2(x2 – 1) = 15x2(x – 1)(x + 1) = 0


So, the critical points are x = –1, x = 0, and x = 1.
The second derivative is f ′′(x) = 60x3 – 30x.
When we compute f ′′(x) at each critical point, we find that
f ′′(–1) = –30 < 0, f ′′(0) = 0, f ′′(1) = 30 > 0.
The second derivative test tells us that f has a local maximum at x = –1, a local minimum
at x = 1. Since f ′′(0) = 0, this test gives no information about the critical point 0. Let us
apply the first derivative test.
x –¥ –1 0 1 +¥

f ¢(x) + – – +

f(x)

We see that f ′ does not change sign at x = 0. So, f does not have a local maximum or
minimum.

Check Yourself 10
Apply the second derivative test to find the local extrema of each function.

1. f (x) = 4x3 + 9x2 – 12x + 7

2. f (x) = 8x5 – 5x4 – 20x3


4
3. f (x) = x +
x

Answers
1. max.: x = –2 2. max.: x = –1 3. max.: x = –2
1 3
min.: x = min.: x = min.: x = 2
2 2

Applications of the Derivative 214


EXERCISES 2 .3
A. Concavity i. f(x) = x +1

1. The graphs of two functions are given. Find the


3
intervals where the second derivative of the j. f(x) = 5– x
function is positive or negative.
ex – e– x
a. y › k. f ( x) =
2

–4 2 4
–2 › l. f(x) = –Arcsin(x – 2)
x

sin x
›› m. f ( x) =
b. 1 − cos x
y

–2
–3 –1 2 x
3. Find a and b, if f(x) = x4 – 4x3 + ax2 + b has an
inflection point at (1, 3).

2. Find the intervals of concavity and the inflection

points for each function. 4. The function f(x) = x4 + kx2 + 7x – 7 has an


inflection point at x = 1. Find the coordinates of
a. f(x) = x2 – 5x + 6
the other inflection point.

b. f(x) = –2x2 + 7x

c. f(x) = x3 + x2

d. f(x) = x3 – 3x2 + 5x – 7 5. Find the coordinates of the inflection points of


›
ex
4 3
e. f(x) = 3x – 16x + 30x + 4 2 the function f ( x) = on the interval (0, ∞).
x2 +1
1 4
f. f(x) = x – 6x2 + 4x – 7
4

g. f(x) = 3x5 – 10x3 + 5x


6. Find the coordinates of the inflection points of
1
h. f(x)= the function f(x) = exsin x on the interval [0, π].
x2
215 Derivatives
7. The graph of the derivative of a function f is shown Mixed Problems
in the figure. Find the intervals of concavity and
the inflection points of f.
10. Given the graph of f(x), plot a rough graph of
›
f ′(x).
y
y
y = f(x)
–1 1 5
–3 3 x

y = f ¢(x)
a b x

11. Show that a cubic function has exactly one


inflection point.
B. The Second Derivative Test

8. Determine whether f has a local maximum or


12. Show that a polynomial function of degree 4 has
minimum at the given value of x, using the
either no inflection point or exactly two inflection
second derivative test.
1 points.
a. f(x) = x3 – 3x2 – 7x + 5, x=7
3
b. f(x) = x4 – 3x2 + 2, x=0
1 4 13. Find the equation of the tangent line to the curve
c. f(x) = 4x + x , 3 3
x = −1 1 3 1
y= x – 2 x2 + 3x + at its inflection point.
3 3
2
d. f(x) = x2 + , x =1
x

14. Find a, b, and c so that f(x) = ax3 – 3x2 + bx + c


has an inflection point at the point (–1, 1) and a
local extremum at x = –2.
9. Find the local extrema of the following functions,
using either the first or second derivative test.
15. The tangent line to the curve
a. f(x) = x3 + 6x2 + 9x + 1
f(x) = x3 + 3x2 + cx + 1 at the inflection point of
4 3 2
b. f(x) = x + 4x + 2x + 1 f is perpendicular to the line y = x + 4. Find c.
x2
c. f(x) =
x–2
2
d. f(x) = ( x + 3) 3 16. Given that g(x) = e–x ⋅ f(x) where f is a differentiable
›
1 function for all real numbers and the function g(x)
e. f(x) =
1 + x2 has an inflection point at x = a, find an expression
f. f(x) = x + sin x for f ′′(a) in terms of f ′(a) and f(a).

Applications of the Derivative 216


In this section we solve applied maximum-minimum problems in which the function is not
given directly. When we face such a problem, we are required to first find the appropriate
function to be maximized or minimized. The following steps will be helpful for solving these
problems.
1. Determine the quantity to be maximized or minimized and label it with a letter (say M for now).
2. Assign letters for other quantities, possibly with the help of a figure.
3. Express M in terms of some of the other variables.
4. Use the data in the problem to write M as a function of one variable x, say M = M(x).
5. Find the domain of the function M(x).
6. Find the maximum (or minimum) value of M(x) with the help of the first derivative.
Such problems where we look for the “best” value are called optimization problems.

Example 43 A man has 40 m of fencing that he plans to use to enclose a


rectangular garden plot. Find the dimensions of the plot that
will maximize the area.

Solution We want to maximize the area A of the rectangular plot.


Let x and y represent the length and width of the rectangle.
Then, since there is 40 m of fencing,
2x + 2y = 40 or x + y = 20.

b
Then we express A in terms of x and y: A = x ⋅ y
a Expressing A as a function of just one variable,
Given a rectangle with
sidelength a and b we get A(x) = x ⋅ (20 – x) = 20x – x2 (since y = 20 – x).
Perimeter = 2(a + b) Since the dimensions will be positive,
Area = a ⋅ b
x > 0 and y = 20 – x > 0 or 0 < x < 20.
The derivative is A′(x) = 20 – 2x. So, the only critical point is x = 10. To investigate this
critical point, we calculate the second derivative. Since A′′(x) = – 2 < 0, the second
derivative test implies that A has a local maximum x = 10.
We can verify that this local maximum is the absolute maximum by showing that the graph of
A is concave down everywhere. Since A′′(x) < 0 for all the values of x in (0, 20), maximum
value of A occurs at x = 10. The corresponding value of y is y = 20 – x = 20 – 10 = 10.
Thus, the garden would be of maximum area (100 m2) if it was in the form of a square with
sides 10 m.

217 Derivatives
Note
Suppose that f has only one critical point c in the interval I. If f ′′(x) has the same sign at all
points of I, then f(c) is an absolute extremum of f on I. This absolute interpretation of the
second derivative test is useful in optimization problems.

Example 44 Find two positive numbers x and y such that their sum is 15 and x2 + 5y is as small as possible.

Solution We have x + y = 15 and we want to minimize M = x2 + 5y. Expressing M as a function of


just one variable we get M(x) = x2 + 5(15 – x) = x2 – 5x + 75 (since y = 15 – x).
Since both numbers are positive, x > 0 and y = 9 – x > 0 or 0 < x < 9.
5
The derivative is M′(x) = 2x – 5. So, the critical point is x = .
2
Since M′′(x) = 2 > 0 for all the values of x in (0 , 9), the second derivative test implies that
5
x = is the minimum value of M(x).
2
5 5 25
Therefore, M gets its minimum value when x = , and y = 15 − x = 15 − = .
2 2 2

Example 45 A manufacturer has an order to make cylindrical cans with a


volume of 500 cm3. Find the radius of the cans that will
minimize the cost of the metal in their production.

Solution In order to minimize the cost of the metal, we minimize the


total surface area of the cylinder.
500
The volume of the can is V = πr2h = 500. So, h = .
πr 2
Hence the surface area of the can as a function of r is
r
500 1000
A( r ) = 2 πr 2 + 2 πr ⋅ ( 2
) = 2 πr 2 + , r > 0.
h πr r
1000 4( πr 3 – 250)
The surface area of a The derivative of A(r) is A′( r ) = 4πr – = .
cylinder is 2πr2 + 2πrh r2 r2
and the volume is πr2h
where r is the radius 250
Thus, the only critical point is r = 3 .
and h the height. π

2000 250
Since A′′( r ) = 4π + 3
> 0 for r = 3 , the second derivative test implies that A gets its
r π
250
minimum value when r = 3 .
π

Applications of the Derivative 218


Example 46 Find the area of the largest rectangle that has two vertices on the x - axis and another two above
the x - axis on the parabola y = 3 – x2.

y
Solution Let (x, y) be the vertex of the rectangle in the
first quadrant. Then the rectangle has sides with
the lengths 2x and y. So, its area is A = 2xy. (x, y)
Using the fact that (x, y) lies on the parabola
y = 3 – x2, the expression to be maximized is –ñ3 ñ3

A(x) = 2 ⋅ x ⋅ (3 – x2) = 6x – 2x3. x

The domain of this function is (0, ñ3). y = 3 – x2

Its derivative is A′(x) = 6 – 6x2.


So, the critical points are – 1 and 1.
Only the positive value x = 1 lies in the
interval (0, ñ3). Since this is the only critical
value in the interval, we can apply the second
derivative test.
The second derivative is A′′(x) = –12x and in
particular A′′(x) < 0 for all the values of x in
(0, ñ3). So, the maximum value of A(x) in this interval is A(1) = 4. Therefore, the area of
the largest rectangle is 4.

Example 47 Find the point on the line y = x + 2 that is the closest to the point (1, 2).

Solution Let (x, y) be a point on y = x + 2 such that the distance y y=x+2


d between (x, y) and (1, 2) is a minimum. (x, y)
d
We have d = ( x – 1) +( y – 2) .
2 2 2 (1, 2)

Distance between If the point (x, y) is on the line, then y = x + 2. To


two points
(x1, y1) and (x2, y2) is rewrite d in terms of the single variable x, substitute
y = x + 2. -2
( x1 – x2 )2 +( y1 – y2 )2 .
1 x
After substitution, d = ( x – 1)2 + x2 = 2 x2 – 2 x +1.

It is clear that the minimum of d occurs at the same point as the minimum of d2. So, we
minimize d2 to simplify calculations by letting M = d2.
M(x) = 2x2 – 2x + 1 has derivative M′(x) = 4x – 2.
1 1
So, M′(x) = 0 when x = . Since M′′(x) = 4 > 0, the minimum value occurs at x = .
2 2
1 5 1 5
Since y = + 2 = , the point ( , ) is the closest point to the point (1, 2).
2 2 2 2

219 Derivatives
Example 48 The swimmer is 40 m from the shoreline. The lifeguard is 100 m from the point on the shore
that is directly opposite the swimmer. The guard can run at a speed of 5 m/s and swim at a
speed of 3 m/s. What path should the guard follow to get to the swimmer in the least time?

Solution Let x be the distance denoted in the given diagram.


swimmer

402 + x2
40 m

guard

x 100 – x

Recall that if travel is at a constant rate of speed, then


(distance traveled) = (rate of travel) ⋅ (time elapsed).
D
In short, D = R ⋅ T or T = .
R
The total time elapsed is
swim distance run distance 40 2 + x2 100 – x
T( x) = (swim time)+( run time) = + = + .
swim rate run rate 3 5

We wish to minimize the total time elapsed. So, we differentiate this equation to get
1 1 −1 1 x 1
T ′( x) = ⋅ ⋅ (40 2 + x2 ) 2 ⋅(40 2 + x2 ) ′ − = − .
3 2 5 3 40 2 + x2 5

T′(x) = 0 gives 5x = 40 2 + x2 ⇒ 25x2 = 9(402 + x2) ⇒ 16x2 = 9 ⋅ 402 ⇒ x = ± 30.


But x ≠ –30 since x measures a distance.
It is left to the student to verify that T′′(30) > 0 which means x = 30 corresponds to a path
of minimum time.

Check Yourself 11
1. Find two positive numbers x and y such that their sum is 9 and x2y is as large as possible.
2. A rectangle has area of 144 m2. What dimensions will minimize its perimeter?
3. An open rectangular box with a square base is to be made from 300 cm2 of material. Find
the dimensions of the box with the maximum volume.
4. Find the minimum distance from the line 2x + 3y = 13 to the origin.
Answers
1. x = 6, y = 3 2. 12, 12 3. 5, 10 4. ò13

Applications of the Derivative 220


How can we explain why a pencil appears to be
broken when it is immersed partially into water or
why objects under water appear to be nearer the
surface than they really are to an observer looking
down? It is an illusion caused by the refraction of
light.
When light passes from one transparent medium to
another(like air and water), it changes speed, and
bends according to the law of refraction which
states:
sin θ1 sin θ2
=
ν1 ν2
where,

ν1 is the speed of light in medium 1,


ν2 is the speed of light in medium 2,
θ1 is the angle between the incident ray and normal to the surface at the point P,
θ2 is the angle between the refracted ray and the normal.
N
incident
ray
q1
P medium 1
medium 2
q2
refracted
ray
The experimental discovery of this relationship is usually credited to Willebrord
Snell (1591-1627) and is therefore known as Snell’s law.
122 Derivatives
Snell’s law can be derived from a physics principle discovered by Pierre de Fermat,
the seventeenth-century mathematician. Fermat’s principle states that a ray of
light travels the path of minimum time. The derivation of Snell’s law from
Fermat’s principle represent an interesting application of the derivative.
A
Suppose a light ray is to travel from A
to B, where A is in the medium 1 and
d1
B is in the medium 2. Using the a
geometry of the figure given above, q1 d–x
medium 1
we see that the time it takes the ray medium 2
x
to travel from A to B is q2
d2
b
d d distance
t= 1 + 2 (time= ),
ν1 ν 2 velocity
d
2 2 2 2
a +x ( d − x) + b
t( x) = + .
ν1 ν2
We obtain the least time, or the minimum value of t, by taking the derivative of t
with respect to x and setting the derivative equal to zero.
x d−x
t′( x) = −
ν 1 a2 + x2 ν 2 ( d − x)2 + b2
The function t is differentiable for all the
values of x. So, the only critical values are the
solutions to the equation t′(x) = 0. This
equation gives the condition that
x d−x
= .
ν 1 a2 + x2 ν 2 ( d − x )2 + b2
x d−x
From the figure, we see that sin θ1 = and sin θ 2 = .
a2 + x2 ( d − x)2 + b2
sin θ1 sin θ2
Using these equations, we obtain = , which is Snell’s law.
ν1 ν2
Applications of the Derivative
123 Derivatives
EXERCISES 2 .4
1. Find two positive numbers such that their sum is 7. A man has 120 metres of fencing. He wishes to
30 and their product is as large as possible. enclose a rectangular garden adjacent to a long
existing wall. He needs no fence along the wall.
What are the dimensions of the largest area he
2. One number is 4 larger than another. How must can enclose?
they be chosen in order to minimize their
product?

8. A closed rectangular
3. The sum of two positive integers is 10. Find the box is to be made with
maximum value of the sum of their squares. 192 cm2 of material.
The length of its base
y
is twice its width.
x
What is the largest 2x
4. Find the minimum possible value of the sum of possible volume of
two positive numbers such that their product is m. such a box?

5. Find the value of m if the sum of squares of the


roots of x2 + (2 – m)x – m – 3 = 0 is to be
9. An open tank with a square base is to be made of
minimum.
sheet iron. Its capacity is to be 4 m3. Find the
dimensions of the tank so that the least amount of
sheet iron may be used.
6. A farmer has 100 m of fencing and wants to build
a rectangular pen for his horse. Find the
dimensions of the largest area he can enclose.

10. A rectangular sheet of x


tin, 30 cm by 48 cm,
x
has four equal squares
cut out at the corners.
48 cm

The sides are then


turned up to form an
open rectangular box.
Find the largest possible
volume of the box. 30 cm

223 Derivatives
11. A closed cylindrical drum with the volume 54 m3 17. In the figure on the right, y

is to be manufactured using the minimum ›


the point (x, y) lies on 4
amount of metal sheet possible. Find the diameter the graph of the ellipse P(x, y)
of the base of the drum. x 2
y 2
+ =1 in
9 16
the first quadrant. O A 3 x
For what value of y will
12. The sum of two non-negative numbers is 10. Find the area of the triangle POA be maximum?
the minimum possible value of the sum of their
cubes.
18. Find the length of the diagonal of the rectangle
›
with the largest area that can be inscribed in an
isosceles triangle of base 24 cm and height 10 cm.
13. Find the area of the
largest rectangle that is D C 19. A right cylinder is inscribed
inscribed in a circle with ›
2R in a right circular cone with
the radius R. ([AC] is the the radius 3 cm and height
A B
diameter of the circle 5 cm. Find the dimensions
because an angle inscribed in a of the cylinder of maximum
semicircle is a right angle). volume.

20. Car A is 125 km directly west of car B and begins


›
14. What is the shortest distance from a point on the moving east at 100 km/h. At the same time, car B
›
curve y2 = 8x to the point (4, 2)? begins moving north at 50 km/h. At what time t
does the minimum distance occur?

15. Find the point on the curve y = lnx that is closest


›
to the line y = x.

16. What is the minimum y


›
possible area of a right
triangle that is formed
(4, 3)
21. Among all tangents to graph of
in the first quadrant by a 3 ›› 45
y = x 3 − 6 x2 + x +1 at positive x-values, the
line passing through the 4
4 x one which intersects y-axis at maximum y-value
point (4, 3) and the
coordinate axes? is chosen. Find that y-value.

Applications of the Derivative 224


A. ASYMPTOTES
When plotting the graph of a function, we need to know the behavior of the function at
infinity and the behavior near points where the function is not defined. To describe these
situations, we define the term “asymptote”. y = f(x)
An asymptote is a line that a curve approaches closer and P
closer until the distance between the asymptote and the
points on the curve approaches zero.
x −1
Consider the graph of the function f ( x) = .
x+ 2 asymptote
Observe that f is increasing on the interval (–2, ∞). So, you
might think that its values f(x) increase without bound as x y
We represent an asymptote
by drawing a dashed line. increases without bound. (f(x) → ∞ as x → ∞). However, we x–1
y=
can see that the graph of f approaches the line y = 1 as x+1

x → ∞. So, we say that y = 1 is a horizontal asymptote of f. y=1


Next, the function f is not defined at x = –2. Let us examine
the behavior of f near –2. We see that the graph of f goes to x

plus infinity as x approaches –2 from the left, whereas the


graph goes to minus infinity as x approaches –2 from the x = –2
right. So, we say that x = –2 is a vertical asymptote.

Definition vertical asymptote


The line x = a is a vertical asymptote of the graph of f(x) if y
either

lim f(x) = ±∞ or lim f(x) = ±∞.


x → a+ x → a–
x

vertical
asymptote

FINDING THE VERTICAL ASYMPTOTE


P( x)
A rational function f ( x) = has a vertical asymptote x = a whenever only the
Q( x)
denominator of f(x) equals zero (that is, Q(a) = 0 but P(a) ≠ 0).

225 Derivatives
Example 49 Find the vertical asymptotes of the function f ( x) = 2
x
x –4
.

Solution Let P(x) = x and Q(x) = x2 – 4.


Note that x = –2 and x = 2 are the roots of the denominator.
Since P(–2) ≠ 0 and P(2) ≠ 0, x = –2 and x = 2 are both vertical asymptotes of the graph of f.

Definition horizontal asymptote


The line y = b is a horizontal asymptote of the graph of f(x) y
if either horizontal
asymptote
lim f(x) = b or lim f(x) = b.
x → –∞ x→∞

FINDING THE HORIZONTAL ASYMPTOTE

an xn + an −1x n −1 +...+ a1x + a0


A rational function f ( x) = has the following limit as
bm xm + bm −1x m −1 +...+ b1x + b0
horizontal asympote:
±∞ if n>m
lim f ( x) = an / bm if n = m.
x→ ∓ ∞

0 if n<m

Example 50 Find the horizontal asymptote of the function f ( x) =


x
x2 – 4
.

Solution To find the horizontal asymptotes we must evaluate lim f ( x).


x →∞

Since the degree of the polynomial in numerator is smaller than the degree of the polynomial
x
in denominator, lim 2 = 0. So, y = 0 is a horizontal asymptote of f.
x →∞ x – 4

Example 51 Find all the vertical and horizontal asymptotes of the function f ( x) = 2
x2 – x
2 x – 5x + 3
.

Solution To find the horizontal asymptotes we must evaluate lim f ( x).


x →∞
x2 – x 1
Since the degree of x2 – x equals the degree of 2x2 – 5x + 3, lim = . So, y = 1/2
x →∞ 2 x2 – 5 x + 3 2
is a horizontal asymptote of f.
Next, to find the vertical asymptotes we must solve 2x2 – 5x + 3 = 0.
2x2 – 5x + 3 = (x – 1)(2x – 3) = 0 gives x = 1 and x = 3/2.
However, x = 1 is also a root of numerator. So, only x = 3/2 is a vertical asymptote of f.

Applications of the Derivative 226


Example 52 Find all the vertical and horizontal asymptotes of the function f ( x) =
2 x2 − 3x +5
x2 +1
.

Solution 2 x2 − 3x +5
lim = 2. So, y = 2 is a horizontal asymptote of f.
x →∞ x2 +1
Since the denominator x2 + 1 is never equal to zero, f has no vertical asymptotes.

Example 53 Find all the asymptotes of the function f ( x) = 2 x3 − 5x2 + 7 x − 12.

Solution The function f is a polynomial function. But we can write it as a rational function with
2 x3 − 5x2 + 7x − 12
the denominator 1 such as f ( x) = .
1
Since the denominator is never equal to zero, f has no vertical asymptotes.
Next, compute lim (2 x3 − 5 x2 + 7 x − 12).
x →±∞

We know that the limit of a polynomial at infinity is the limit of the term of highest degree.
So, lim (2 x3 − 5 x2 + 7 x − 12) = lim 2 x3 = 2 ⋅( ∞) 3 = ∞.
x →+∞ x→+∞

lim (2 x − 5 x + 7 x − 12) = lim 2 x3 = 2 ⋅( −∞) 3 = −∞.


3 2
x →−∞ x→−∞

In other words, lim f ( x) and lim f ( x) do not exist. Therefore, f has no horizontal asymptote.
x →+∞ x →−∞

Note
A polynomial function has no vertical or horizontal asymptotes.

Definition oblique asymptote


The line y = mx + n is an oblique asymptote of the graph of f(x) y oblique asymptotes
if either

lim[ f ( x) − ( mx + n)] = 0 or lim[ f ( x) −( mx + n)] = 0.


x →∞ x →−∞
x

To find the equation of an oblique asymptote, we use long division.


P( x)
For a rational function f ( x) = for which the degree of P is exactly one more than the
Q( x)
c
degree of Q, by dividing Q(x) into P(x), we get f ( x) = mx + n + .
Q( x)
c
In this case, we have lim[ f ( x) − ( mx + n)]= lim = 0.
x →±∞ x→±∞ Q( x )

So, the line y = mx + n is an oblique asymptote of the graph of f(x).

227 Derivatives
x2 + x
Example 54 Find all the asymptotes of the graph of f ( x) =
x–2
.

Solution x = 2 is a vertical asymptote of the graph of f because 2 makes only the denominator zero.
Note that the degree of the numerator is one more than the degree of the denominator. So,
the graph of f has an oblique asymptote.
x2 + x 6
By long division of x – 2 into x2 + x, we can find that f ( x) = = x + 3+ .
x–2 x−2
So, y = x + 3 is an oblique asymptote of f.

Check Yourself 12
Find all the asymptotes of the graph of each of the following functions.
x −1 5x x2 − 9 x2 − x − 2
1. f ( x) = 2. f ( x) = − 3. f ( x) = 4. f ( x) =
2x + 3 3 + x2 −3+7 x − 2 x2 x −1
Answers
3 1 1 1
1. x = − , y = 2. y = 0 3. x = ,y=− 4. x = 1, y = x
2 2 2 2

B. CURVE PLOTTING
Curve plotting is the final part of our study of the derivatives. So far we have seen how to use
the derivatives to find the most interesting features of a graph. With the use of all the
information about the graph of a function, we can easily draw it.

STEPS OF CURVE PLOTTING

1. Domain: Find where f(x) is defined.


2. Intervals of Increase and Decrease: Construct the sign chart of f ′(x) to determine the
intervals where f(x) is increasing and where f(x) is decreasing.
3. Local Extrema: Find the critical points of f and classify each as a maximum, a
minimum, or neither by using the First Derivative Test.
4. Concavity and Inflection Points: Construct the sign chart of f ′′(x) to determine the
intervals where f(x) is concave up and where f(x) is concave down. With the help of
the chart, find the inflection points.
5. Intercepts: In y = f(x) setting x = 0 gives the y-intercept and y = 0 gives the
x-intercept(s). To find the x-intercept(s) may be difficult, in which case we do not use
this information.
6. Behavior at Infinity: Consider lim f(x) and lim f(x) to see how the graph of f
x → +∞ x → –∞
behaves as x → ±∞.
7. Asymptotes: Find all the asymptotes of the graph and draw the asymptotes in a
coordinate plane by using dashed lines.
8. Graph: Start graphing by plotting the local extrema, inflection points, and intercepts.
Then, using the rest of the information, complete the plot by joining the plotted points.

Applications of the Derivative 228


Example 55 Plot the graph of the function f(x) = x3 – 3x – 2.

Solution 1. Domain: Recall that the domain of a polynomial function is all real numbers. So, f is
defined for all the values of x.

2. Intervals of Increase and Decrease: f ′(x) = 3x2 – 3 = 3(x + 1)(x + 1)


When f ′(x) = 0 we have x = –1 and x = 1. x –¥ –1 1 +¥
The sign chart of f ′ shows that f is increasing
f ¢(x) + – +
on (–∞, –1) and (1, ∞) and decreasing on (–1, 1).
f(x)

(max) (min)

3. Local Extrema: We have learned that a polynomial function is differentiable everywhere.


So, the critical points of f(x) are the roots of f ′(x) = 0.
From the results of Step 2, we say that f has a local maximum at x = –1 and a local
minimum at x = 1.

4. Concavity and Inflection Points: f ′′(x) = 6x = 0


When f ′′(x) = 0 we have x = 0. x –¥ 0 +¥

The sign chart of f ′′ shows that f is concave f ¢¢(x) – +


down on (–∞, 0) and concave up on (0, ∞). concave concave
f(x)
down up
So, f has an inflection point at x = 0.

5. Intercepts: x = 0 ⇒ y = –2 (y-intercept)
Setting y = 0 leads to a cubic equation. Since the solution is not readily found, we will
not use this information.

6. Behavior at Infinity: Recall that the limit of a polynomial function at infinity is the limit
of the term of highest degree.
lim f(x) = lim (x3 – 3x – 2) = lim x3 = (–∞)3 = –∞
x → –∞ x → –∞ x → –∞

(This means that f(x) decreases without bound as x decreases without bound. So, the
graph of f goes to plus infinity as x → –∞)
lim f(x) = lim (x3 – 3x – 2) = lim x3 = (+∞)3 = +∞
x → +∞ x → +∞ x → +∞

(This means that f(x) increases without bound as x decreases without bound. So, the
graph of f goes to plus infinity as x → +∞)

7. Asymptotes: A polynomial function has no asymptotes.

229 Derivatives
8. Graph: We can find f(–1) = 0, y

f(1) = –4, and f(0) = –2. local f(x) ® +¥


Plot a local maximum at (–1, 0) a local maximum
as x ® +¥
minimum at (1, –4), an inflection –1 1
point at (–1, 0), and the y-intercept at x
y = –2. Finally, complete the graph by
passing a smooth curve through the y = f(x)
inflection
plotted points. –2 point

f(x) ® –¥
as x ® –¥
–4 local
minimum

It is clear from the graph of f that x = –1 is a root of f(x) = 0. So, x + 1 is a factor of


x3 – 3x – 2. The other factor can be found by division:
f(x) = (x + 1)(x2 – x – 2) = (x + 1)2(x – 2).
Hence x = 2 is also a root of f(x) = 0 and the graph crosses the x-axis at this point. But note
that x = –1 is a “double root” of f(x) = 0 and the graph is tangent to the x-axis at x = –1.

Example 56 Plot the graph of the function f(x) = –x4 + 8x2 – 7.

Solution 1. Domain: Since f is a polynomial, it is defined for all the values of x.

2. Intervals of Increase and Decrease:


x –¥ –2 0 2 +¥
f ′(x) = –4x3 + 16x = –4x(x2 – 4)
When f ′(x) = 0 we have x = –2, x = 0, f ¢(x) + – + –
and x = 2. f(x)
f is increasing on (–∞, –2) and (0, 2), and
(max) (min) (max)
decreasing on (–2, 0) and (2, +∞).

3. Local Extrema: From the sign chart of f ′(x), f has local maximum at x = –2 and
x = 2, a local minimum at x = 0.

4. Concavity and Inflection Points: 2 2


–
x –¥ ñ3 ñ3 +¥
f ′′(x) = –12x2 + 16 = 0
2 f ¢¢(x) – + –
When f ′′(x) = 0 we have x = ± .
3 concave concave concave
f(x)
2 2 down up down
f has inflection points at x = − and x = .
3 3
5. Intercepts: x = 0 ⇒ y = –7 (y-intercept)
y = 0 ⇒ x1 = –1, x2 = 1, x3 = –ñ7, x4 = ñ7 (x-intercepts)

Applications of the Derivative 230


6. Behavior at Infinity:
lim f(x) = lim (–x4) = –(–∞)4 = –∞ and lim f(x) = lim (–x4) = –(+∞)4 = +∞
x → –∞ x → –∞ x → +∞ x → +∞

The graph goes to –∞ to the left and to the right.


7. Asymptotes: Since f is polynomial, it has no asymptotes.

2 5 2 5
8. Graph: f(–2) = 9, f(0) = –7, f(2) = 9, f ( − )= , f( )= .
3 3 3 3

y
local (and absolute) local
maximum (and absolute) maximum
9

inflection point (– 2 , 5 )
ñ3 3 ( 2 , 5 ) inflection point
ñ3 3
5
3

–ñ7 –2 –1 1 2 ñ7 x

y = f(x)

f(x) ® –¥ –7 f(x) ® –¥
as x ® –¥ local as x ® +¥
minimum

Check Yourself 13
Plot the graph of each function.
1. f(x) = 2x3 – 3x2 – 12x 2. f(x) = x2(x – 2)2

Answers
1. y 2. y

7 y = f(x) y = f(x)

2
–1 x 1

1 2 x

–20

231 Derivatives
Example 57 Plot the graph of the function f ( x) =
x−3
2x + 4
.

Solution 1. Domain: Recall that the domain of a rational function is all real numbers except the
numbers that make the denominator zero. So, f is defined everywhere except x = –2.
1 ⋅ (2 x + 4) − ( x − 3) ⋅ 2 10
2. Intervals of Increase and Decrease: f ′( x) = 2
=
(2 x + 4) (2 x + 4)2
Since f ′(x) > 0 for all the values of x except –2. So, f is always increasing in its domain.
3. Local Extrema: Note that f ′ does not change its sign. By the first derivative test, we say
that f has no local extrema.
−40
4. Concavity and Inflection Points: f ′′( x) =
(2 x + 4)3
The sign chart of f′′ shows that f is concave up x –¥ –2 +¥
on (–∞, –2) and concave down on (–2, –∞).
f ¢¢(x) + –
Observe that f ′′ changes its sign at x = –2. But
at this point f is not defined. Therefore, there is f(x) concave concave
up down
no inflection point.

3
5. Intercepts: x = 0 ⇒ y = − ( y - intercept)
4
y = 0 ⇒ x=3 ( x - intercept)

6. Behavior at Infinity:
x−3 1
lim f(x) = lim = .
x → ±∞ x → ± ∞ 2x + 4 2

1
7. Asymptotes: From Step 6, y = is a horizontal asymptote of the graph of f.
2
Also, x = –2 is a vertical asymptote of the graph of f because –2 makes the denominator zero.
8. Graph: y

y = f(x) y = 1 horizontal asymptote


2
1
2

–2 3 x
–3
4
x = –2
vertical asymptote

Applications of the Derivative 232


Example 58 Plot the graph of the function f ( x) = 2
x
x –4
.

Solution 1. Domain: The domain of f is all real numbers except x = –2 and x = 2.

1 ⋅ ( x2 – 4) – x ⋅ 2 x – x2 – 4 –( x2 + 4)
2. Intervals of Increase and Decrease: f ′( x) = 2 2
= 2 2
= 2
( x – 4) ( x – 4) ( x – 4) 2
Since f ′(x) < 0 for all the values of x except –2 and 2, f is always decreasing in its domain.

3. Local Extrema: f has no local extrema.

4. Concavity and Inflection Points: x –¥ -2 0 2 +¥


2
2 ⋅ x( x +8) f ¢¢(x) – + – +
f ′′( x) =
( x2 – 4)3
f ′′(x) = 0 only when x = 0. f(x) concave concave concave concave
down up down up
f ′′ is not defined at x = –2 and x = 2. (inf)

Thus, f is concave up on (–2, 0) and (2, ∞) and concave down on (–∞, –2) and (0, 2). The
sign of f ′′(x) changes at the points x = –2, 0, and 2. But the only inflection point is
x = 0 because f is not defined at –2 and 2.
5. Intercepts: x = 0 ⇒ y = 0 and y = 0 ⇒ x = 0.
The point (0, 0) is the only intercept.
6. Behavior at Infinity:
x
lim f(x) = lim 2
= 0.
x → ±∞ x → ±∞ x –4

7. Asymptotes: From Step 6, y = 0 (the x – axis) is a horizontal asymptote of the graph of f.


Next, x = –2 and x = 2 are vertical asymptotes of the graph of f because they make the
denominator zero.
8. Graph: y

x=–2
vertical
asymptote y = f(x)

–2
2 x
y=0
horizontal
asymptote
x=2
vertical
asymptote

233 Derivatives
Example 59 Plot the graph of the function f ( x) =
x2 − x + 4
x –1
.

Solution 1. Domain: f is defined for all the values of x except x = 1.

(2 x − 1) ⋅ ( x − 1) − ( x2 − x + 4) x 2 − 2 x − 3
2. Intervals of Increase and Decrease: f ′( x) = =
( x – 1)2 ( x − 1)2
When f ′(x) = 0, x = –1 and x = 3.
x –¥ –1 1 3 +¥
Also, note that f ′(x) is not defined at x = 1.
From the sign chart of f ′, f is increasing on f ¢(x) + – – +
(–∞, –1) and (3, ∞) and decreasing on f(x)
(–1, 1) and (1, 3).
(max) (min)
3. Local Extrema: f has a local maximum at x = –1 and a local minimum at x = 3.

x –¥ 1 +¥
8
4. Concavity and Inflection Points: f ′′( x) =
( x − 1)3 f ¢¢(x) – +
We conclude that f is concave down on (–∞, –1) and
concave concave
concave up on (1, ∞). But it has no inflection point f(x)
down up
because –1 is not in the domain of f.
5. Intercepts: x = 0 ⇒ y = –4 (y-intercept)
y = 0 ⇒ x2 – x + 4 = 0 ⇒ ∆ < 0 (no x – intercepts)
2
6. Behavior at Infinity: lim f(x) = lim x – x + 4 = ±∞.
x → ±∞ x → ±∞ x–1
7. Asymptotes: Note that the degree of the numerator of f is exactly one more than the
degree of the denominator of f. y
(3, 5)
So, f has an oblique asymptote. local min.

4
By long division, we have f ( x) = x + .
x–1 5 y=x
So, y = x is an oblique asymptote of f. oblique
asymptote
Next, x = 1 is a vertical asymptote of f.
–1
8. Graph: f(–1) = –3 and f(3) = 5. 3 x

–3
–4 x=1
vertical
asymptote
(–1, –3)
y = f(x) local max.

Applications of the Derivative 234


Check Yourself 14
Plot the graph of each function.
x x +1
1. f ( x) = 2. f ( x) =
x +1 2
2−x
Answers
1. y
y = f(x)
1/2
–1
1 x
–1/2

2. y
y = f(x)

–1 1/2 2
x
–1

Fill in the 3 × 3 field of squares such that


the graph of the derivative is located
1 2 below each graph.
a b c

3 4

d e f

5 6
EXERCISES 2 .5
A. Asymptotes 5. The graph of the y
function
1. Find all the asymptotes of the graph of each function. 3 y = f(x)
f(x) = a(x – 2)2(x + b)
2 1
a. f ( x) = b. f ( x) = is shown in the figure.
x +1 ( x − 1)3
Find a and b. 2 3 x
3 2 3x + 2
c. f(x) = x – 4x – 5x + 6 d. f ( x) =
4−x 6. The graph of the function y
y = f(x)
f(x) = a(x + b)3(x + c) is
−3 x 2
e. f ( x) = f. f ( x) = x +1 shown in the figure. f has
( x + 3)2 1 − x2
an inflection point at –1
2 2
g. f ( x) = ( x − 1) h. f ( x) = 4 − x − 3x x = –1. 2 x
2 2
x + 2x − 3 2 x + 3x − 9 Find the sum a + b + c.
–2
3
i. f ( x) = 2 x − 3 j. f ( x) = 2x
x − 5x − 6 x +9 y
7. Find the equation of a y = f(x)
2
k. f ( x) = x + 2 x + 3 polynomial function of 4
x −1 degree 4 whose graph is
shown in the figure. –1
–2 2 x
3x − 1
2. The curve y = 2
has exactly one vertical
x + x+ m
8. Plot the graph of each rational function.
asymptote. Find m.
x −1 1
a. f ( x) = b. f ( x) = 2
B. Curve Plotting x +1 x +1
3. Plot the graph of each polynomial function. x −1 2
c. f ( x) = 2
d. f ( x) = 2( x + 1)
a. f(x) = x3 – 6x2 x − 2x − 3 x − 4x + 3
b. f(x) = (x – 1)2(x + 3) 2 2
3 2 e. f ( x) = x2 − 9 f. f ( x) = x + x
c. f(x) = x – 2x + x – 2 x + 3x x−2
d. f(x) = (x2 – 4)(3 – x)
e. f(x) = x4 – 2x2 + 1 Mixed Problems
f. f(x) = x(x – 1)(x + 1)2 9. Plot the graph of each function.
›
4. In the figure the graph of y x–2
y = f(x) a. f ( x) = −x 4 − x2 b. f ( x) =
a cubic function y = f(x) x+ 2
x −1
−x
is given. Find the local c. f ( x) = e x− 2 d. f ( x) = x
–1 1 e
minimum value of f. x
e. f(x) = ln(x2 + 4) f. f(x) = sin x + cos2x

–2 10. State how many solutions the equation


› 2x3 + 3x2 – 12x + 3 = a has for each value of a.

Applications of the Derivative 236


CHAPTER SUMMARY
1. L’Hospital’s Rule • To find the absolute extrema of a continuous function on
a closed interval [a, b], use the Closed Interval Method.
• If we have the indeterminate forms 0/0 or ∞/∞.
f ′( x) 1. Find the critical points of f on the interval [a, b].
L’Hospital’s Rule says that if lim exists, then
x → a g′( x) 2. Evaluate f(x) at each critical point.
f ( x) f ′( x)
lim = lim . 3. Evaluate f(a) and f(b).
x →a g ( x) x →a g ′( x )
4. The largest of the values of f found in previous is the
2. Applications of the First Derivatives absolute maximum, shown by fmax[a, b]; the smallest of
these values is the absolute minimum, fmin[a, b].
• A function f is increasing on an interval I if f(x) increases
as x increases for every x in I. 3. Applications of the Second Derivatives
Similarly, f is decreasing on an interval I if f(x) decreases • A function f is concave up on an interval I if the graph of
as x increases for every x in I. f lies above all of its tangent lines on the interval I.
• If f ′(x) > 0 for all the values of x in the interval I, then Similarly, f is concave down on I if the graph of f lies
f(x) is increasing in I. below all of its tangent lines on I.
If f ′(x) < 0 for all the values of x in the interval I, then • If f ′′(x) > 0 for all the values of x on the interval I, then
f(x) is decreasing in I. f is concave up on I.

• A function f has an absolute maximum at c if If f ′′(x) < 0 for all the values of x on the interval I, then
f is concave down on I.
f(c) ≥ f(x) for all the values of x in the domain of f.
• An inflection point is a point where a graph changes its
Similarly, f has an absolute minimum at c if direction of concavity.
f(c) ≤ f(x) for all the values of x in the domain of f. • To find the inflection points of a function, follow the steps:
• A function f has an local maximum at c if 1. Find the points where f ′′(x) = 0 and f ′′(x) does not
f(c) ≥ f(x) for all the values of x in an interval I containing c. exist. These points are the possible inflection points of
the function f.
Similarly, f has a local minimum at c if
2. Construct the sign chart of f ′′(x). If the sign of f ′′(x)
f(c) ≤ f(x) for all the values of x in an interval I containing c.
changes across the point x = a, then (a, f(a)) is an
• The number c in the domain of f is called a critical point inflection point of f.
if either f ′(c) = 0 or f ′(c) does not exist. • The Second Derivative Test is an alternative test for
• If a function f has a local extremum at c, then c is a determining whether a critical point of f is a local
critical point of f. maximum or a local minimum. Let f be twice
• Let c be a critical point of a function f. The First differentiable on an interval I and c be a critical point of
Derivative Test says that: f in I such that f ′(c) = 0.
1. If f ′′(c) > 0, then f has a local minimum at x = c.
1. if f ′ changes from positive to negative at c, then f has
a local maximum at c. 2. If f ′′(c) < 0, then f has a local maximum at x = c.

2. if f ′ changes from negative to positive at c, then f has 4. Optimization Problems


a local minimum at c. • To solve optimization problems, follow the steps:
3. If f ′ does not change sign at c, then f has no local 1. Determine the quantity to be maximized or minimized
maximum or minimum at c. and label it with a letter (say M for now).
• If a function f is continuous on a closed interval [a, b], 2. Assign letters for other quantities, possibly with the
then f has both an absolute maximum and an absolute help of a figure.
minimum on [a, b]. 3. Express M in terms of some of the other variables.

237 Derivatives
4. Use the information given in the problem to write M 5. Intercepts: In y = f(x) setting x = 0 gives the y-intercept
as a function of one variable x, say M = M(x). and y = 0 gives the x-intercept(s). To find the
5. Find the domain of the function M(x). x-intercept(s) may be difficult, in which case we do not
use this information.
6. Find the maximum (or minimum) value of M(x).
6. Behavior at Infinity: Find lim f(x) and lim f(x) to
5. Plotting Graphs x → +∞ x → –∞

• An asymptote is a line that a curve approaches more and see how the graph of f behaves as x → ±∞.
more closely until the distance between the asymptote 7. Asymptotes: Find all the asymptotes of the graph and
and the points on the curve must approach zero. draw the asymptotes in a coordinate plane by using
• The line x = a is a vertical asymptote of the graph of f(x) dashed lines.

if either lim+ f(x) = ±∞ or lim– f(x) = ±∞. 8. Graph: Start graphing by plotting the local extrema,
x→ a x→ a inflection points, and intercepts. Then, using the rest
P( x) of the information, complete the plot by joining the
• A rational function f ( x) = has a vertical
Q( x) plotted points.
asymptote x = a whenever only the denominator of f(x)
Concept Check
equals zero (that is, Q(a) = 0 but P(a) ≠ 0).
• What does L’Hospital’s Rule say? How do we know how
• The line y = b is a horizontal asymptote of the graph of
many times to use it in a given problem?
f(x) if either lim f(x) = b or lim f(x) = b. • How can we use L’Hospital’s Rule if we have
x→ – ∞ x→+∞

• To find the horizontal asymptote of a rational function, the indeterminate forms ∞ ⋅ 0 and ∞ – ∞?
apply the rule: • What is the relation between the first derivative and the
increasing and decreasing behavior of the function?
⎧ ±∞, n > m
an xn + an −1x n −1 +...+ a1x + a0 ⎪⎪ • Explain the difference between an absolute extremum
lim = ⎨ an / bm , n = m and a local extremum.
x →±∞ b x m + b m −1
m m −1x +...+ b1x + b0 ⎪
⎪⎩ 0, n < m • Define a critical point of a function.
• How can we determine that a critical point of a function
• The line y = mx + n is an oblique asymptote of the is a maximum, a minimum, or neither?
graph of f(x) if either
• In what conditions does a function have both the
lim[ f ( x) − ( mx + n )] = 0 or lim [ f ( x) − ( mx + n)] = 0. absolute maximum and absolute minimum?
x →∞ x →−∞
• Discuss the significance of the sign of the second
• If the degree of the numerator of a rational function is derivative.
exactly one more than the degree of its denominator,
• What is an inflection point?
then the graph of the function has an oblique asymptote.
• What does a curve y = f(x) on which f ′ < 0 and
• A polynomial function has no asymptotes. f ′′ > 0 look like?
• To plot the graph of a function, follow the steps: • Let n be a positive integer. For what values of n does the
1. Domain: Find where f(x) is defined. function f(x) = xn have an inflection point at the origin?
2. Intervals of Increase and Decrease: Construct the • What are the relative advantages and disadvantages of
sign chart of f ′(x) to determine the intervals where the first and second derivative tests?
f(x) is increasing and where f(x) is decreasing. • Describe the procedure to solve optimization problems.
3. Local Extrema: Find the critical points of f and • What is an asymptote?
classify each as a maximum, a minimum, or neither • How many different asymptotes are there?
by using the First Derivative Test. • Give an example of a function whose graph has an
4. Concavity and Inflection Points: Construct the sign oblique asymptote.
chart of f ′′(x) to determine the intervals where f(x) • Describe the procedure to plot the graph of a function.
is concave up and where f(x) is concave down. With • When plotting the graph of a function, how do we know
the help of the chart, find the inflection points. the behavior of the function at infinity?

Chapter Summary 238


CHAPTER REVIEW TEST 2A
x2 + x – 2 5. For what values of k is f(x)=x3 +(k+1)x2 +3x+2
1. Find lim 2 .
x →1 x – 4 x + 3 always increasing?
3 3 2
A) 1 B) − C) 0 D) – E) A) –6 < k < 3 B) k > 0
2 4 3
C) –4 < k < 0 D) –3 < k < 2

E) –4 < k < 2

6. Which of the following is a local extremum of


x
1 – sin f(x) = x3 – 3x2 + 3x + 1?
2. Find lim 2.
x →π π–x
A) –6 B) –1 C) 0
1 1
A) 1 B) C) 0 D) – E) –1
2 2 D) 2 E) no extremum

7. Which of the following is false y


y = f(x)
for the graph of the
function y = f(x)? 1
3. Find the interval on which f(x) = x – 6x + 2 is2 –2 –1 2 x

decreasing.
A) f(2) = 0 B) f ′(–1) = 0
A) (–∞, 3) B) (–3, 3) C) (3, ∞)
C) f ′′(1) > 0 D) f ′′(–1) > 0
D) (0, ∞) E) (–∞, 6) E) f ′(0) < 0

8. Find the interval on which y = (x + 2)3 is


concave up.
4. Find the maximum value of the function
2
f(x) = e4x – x . A) (–2, ∞) B) (–∞, –2) C) (–2, 2)

A) 1 B) e C) e2 D) e4 E) e8 D) (–∞, ∞) E) (2, ∞)

239 Derivatives
9. For what value of m does the polynomial 13. Let x1 and x2 be the roots of the equation
P(x) = x4 + x3 + (m – 1)x2 have an inflection y = x2 – (m + 1)x + 2m – 1 = 0. Find the value
point at x = –1? of m that minimizes x12 + x22.

A) –3 B) –2 C) –1 D) 0 E) 1 A) 0 B) 1 C) –1 D) 2 E) –2

x2
14. Find the point on the parabola y = that is
10. The graph of the derivative y 2
y = f ¢(x) 3
of the function closest to the point (– , 0).
2
f(x) = x3 + ax2 + bx + 1 is 1 1
given in the figure. A) ( –1, ) B) (1, ) C) (0, 0)
2 2 2
Find a + b. x 1 1 1 1
–1 D)(– , ) E) ( , )
2 8 2 8

A) 17 B) 11 C) 5 D) –17 E) –10

15. Given the graph of a y

cubic function f, find 5


2
f(2).
11. Find the intersection point of the asymptotes of
3– x
y= . –1 1 x
x+ 2
y = f(x)
A) (–2, 3) B) (3, –1) C) (–2, –1) 3 5
A) B) 0 C) –1 D) –2 E) –
2 2
D) (2, 1) E) (–1, 3)

16. Which one of the following graphs could be the


graph of y = x4 – 2x2?
12. Which of the following is true for the function
A) y B) y C) y
f(x) = 2x3 + 3x2 + 12x + 4?

A) f has a local minimum at x = 0.


x x x
B) f′(2) < 0
1 D) y E) y
C) f is concave up on ( −∞, − ).
2
D) f is always increasing. x

E) f has a local maximum at x = –1. x

Chapter Review Test 2A 240


CHAPTER REVIEW TEST 2B
3
x –1 6. Given that f(x) = a ln x + bx2 + x – 2 has a local
1. Find the value of the limit lim . minimum at x = 1 and a local maximum at
x →1 x2 – 1
1 1 1 x = 2, find a + b.
A) 0 B) C) 1 D) E)
2 3 6
5 2 1 5 4
A) – B) – C) – D) – E) –
6 3 6 3 3

3x3 +5 x + 4 7. Find the sum of the maximum and minimum


2. Find the value of the limit lim .
x →∞ e x + 3x x–2
values of f ( x) = on the interval [–4, 1].
3 5 x–3
A) 3 B) 0 C) D) ∞ E)
e 3
19 9 17 8
A) B) C) 1 D) E)
14 7 14 7

3. Let f be an increasing function on the closed x x


a
8. Given that lim a – b = 2, find .
interval [–4, 4]. Which of the following is definitely x →0 x b
true?
A) 0 B) 1 C) e D) e2 E) e2 + 1
A) f(3) > 0 B) f(–2) < 0

C) f ′ (1) < 0 D) f(2) < f(–2)


9. Which of the following is y
E) f(–1) < f(1)
false for the graph of the
function y = f(x)? –3 –1 4
–4 2 x
y = f(x)

x2 + x
4. If the function f ( x) = has a local extremum 11
x+ a A) f ′(2) = 0 B) f ′′ ( ) <0
at x = 2, find a. 5

4 2 C) f(–1) = 0 D) f ′(1) > 0


4 2
A) B) C) –1 D) – E) – 13
5 5 5 5 E) f ′′ ( – ) < 0
4

10. If x = a and y = b are the asymptotes of the graph


5. Find the local minimum value of f ( x) = x + 84 . x2 + 3x + 2
x of f ( x) = , find a + b.
x2 – 2 x – 3
5 3 5 3
A) 1 B) C) D) E) A) 1 B) 2 C) 3 D) 4 E) 5
2 4 4 2

241 Derivatives
11. Find the area of the largest 15. Which of the following is y
D C
rectangle that can be the function whose graph
2
inscribed in a semicircle of is given in the figure?
radius 3. A O B
A) f(x) = (x + 1)2(x – 2) –1 2 x
A) 2ñ2 B) 3ñ2 C) 9 B) f(x) = (x + 1)(2 + x) y = f(x)

D) 8ñ2 E) 9ñ2 C) f(x) = (x + 1)(2 – x)


D) f(x) = (x + 1)2(2 – x)
E) f(x) = (x – 1)2(x – 2)

12. Find the maximum possible area of a right


triangle whose hypotenuse is 6 cm.

A) 36 B) 12 C) 3 D) 18 E) 9
16. Which one of the following could be the graph of
2x
the function f ( x) = ?
x2 – 1
A) y B) y

13. Given the graph of the y

function f, find the slope of 3


–1 –1
the tangent line to the 1 1
–4 x x
graph of g(x) = x ⋅ f(x) at
–3 x
x = – 3.
y = f(x)

A) 3 B) –4 C) –6 D) 0 E) –3 y
C) D)
y

–1
–1 1 x
14. y 1 x
y = f ¢(x)

–3 –1 5 7
–5 2 4 x y
E)

Given the graph of the derivative of a function f,


–1
what is the sum of the abscissa of inflection 1 x
points of f?

A) –9 B) –6 C) 4 D) 7 E) 16

Chapter Review Test 2B 242


CHAPTER REVIEW TEST 2C
ln(2 x2 − 1) 5. Find m if the inflection point of the function
1. Find lim . 1 3 2
x →1 sin( x − 1) f ( x) = x − x2 + mx + is on the parabola
3 3
A) 2 B) 4 C) 8 D) 16 E) 32 2
y = x – 2x + 3.

A) 2 B) 1 C) 0 D) –1 E) –2

6. y

2. Given that the function f(x) = x4 – ax3 + bx2 – 2x + 3


has a local minimum at the point (1, 2), find a.
–1 1
A) –2 B) –1 C) 0 D) 1 E) 2 –5 –3 –2 4 6 x

y = f ¢(x)

The graph of the derivative of the function f is


given in the figure. For what value of x does f have
a local maximum?

A) –5 B) –4 C) –3 D) –2 E) –1
3. Find the interval on which y = xln x is increasing.

A) (–∞, –e) B) (–e, 0) C) (1, e)


1 1
D) (–∞, ) E) ( , ∞) 7. At what point does the tangent to the graph
e e
f(x) = x3 – 3x2 – 4x – 5 have the smallest slope?

A) (–2, 7) B) (–1, 1) C) (0, –5)

D) (1, –11) E) (2, 7)

4. Let A(x1, y1) and B(x2, y2) be the extremum 8. The length of a line segment [AB] is 10 cm. Take a
2
x +1 point C on [AB]. What should the value of |AC| so
points of y = . Find the distance between
x that |AC|2 + |BC| is a minimum?
these points.
1 1 1 1 2
A) B) C) D) E)
A) ñ5 B) 2ñ5 C) 3ñ5 D) 4ñ5 E) 5ñ5 5 4 3 2 3

243 Derivatives
9. Which one of following could be the value of m if 14. A right cone is inscribed in a sphere of diameter 6 cm.
2
x −1 What is the maximum possible volume of the cone?
the function f ( x) = has no local extrema?
mx + 3
32π 24π 16 π 12 π 8π
A) –2 B) –1 C) 0 D) 2 E) 4 A) B) C) D) E)
3 3 3 3 3

10. The function f(x) is a positive valued increasing


function on the interval (a, b). Which one of the 15. The graph of the derivative y
following functions is decreasing on the same of a function f is given
interval? in the figure. Which d
a b c x
1 of the following
A) f 2(x) B) − C) f 3(x) y = f ¢(x)
f ( x) statements is false?
1
D) x + f(x) E) 2 A) f ′(a) = 0
f ( x)
B) f′′(0) = 0
11. Given that y = 3x – 3 is an oblique asymptote of C) f has a local minimum at x = a.
( m − 3)x3 − 4 x +1 D) f has a local maximum at x = d.
the graph of y = , find m.
x2 + x − 3
E) f has a local maximum at x = 0.
A) 3 B) 4 C) 5 D) 6 E) 7

12. Which of the following is y 16. The graph of a function f y


the function whose graph is given in the figure.
y = f(x)

is given in the figure? 1


3 Which of the following
–1 1
–1 1 3 could be the graph of –2 2 x
x
its derivative? –1

A) y B) y

x +1 x −1
A) y = 2
B) y = 2 –1 1
x − 2x − 3 x − 2x − 3 1 x –1 x
x +1 x +1
C) y = 2 D) y = 2
x + 2x + 3 x +3
C) y D) y
−1
E) y = 2
x − 2x − 3
–1 1 x –1 1 x
13. Let f(x) = (x – a)(x – b)(x – c) and a < b < c.
Which one of the following statements is false? y
E)
A) f ′(a) > 0 B) f ′(b) <0 C) f ′(c) > 0
–1 1
D) f ′′(a) < 0 E) f ′′(c) < 0 x

Chapter Review Test 2C 244


CHAPTER 7

INDEFINITE INTEGRAL
1 ANTI-DERIVATIVE AND THE INDEFINITE INTEGRAL
A. DEFINITION OF THE INDEFINITE INTEGRAL
The set of all antiderivatives of a function is called the indefinite integral of the given
function.

Definition
Let F(x) be a differentiable function such that F′(x) = f(x) then F(x) is called the primitive
of function f(x) and the expression F(x) + c is called as the indefinite integral of f(x).

We show the indefinite integral by: ∫ f ( x) dx = F( x)+ c


In the given expression:

Integral Sign The Primitive of f(x)

∫ f ( x)dx = F( x)+ c
Integrand Differential Constant of Integration

“dx” is used to show which term is used for variable.

We can read the expression: ∫ f ( x) dx as “the integration of f(x) with respect to x”


To find the integral of f(x) we will answer the question: “Derivative of which function is f(x)?”

EXAMPLE 1 ∫ 2x dx = x
2
+ c.

(The derivative of x2 is 2x so, ∫ 2x d( x) = x


2
+ c. )

EXAMPLE 2 ∫ 4x
3
dx = x4 + c.

(The derivative of x4 is 4x3 so, ∫ 4x


3
dx = x4 + c. )

246 Integral
EXAMPLE 3 ∫ sin x dx = – cos x+ c.

(The derivative of –cos x is sin x so, ∫ sin x dx = – cos x+ c. )


In the given examples we use “+c”. Why do we use it? Before answering this question look
at the derivatives of given functions :
y = x2, y′ = 2x
2
y = x + 3, y' = 2x
2
y = x – 6, y' = 2x
As you see, the derivatives of all given three functions are the same but the primitive
functions are different. The only difference is the constant numbers of given functions, the
variables are the same. This means, for antiderivatives we cannot find exact constant parts.
For this reason we wrote “+c” after the derivative of given function to show any constant
number.

Note
In every indefinite integral we must use the constant of integration.

EXAMPLE 4 If f ′(x) = 2x and f(3) = 7 then find f(x).

Solution
∫ f ′( x) dx = ∫ 2 x dx = x
2
If f ′(x) = 2x then f(x) = +c
f(3) = 32 + c = 7
c = –2
2
So, f(x) = x – 2.

EXAMPLE 5 ∫ dx = ?

Solution We know that: dx = 1 ⋅ dx and y = x, y′ = 1


So, ∫ dx = ∫ 1 ⋅ dx = x + c.

EXAMPLE 6 ∫ dy = ?
Solution By the same method of example 5 we can say that the
variable of this example is y, so, ∫ dy = y + c.

The Indefinite Integral 247


EXAMPLE 7 ∫ d(tan x) = ?

Solution Use tan x as variable then ∫ d(tan x) = ∫1 ⋅ d(tan x) = tan x+ c

B. THE PROPERTIES OF THE INDEFINITE INTEGRAL


1. Differentiation of the indefinite integral is equal to the expression under integral sign:
d ∫ f ( x) dx = f ( x) dx
d d
2. Derivative of indefinite integral is equal to the integrand:∫ f ( x) dx= ∫ f ( x) dx= f ( x)
dx dx
Integral and derivative are inverse operations so we can simplify them.
3. The indefinite integral of differentiation of a function is the same function with a constant
term added: ∫ dF( x) = F( x )+ c
4. Constant multipliers can be taken outside of integral sign: ∫ a ⋅ f ( x) dx = a ⋅ ∫ f (x) dx
5. Integration of sum or difference of two functions is equal to the sum or difference of
integrations of given functions.

∫ [ f ( x) ± g( x)] dx = ∫ f (x) dx ± ∫ g(x) dx


Proof To make proofs we will use the definition of integration: F′(x) = f(x) then
∫ f ( x) dx = F( x) + c.
1. We will differentiate both sides of:

∫ f ( x) dx = F( x) + c; d ∫ f ( x) dx = dF( x) + c = F ′(x) dx + dx = f (x ) dx
2. ( ∫ f ( x) dx)′ = (F(x) + c)′ = F′(x) + c′ = f(x) + 0 = f(x)

3.and 4. left as exercises for the students.


5. Let's take the derivative of both sides :
( ∫ [ f ( x) ± g( x)] dx)′ = ( ∫ f ( x) dx ± ∫ g( x) dx) ′

f(x) ± g(x) = ( ∫ f ( x) dx)′ ± ( ∫ g( x) dx)′ = f(x) + g(x)

EXAMPLE 8 f ( x) = ∫ d( x3 – 2 x2 +1). Find f(2) if f(1) = 0.

Solution (By 3rd property )


f ( x) = ∫ d( x3 – 2 x2 +1) = x3 – 2x2 + 1 + c
f(1) = 13 – 2 ⋅ 12 + 1 + c = 0 then c = 0 and f(x) = x3 – 2x2 + 1
f(2) = 23 – 2 ⋅ 22 + 1 = 8 – 8 + 1 = 1

248 Integral
EXAMPLE 9 ∫x
2
⋅ f ( x) dx = 3x4 + 4x3 – x2 then find f(x). (x ≠ 0)

Solution ( By 2nd Property ) Take the derivative of both sides:


d d
dx ∫ x2 ⋅ f ( x) dx = (3x4 + 4 x3 – x2 )
dx
x2 ⋅ f ( x) =12 x3 +12 x2 – 2 x
2
f ( x) =12 x +12 –
x

EXAMPLE 10 Evaluate the integral ∫ (sin x – 3x


2
+ e x ) dx.

(By 5th Property ) ∫ (sin x – 3x + e x ) dx = ∫ sin x dx – ∫ 3 x 2 dx + ∫ e x d


2
Solution
(we know, (cos x)' = –sin x, (x3)′ = 3x2, (ex)′ = ex) = –cos x – x3 + ex + c

EXAMPLE 11 If f ′(x) = 12x3 + 6x2 – 4x – 5 and f(2) = 5 then find the value of f(1).

Solution (By 2nd,4th and 5th Properties ) Integrate both sides.

∫ f ′( x) dx = ∫ (12 x
3
+6 x2 – 4 x – 5) dx

f ( x) = ∫ 12 x3 dx + ∫ 6 x2 dx – ∫ 4 x dx – ∫ 5 dx

= 3 ⋅ ∫ 4 x3 dx + 2.∫ 3x2 dx – 2 ∫ 2 x dx – 5 ∫ dx

= 3 ⋅ x4 + c1 + 2 ⋅ x3 + c2 – 2 ⋅ x2 + c3 – 5x + c4
= 3x4 + 2x3 – 2x2 – 5x + (c1 + c2 + c3 + c4 )
(Instead of c1 + c2 + c3 + c4 we can write just c, because c is any constant real number like
c1 + c2 + c3 + c4 )
So, f(x) = 3x4 + 2x3 – 2x2 – 5x + c
f(2) = 5 so 5 = 3 ⋅ 24 + 2 ⋅ 23 – 2 ⋅ 22 – 5 ⋅ 2 + c = 48 + 16 – 8 – 10 + c = 46 + c
So c = –41 and f(x) = 3x4 + 2x3 – 2x2 – 5x – 41
So, f(1) = 3 ⋅ 14 + 2 ⋅ 13 – 2 ⋅ 12 – 5 ⋅ 1 – 41 = –43

The Indefinite Integral 249


Check Yourself 1
1. Evaluate given integrals by using the definition of integration:

∫ 6x ∫ 3sin x dx ∫ 4e
5 2x
a. dx b. c. dx

2. By using the properties of indefinite integral calculate the following :


⎛ 1 ⎞
a. ∫ ( x3 + 4 x2 − 3x − 1) dx b. ∫ ⎜ 3 + 4cos x − e x ⎟ dx
⎝x ⎠

∫x
3
3. If ⋅ f ( x) dx = x5 − 4x3 + 2 x2 +1 then find f(x).
Answers

1. a. x6 + c b. –3cos x + c c. 2e2 + c
X

x4 4x3 3x2 1 12 4
2. a. + − − x+ c b. − + 4sin x − e x + c 3. f ( x) = 5x − +
4 3 2 2 x2 x x2

C. BASIC INTEGRATION FORMULAS


By using rules and methods of derivative we can get the following formulas for indefinite
integral:

xn+1
1. ∫ x dx =
n
a. +c n ≠ –1
n +1

b. ∫ a dx = ax + c for a ∈ R

Note
For proving the basic integral formulas we will take the derivative of right side and at the end
we will try to get the integrand. If we can get the integrand after differentiation then it will
be the end of the proof.

EXAMPLE 12 ∫x
3
dx = ?

x3+1 x4
Solution ∫x
3
By using the first rule we get: dx = + c = + c.
3+1 4

250 Integral
EXAMPLE 13 ∫x
1
4
dx = ?

Solution 1 x−4+1 x−3 1


∫x 4
dx = ∫ x−4 dx =
−4 + 1
+c=
−3
+ c = – 3 + c.
3x

EXAMPLE 14 ∫ 3x
5
dx = ?

Solution x6 x6
∫ 3x dx = 3∫ x dx = 3 ⋅
5 5
+ c = + c.
6 2

EXAMPLE 15 ∫ (3x
4
+ 4x3 – 2 x2 + x – 5) dx = ?

Solution x5 x4 x3 x2
∫ (3x + 4x − 2x + x − 5) dx = 3
4 3 2
+4 −2 + −5 x + c
5 4 3 2
3x5 2 x3 x2
= + x4 – + − 5 x + c.
5 3 2

Check Yourself 2
Evaluate the following integrals
x3 −3
∫x ∫(x
2
a. ∫ dx b. 2
dx c. – x3 + x4 ) dx
2

∫ ( x +1) ∫ (3x
2 3
dx + 4x2 − x) dx
d. ∫ ( x − 1) x dx e. f.

Answers
x4 3 x3 x4 x5
a. +c b. +c c. − + +c
8 x 3 4 5
2 x5 / 2 2 x3 / 2 x3 3x4 4x3 x2
d. – +c e. + x2 + x + c f. + – +c
5 3 3 4 3 2
The Indefinite Integral 251
2. 1
a. ∫x dx = ln| x |+ c

b. u '( x)dx
∫ u( x)
= ln| u( x)|+ c

EXAMPLE 16 ∫y
1
dy = ?

Solution 1
∫y dy = ln| y |+ c ( y′ =1)

EXAMPLE 17 ∫x
2x
2
dx = ?

Solution 2x
∫x 2
dx = ln|x2| + c = ln x2 + c ((x2)′ = 2x and x2 ≠ 0)

EXAMPLE 18 cos x
∫ sin x dx

Solution cos x
∫ sin x dx = ln|sin x| + c ((sin x)′ = cos x)

EXAMPLE 19 ∫ x−3
4
dx = ?

Solution 4
∫ x−3 dx = 4 ⋅ ln|x – 3| + c ((x – 3)′ = 1)

Check Yourself 3
Evaluate the following integrals
⎛1 1 1 ⎞ 5x5 + 2 x2 + 3x – 5 3
a. ∫ ⎜⎝ x – x2 – x3 ⎟⎠ dx b. ∫ x2
dx c. ∫ 1 + 5x dx

Answers
1 1 5 x4 5 3
a. ln| x |+ + 2 + c b. + 2 x + 3 ln| x|+ + c c. ln|5 x +1|+c
x 2x 4 x 5
252 Integral
∫e
x
3. a. dx = e x + c

ax
b. ∫ a dx =
x
+c
ln a

EXAMPLE 20 ∫3
x
dx = ?
ex is the only function whose
primitive (integration) is
equal to it self when c = 0

Solution 3x
∫ 3 dx =
x
+c
ln 3

EXAMPLE 21 ∫4⋅5
x
dx = ?

Solution 4.5 x
∫ 4 ⋅ 5 dx = 4∫ 5 dx =
x x
+c
ln5

EXAMPLE 22 ∫ 4e
x
dx = ?

∫ 4e
x
Solution dx = 4e x + c

23 ∫7
4 x− 3
EXAMPLE dx = ?

Solution (7 4 )x (7 4 )x 7 4x−3
∫7 dx = ∫
4 x− 3
3
dx = 3 4
+c= +c
7 7 ⋅ ln7 4 ⋅ ln7

EXAMPLE 24 ∫ 5e
x +1
dx = ?

∫ 5e dx = 5e ⋅ ∫ e x dx = 5e ⋅ e x + c = 5e x +1 + c
x +1
Solution

The Indefinite Integral 253


EXAMPLE 25 ∫e
3 x +1
dx = ?

∫e dx = e ⋅ ∫ ( e 3 )x dx
3 x +1
Solution

Let a = e 3. Then

( e3 )x
=e⋅ +c
ln( e 3 )

e3 x+1
= +c
3

Note
We can make the following generalizations:
1 ax + b
∫e
ax + b
1. dx = e +c
a
1 amx + n
∫a
mx + n
2. dx = +c
m ln a

EXAMPLE 26 ∫e
4 x −1
dx = ?

Solution 1
∫e
4 x −1
dx = e4 x−1 + c
4

EXAMPLE 27 ∫5
3 x +1
dx = ?

Solution By the previous rule;


1 53 x +1 53 x +1
∫5
3 x +1
dx = +c= +c
3 ln5 3 ln5

254 Integral
Check Yourself 4
Evaluate the following integrals

a. ∫ 5 ⋅ 3x dx b. ∫ (2 x − 3x ) dx c. ∫ 65 x +1 dx d.
⎛ 4x + 3 ⎞
∫ ⎜⎝
x+3
3 ⋅ 5 + ⎟ dx
3 ⎠
e. ∫ 3e3 x dx f. ∫ 2e
(2 x − 3)
dx g. ∫2
4x−4
dx h. ∫4
ex + π
dx

Answers
x x x 5 x +1 x+3 x+3
a. 5 ⋅ 3 b. 2 – 3 + c c. 6 +c d. 3 ⋅ 5 + 4 +c
ln 3 ln 2 ln 3 5 ln6 ln5 3 ln 4
4x−6 2 ex + 2π −1
e. e3x + c f. e2x-3 + c g. 2 +c h. 2 +c
ln 2 e ⋅ ln 2

4. a. ∫ sin x dx = − cos x + c

1
b. ∫ sin( ax + b) dx = – cos( ax+ b)+ c
a

c. ∫ cos x dx = sin x + c

1
d. ∫ cos( ax + b) dx =
a
sin( ax + b)+ c

e. 1
∫ cos 2
x
dx = ∫ sec 2 x dx = ∫ (1+ tan 2 x) dx = tan x+ c

1 1
f. ∫ cos ( ax + b)
2
dx = ∫ sec 2 ( ax + b) dx =∫ (1+ tan 2( ax+ b) dx = tan( ax+ b)+ c
a

1
g. ∫ sin 2
dx = ∫ csc 2 xdx =∫ (1+ cot 2 x)dx = − cot x+ c
x

1 1
h. ∫ sin ( ax + b) dx = ∫ csc ( ax + b) dx = ∫(1+ cot ( ax+ b) dx = − a cot( ax+ b)+ c
2 2
2

1
i. ∫ 1 − x2
dx = arcsin x + c1 = − arccos x+ c2

1
j. ∫ 1+ x 2
dx = arctan x + c1 = − arc cot x+ c2

The Indefinite Integral 255


EXAMPLE 28 ∫ 4sin x dx = ?

Solution 4∫ sin x dx = −4cos x+ c

EXAMPLE 29 ∫ sin 3x dx = ?

Solution 1
∫ sin 3x dx = − 3 cos 3x + c

EXAMPLE 30 ∫ cos(7 x + 3) dx = ?

Solution 1
∫ cos(7 x+ 3) dx = 7 sin(7 x+ 3)+ c

EXAMPLE 31 ∫ 5 ⋅ sin (2 x – 1) dx?

Solution ∫ 5 ⋅ sin (2 x – 1) dx = 5 ⋅ ∫ sin ( x – 1) dx


1
= 5⋅ ⋅ (– cos(2 x – 1)) + c
2
5
= – cos(2 x – 1)+ c
2

EXAMPLE 32 ∫ cos
1
2
3x
dx

Solution 1 1
∫ cos 2
3x
dx =
3
⋅ tan 3x + c

256 Integral
EXAMPLE 33 ∫ csc (5 x − 3) dx
2

1
Solution ∫ csc (5 x − 3) dx = − 5 ⋅ cot(5 x − 3)+ c
2

EXAMPLE 34 ∫ cos
2
x dx

1+ cos 2 x
∫ cos x dx = ∫
2
Solution dx
2
1 cos 2 x
= ∫ dx + ∫ dx
2 2
x 1 1
= + c1 + . sin 2 x+ c2
2 2 2
x sin 2 x
= + +c
2 4

EXAMPLE 35 ∫ cot
2
x dx = ?

Solution ∫ cot x dx = ∫ (cot 2 x +1 − 1) dx


2

= ∫ (cot 2 x +1) dx − ∫ dx

= − cot x + c1 − x + c2

= − cot x − x − c

EXAMPLE 36 ∫ cos
2
3 x dx − ∫ sin 2 3 x dx = ?

Solution ∫ cos 3 x dx − ∫ sin 2 3 x dx = ∫(cos 2 3 x − sin 2 3 x) dx


2

= ∫ cos6 x dx

1
= sin6 x + c
6

The Indefinite Integral 257


EXAMPLE 37 ∫ 3tan (2 x+1)
2
dx = ?

Solution ∫ 3tan (2 x + 1)
2
dx = 3 ∫ tan 2(2 x +1) dx

= 3∫ (1 + tan 2(2 x + 1) −1) dx

1
= 3( tan(2 x+1) – x)+ c
2
3
= tan(2 x +1) − 3 x+ c
2

Check Yourself 5
Evaluate the following integrals
3 ⎛ 1 1 ⎞
a. ∫ (sin x − cos x) 2 dx b. ∫ (2 sin x − 3cos x) c. ∫ 4 − 4 x2
dx d. ∫ ⎜⎝ cos 2
x
+ ⎟ dx
sin 2 x ⎠
e. ∫ sin 4x dx f. ∫ cos(5 x − 1) dx g. ∫ tan
2
2x h. ∫ cot (3x+1)
2
dx

5
i. ∫ sin j. ∫ 1+ x
2
x dx 2
dx

Answers
1 3
a. x + cos 2 x + c b. –2cos x – 3sin x + c c. arcsin x + c d. tanx – cotx + c
2 2
1 1 tan 2 x cot(3 x +1)
e. − cos 4 x + c f. sin(5 x − 1)+ c g. − x+ c h. − − x+ c
4 5 2 3
x sin x cos x
i. − +c j. 5arctan x + c
2 2

258 Integral
EXERCISES 1 .1
A. Definition of the Indefinite Integral 5. If f ′(x) = 5x2 – 4x + 1 and f(1) = 3 then find f(3).
1. Evaluate the following integrals
a. ∫ dw b. ∫ dz

c. ∫ d cos x d. ∫ d( x
3
+ 3x2 − 1)

6. Evaluate the following integrals.


a. ∫ (cos x + 4 x3 − 2 e2 x ) dx

b. (5x3 − 3x2 +5) dx c. 5dx


∫ ∫ 1+ x 2

d. e.
∫ 7x ∫ cos 4x dx
8
dx
B. The Properties of the Indefinite
Integral f.
∫ 7 ⋅ sin x dx g.
∫5⋅ e
3x
dx
2. If f(x) = ∫ d( x − 1) and f(1) = 2 then find f(5).
2

h. (2 cos 3 x + 4sin x − 4 e5 x ) dx

3. If ∫ x ⋅ f ( x )dx = x 2 +5x +1 then find f(x).


C. Basic Integration Formulas
7. Evaluate the following integrals by using basic
integration rules and properties.

a. b.
∫x ∫ 4 dx
5
dx
1
c.
∫x
−3
dx d. ∫x 5
dx

1 1 1
e. f. ∫ (
∫ 3x + − ) dx
7
dx
x3 x6 x9
4. If ∫ x3 ⋅ f ( x)dx = x5 − 4x3 − x +1 then find f(2). g.
∫8 ⋅ e
2x
dx h. (3x2 + 4x − 1) dx

259 Integral
8. Evaluate the following integrals by using the basic 10. Evaluate the following integrals by using basic
integration rules. integration formulas.
x3
a.
∫ dx a. ∫ sin 4x dx b. ∫ cos5 x dx
x3 4 5
3 x2
c. ∫ cos 2
x
dx d. ∫ sin 2
2x
dx
b. ∫ 3 dx
x
∫ 4 ⋅ sec ∫ tan
2 2
e. 4 x dx f. x dx
1 1
c. ∫ ( 2 + +1+ x) dx 3
x x g. ∫ (cot h. ∫
2
x + 2) dx dx
1 − x2
d. ∫ (sin x + cos x) dx 4
i. ∫x dx j. x2 +5
2
+1 ∫x 2
+1
dx
x dx
e. ∫ x2 k. ∫ 5 ⋅ cos(8 x − 4) dx l.

1
dx
1 − 4 x2
5 5
f. ∫ x +1 dx m.
∫ 9x 2
+1
dx n. ∫ sin
2
x dx

1 o. ∫ cot p. ∫ (tan 2 x − 1) dx
2
x dx
g. ∫ x − 1 dx
4x3 + 3x2 − 4x +1
h. ∫ x2
dx

9. Evaluate the following integrals by using basic


integration formulas.

a. ∫ e dx b. ∫ e dx
2x 5x

c. ∫ 3 ⋅ e dx 2x
d. ∫ 5 ⋅ e7 x+2
dx

e. ∫ 7 ⋅ e x−2
dx f. ∫ 2
2 x+1
dx

g. ∫ 5 dxx
h. ∫ 6
2 x −1
dx

i. ∫ 4 3x− 4
dx j. ∫ 3 dx
3x

k. ∫ 10 dx x +1
l. ∫ 4 ⋅ 32 x –1
dx

The Indefinite Integral 260


2 METHODS OF INTEGRATION
For some questions using just the basic integration rules will not be enough to get the answer.
Because of this reason we must know some more rules or methods to calculate the integral. For
finding the integral of a function we have some other methods. Now, we will learn these methods.

A. METHOD OF CHANGING VARIABLE (CHAIN RULE OR


SUBSTITUTION METHOD)
For some questions, by using x as variable will not be useful to integrate the function. In these
type of questions we can change the variable. This method is called the method of changing
variable. The following theorem states the method for this operation.

Theorem
Let's say that F(u) and u(x) be two functions which are differentiable with respect to u and
x, respectively. So,

∫ f ( u( x)) ⋅ u′( x) dx = F( u)+ c.


Proof We know from the Chain Rule that, when F′(x) = f(x) then dF( u( x)) = F '( u( x)) ⋅ u '( x).
dx
dF( u( x))
By integrating both sides with respect to x we get: ∫ dx
dx = ∫ f ( u( x)) ⋅ u '( x)dx which

implies.

F(u(x)) + c = ∫ f ( u( x)) ⋅ u′( x) dx which completes the proof.

Note
In general we use the Chain Rule by the following way:
In the given expression do the following two substitutions:
du
u = g(x) by differentiating both sides we get = g′( x) or du = g′(x) dx.
dx
After these substitutions we get a simpler expression and by using basic integration formulas
we can easily integrate the function.
In this method the selection of u is important.
261 Integral
38 ∫ ( x − 1)
2
EXAMPLE dx = ?

Solution 1st Way


By using the power rule we can get the answer;
x3
∫ ( x − 1) dx = ∫ ( x − 2 x + 1) dx =
2 2
− x 2 + x + c1
3
2nd Way
By using changing variable method we can calculate the answer;
Let u = x – 1 the du = dx.
Use this substitution in the question
u3 ( x − 1)3 x 3 − 3 x2 + 3 x − 1 x3 1
∫ ( x − 1) dx = ∫ u 2 du =
2
+ c2 = + c2 = + c2 = −x2 + x + − + c2
3 3 3 3 3
1
We have – + C2 instead of C1 but we say the answers are the same. Why?
3

EXAMPLE 39 ∫ (1 − x)
9
dx = ?

Solution Let u = 1 – x then du = –dx. Substitute it in the question;


u10 (1 – x)10
∫ (1 – x) dx = ∫ u 9 (– du ) = – ∫ u 9 du = –
9
+ c= – +c
10 10

EXAMPLE 40 ∫ 3x
2
cos x3 dx = ?

Solution Let u = x3 then du = 3x2dx. Substitute it into the integral sign then:

∫ 3x cos x3 dx = ∫ cos u du = sin u + c = sin x3 + c


2

The Indefinite Integral 262


EXAMPLE 41 ∫(x
2
+5)7 ⋅ x dx = ?

Solution du
Let u = x2 + 5 then du = 2x dx or = x dx.
2
By substituting these in the question we have:
du 1 7 1 u8 ( x2 + 5)8
∫ ( x +5) ⋅ xdx = ∫ u = ∫ u du = ⋅ + c =
2 7 7
+c
2 2 2 8 16

EXAMPLE 42 ∫ (2x
2
+1)(2 x3 + 3 x)7 dx = ?

Solution Let u = 2x3 + 3x then du = (6x2 + 3 ) dx or du = (2x2 + 1) dx


3
du 1 7 1 u8 (2x3 + 3x )8
∫ (2x +1)(2 x3 + 3 x)7 dx = ∫ u7 = ∫ u du =
2
+ c= +c
3 3 3 8 24

EXAMPLE 43 ∫
ln x
x
dx = ?

Solution 1
Let u = ln x then du = ⋅ dx.
x
ln x u2 ln 2 x
So, ∫ dx = ∫ u du = +c = + c.
x 2 2

EXAMPLE 44 ∫f
5
( x) ⋅ f '( x) dx = ?

Let u = f(x) then du = f ′(x) dx.


Solution
u6 f 6 ( x)
∫ u du =
5
So, +c= +c
6 6

263 Integral
EXAMPLE 45 ∫ 1 − 3x dx = ?

du
Solution Let u = 1 – 3x then du = –3dx or dx = − .
3
du
So, ∫ 1 − 3x dx = ∫ u ⋅ ( −
3
)

3
1 1 1 u 2
= − ∫ u 2 du = − ⋅ +c
3 3 3
2
2 3
= − ⋅ (1 − 3 x) 2 + c
9

EXAMPLE 46 ∫ sin
3
x ⋅ cos x dx = ?

Solution Let u = sin x then du = cosx dx.

u4 sin4 x
∫ sin x ⋅ cos x dx =∫ u 3 du =
3
So, +c= +c
4 4

EXAMPLE 47 ∫ cos
3
x dx = ?

Solution Let u = sin x then du = cos x dx. So,

∫ cos x dx = ∫ cos 2 x ⋅ cos x dx


3

= ∫ (1 − sin 2 x)cos x dx

= ∫ (1 − u 2 ) du

u3
=u− +c
3
sin 3 x
= sin x − +c
3
The Indefinite Integral 264
EXAMPLE 48 ∫
ex
1 − e2 x
dx = ?

Solution Let u = ex and du = ex dx. So,

ex du
So, ∫ dx = ∫
2
1− e x 1 − u2
= arcsin u + c1

= arcsin e x + c1

or

= − arccos u + c2

= − arccos e x + c2

EXAMPLE 49 ∫3
3 x3 + 4 x2 −1
⋅ (9x2 + 8 x) dx = ?

Solution Let u = 3x3 + 4x2 – 1 then du = (9x2 + 8x) dx

3 x3 + 4 x2 −1
∫3 ⋅ (9 x2 +8 x) dx = ∫ 3 u du

3u
= +c
ln 3
3 2
33 x + 4 x −1
= +c
ln 3

EXAMPLE 50 ∫
1
1 − 9 x2
dx = ?

du
Solution Let u = 3x then du = 3 dx or dx = . By substitution we get;
3
1 1 du
∫ 1 − 9x 2
dx = ∫
1−u 3 2

1 1
= arcsin u + c1 = − arccos u+ c2
3 3
1 1
= arcsin 3 x+ c1 = − arccos 3 x+ c2
3 3

265 Integral
EXAMPLE 51 ∫ 1+ 4x
1
2
dx = ?

du
Solution Let u = 2x then du = 2 dx or dx = . By substitution;
2
1 1 du
∫ 1 + 4x 2
dx = ∫1+ u 2
2
1 1
= arctan u + c1 = − arc cot u + c2
2 2
1 1
= arctan 2 x + c1 = − arc cot 2 x + c2
2 2

EXAMPLE 52 ∫
sin(ln x)
x
dx = ?

1
Solution Let u = ln x then du = dx. So,
x
sin(ln x)
∫ x
dx = ∫ sin u du

= – cos u + c

= – cos(ln x)+ c

EXAMPLE 53 ∫ ( x + 2)( x − 1)
4
dx = ?

Solution As you see by using direct substitution we cannot get the answer.
So we will substitute both of the terms.
Let u = x – 1 and du = dx
We can get x = u+1 then x+2 = u + 3. So,

∫ ( x + 2)( x − 1) dx = ∫ ( u + 3) ⋅ u 4 du
4

= ∫ ( u 5 + 3u 4 ) du

u6 u5
= +3⋅ +c
6 5
( x − 1)6 3( x − 1)5
= + +c
6 5

The Indefinite Integral 266


Check Yourself 6
Evaluate the following integrals
5
a. ∫ sin(1 − x) dx b. ∫ (1 − x3 )5 ⋅ x2 dx c. ∫ x ⋅ sin(5x − 1) dx d. ∫ 1 + 9x
2
2
dx

sin x
e. ∫e f. ∫ 1 + cos g. sin(7 x + 1) dx h.
cos x x2 + 4 x − 2
sin x dx 2
dx ∫ ∫5 ⋅ ( x + 2) dx
x
1 3 xdx
i. j. ∫ k. l. ∫ sin x cos
3
∫ 1+ 2 x 2
dx
1 − 4x 2
dx ∫ 1+ x 4
x dx

Answers
3 6 2
5arctan 3 x
a. cos (x – 1) + c b. − ( x − 1) + c c. − cos(5 x − 1) d. +c
18 10 3
2
1 5x + 4 x − 2
e. –e cos x + c f. –arctan(cos x) + c g. − cos(7 x +1)+ c h. +c
7 2 ln5

j. 3arcsin 2 x + c
2 4
i. 2 arctan( x 2 )
+c k. arctan x + c l. − cos x + c
2 2 2 4

B. METHOD OF INTEGRATION BY PARTS


In general, we have two types of solution methods. One is by substitution and second is by
using the integration by parts. The other methods are used by changing given expression to
these two methods. So as being the main method we must know integration by parts very well.
An integral of the form ∫ f ( x) ⋅ g( x) dx sometimes may be hard to evaluate. In these type of
questions we may use another method to find the answer. The following theorem states the
solution method of integration by parts.

Theorem
Let u = f(x) and v = g(x) be two differentiable functions with respect to x, then

∫ u ⋅ v′ dx = u ⋅ v − ∫ v ⋅ u ′ dx
Proof We know from derivative that d(u ⋅ v) = v ⋅ du + u ⋅ dv
If we take the integral of both sides we get: ∫ d(u ⋅ v) = ∫ vdu + ∫ udv ;
u ⋅ v = ∫ v du + ∫ u dv then we get;

∫ u dv = u ⋅ v – ∫ v du which is required.
267 Integral
Note
In this formula there is no certain rule for selection of u and v′ but generally we choose the
function which is reducing when we take its derivative as u. Also logarithmic and inverse
trigonometric functions may be chosen as u. And the functions like ex, sin x, cos x, … are
chosen as v′.

EXAMPLE 54 ∫ x ⋅ e dx = ?
x

Solution Let u = x and v′ = ex then , u′ = 1 and v = ex.

∫ x ⋅ e dx = x ⋅ e – ∫ e dx = x ⋅ e
x x x x
So, – ex + c

EXAMPLE 55 ∫x
2
⋅ ln x dx = ?

1 3
Solution Let u = ln x and v′ = x2 then, u′ = and v = x .
x 3
3 3
x x 1 x3 1 x3 x3 1
So, ∫ x ⋅ ln xdx = ln x ⋅ −∫
2
⋅ dx = ln x ⋅ − ⋅ + c = (ln x − )+ c.
3 3 x 3 3 3 3 3

ln x
EXAMPLE 56 ∫ x4
dx = ?

1 1 x3
Solution Let u = lnx and v′ = . Then, u′ = and v = − .
x4 x 3
ln x 1 1 1
So, ∫ 4 dx = − 3 ⋅ ln x – ∫ − 3 ⋅ dx
x 3x 3x x
ln x 1 −4
3 x3 3 ∫
=– + x dx

ln x 1 1
=– 3
+ ( − 3 )+ c
3x 3 3x
ln x 1
=– 3
− 3 +c
3x 9x

The Indefinite Integral 268


EXAMPLE 57 ∫ x ⋅ cos x dx = ?

Solution Let u = x and v′ = cos x then, u′=1 and v = sin x.


So, ∫ x ⋅ cos x dx = x ⋅ sin x −∫ sin x dx = x ⋅sin x + cos x + c.

EXAMPLE 58 ∫ arctan x dx = ?

Solution 1
Let u = arctan x and v′ = 1 then, u′ = and v = x.
1+ x2
x
So, ∫ arctan xdx = x ⋅ arctan x − ∫ dx, in this part we will use substitution.
1 + x2
dw
Let w = 1+ x2 then dw = 2x dx and x ⋅ dx = then,
2
1
∫ arctan x dx = x ⋅ arctan x − ∫ 2 w dw

1
= x ⋅ arctan x − ln| w|+c
2
1
= x ⋅ arctan x − ln(1+ x2 )+ c
2

EXAMPLE 59 ∫ x ⋅ sin 4x dx = ?

1
Solution Let u = x and v′ = sin 4x. Then, u′ = 1 and v = − cos 4 x.
4
x ⋅ cos 4 x cos 4 x
∫ x ⋅ sin 4x dx = − 4 – ∫ − 4 dx
x ⋅ cos 4 x 1
=− + ∫ cos 4 x dx
4 4
x ⋅ cos 4 x sin 4 x
=− + +c
4 16

269 Integral
EXAMPLE 60 ∫x x − 2 dx = ?

2 3
Solution Let u = x and v′ = x − 2. Then, u′ = 1 and v = ( x − 2) 2 .
3
2 3 2 3
∫ x x − 2 dx = 3 (x − 2) 2 x – ∫ 3 ( x − 2) 2 dx
5
2 3 2 ( x − 2) 2
= ( x − 2) 2 x – ⋅ +c
3 3 5
2
5
2 3 4( x − 2) 2
= ( x − 2) 2 x – +c
3 15

Note
When we integrate a function by using different methods we may get different answers. But
if the method is correct then we say both of the solutions are correct. Because, we know that
integration is anti-derivative and derivative of two different functions may be the same.

EXAMPLE 61 ∫e
2x
⋅ sin e x dx = ?

Solution Let u = ex and v′ = ex sin ex. Then, u′ = ex. To find v we will integrate ex sin ex.
Let t = ex then dt = ex dx. So, v = ∫ e sin e dx = ∫ sin t dt = – cos t = – cos e .
x x x

∫e ⋅ sin e x dx = – e x cos e x – ∫ −e x cos e x dx


2x

= −e x cos e x + ∫ e x cos e x dx

For the second part use k = ex and dk = ex dx.

∫e cos e x dx = ∫ cos k dk = sin k + c = sin e x + c


x
By substitution;

∫e
2x
⋅ sin e x dx = – e x cos e x + sin e x + c.

The Indefinite Integral 270


EXAMPLE 62 ∫ sin(ln x) dx = ?

Solution cos(ln x)dx


Let u = sin(ln x) and v′ = 1 then u ' = and v = x.
x
cos(ln x)
∫ sin(ln x) dx = x ⋅ sin(ln x) – ∫ x ⋅ x dx = x ⋅sin(ln x) −∫cos(ln x) dx (*)
Now, we will evaluate ∫ cos(ln x)dx separately:
sin(ln x)dx
Lets say u = cos (ln x) and dv′ = 1 then du = − and v = x.
x
sin(ln x)
∫ cos(ln x) dx = x ⋅ cos( ln x) – ∫ x ⋅( −
x
) dx

= x ⋅ cos(ln x) + ∫ sin(ln x) dx (**)

Now, we will replace (**) in (*) then we have:

∫ sin(ln x) dx = x ⋅ sin( ln x) – ( x ⋅cos(ln x)+ ∫ sin(ln x) dx )


= x ⋅ sin(ln x) – x cos(ln x) – ∫ sin(ln x) dx

2 ⋅ ∫ sin(ln x) dx = x ⋅ sin(ln x) – x ⋅cos(ln x),

x(sin(ln x) − cos(ln x))


∫ sin(ln x) dx = 2
+ c.

EXAMPLE 63 ∫e
x
sin x dx = ?

Solution Let u = ex and v′ = sinx. Then, u′ = ex and v = –cos x.

∫e sin x dx = – e x cos x + ∫ e x cos x dx


x

(For the second part we will use integration by parts one more time. Let u1 = ex and

∫e cos x dx = e x sin x – ∫ e x sin x dx)


x
v1′ = cos x. Then, u1′ = ex and v1 = sinx. So,

∫e sin x dx = – e x cos x + ∫ e x cos x dx


x

= – e x cos x + e x sin x – ∫ e x sin x dx ;

2 ∫ e x sin x dx = – e x cos x + e x sin x

ex
∫e
x
sin x dx = (sin x – cos x)+ c
2

271 Integral
Check Yourself 7
Integrate the following integrals

a. ∫ ln x dx b. ∫x c. ∫ x ⋅ ln x dx d. ∫x
2 2
⋅ sin x dx ⋅ e2 x dx

e. arcsin x dx f. g. h.
∫ ∫ log( x + 3) dx ∫ (2x + 1)sin 3x dx ∫e
3x
⋅ sin x dx

i. j. k. l.
∫e ∫ ( x − 1) ⋅ ln x dx ∫x ∫ 3x ⋅ sin x ⋅ cos x dx
(2 x +1)
⋅ cos( x − 1) dx x +1 dx

Answers

a. x ln x – x + c b. (2 – x2) cos x + 2x sin x + c


x2 ln x x2 2x 2
c. − +c d. e (2 x − 2 x +1) + c
2 4 4
x
e. x arcsin x + 1 − x2 + c f. (x + 3) ⋅ log (x + 3) – +c
ln10
2 sin 3 x (2 x + 1)cos 3 x 3sin x cos x
g. − +c h. e3 x ( − )+ c
9 3 10 10
2 cos( x − 1) sin( x −1) 2
i. e2 x+1( + )+ c j. ( x − x)ln x − x( x − 4) + c
5 5 2 4
3/ 2
k. 2( x +1) (3x − 2) + c l. 3sin x cos x 3 x sin 2 x 3 x
+ − +c
15 4 2 4

C. INTEGRATION BY PARTIAL FRACTIONS


P( x)
We use this method to calculate the integral of the form ∫ Q( x) dx , where P(x) and Q(x) are
polynomials and Q(x) ≠ 0.
For different type of partial fractions we should use different methods. Now, in this section
we will learn the most useful methods for evaluating the integration of partial fractions.

P( x) dx and deg P x = deg Q x


1. ∫ Q(x) [ ( )] [ ( )] – 1
In this condition we will use the substitution u = Q(x) and try to find du in terms of P(x) dx.
After that we will try to find the answer.

The Indefinite Integral 272


3
EXAMPLE 64 ∫ 5x +1 dx = ?
du
Solution Let u = 5x+1 then du = 5 dx or dx = .
5
3 3
So, ∫ dx = ∫ du
5x + 1 5u
3
= ⋅ ln| u |+ c
5
3
= ⋅ ln|5 x +1|+ c
5

2x − 3
EXAMPLE 65 ∫x 2
− 3x − 1
dx = ?

Let u = x2 – 3x – 1 then du = 2x – 3.
Solution
2x − 3 du
So, ∫ dx = ∫
x2 − 3x − 1 u
= ln| u |+ c

= ln| x2 − 3 x − 1|+ c

7
EXAMPLE 66 ∫ (5x + 2) 9
dx = ?

du
Solution Let u = 5x+2 then du = 5 dx or dx = .
5
7 7
So, ∫ dx = ∫ 9 du
(5x + 2)9 5u
7 −9
5∫
= u du

7 u −8
= ⋅ +c
5 −8
7
=− +c
40 u8
7
=− +c
40(5 x + 2)8

273 Integral
dx
EXAMPLE 67 ∫x 2
+ 2 x +1
=?

Solution dx dx
∫x 2
+ 2 x +1
=∫
( x +1)2
Let u = x + 1 then du = dx.
dx du
∫ ( x +1) 2
=∫
u2

= ∫ u −2 du

1
=− +c
u
1
=− +c
x +1

Check Yourself 8
Evaluate the following integrals.
4 2x + 3 3 dx x2 − 2x
a. ∫ 7x − 6 dx b. ∫ x2 + 3x − 1 dx c. ∫ ( x +1)4 dx d. ∫ x2 +8 x +16 e. ∫ x3 − 3x2 − 1 dx
Answers
1 1
a. 4 ln|7 x − 6| + c b. ln|x2 + 3x – 1|+ c c. − +c d. − +c
7 ( x +1)3 x+ 4
3 2
e. ln| x − 3x − 1|
3

P( x) dx deg P x < deg Q x


2. ∫ Q(x) [ ( )] [ ( )] and Q(x) is reducible on R

If Q(x) is linear then by using “ln x” formula we can easily get the answer but if the degree
of Q(x) is bigger then or equal to 2 then we will try to write given expression as the sum or
difference of two or more partial fractions. Then we will use the methods to evaluate the
integral.
In this method we have some rules to write given expression as the sum or difference of two
or more partial fractions. The rules are given below.

The Indefinite Integral 274


Rule

1. P( x) A B
= +
( ax + b) ⋅ ( cx + d ) ax + b cx + d

2. P( x) A B C
n
= + 2
+ +...
( ax + b ) ax + b ( ax + b ) ( ax + b )3

3. P( x) A Bx + C
= +
( ax + b) ⋅ ( cx2 + dx + e ) ax + b cx 2 + dx + e

Notice that the numbers of partial fractions are the same as the number of factors of
denominator

5x +7
EXAMPLE 68 ∫ ( x − 2)(3x +1) dx = ?

Solution 5x +7 A B 3Ax + A + Bx − 2B (3A + B)x +( A − 2 B)


= + = =
( x − 2)(3 x +1) x − 2 3 x+1 ( x − 2)(3 x+1) ( x −2)(3 x+1)

3A + B = 5
A – 2B = 7
17 16
The solution of this system gives as A = and B = – .
7 7
5x +7 17 16
So, = − .
( x − 2)(3 x +1) 7 ⋅( x − 2) 7 ⋅(3 x+1)
5x +7 17 16
Integrate both sides: ∫ ( x − 2)(3x +1) dx = ∫ 7 ⋅( x − 2) dx − ∫ 7 ⋅(3 x+1) dx
17 16
= ⋅ ln| x − 2| − ⋅ ln| 3 x+1|+c.
7 21

Note
In these type of examples as another method we can find A and B by the following method:
5x +7 A B
= + multiply both sides by (x – 2);
( x − 2)(3 x +1) x − 2 3 x+1
5x +7 ( x − 2) ⋅ B replace x = 2 (to make x – 2 = 0);
= A+
3x +1 3x +1
5 ⋅ 2+7 0 ⋅ B so we can find A = 17 .
= A+
3 ⋅ 2+1 3x +1 7
By the same method we can find B and the rest of the solution will be the same.

275 Integral
11x + 4
EXAMPLE 69 ∫ 2x 2
+x−3
dx = ?

Solution Firstly we will factorize denominator.


2x2 + x – 3 = (x – 1) ⋅ (2x + 3).
Then; 11 2
x+ 4
=
A
+
B
=
2 Ax + 3 A + Bx − B (2 A + B)x +(3 A − B)
= .
2x + x − 3 x − 1 2x + 3 ( x − 1)(2 x + 3) 2 x2 + x − 3
2A + B = 11
3A – B = 4
The solution of this system will lead us to the solution: A = 3 and B = 5.
So we have: 11x + 4 3 5
2
= + .
2x + x − 3 x − 1 2x + 3

Integrate both sides: 11x + 4 3 5 .


∫ 2x 2
+x−3
dx = ∫
x −1
dx + ∫
2x+ 3
dx

5
= 3 ⋅ ln| x − 1|+ ln| 2 x+ 3|+ c.
2

x +1
EXAMPLE 70 ∫x 3
−1
dx = ?

Solution Factorize the denominator: x3 – 1 = (x – 1) ⋅ (x2 + x + 1).


Then we have:
Then; x +1 = x +1 A Bx + C Ax 2 + Ax + A + Bx 2 − Bx + Cx − C
= + =
x3 − 1 ( x − 1)( x2 + x +1) x − 1 x2 + x+1 x3 −1

( A + B)x2 + ( A − B + C )x + ( A − C )
=
x3 − 1
A+B=0
A–B+C=1
A–C=1
2 2 1
Solution of this system is: A = , B = − , C = − .
3 3 3
x +1 2 1 2 x +1
So we have, 3
= − ⋅ 2 .
x 1− 3( x − 1) 3 x + x +1

x +1 2 1 2 x +1
Integrate both sides: ∫x 3
−1
=∫
3( x − 1)
dx + ∫ − ⋅ 2
3 x + x +1
dx.

The Indefinite Integral 276


In the second part use substitution u = x2 + x + 1 and du = (2x + 1)dx:
x +1 2 1 du 2 1
∫x 3
−1
dx = ⋅ ln| x − 1|+ c1 − ∫
3 3 u
= ⋅ ln| x −1|+ c1 − ⋅ln| u|+ c2
3 3
2 1
= ln| x − 1| − ⋅ ln| x2 + x+1|+ c
3 3
x2 − 2 x +1 13
= ln| | +c
x2 + x +1

5x − 1
EXAMPLE 71 ∫ (2x + 3) 2
dx = ?

Solution 5x − 1 A B 2 Ax + 3A + B
2
= + 2
=
(2 x + 3) 2 x + 3 (2 x + 3) (2 x+ 3) 2

2A = 5
3A + B = –1
5 17
Solution of this system is: A = and B = – .
2 2
5x − 1 5 17
So, ∫ (2 x + 3) 2
dx = ∫
2 ⋅ (2 x + 3)
dx − ∫
2(2 x+ 3) 2
dx.

Use substitution u = 2x + 3 and du = 2dx in the second part.


5x − 1 5 17 1 −2 5 17
∫ (2x + 3) 2
dx =
4
ln| 2 x + 3|+ c1 − ∫
2 2
⋅ u du = ln| 2 x + 3|+
4 4(2 x + 3)
+c

ex
EXAMPLE 72 ∫ (e x
+ 2)( e x − 3)
dx = ?

Solution Firstly we will change given expression to rational function then by using the previous
method we will evaluate the integral. Let u = ex then du = ex dx. So the integration will
change.
ex du
∫ (e x x
+ 2)( e − 3)
dx = ∫
( u + 2)( u − 3)
1 A B ( A + B)u − 3 A + 2 B
= + = . Then
( u + 2)( u − 3) u + 2 u − 3 ( u + 2)( u − 3)
A+B=0
–3A + 2B = 1

277 Integral
1 1
Solving this system we will get A = – and B = .
5 5
du −1 1 1 1
∫ ( u + 2)( u − 3) = ∫ 5( u + 2) du + ∫ 5( u − 3) du = − 5 ln| u + 2|+ 5 ln| u − 3|+ c
So we get the result;
ex 1 1 1 ex − 3
∫ (e x x
+ 2)( e − 3)
dx = − ln| e x + 2|+ ln| e x − 3|+ c = ln| x
5 5 5 e +2
|+ c

P( x)
3. ∫ Q(x) dx deg[P(x)] <deg[Q(x)] and Q(x) is not reducible on R
dx
The expression ∫ ax
+ bx + c
is given. If ∆= b2 – 4ac < 0 then we will use the following
2

method to evaluate the integral:


dx dx 1 dx
∫ ax 2
+ bx + c ∫ ( mx + n )2 + r 2 r 2
= = ∫ ⎛ mx + n ⎞ 2

⎜ ⎟ +1
⎝ r ⎠
1 r mx + n 1 mx + n
= 2
⋅ ⋅ arctan( )+ c1 = arctan( )+ c1
r m r rm r
1 mx + n
or = − arc cot( )+ c2.
rm r

EXAMPLE 73 Evaluate the integral: ∫x


2
dx
+ 4x +5

dx dx dx
Solution ∫x 2
+ 4x +5
=∫ 2
x + 4x + 4+1
=∫
( x + 2)2 +1
= arctan( x+ 2)+ c

or = –arccot(x + 2) + c

P( x) dx and deg P x ≥ deg Q x


4. ∫ Q(x) [ ( )] [ ( )]

In this condition, firstly we will divide the numerator by denominator then calculate the
integrals separately by using lnx formula.

The Indefinite Integral 278


4x2 +14 x + 3
EXAMPLE 74 ∫ x2 + 3x dx = ?

Solution deg [numerator] = deg [denominator].


So, we will divide the numerator by the denominator.
4x2 +14 x + 3 2 x+ 3
= 4+ 2
x2 + 3x x + 3x
Integrate both sides:

4x2 +14 x + 3 2x + 3 2 x+ 3
∫ x2 + 3x dx = ∫ 4 dx + ∫ x2 + 3x dx = 4x + c1 + ∫x 2
+ 3x
dx

For the second part we will use substitution. u = x2 + 3x and du = (2x + 3) dx


2x + 3 du
∫x 2
+ 3x
dx = ∫
u
= ln| u |+ c2.

4x2 +14 x + 3
∫ x2 + 3x dx = 4x + c1 + ln|u| + c2 = 4x + ln|x + 3x| + c.
2
So,

x3 + 3x
EXAMPLE 75 ∫ x2 +1
dx = ?

Solution Deg (numerator) > Deg (denominator).


So, we will divide the numerator by denominator.
x3 + 3x 2x
2
= x+ 2
x +1 x +1

Integrate both sides:


x3 + 3x 2x x2 2x
∫ x2 +1 dx = ∫ x dx + ∫ x2 +1 dx =
2
+ c1 + ∫ 2
x +1
dx

For the second part we will use substitution u = x2 + 1 then du = 2x dx.

x3 + 3x x2 x2
So, ∫ 2
dx = + c1 + ln| u |+ c2 = + ln( x2 +1)+ c.
x +1 2 2

279 Integral
EXAMPLE 76 3x − 1
∫ x+ 2 dx = ?

Solution As another method we can use normal substitution twice.


Let u = x + 2 and du = dx.
Also x = u – 2 then 3x – 1 = 3(u – 2) – 1 = 3u – 7.
3x − 1 3u − 7 7
So, ∫ dx = ∫ du = ∫ (3 − ) du
x+ 2 u u
= 3u – 7 ln| u | + c

= 3( x + 2) – 7 ln| x+ 2|+ c

Check Yourself 9
Evaluate the following integrals.
dx 2x − 1 x −1
a. ∫ dx b. ∫ dx c. ∫ ( x +1)( x − 2) 2
dx
( x − 1)( x + 2) ( x +1)( x + 2)
x2 + 3 3x − 1 dx
d. ∫ x2 +1 dx e. f.
∫x2
−1
dx ∫x 2
+ 2 x +5
dx

g. x2 − 1 h. 1− x i. 3
∫ x2 − 16 dx ∫x 2
+ 3x
dx ∫e x
−2
dx

Answers
2 x−2 1
a. 1 ln| x − 1 |+ c b. 5ln|x + 2| – 3ln|x + 1| + c c. ln| |− +c
3 x+ 2 9 x +1 3( x − 2)
1 x +1
d. 2arctan x + x + c e. ln |x – 1| + 2 ⋅ ln|x + 1| + c f. arctan( )+ c
2 2
15 x−4 1 4 3 3x
g. ln| |+x + c h. ln| x | − ln| x+ 3|+ c i. ln| e x − 2| − + c
8 x+ 4 3 3 2 2

D. INTEGRATION OF RADICAL FUNCTIONS


For the integration of the functions of the form ∫ f ( x) dx, ∫ n f ( x) dx, ∫ a 2 ± u 2 we have
some special methods. Now, we will learn these methods.

1. Simple Radical Functions


In these type of expressions we will try to simplify the radical sign. For this reason we will
make substitutions with u2, u3 … related with the degree of root.

The Indefinite Integral 280


EXAMPLE 77 ∫ 3x +1 dx = ?

Solution 1 2u du
To simplify the root we will say u2 = 3x + 1, (x ≥ – ) and then 2u du = 3 dx or dx = .
3 3
3
2udu u ⋅ 2udu 2 2 2 u3 2(3x +1) 2
∫ 3 x+1dx = ∫ =∫ = ∫ u du = ⋅ + c =
2
So, u ⋅ + c.
3 3 3 3 3 9

EXAMPLE 78 ∫
3
5x − 2 dx = ?

Solution For simplifying third degree root we will choose u3 = 5x – 2 then,


3u 2
3u2 du = 5 dx or dx = du.
5
4
3u 2 3u 3 3 u4 3(5x − 2) 3
∫ 5 x − 2 dx = ∫ du = ∫
3 3 3
Then u ⋅ du = ⋅ + c = + c.
5 5 5 4 20

EXAMPLE 79 ∫
x
2
x +5
dx = ?

Solution Let, u2 = x2 + 5 and 2u ⋅ du = 2x ⋅ dx or u ⋅ du = x ⋅ dx.

x udu
∫ 2
x +5
dx = ∫
u2
= ∫ du = u + c = x 2 +5 + c.

EXAMPLE 80 ∫
4x
x −1
dx = ?

Solution Let u2 = x – 1, (x > 1 ) and 2u du = dx.


4x2u
But, when we make this substitution we will have ∫ u
du .

We should write every term related with u.

So to cancel x we will say x = u2 + 1 from the substitution u2 = x – 1. So,


3
⎛ 1 ⎞
4x 4 ⋅ ( u 2 +1) u 3
( x −1) 2

∫ dx = ∫ ⋅ 2 ⋅ udu = ∫ 8( u 2 +1)du = 8 ⋅( + u )+ c = 8 ⋅ ⎜⎜ +( x −1) 2 ⎟+ c



x −1 u2 3 ⎝ 3 ⎠

281 Integral
Check Yourself 10
Evaluate the following integrals.
5x
a. b. ∫ c. ∫
5
∫ 1+ 4x dx 4x − 3 dx dx
2 x2 + 3

d. x2 e. 3x f. x +1+1
∫ 3
x −2
dx ∫ x −1
dx
∫ x +1
dx

Answers
3/ 2 6/5
a. (4 x +1) + c b. 5(4 x − 3) + c
2
c. 5 2 x + 3
+c
6 24 2
3
d. 2 x − 2 e. 2( x + 2) x − 1+ c f. 2 x +1+ x + c
+c
3

2. Integral Of The Form ∫ a2 ± u2 dx or ∫ u2 ± a2 dx


In this part we will use trigonometric substitution.
We will draw a right triangle and give names to sides as a, u and a2 ± u 2 or u 2 ± a2 then
solve the question.

EXAMPLE 81 ∫ 1 − x2 dx = ?

Solution Let sin α = x


cos α dα = dx
1
α = arcsin x x

∫ 1 − x dx = ∫ 1 − sin α cos α dα
2 2
Then a
1 – x2
1+ cos 2α
= ∫ cos α dα = ∫
2

2
1 1 α 1 1
=
2 ∫ dα + ∫ cos 2α dα = + ⋅ ⋅ sin 2 α + c
2 2 2 2
α 1 arcsin x 1
= + ⋅ 2 ⋅ sin α ⋅ cos α + c = + ⋅ x ⋅ 1 − x2 + c.
2 4 2 2

The Indefinite Integral 282


EXAMPLE 82 ∫
4 x2 − 1
x
dx = ?

Solution Let 2x = sec α then 2 dx = tan α ⋅ sec α dα and α = arcsec 2x

4 x2 − 1 sec 2 α − 1 tan α sec α A


∫ x
dx = ∫
sec α

2

2 2x
4x2 – 1
= ∫ tan α ⋅ tan α dα = ∫ tan 2 α da
a
C 1 B
= ∫ (tan α +1 − 1) dα
2

= ∫ (tan 2 α +1) dα − ∫ dα

= tan α – α + c = 4 x2 − 1 − arc sec 2 x+ c.

EXAMPLE 83 ∫x 2
dx
⋅ 9+ 4x2
=?

A
2x
Solution Let, = tan α
3 9 + 4x2
2x
3
x = tan α
2 a
3dα C 3 B
dx =
2 cos 2 α
dx 3dα
∫x 2
⋅ 9+ 4x 2
=∫
9 9
2 ⋅ cos 2 α ⋅ ⋅ tan 2 α ⋅ 9+ 4 ⋅ ⋅ tan 2 α
4 4
3da
=
9
cos 2 α ⋅ tan 2 α ⋅ 3 ⋅ 1+ tan 2 α
2
2dα 2 cos α
=∫ 2
= ∫ dα
2 sin α 1 9 sin 2 α
9cos α ⋅ ⋅
cos 2 α cos α

(use substitution u = sin α and du = cos α dα)

2 du 2 2
= ∫
9 u 2
=−
9u
+c= −
9sin α
+c

2 ⋅ 9 + 4 x2 9 + 4 x2
=− +c= − + c.
9 ⋅ 2x 9x

283 Integral
Check Yourself 11
Evaluate the following integrals.
dx x
a. ∫ 9 − x2 dx b.
∫ c. ∫ 9 x2 + 4
dx
9 − x2

Answers
9 x x 9 − x2 x 9 x2 + 4
a. arcsin( )+ +c b. arcsin( )+ c c. +c
2 3 2 3 9

E. INTEGRATION OF TRIGONOMETRIC
FUNCTIONS
Basic Formulas
∫ sin x dx = − cos x+ c
∫ cos x dx = sin x + c
sin x 1
∫ tan x dx = ∫ cos x dx = − ln| cos x|+ c = ln|
cos x
|+ c.

m x ⋅ cos n x dx ( m, n ∈N )
1. Integrals of the Form ∫ sin
a. If m and n are odd numbers
Let, m = 2k + 1 and n = 2t + 1 then we will write given expression as
∫ sin
2 k +1
x ⋅ cos 2 t x ⋅ cos x dx then by using substitution u = sin x,

du = cos x dx and cos2t x = (1 – sin2 x)t we can evaluate the integral.

(or we may write ∫ cos 2t +1 x ⋅ sin 2 k x ⋅ sin x dx and make substitution

u = cos x, du = –sin x dx and sin2k x =(1 – cos2 x)k .

EXAMPLE 84 Evaluate ∫ cos 7 x ⋅ sin 3 x dx

∫ cos x ⋅ sin 3 x dx = ∫ cos 7 x ⋅ sin 2 x ⋅sin x dx = ∫ cos 7 x ⋅ (1 − cos 2 x) ⋅ sin x dx


7
Solution

The Indefinite Integral 284


Use the substitution u = cos x and du = – sin x dx. Then,

∫ cos x ⋅ sin 3 x dx = −∫ u 7.(1 − u 2 ) du


7
Then

= − ∫ ( u7 − u 9 ) du

u8 u10
=− + +c
8 10
cos 8 x cos10 x
=− + + c.
8 10

b. If one of m or n is odd number


We will write the term with odd power as the multiplication of the term with even power and
the same term with first power then by using the substitution u =sin x or u = cos x we can
evaluate the integral like in the previous method.

EXAMPLE 85 Evaluate the integral: ∫ sin


6
x ⋅ cos 3 x dx.

∫ sin x ⋅ cos 3 x dx = ∫ sin 6 x ⋅ cos 2 x ⋅cos x dx = ∫ sin 6 x ⋅(1 −sin 2 x) ⋅cos x dx


6
Solution

Use the substitution u = sin x and du = cos x dx

u7 u 9 sin7 x sin9 x
∫ sin x ⋅ cos x dx = ∫ u ⋅(1 − u ) du = ∫( u − u ) du =
6 3 6 2 6 8
− + c= − + c.
7 9 7 9

c. If m and n are even numbers


2 1 − cos 2 x 2 1+ cos 2 x
In this part we will use the rule sin x = and cos x = to find the answer.
2 2

EXAMPLE 86 Evaluate the integral: ∫ sin


2
x ⋅ cos 2 x dx.

Solution 1 − cos 2 x 1 + cos 2 x 1


∫ sin x ⋅ cos 2 x dx = ∫ dx = ∫ (1 −cos 2 2 x) dx
2

2 2 4
1 1 1 − cos 4 x 1 1
=
4 ∫ sin 2 2 x dx = ∫
4 2
dx = ∫ dx − ∫ cos 4 x dx
8 8
x 1 1 x sin 4 x
= − ⋅ ⋅ sin 4 x + c = − + c.
8 8 4 8 32

285 Integral
2. Integral of the Form ∫ sin mx ⋅ cos nx dx,
∫ sin mx ⋅ sin nx dx, ∫ cos mx ⋅cos nx dx
For the solution of these type of integrals we will use inverse conversion formulas:
1
sin a ⋅ sin b = − [cos( a+ b) − cos( a − b) ]
2
1
sin a ⋅ cos b = [sin( a+ b)+ sin( a − b) ]
2
1
cos a ⋅ cos b = [cos( a+ b)+ cos( a − b) ].
2

EXAMPLE 87 ∫ sin 3x ⋅ cos9 x dx = ?


1 1
Solution ∫ sin 3x ⋅ cos9 x dx = ∫ 2 [sin12 x+ sin( −6 x) ] dx = 2 ∫(sin12 x −sin6 x) dx
1 1 1 1 cos12 x cos6 x
=− ⋅ cos12 x+ ⋅ ⋅ cos6 x+ c = − + + c.
2 12 2 6 24 12

EXAMPLE 88 ∫ cos6 x ⋅ cos 2 x dx = ?


1 1
Solution ∫ cos6 x ⋅ cos 2 x dx = ∫ 2 [cos8 x+ cos 4 x] dx = 2 ∫(cos8 x+ cos 4 x) dx
1 1 1 1 sin8 x sin 4 x
= ⋅ ⋅ sin8 x + ⋅ ⋅ sin 4 x+ c = + + c.
2 8 2 4 16 8

Check Yourself 12
Evaluate the following integrals.

a. ∫ sin b. ∫ sin c. ∫ sin


3 4
x ⋅ cos x dx x ⋅ cos5 x dx 4
x ⋅ cos 4 x dx

d. ∫ cos 2 x ⋅ cos x dx e. ∫ cos 4 x ⋅ sin5 x dx f. ∫ sin 3x ⋅ sin5 x dx


Answers
5 7 9
b. sin x − 2 sin x + sin x + c c. 3x − sin 4 x + sin8 x + c
4
a. sin x + c
4 5 7 9 128 128 1024
sin 3x sin x sin 2 x sin8 x
d. + +c e. − cos9 x − cos x + c f. − +c
6 2 18 2 4 16
The Indefinite Integral 286
x
3. tan Substitution
2
Sometimes in the function we may have an expression with first power of sin x and cos x. In
this condition we may use a different method for evaluating the integral which is called the
x
tan method. For using this method we will do the following substitutions:
2
x 2dt
tan = t and x = 2 arctan t then, dx =
2 1+ t2 A

x 2⋅t x 1
sin = and cos = . 1 + t2
2 1+ t 2 2 1+ t 2 t

And by using these expressions we have; x


2
x x t 1 2t C 1 B
sin x = 2 ⋅ sin ⋅ cos = ⋅ =
2 2 1+ t 2
1+ t 2 1+ t2

x x 1 t2 1 – t2
cos x = cos 2 – sin 2 = – = .
2 2 1+ t2 1+ t2 1+ t2
So we have;
2
sin x = 22tt , cos x =11––tt2 and dx = 22 dt .
sin x = , sin x = and dx = dt
1+tt22
1+ 1+tt22
1+ 1+tt22
1+

EXAMPLE 89 sin x
∫ 1+ cos x dx = ?

2t 2dt 4tdt
⋅ 2 2
1+ t 1+ t = (1+ t )(1+ t ) = 2tdt
2 2
Solution ∫ 1− t 2 ∫ 2 ∫ 1+ t2
1+ 2
1+ t2 1+ t
Lets use the substitution u = 1 + t2 and du = 2t dt

du x 2
∫ u
= ln| u |+ c = ln|1+ t 2 |+ c = ln|1+ tan 2 |+ c = ln|
2 1+ cos x
|.

EXAMPLE 90 dx
∫ sin x +1 dx = ?
287 Integral
2 dt 2 dt
1+ t 2 = 1+ t 2 = 2 dt .
Solution ∫ 2t ∫ t2 + 2t +1 ∫ (t +1)2
2
1+ t +1 1+ t2
Lets use the substitution u = 1 + t and du = dt. Then,
2dt 2du 2 2 2
∫ (t +1) 2
=∫
u2
= 2 ∫ u −2 du = − + c = −
u 1+ t
+c= −
x
+ c.
1+ tan
2

EXAMPLE 91 dx
∫ 1+ sin x − cos x = ?
2 dt
dx 1+ t2 2dt 2dt dt dt
Solution ∫ 1+ sin x − cos x = ∫ 2t 1− t 2
=∫ 2
1+ t + 2t − 1+ t 2
=∫ 2
2t + 2t
=∫ 2
t +t
=∫
t( t +1)
.
1+ −
1+ t2 1+ t2
Let's use partial fractions:

1 A B At + A + Bt ( A + B)t + A
= + = =
t(t +1) t t +1 t(t +1) t( t +1)
A+B=0
A=1
Then B = –1.

dt dt dt x x
So ∫ t(t +1) = ∫ t ∫ t +1
− = ln| t |+ c1 − ln| t+1|+ c2 = ln| tan | −ln| tan +1|+ c
2 2

x
tan
= ln 2 + c.
x
tan +1
2

Check Yourself 13
Evaluate the following integrals.
1 − sin x cos x 1 − cos x dx
a.
∫ 1+ cos x dx b. ∫ 1+ sin x dx c.
∫ 1+ cos x dx d. ∫ 1 – sin x
Answers
x 2 x 2
a. tan – ln +c b. ln(sin x + 1)+ c c. 2 ⋅ tan – x+ c d.
2 1+ cos x 2 x
1 – tan
2

The Indefinite Integral 288


EXERCISES 1 .2
A. Method of Changing Variable B. Method of Integration by Parts
1. Evaluate the following integrals by using the 3. Evaluate the following integrals by using the
method of integration by changing variable. method of integration by parts.
dx cos π
a. b. ∫ a. ∫ e x ⋅ x dx b. ∫x
2
⋅ e x dx
∫ x−3 x
dx

c. ∫x d. ∫ x ⋅ sin x dx
3
x dx 5 ⋅ e x dx
c.
∫ x2 d. ∫ 3x +1 dx
x2
∫ sin
2
e. ∫ ex dx ››f. x ⋅ e2 x dx
e. ∫ sin(4 x +1) dx
f. ∫ (1+ x + x ) ⋅ (2 x+ 3 x ) dx g. ∫ ln( x +5) dx h. ∫ log x dx
2 3 8 2

g. ∫ (1 − x ) ⋅ x dx h. ∫ x ⋅ cos( x
2 7 2
− 5) dx

›i. cos(ln x) dx j. ∫ arc cot x dx
1
i. ∫ j. ∫e
sin x
dx ⋅ cos x dx
1 − 16 x 2 k. ∫ arccos x dx
cos x
k. ∫ 1+ sin 2
x
dx l.
∫x⋅ 1+ x2 dx

m. ∫ ( x4 + x2 ) ⋅ (2 x3 + x) dx
C. Integration by Partial Fractions
4. Evaluate the following integrals by the method of
integration by partial fraction.
5 9
a. ∫ 3x +1 dx b. ∫ (3x +1) 4
dx

c. x4 + 2 x2 + x d.
2 x +1
∫ x3
dx ∫x 2
+ x −1
dx

3x4 – 16 x3 +19 x2 − 5 x – 4
2. Evaluate the following integrals by using the ›e. ∫ dx
x3 − 4x2 +1
method of substitution.
4x2 +5 x + 4
a. ∫ x ⋅ cos x
2
dx b. ∫ x ⋅ sin(5x
2
+7) dx f. ∫ x2 +1 dx
ln x
c. ∫x
dx d. ∫ cot x dx dx dx
dx x
g.
∫x 2
+ 4x + 4
h. ∫ ( x +1) 3
e.
∫ (1 − 3x)4 ›f. ∫ 5x − 1
dx
3x +1 x −1
g. ∫ x
ex
dx h. sin x i.
∫ ( x +1)( x + 2) dx j. ∫x dx

2
dx − 2x − 3
e −3 5
cos x
x −x
i. e −e 2x − 1 x2 + 2 x +1
∫e x
+ e−x
dx k.
∫x
2
−1
dx ›l.
∫ x3 − 1
dx

The Indefinite Integral 289


x2 + 3x − 1 1− x E. Integration of Trigonometric
m. ∫ ( x − 1)3 dx n. ∫ (1+ x)2 dx Functions
x2 − x +1 7. Evaluate the integrals of the following trigono-
o.
∫ ( x − 1)( x 2
+ x + 2)
dx
metric functions.
2x + 3
∫ sin ∫ sin x ⋅ cos x dx
2
››p.
a. x ⋅ cos x dx b.
∫ ( x +1)( x 2
+ 2)
dx

∫ cos ∫ cos
3
c. x ⋅ sin 5 x dx d. 2
x ⋅ sin x dx
q. 7x − 6 r. dx
∫ 3x2 − 4x +1 dx ∫ x2 +8 x +15
∫ sin ∫ sin
3
e. x ⋅ cos5 x dx f. 4
x ⋅ cos 3 x dx
dx xdx
›s. ∫ 1 − 4x + x ›t. ∫ 3+ x ∫ cos ∫ sin
4
2 4 g. x ⋅ sin 3 x dx ›h.
6
x ⋅ cos6 x dx

∫ sin ∫ sin 3x ⋅ cos 4 x dx


5
i. x ⋅ cos7 x dx j.

k. ∫ sin7 x ⋅ sin5 x dx l. ∫ sin 3x ⋅ cos8 x dx


D. Integration of Radial Functions
m. ∫ cos 2 x ⋅ sin 4 x dx n. ∫ cos5x ⋅ sin x dx
5. Evaluate the integral of given radical functions.
a. b. ∫ 1 − x dx
o. ∫ cos x ⋅ cos 4 x dx p. ∫ cos 4 x ⋅ cos 3 x dx
∫ 5x − 1 dx

c. ∫x⋅ 1+ x2 dx d. ∫
3
x +1 dx
x 5x
e. ∫ 1− x 2
dx f.
∫ dx
5 x2 + 3 8. Evaluate the integral of the following functions
x2 x
g. ∫ h.
5
1+ x dx ∫ dx by using tan substitution.
3
1+ x3 2
i. x − 2 +3 1+ sin x dx
∫ x−2
dx ›a. ∫ 1 − sin x dx b. ∫ tan x + sin x
dx 3 ⋅ sin x
c. ∫ sin x d. ∫ 1 − cos x dx

6. Evaluate the integrals of given functions. e. sin x


∫ 3+ cos x dx
∫ ∫
2
a. 1 − 4x dx ›b.
2
x − 9 dx

dx x
›c. ∫ x −92
d. ∫ 16 x2 +1
dx

16 x2 − 9
›e. ∫ x
dx f. ∫ 16 − 9 x2 dx

dx
›g.
∫ 9x2 +1 dx h. ∫ 1 − x2
1
i. ∫ dx
1+ x2

290 Integral
CHAPTER SUMMARY
• F(x) is a function such that F′(x) = f(x) then F(x) is Methods of Integration
called the primitive of function f(x) and the expression 1. Method of Changing Variable
f(x) + c is called as the indefinite integral of F(x).
Let's say that f(u) and u(x) be two functions which are
• In every indefinite integral we must use the constant of
differentiable with respect to u and x, respectively.
integration.
Properties of Indefinite Integral So, ∫ f (u( x)) ⋅ u′( x)dx = F(u )+ c.
1. d ∫ f ( x) dx = f ( x ) dx 2. Integration by Parts

2. d f ( x)dx = d f ( x )dx = f (x ) Let u and v be two differentiable functions with respect to


dx ∫ ∫ dx x, then ∫ u ⋅ dv = u ⋅ v − ∫ v ⋅ du.
3. ∫ dF( x) = F( x)+ c 3. Integration by Partial Fractions
4. ∫ a ⋅ f ( x) dx = a ⋅ ∫ f (x ) dx, a ∈R
a. P( x)
∫ Q( x) dx and deg(P(x)) < deg(Q(x))
5. ∫ [ f ( x) ± g( x)] dx = ∫ f (x) dx ± ∫ g(x) dx In this condition we will use the substitution u = Q(x)
Basic Integration Formulas and try to find du in terms of P(x)dx. After that we will
n +1
1. a. xn dx = x + c, n ≠ −1 try to find the answer.
∫ n +1
b. a dx = ax + c for a ∈ R b. P( x)
∫ ∫ Q( x) dx deg(P(x)) < deg(Q(x)) and Q(x) is
2. a. 1
∫ x dx = ln| x |+ c Reducible on R.

u′( x) dx If Q(x) is linear then by using “ln x” formula we can


b.
∫ u( x)
=ln| u( x)|+ c
easily get the answer but if the degree of Q(x) is bigger
then or equal to 2 then we will try to write given
3. a. ∫ e dx = e
x x
+c
expression as the sum or difference of two or more
b. ax
∫ a dx = ln a + c
x
simple fractions. Then we will use the previous

a u( x ) methods to evaluate the integral.


c. au( x )dx =
∫ u '( x) ⋅ ln a
+c
In this method we have some rules to write given

4. a. expression as the sum or difference of two or more


∫ sin xdx = − cos x + c partial fractions. The rules are given below.
b. ∫ cos xdx = sin x + c
RULES:
c. 1
∫ cos 2
x
dx = ∫ sec 2 x dx = ∫(1+ tan 2 x)dx = tan x+ c
P( x) A B
1 = +
d. ∫ sin2 x dx = ∫ csc xdx = ∫(1 + cot x)dx = − cot x+ c
2 2
( ax + b ) ⋅ ( cx + d ) ax + b cx + d
P( x) A B C E
= + + +...+
1 ( ax+ b )n ax + b (ax + b )2 (ax + b )3 (ax + b ) n
e. ∫ dx = arcsin x + c1 = − arccos x+ c2
1 − x2
P( x) A Bx + C
f. 1 = + 2
∫ 1+ x2 dx = arctan x+ c1 = − arc cot x+ c2 2
( ax + b ) ⋅ ( cx + dx + e ) ax + b cx + dx + e

Chapter Summary 291


P( x) If m and n are even numbers
c. ∫ Q( x) dx deg(P(x)) < deg(Q(x)) and Q(x) is not 1 − cos 2 x
In this part we will use the rule sin 2 x =
2
dx 1+ cos 2 x
Reducible on R. The expression ∫ ax is given. 2
and cos x = to find the answer.
2
+ bx + c 2
If ∆ = b2 – 4ac < 0 then we will use the following b. Integral of the Form ∫ sin mx ⋅ cos nx dx ,
method to evaluate the integral:
dx dx 1 dx
∫ sin mx ⋅ sin nx dx , ∫ cos mx ⋅ cos nx dx
∫ ax2 + bx + c = ∫ (mx + n )2 + r 2 = r 2 ∫ ⎛ mx + n ⎞ 2 For the solution of these type of integrals we will use
⎜ ⎟ +1 inverse conversion formulas:
⎝ r ⎠
1
1 r mx + n 1 mx + n sin a ⋅ sin b = − [cos( a+ b) − cos( a − b) ]
⋅ ⋅ arctan( )+ c1 = arctan( )+ c1 2
2
r m r rm r 1
1 mx + n
sin a ⋅ cos b = [sin( a+ b)+ sin( a − b) ]
2
or =− arc cot( )+ c2
rm r 1
cos a ⋅ cos b = [cos( a+ b)+ cos( a − b) ]
P( x) 2
d. ∫ dx , and deg P(x) ≥ deg Q(x)
Q( x)
Concept Check
In this condition, generaly we will divide the numera-
• What is the relation between derivative and integral?
tor by denominator then calculate the integrals sepa-
• What is the primitive of a function?
rately by using lnx formula.
• Why do we need to add “+c” at the and of the indefinite
4. Integral of Radical Functions integral of a function?
a. Simple Radical Functions • Why do we need to write “dx” when we integrate a function?
In these type of expressions we will try to simplify the • Can we integrate a function with respect to a variable dif-
radical sign. For this reason we will make substitutions ferent then x?
with u2, u3 … related with the degree of root. • What are the properties of the indefinite integral?
• Is there a function whose integral is the same function?
b. Integral of the Form ∫ a2 ± u 2 dx or ∫ u 2 ± a2 dx. • Can we directly integrate trigonometric functions?
In this part we will use geometric substitution. We will • Can we get different answers when we integrate a
function by two different methods? Explain your answer.
draw a right triangle and give names to sides as a, u and
• If we cannot integrate a function by using basic
a2 ± u 2 or u 2 ± a2 then solve the question.
integration formulas which rules we can use?
5. Integral of Trigonometric Functions • What are the methods of integration?
• What is the substitution method of integration?
a. Integrals of the form ∫ sin
m
x ⋅ cos n x dx.
• In the method of integration by parts what is rule for
If m and n are odd numbers selecting u and v'?
Firstly we will write given expression as • By which methods we can integrate a rational function?
∫ sin
2 m +1
x ⋅ cos2 n x ⋅ cos x dx (we may write also sin2nx) • In a rational fraction if the degree of numerator is greater
than the degree of denominator then how can we
then by using substitution u = sin x and
integrate it?
1+ cos 2 x n we can evaluate the integral. • How can we use trigonometric substitution for
cos 2 n x = ( )
2 integrating a radical function?
If one the m and n is odd number
• When we have a trigonometric function with even powers
We will change the term with odd power to the term of sin x and cos x how can we integrate the function?
with even power then by using the substitution x
• What is tan substitution?
u = sin x or u = cos x we can evaluate the integral. 2

292 Integral
CHAPTER REVIEW TEST 1A
1. If f ( x) = ∫ ( x2 − x + 3) dx then what is f′(2)? 5. f ′(x) = 3x2 + 2x + 4 and f(1) = 3 what is f(x)?

8 23 1
A) 1 B) 3 C) D) E) 5 A) 7 B) 6 C) D) 27 E) 45
3 3 3

2. ∫ (3x2 + 4x − 5) dx ? 6. What is the primitive of the function


F(x) = 3 cos x – 4 sin x.
A) x3 + x2 – 5x B) x3 + 2x2 – 5x + c

C) 3x3 + 4x2 – 5x + c D) 6x – 4 + c A) 3 sin x + 4 cos x + c

E) 3x3 – 4x2 – 5x + c B) 3 sin x – 4 cos x + c


C) –3 sin x – 4 cos x + c
D) –3 sin x + 4 cos x + c
3cos2 x
E) –2 sin 2x + c
2
x + x x − x2
3. f ( x) = ∫ dx then find f(4) if the
x
constant of the integration is 0.
9 17 8
A) 1 B) 2 C) D) E) 3
2 3 15 7. ∫2+x dx = ?

2
A) ln|x| + c B) 3 ln |2 + x| + c
3
4. What is the primitive of the function 1 1 3
C) ln|2 + x| + c D) ln + c
2 3 2 x
f(x) = 5x2 + 3x – ?
x 3
E) +c
ln| 2+ x |
5x3 3x3 2
A) + + +c
3 2 x
3 2
B) 4x + 3x – 22 + c
3 2 x
8. ∫ (cot 2 x+1) dx = ?
5 x 3 3 x2
C) + − 2 ln| x|+ c
3 2 A) –cot x + c B) cot x + c
2
D) 10x + 3 + 2 + c C) sin x + c D) tan x + c
x 3
cot x
2 E) + x+ c
E) 5x2 + 3x – +c 3
x
Chapter Review Test 1 293
9. 13. ∫ 9 ⋅ sin
9
∫ cot x dx = ? x ⋅ cos x dx = ?

A) ln |sin x| + c B) ln |cos x| + c 10 10
A) sin x + c B) cos x + c
10 10
C) –cot 2x + c D) sec 2x + c
10 10

E) –csc 2x + c C) 9sin x + c D) 9cos x + c


10 10
10 2
E) 9sin x ⋅ cos x + c
20

4x – 1
14. ∫ 2x dx = ?
10. ∫ arccos x dx = ? 2
– x +5
A) x arccos x + x + c
A) 3ln|x – | + c
B) x arccos x + 1 − x2 + c
B) 2ln|x + 2| + 3ln|x – 5| + c
C) arccosx + x arccos x + c
1
D) x arccos x – 1 − x2 + c C) +c
4x +1
E) x arccos x + 1+ x2 + c
4
D) +c
4x − 1
E) ln |2x2 – x + 5| + c

11.
5
∫ 3x +1 dx ?
15. ∫ cos 4 x ⋅ cos 2 x dx = ?

cos 4 x cos 2 x sin 4 x ⋅ sin 2 x


5 A) + +c B) +c
A) 5 ln |3x + 1| + c B) ln|3x + 1| + c 4 2 8
3
C) 15 ln|3x + 1| + c D) ln|3x + 1| + c cos6 x cos 2 x sin6 x sin 2 x
C) + +c D) + +c
12 4 12 4
E) 3 ln|3x + 1| + c cos x sin x
E) + +c
2 3

16. ∫ cos
2
x dx = ?

12. ∫ (cos 2 x − 3) dx = ? A)
sin2 x
+c B)
x sin 2 x
+ +c
2 2 4
sin 2 x sin 2 x
A) +c B) – 3x + c 3
2 2 C) cos x + c D) cos x + c
C) 2 sin x + c D) 2 cos x + c 3
cos 2 x x cos 2 x
E) + x+ E) + +c
2 2 4

294 Integral
CHAPTER REVIEW TEST 1B
1. If f ( x) = ∫ d( x +1) and f(1) = 2 then what is 5. ∫(x
2
+ 4x)5 ⋅ ( x + 2) dx = ?
f(2)?
( x2 + 4x)6 ( x2 + 4x)6
A) +c B) +c
A) 0 B) 1 C) 2 D) 3 E) –2 6 12
( x2 + 4x)6 2x + 4
C) +c D) +c
3 6
6
⎛ x3 2 ⎞ ⎛ x2 ⎞
⎜ + x ⎟ ⎜ + x⎟

3 ⎠ ⎝ 2
E) ⎝ ⎠+c
x3 + 4 x2 − 3 x 6
2. ∫ dx ?
x2

x2 x2
A) + 4x − 3+ c B) + 4x − 3 ln x + c
2 2
x4 4x3 3x2 1
+ −
2 + c D) x + 4 − 3 + c
6. ∫2 x
dx ?
C) 4 3
x 3
x 1 x
3 A) +c B) ñx + c C) +c
2x 2
E) x3 + 4 x2 − 3x x x 1
D) +c E) +c
3 3 x

f ′( x) 7. ∫ sin 3x dx = ?
3. ∫ f ( x)
dx ?
1
A) cos 3x + c B) sin3x + c
2
A) f (x) + c B) f(x) + c 3
1
C) ln|f(x)| + c D) –ln(f 2(x)) + c C) – cos 3x + c D) sin 3x + c
3
E) f 2(x) E) 0

d(sin x)
8. ∫ dx = ?
f ( x) sin x
4. ∫ dx = x3 + 4 x2 +5 x – 1 is given. A) sin x + c B) cos x + c
x2
What is f(1)? C) ln |sin x| + c D) –ln |cos x| + c

A) –7 B) 3 C) –2 D) 16 E) 20 E) ln |cos x| + c

Chapter Review Test 1B 295


9. ∫ x sin x dx ? 13. ∫ sin 2 x ⋅ cos 4 x dx = ?
A) x cos x + sin x + c B) x cos x – sin x + c cos 2 x cos6 x
A) − +c
C) x2 cosx + x sin x + c D) –xcosx + sinx + c 4 12
sin 2 x − cos6 x
E) x cos x – sin x + c B) +c
12
sin2 2 x cos 4 4 x
C) + +c
2 4
sin6 x cos 2 x
D) − +c
10. ∫e
x
cos x dx = ? 6 2
ex
⋅(cos x + sin x)+ c
A)
2 E) cos 2 x − 2 sin 4 x + c
4
B) x arccos x + 1 − x2 + c

C) arccosx + x arccos x + c
14. ∫x
2
D) sec 2x + c ⋅ f ( x) dx = 5x 4 + 2 x 2 – 1. What is f(x)?

E) x arccos x + 1+ x2 + c 4
A) 20x + B) 20x3 + 4x + c
x
5 x3
C) – 2x + c D) 5x2 + 2 + c
3
2
E) + 5x + c
x
dx
11. ∫x 2
=?
+ 2x + 2

A) arctan (x – 1) + c B) arctan (x + 1) + c
15. ∫4⋅e
4x+4
dx = ?
2
C) arccot (x + 1) + c D) ln |x + 2x + 2| + c
A) e4x+4 + c B) 4e4x+1 + c
E) 2 ln|x + 1| + c e4 x +1
C) +c D) 4e4x+3 + c
4
E) e4x+1 + c

12. ∫ sin
3
x ⋅ cos5 x dx?
cos(ln x)
16. ∫ dx = ?
A) 15sin2x cos4 x + c B) 8sin2 x + 8cos4x + c x

sin4 x cos5 x cos8 x cos 6 x A) sin x + c B) cos x + c


C) + +c D) − +c
4 5 8 6
C) cos (ln x ) + c D) sin(ln x ) + c
sin7 x sin 5 x
E) − +c E) cos(sin x ) + c
7 5

296 Integral
CHAPTER REVIEW TEST 1C
3 x2 + 4
1. f ( w ) = ∫ ( xw − w ) dw. What is f(w) ? 5. ∫e ⋅ x dx = ?
2

x2 w xw2 e3x + 4 e6 x
A) − wx + c B) − wx + c A) +c B) +c
2 2 3x2 + 4 6x
2
xw2 w2 C) e
3x +4
D) 6 x ⋅ e3 x
2
+4
C) − +c 2 2
D) x w + w + c 2
+c +c
2 2 6
2

w2 e3 x + 4
E) xw − +c E) +c
2 x

x
6. ∫ (x 2
− 1)2
dx = ?

2. If f ( x) = ∫ ( x2 + x − 2) dx and f(1) = 2 then what A) x


3
B)
( x2 − 1)3
+c
is f(2) ? 3 4
−1
23 17 9 C) ln| x2 − 1|+ c D) +c
A) B) C) D) 21 E) 13 2( x2 − 1)
6 3 2
x2 − 1
E) +c
x

7. ∫5 ⋅ e
7 x−2
dx = ?
3. ∫e
sin x
cos x dx = ?
7 x− 2

A) sinx + c B) esin x + c A) 35e7x–2 + c B) 7e +c


5
C) ecos x + c D) cos x + c 5e 7 x − 2
C) 35e7 + c D) +c
1 7
E) + c
e 5e 7
E) +c
7

8. ∫ 3e
sin 2 x
sin 2 x dx = ?
3
4. ∫ 1 − 9 x2
dx = ?
1 1 sin2 x
A) sin 2 x + c B) e +c
2 3
A) arcsin x + c B) arcos x + c
1 2 2

C) arcsin 3x + c D) arctan 3x + c C) − esin x + c D) 6esin x + c


3
2
E) arcos 3x + c E) 3esin x + c

Chapter Review Test 1C 297


9. 2x
∫e
3x
x2 dx = ? 13. ∫ 1 − x2
dx ?

A) e3 x + x3 + c
A) ln|1 – x2| + c B) arcsin x + c
2 3x 3x 3x
xe 2xe 2e
B) − + +c 2 2
3 9 27 C) –2 1 − x + c D) 2 1 − x + c
xe 3 x x2 e 3 x x3 e 3 x
C) + + +c E) 1 ln|1 − x2 |+ c
3 9 27 2
e3 x x3
D) + +c
3 3
e3 x x2
E) − 6 xe 3 x + e 3 x + c
3
14. 1 – cos x
10. ∫ x ln x dx = ?
3
∫ sin 2 x
dx = ?

x2 ln x x x ln x x2 ln x x x
A) + +c B) + +c A) 2 tan + c B) tan + c
3 9 2 4 2 2
x4 ln x x4 x3 ln x x3 x x
C) − +c D) + +c C) 2 tan + 2 sin + c D) sin x + cos x + c
4 16 4 16 2 2
x4 ln x x4
E) + +c x x
4 16 E) sin + cos + c
2 2
x −1
11. ∫x 3
dx ?
+1
A) ln|x3 + 1| + c
x +1
B) ln 2 +c 15. ∫ cos(cos x) ⋅ sin 2 x dx = ?
x − x +1
C) ln (|x + 1|3|x2 – x + 1|2) + c A) cos x + c
3 B) sin(cos x) + cos(sin x) + c
D) ln| x3 +1|+ c
4
2
C) sin x + c
E) 1 ln| x − x +1 |+ c D) sin(cos x) +cos(sin x) + c
3 x2 + 2 x +1
E) –2(cos(cos x) + cos x sin(cos x)) + c
3x +1
12. ∫ 2 dx ?
x + 3x − 4
A) ln |x + 4| + ln|x – 1| + c
x+ 4
B) ln +c
x −1
16. ∫ e ( x – 1)
x
dx = ?
11 4
ln(| x + 4| | x − 1| )
C) +c
5 A) ex(x – 1) + c B) ex(x + 1) + c
D) ln((x + 4)11(x – 1)4) + c C) xex + ex + c D) ex(ex + 1) + c

E) ln |x2 + 3x – 4| + c E) ex(x – 2) + c

298 Integral
CHAPTER 8

DEFINITE INTEGRAL
1 EVALUATING THE DEFINITE INTEGRAL
A. DEFINITION OF THE DEFINITE INTEGRAL
Definition
Let f(x) be a continuous function defined on the interval [a, b]. The area between the graph
of f(x) and x-axis is called the definite integral of f(x) from a to b.

y
In the figure the given area is the integral of f(x) on
y = f(x)
the interval [a, b].
b
We can write this expression as A = ∫ f ( x) dx.
a A

x
upper limit a b

∫ f ( x) dx
a
integrand
lower limit

y
Note
If the graph is under x-axis then its integral will be a b
x
negative.
A
The area is a positive value so in the figure
b
A = – ∫ f ( x) dx.
a

Note
If some parts of the graph is under x-axis and the y

other parts are above the x-axis then the integral will
be algebraic sum of the areas.
In the figure all of the areas A, B, C are positive A bC
numbers. a b
x
b B
So, ∫ f ( x) dx = A – B + C
a

300 Integral
Note
In the linear functions we can use geometric methods to calculate the area under a curve.

EXAMPLE 1 Find the area of region between the graph of y = 3x and x-axis on the interval [0, 4].

y
4
1
Solution The shaded area is = A = ∫ 3x dx = ⋅ 4 ⋅ 12 = 24 12
0 2
unit square.

x
0 4

EXAMPLE 2 In the following figure the area of parts A, B and C y


are given as A = 7, B = 9 and C = 8. Find the area
of the shaded region and evaluate the integral on the
interval [a, b]. C
A
x
a B b
Solution Total Area = A + B + C = 7 + 9 + 8 = 24
The integral on [a, b] = A – B + C = 7 - 9 + 8 = 6

B. THE FUNDAMENTAL THEOREM OF CALCULUS


Let f(x) be a function such that f : [a, b] → R. If F′(x) = f(x) and ∫ f(x) dx = F(x) + c.

b b
then .
∫ f ( x) dx = F( x)+ c| = F( b) − F( a)
a a

Note b
In this part we wrote: ∫ f ( x) dx = F( x)+ c .
a
But for definite integral we don't need to write “+c” because by the fundamental theorem of
b
calculus we have: ∫ f ( x) dx = ( F( b)+ c) − ( F( a)+ c) = F(b) + c – F(a) – c = F(b) – F(a).
a

The Definite Ýntegral 301


1
EXAMPLE 3 ∫ x dx = ?
0

1
x2 1 12 02 1
Solution ∫ x dx =
0
|= –
2 0 2 2
= .
2

5
EXAMPLE 4 ∫x
3
2
dx = ?

5
x3 5 53 33 125 − 27 98
∫ x dx =
2
Solution |= – = = .
3
3 3 3 3 3 3

π
EXAMPLE 5 ∫ sin x dx = ?
0

π π
Solution ∫ sin x dx = − cos x| = (–cos π) – (cos 0) = (–(–1) – (–1) = 1 + 1 = 2.
0 0

1
EXAMPLE 6 ∫e
0
3x
dx = ?

1
1 1 e 3 e0 e 3 − 1
∫e
3x
Solution dx = e 3 x | = − =
0
3 0 3 3 3

Check Yourself 1
Evaluate the following definite integrals:
5 π e e
1
∫ x dx ∫ cos x dx ∫ e dx ∫x
3
a. b. c. 2x
d. dx
1 0 0 1

Answers
2e
a. 156 b. 0 c. e − 1 d. 1
2

302 Integral
C. THE PROPERTIES OF THE DEFINITE INTEGRAL
Let f: [a, b] → R and g: [a, b] → R be two integrable functions.
a
1. ∫ f ( x) dx = 0
a
a
Proof ∫ f ( x) dx = F( a) − F( a) = 0.
a

b a
2. ∫
a
f ( x) dx = − ∫ f ( x) dx
b

b a
Proof

a
f ( x) dx = F( b) − F( a) = −( F( a) − F( b)) = −∫ f ( x) dx.
b

b b
3. ∫ c ⋅ f ( x) dx = c ⋅ ∫ f (x) dx
a a
b b b
Proof c ⋅ ∫ f ( x) dx = c ⋅ (F( b ) − F( a ) )= c ⋅ F( b ) − c ⋅ F(a ) = c ⋅F( x)| = ∫ c ⋅ f ( x ) dx.
a a a

b b b
4. ∫ [ f ( x) ± g( x)] dx = ∫ f (x) dx ± ∫ g(x ) dx
a a a

b b
Proof
∫ f ( x) dx ± ∫ g( x) dx =[F(b) – F(a)] ±[G( b) – G( a)]
a a
= [ F( b) ± G( b)] – [ F( a) ±G( a)]

= [ F ± G]( b) – [ F ± G]( a)
b
= ∫ [ f ( x) ± g( x)] dx.
a

c b c
5. Let, a ≤ b ≤ c then ∫
a
f ( x) dx = ∫ f ( x) dx + ∫ f ( x) dx
a b

c
Proof
∫ f ( x) dx = F( c) – F( a) = F( c) – F( a)+ F( b) – F( b)
a
= [ F( b) – F( a)]+[ F( c) – F( b)]

b c
= ∫ f ( x) dx + ∫ f ( x) dx
a b

The Definite Ýntegral 303


b b
6. f(x) ≥ g(x), ∀x ∈ [a, b] then ∫ f ( x) dx ≥ ∫ g( x) dx
a a

Proof If f(x) ≥ g(x) then f(x) – g(x) ≥ 0.


Let's say h(x) = f(x) – g(x) so, h(x) ≥ 0 for every x on [a, b]
b
We know that ∫ h( x) dx is the area between the graph of h(x) and x-axis.
a
b
If h(x) ≥ 0 then ∫ h( x) dx ≥ 0.
a

b b
7. ∫ f ( x) dx ≤
a
∫ a
f ( x) dx

b
Proof We know ∫a f ( x) dx is the area between the graph of f(x) and x-axis on the interval [a; b].
So we have two conditions:
a. f(x) ≤ 0, ∀x ∈ [a; b] or f(x) ≥ 0, ∀x ∈ [a; b].

y y

a b
x

A A

x
a b

b b b
In the first figure A = ∫a f ( x) dx or ∫
a
f ( x) dx = – A and | – A|=| A|= ∫
a
f( x) dx
b b
In the second figure A = ∫ f ( x) dx = ∫ f ( x) dx.
a a

b. If f(x) ≤ 0 for x ∈ [a; c] and f(x) ≥ 0 for x ∈ [c; b] y


b
where c ∈ [a; b] then ∫
a
f ( x) dx = A2 – A1 and
A2
b a
∫ f ( x) dx = A2 – A1 but x
a c b
b A1
∫a
f ( x) dx =| A2 |+| A1 | and

A2 – A1 ≤ A2 + A1 by triangle inequality.
b b
So, ∫ f ( x) dx ≤ ∫ f ( x) dx.
a a

304 Integral
7
3
EXAMPLE ∫ 3
x dx = ?

Solution a 3
We know ∫a
f ( x) dx = 0 then ∫
3
x dx = 0 .

EXAMPLE 8 ∫
4

4
5x2 sin3 4 x dx

Solution 4
By the same rule in example 7 we can say ∫
4
5x2 sin3 4 x dx = 0.

9
3
EXAMPLE ∫ 1
(2 x2 + 3x − 4) dx = ?

Solution 3 3 3 3 x3 3 x2 3 3

∫ 1
(2 x2 + 3x − 4) dx = 2 ⋅ ∫ x2 dx + 3 ⋅ ∫ x dx − 4 ⋅ ∫ dx = 2 ⋅(
1 1 1 3 1
|)+ 3 ⋅( |) −4 ⋅( x|)
2 1 1

1 9 1 52 64
= 2 ⋅ (9 − )+ 3 ⋅ ( − ) − 4 ⋅(3 − 1) = +12 − 8 =
3 2 2 3 3

10 x2 − 5x +1
e
EXAMPLE ∫ 1
(
x
) dx = ?

Solution e x2 − 5x + 1 e 1 e e e1 x2 e e e

∫ 1
(
x
) dx = ∫ ( x − 5+ ) dx = ∫ x dx − 5 ⋅ ∫ dx + ∫
1 x 1 1 1 x
dx = | − 5x| + ln x|
2 1 1 1

e2 1 e2 − 10e +11
=( − ) − 5 ⋅ ( e − 1)+(ln e − ln1) =
2 2 2

1
EXAMPLE 11 ∫e
0
3 x +1
dx = ?

Solution
1
e3 x+1 1 e4 − e
∫e
3 x +1
dx = |= .
0 3 0 3

π
EXAMPLE 12 2

∫ (sin x + cos x) dx = ?
0

π
Solution 2 π
2 π π
∫ (sin x + cos x) dx = −cos x + sin x | = ( −cos
0 0 2
+ sin ) – ( −cos0° + sin0°)
2
= (–0+1 ) – (–1+0) = 2

The Definite Ýntegral 305


1
EXAMPLE 13 ∫
0
3x +1 dx = ?
In definite integral we
can write the boundaries
by two ways. First find
Solution 1 Let u = 3x + 1 and du = 3 dx then integrate given expression.
the boundaries of u and
The lower bound is 3 ⋅ 0 + 1 = 1 then use it in the answer.
Second integrate the
The upper bound is 3 ⋅ 1 + 1 = 4 function and write the
3 3 3 3 initial boundaries.
1
14 1 u 2 4 2u 2 4 2 ⋅4 2
2 ⋅1 2


0
3x +1 dx = ∫ u du =
31 3 3 1
|=
9
|=
1 9

9
2
16 − 2 14
= = .
9 9
1
2 3 1 2 2 16 − 2 14
Solution 2 ∫
0
3x +1 dx = (3 x +1) 2 | = ⋅ 4 3 / 2 − ⋅13 / 2 =
9 0 9 9 9
= .
9

e
EXAMPLE 14 ∫ x ln x
1
dx = ?

1 x2
Solution Let u = ln x and v′ = x then u′ = and v = .
x 2
e
x2 1 x2 x2 x2 e e2 e2 1 1
∫ x ln x dx =
1 2
ln x – ∫ ⋅
x 2
dx = ( ln x –
2
) |= ( ln e − ) −( ln1 − )
4 1 2 4 2 4

e2 e2 1 e2 +1
= − + =
2 4 4 4

3
EXAMPLE 15 ∫x
−1
x+ 4
2
+ 2x
dx = ?

3 3
Solution x+ 4 x+ 4
∫x
−1
2
+ 2x
dx = ∫
−1 x( x + 2)
dx

x+ 4 A B ( A + B)x + 2 A
= + = ; A + B = 1 and 2 A = 4.
x( x + 2) x x + 2 x( x + 2)

So A = 2 and B = –1.
3
x+4 3
2 3
1 3 x2 3
∫ 2
−1 x + 2 x
dx = ∫
−1 x
dx − ∫
−1 x + 2
dx = 2 ln| x | – ln| x + 2| |
−1
= ln |
x + 2 −1
9 9
= ln – ln1= ln .
5 5

306 Integral
Check Yourself 2
Evaluate the following definite integrals.
2 4 e
x3 + 4x2 +5 x − 1
a. ∫x
5
⋅ cos 4 x dx b. ∫ ( x3 + 4 x2 − 3x − 2) dx c. ∫1 dx
2 1 x
3 π 4
x−3
d.
∫ ( x + x − 2) dx
2 e. ∫ (2 cos x − sin 2 x) dx
π
f.
∫ 2x 2
+x
dx
−2 2 1

2
3 e
ln(ln x)
g. ∫
−1
2 x + 3 dx h. ∫
1
x
dx

Answers
477 3
25 7 ln 3 26
a. 0 b. c. e + 2 e2 +5 e − 25 d. e. –1 f. − 6 ln 2 g. h. ln4 – 2
4 3 3 6 2 3

D. LEIBNIZ RULE
x
f : [a; b] → R is a continuous function and F( x) = ∫ f (t ) dt then,
a
d ⎛x ⎞
F ′( x) = ⎜ ∫ f ( t) dt ⎟ = f ( x).
dx ⎝ a ⎠
Applications:
v( x )
1. F( x) =
∫ f ( t) dt then F ′( x) = f (v( x) ) ⋅ v ′( x).
a
v( x )
2. F( x) = ∫ f (t ) dt then F ′( x) = f (v( x) ) ⋅ v ′( x) − f (u(x )) ⋅ u ′(x ).
u( x )

16
x
EXAMPLE If, F( x) = ∫ cos t 2 dt. then find F ′(
π
).
1 2
v( x )

Solution We know, when F( x) = ∫a


f ( t) dt then F′(x) = f(v(x)) ⋅ v′(x).

x
If, F( x) = ∫ cos t 2 dt and f(t) = cos t2, then F′(x) = cos x2 ⋅ (x)′ = cos x2.
1

π π 2 π 2
So, F ′( ) = cos( ) = cos = .
2 2 4 2

The Definite Ýntegral 307


17
x2
EXAMPLE If F( x) = ∫ ( t 2 − 4t +1) dt then evaluate F′(2).
x

v( x )

Solution We know, F( x) = ∫
u( x )
f (t ) dt then F ′( x) = F( v( x)) ⋅ v ′( x) – f ( u( x)) ⋅ u (′ x).

x2
So, if we have F( x) = ∫ ( t 2 − 4t +1) dt then
x

F′(x) = f(x2) ⋅ (x2)′ – f(x) ⋅ (x)′ where f(t) = t2 – 4t + 1


F′(x) = (x4 – 4x2 +1 ) ⋅ (2x) – (x2 – 4x + 1 ) ⋅ 1
F′(2) = (24 – 4 ⋅ 22 + 1) ⋅ (2 ⋅ 2) – ( 22 – 4 ⋅ 2 + 1)
= (16 – 16 + 1 ) ⋅ 4 – ( 4 – 8 + 1 )
=4+3
= 7.

E. THE MEAN VALUE THEOREM (MVT)


Let f: [a, b] → R be a continuous function then there exist at least one real number
c ∈ [a, b] such that
b

2t ∫a fx( x=) 1dx– t2 and dx =.


sin x = f ( c2),=sin 2
1+ t b − a1+ t

In the given formula f(c) is called the mean value of f(x) on the interval [a, b].

EXAMPLE 18 Find the mean value of f(x) = x2 – 4x on the interval [0, 4].

Solution Let, f(c) is the mean value of f(x) in [0, 4] then by Mean Value Theorem we have;
4
x3 x2 4 64 64
∫(x
2
− 4x)dx (
− 4 )| −
f ( c) = 0
= 3 2 0= 3 2 = − 64 = − 8 .
4−0 4 4 24 3

Note
1. All continuous functions have integrals on a closed interval [a, b].
2. A function with countable number of points of discontinuity has an integral on the closed
interval [a, b]. If the points c1, c2, …, cn ∈ [a, b] are points of discontinuity of f(x) then;
b c1 c2 b

∫ f ( x) dx = ∫ f ( x) dx + ∫ f ( x) dx +...+ ∫ f ( x) dx.
a a c1 cn

308 Integral
Check Yourself 3
x
π
1. If F( x) = ∫ sin t 3 dt then find F ′( 3 ).
2
3
x3
2. If F( x) = ∫ ( t 2 + 4t − 1) dt then find F′(3).
x2

x3 + 2 x2 +1
3. Find the mean value of f ( x) = on the interval [1, 3].
x2
Answers
3 13
1. 2. 21876 3.
2 3

The Definite Ýntegral 309


EXERCISES 2 .1
4
A. Definition of the Definite Integral 1
e. ∫ (3x4 + 4x2 + + x ) dx
x
1. Evaluate the area between the graph of f(x) = x 1

π
and x-axis on the interval [0, 3].
f. ∫ (sin 3x+ 4cos 2 x) dx
2. Find the integral of the function f(x) = 3x – 2 on 0

[0, 4] and find the area between the graph of f(x) e


x3 + x +1
and x-axis on the same interval.
g. ∫1 x dx
3. In the figure A = 5, y
f(x)
B = 4 and C = 7.
5 3
Find the area of the
C 6. If ∫ f ( x) dx = 7 then evaluate ∫ f ( x) dx.
shaded region and A
3 5
x
evaluate the integral of a B b
f(x) on the interval [a, b].
5 5
B. Fundamental Theorem of Calculus 7. If ∫ f ( x) dx = 5 and ∫ f ( x) dx = 8 evalute
4. Evaluate the following integrals. 1 3
3
3 5
a. ∫x
2
dx b. ∫ 3x dx ∫ f ( x) dx.
1
2 1
4 e
ln x D. Leibniz Rule
c. ∫x d. ∫
3
dx dx
x x
dF
0 1
8. a. If F( x) = ∫ cos t dt then find .
ln 2 π
1
dx
e. ∫e f. ∫ cos x dx
x
dx
1
0 0 dF
4 a b. If F( x) = ∫ ( t 2 + 2t) dt then find .
1 dx
g. ∫ ( x + ) dx h. ∫ ( ax2 + a2 x ) dx 1
x
1 x 0 x2
dF
7
c. Let F( x) = ∫ sin t dt then find .
i. ∫
0
x + 9 dx 0
dx
x3
dF
C. Properties of the Definite Integral d. Let F( x) = ∫ ( x + 4) then find .
x
dx
5. Evaluate the following integrals.
2 E. The Mean Value Theorem
a. ∫ (5x2 + 4x + cos x + e ln 4 ) dx
2
9. Find the mean value of the following functions in
1 given intervals.
b. ∫4
sin(ln(cos x)
dx
1 a. f(x) = x + 1 on [0, 5]
4
b. f(x) = x3 + 1 on [–1, 2]
c. ∫ 3x
2
dx
3 x2 + 2 x + 4
c. f ( x) = on [–3, 3]
4 x
d. ∫ (5x
3
+ 4 x2 + 3x − 5) dx
−3
d. f(x) = sinx on [0, 2π]

310 Integral
F. INTEGRAL OF ABSOLUTE VALUE FUNCTION

⎧⎪ f ( x), f ( x) ≥ 0
f ( x) = ⎨
⎪⎩ − f ( x), f ( x) ≤ 0

For integrating absolute value functions firstly we will find the positive and negative parts of
given function. For the intervals where f(x) change its sign we will use fifth property of defi-
nite integral and by using this method we will change negative parts to positive.

2
EXAMPLE 19 ∫ | x | dx = ?
−3

⎧⎪ x, x≥0
Solution |x| = ⎨ y
⎪⎩ − x, x≤0 y = |x|

2 0 2 x2 0 x2 2
So ∫
−3
| x| dx = ∫ ( − x) dx + ∫ x dx = −
−3 0
|+
2 −3 2 0
|
x
⎛ 02 −( −3)2 ⎞ ⎛ 2 2 0 2 ⎞ 13 –3 2
=⎜− − ⎟+⎜ − ⎟=
⎝ 2 2 ⎠ ⎝2 2 ⎠ 2

3
EXAMPLE 20 ∫ x−2
0
dx = ?

Solution x – 2 = 0; x = 2 is the root of x – 2.


y
⎧⎪−( x − 2) if x < 2 y = |x – 2|
So f ( x) = ⎨
⎪⎩ x − 2 if x ≥ 2
3 2 3
Then ∫ x−2
0
dx = ∫ −( x − 2) dx+ ∫( x − 2) dx
0 2 2 3
x

x2 2 x2 3
=− + 2 x|+ − 2 x|
2 0 2 2

9 5
= −2+ 4+ − 6 − 2+ 4 =
2 2
The Definite Ýntegral 311
3
EXAMPLE 21 ∫| x
−3
2
− 1| dx = ?

Solution If x2 – 1 = 0, then x = 1 or x = –1.


⎧⎪ x2 − 1, x ≥ 1 or x ≤ 1 y y = |x2 – 1|
x2 − ⎨
⎪⎩1 − x2 , −1 < x < 1
3 −1 1 3

∫ | x − 1| dx = ∫ ( x − 1) dx + ∫ (1 − x ) dx + ∫( x −1) dx
2 2 2 2

−3 −3 −1 1
x
–3 –1 1 3
⎛ x3 ⎞ −1 x3 1 ⎛ x3 ⎞ 3 44
=⎜ − x ⎟ |+( x − ) |+ ⎜ − x ⎟|= .
⎝ 3 ⎠ −3 3 −1 ⎝ 3 ⎠1 3

π
EXAMPLE 22 ∫| cos x| dx
0

Solution ⎧ π y

⎪⎪ cos x, on [0, 2
]
y = |cos x|
x ⎨ 1
⎪− cos x, on [ π , π]
⎪⎩ 2 x
p p
π
2
π 2 π


0
cos x dx = ∫ cos x dx + ∫ − cos x dx
0 π
2

π /2
x2 ⎛ x ⎞ ⎛− x
π

⎜ ⎟ ⎜ ⎟
⎝ 0 ⎠ ⎝ π /2 ⎠

π π
= (sin − sin0)+( −sin π + sin ) =1+1.
2 2

Check Yourself 4
Evaluate the definite integrals of the following absolute value functions.
2 4 π
a. ∫ | x +1| dx b. ∫ | x − 3x + 2| dx c. ∫ | sin x | dx
2

−2 −2 −π

3 3 4
d. ∫ x2 − 2 x +1 dx e. ∫| x2 − x | dx f. ∫| x
2
− 9| dx
−1 −1 −3

Answers
55 118
a. 5 b. c. 4 d. 4 e. 1 f.
3 3

312 Integral
CHAPTER 9

APPLICATION OF
DEFINITE INTEGRAL
1 INTEGRAL AS THE AREA
The definite integral has a lot of applications such as : the area of region bounded by any
curves or graphs, the volume of revolution, the arclength of a curve, the mean value of any
function, the center of mass, the displacement, the calculation of the work, …etc.
Most of these applications are directly related with mathematics or physics. In this chapter
we will learn the applications which are related with mathematics.

Theorem
Let f : [a, b] → R be a continuous, positively y
f(x)
defined function ( f(x) ≥ 0) then the area of the
region between y = f(x) and x-axis on the
interval [a, b] is
A
b x
a b
A = ∫ f ( x) dx .
a

Conclusion
1. If f(x) ≤ 0 then the area of the region between y

y = f(x) and x-axis on the interval [a; b] will be a b


x
b b
A = ∫ − f ( x) dx = – ∫ f ( x) dx.
a a
A
f(x)

More Generally
Let y = f(x) be any continuous function on the interval [a, b] then the area between
y = f(x) and x-axis is;
b
A = ∫| f ( x)| dx.
a

2. In the figure the total area of the region will be y


c d b
f(x)
Area=A+B+C = ∫ f ( x) dx − ∫ f ( x) dx + ∫ f ( x) dx.
a c d A
C
x
a c B d b

314 Integral
3. The area between y = f(x), y-axis and the lines y
n
y = m and y = n is A = ∫ f ( y) dy. n
m

x
O

y = f(x)

Important Note
y
a. The definite integral of f(x) on [a, b] is f(x)
b
A
∫ f ( x) dx = A – B.
a
x
a b
B
b. The area between f(x) and x-axis on [a, b] is
Area = A + B.
4. The Area Between Two Curves
a. y y y
g(x)
f(x) f(x)
A
f(x)
A
x
g(x) a b
x
x g(x) a b
a b

Let f(x) and g(x) be two curves then the area between f(x) and g(x) on the interval
b

[a, b] is A = ∫ [ f ( x) − g( x)] dx.


a

b. y
g(x)

B C
A f(x)

x
a b c d

The area between f(x) and g(x) is


b c d
A + B + C = ∫ ( f ( x) − g( x) ) dx + ∫ (g( x) − f ( x ) ) dx + ∫ (f (x ) − g(x ) ) dx.
a b c

Applications of the Definite Integral 315


EXAMPLE 1 Find the area of the region bounded by y = x – 1, x-axis, x = 2 and x = 4 lines.

y
4
x2 4 y=x+1
Solution A = ∫ ( x + 1) dx = + x|
2 2 2

= (8 + 4) – (2 + 2)
=8

x
–1 2 4

EXAMPLE 2 Find the area bounded by f(x) = –3x + 1, x-axis, x = 1 and x = 3 lines.

3
3 x2 3 y
Solution A = ∫ −( −3x +1) dx = − x|
1 2 1

1
27 3 1 3
=( − 3) − ( − 1) x
1
2 2 3
21 1
= −
2 2
=10 y = –3x + 1

EXAMPLE 3 What is the area of the region between f(x) = 1 – x2 and x-axis.

Solution f(x) = 1 – x2 = 0 ⇒ x = ± 1. y
1 3
x 1
A = ∫ (1 − x2 ) dx = x − | 1
−1 3 −1
–1 A 1
1 1 x
= (1 − ) − ( −1 + )
3 3
4
= y = 1 – x2
3

316 Integral
EXAMPLE 4 What is the area of the region between the graph y = 2 – 3x + x2 and x-axis.
y
Solution 2 – 3x + x2 = 0 ⇒ x1 = 1, x2 = 2 y = 2 – 3x + x2

2
3 x 2 x3 2
A = ∫ −(2 − 3x + x2 ) dx = −2 x +
2

3
| 2
1 1

8 3 1
= (– 4+6 – ) – ( –2+ – )
3 2 3 x
1 A 2
1
= .
6

EXAMPLE 5 Find the area of the region bounded by y2 – 1 = x and y-axis.


y
Solution y2 – 1 = 0 ⇒ y = ± 1
y2 – 1 = x
1 3
y 1
1
A = ∫ −( y2 − 1) dy = −( + y) |
−1 3 −1
A
1 1 x
= −( − 1)+( − +1) –1
3 3
–1
4
= .
3

EXAMPLE 6 Find the area between the graphs of y = x2 + 1 and y = 5.

Note
In these type of questions (the limits are not given) we must find the limits firstly. To find
the limits we will solve the equations simultaneously (say y1 = y2). These common solutions
will be the limits. After that we will decide which function is upper and which one is lower
(or just use absolute value). Then we can calculate the area by using definite integral.
y
Solution y1 = x2 + 1 y = x2 + 1

y2 = 5 5 y=5
y1 = y2 ⇒ x2 + 1 = 5 ⇒ x = ± 2
A
2 2
x3 2
A = ∫ (5 − ( x2 +1)) dx = ∫ (4 − x2 ) dx = 4 x − |
−2 −2 3 −2
1
x
8 8 32 –2 2
= (8 − ) − ( −8+ ) = .
3 3 3

Applications of the Definite Integral 317


EXAMPLE 7 Find the area of the region bounded by y = x2 + 2 and y = x + 4.
y y = x2 + 2
Solution y1 = y2 ⇒ x2 + 2 = x + 4 y=x+4

x2 – x – 2 = 0
x1 = 2 and x2 = –1 4
2 2
A = ∫ ⎡⎣( x + 4) − ( x2 + 2) ⎤⎦ dx = ∫ ( −x2 + x+ 2) dx
−1 −1
2
–4
x 3 x2 2 8 1 1 x
=− + + 2 x | = ( − + 2+ 4) −( + −2) –1 2
3 2 −1 3 3 2
9
=
2

EXAMPLE 8 Find the area of the region bounded by f(x) = x2 – x and g(x) = 3x – x2.

Solution f(x) = g(x) y


x2 – x = 3x – x2 f(x) = x2 – x
2x2 – 4x = 0
x1 = 0, x2 = 2.
2 2 A
A = ∫ ((3x − x2 ) − ( x2 − x) ) dx = ∫ ( −2 x 2 + 4 x) dx 2 3
x
0 0

3
2x 216 8 g(x) = 3x – x2
=− + 2 x2 | = − +8 − 0 = .
3 0 3 3

EXAMPLE 9 Find the area between the graphs of y = x3, y = 0, x = –2 and x = 1.

Solution The graph is intersecting x-axis at the point x = 0. y


So some parts in the domain is under x-axis and y = x3
some of them is below x-axis. So we will divide the
region into two parts. –2
x
0 1 1
x4 0 x4 1
A = ∫ − x3 dx + ∫ x3 dx = − |+ |
−2 0 4 −2 4 0
1 17
= 0 − ( −4)+ −0= .
4 4

318 Integral
EXAMPLE 10 Find the area of the region bounded by y = x3, x = 0, y = –2 and y = 2.

Solution y = x3 ⇒ x = 3 3 2. y
y = x3
4
2 2 1 2
3y 3 2 33 2 y=2
A = ∫ 3 y dy = ∫ y 3 dy = |=
0 0 4 0 2
x
4
0 3
3y 3 2
3 0
B= ∫
−2
3 y dy = − |=
4 −2 2 –2
y = –2

33 2 33 2
Total Area = A + B = + = 3 3 2.
2 2

Note
These two areas are symmetric. So if we multiply A by 2 it will give us the total area.

EXAMPLE 11 What is the area of the region bounded by y = x2 + 3x, x-axis and x = 3 line?

Solution 0 3
y
A = ∫ −( x2 + 3x) dx + ∫ ( x2 + 3x) dx y = x2 + 3x
−3 0

3 2 0 3
x 3x x 3x2 3
=− − |+ + |
3 2 −3 3 2 0
27 27 B
= 0 − (9 − )+9+ −0 x
2 2 –3 A 3
= 27

EXAMPLE 12 What is the area of the region between y = cos x and x-axis on the interval [0, 2π].

Solution In the figure the area of all these 4 parts are equal. y
So we can get the answer as follows:
π
2 π y = cos x
2
TotalArea = 4 ⋅ ∫ cos x dx = 4 ⋅sin x |
0 0
A D
p
π 3p
x
= 4sin − 4sin0 p 2p
2 2 B C 2

=4

Applications of the Definite Integral 319


EXAMPLE 13 What is the area of the region bounded by the graphs of the functions y = x3 and y = x.

y
Solution Firstly we will find the intersection points.
y=x
y = x3
y1 = y2
x3 = x; x3 – x = 0; x (x – 1)(x + 1 ) = 0
The intersection points are; –1, 0, 1.
–1
x
0 1 1
A = ∫ ( x − x) dx + ∫ ( x − x ) dx
3 3

−1 0

4
⎛x x ⎞ ⎛ x2 x4 1 ⎞ 1
2 0
=⎜ − | ⎟+ ⎜ − |⎟ = .
⎝ 4 2 −1 ⎠ ⎝ 2 4 0⎠ 2

Check Yourself 1
1. Find the area of the region bounded by y = x2 + 1 and y = 3x – 1.
2. Find the area between the graph of y = x2 – 4 and x-axis.
3. What is the area of the region bounded by y = 3x2 + 1 and y = 4.
4. Find the area of the region bounded by y = 2x2 + 3x and y = –x2 – 3x + 24.
5. Find the area of the region bounded by y = 2x2 – 4x + 5, x-axis, x = 2 and x = 3 lines.
6. What is the area of the region bounded by y = 3x + 5, x-axis, x = 1 and x = 4 lines.
7. Find the area of the region bounded by the graphs of y = 5x + 1, y-axis, y = 2 and y = 3 lines.
8. Find the area of the region bounded by the graph of y2 = x + 9 and y-axis.
9. What is the area of the region bounded by the graphs of y = –6x2 – 4x – 29 and y = 5 - x3.
10.What is the area of the region bounded by y = 2x, x = 1, x = –1 and y = 0 lines.

Answers
1 32 23 75 3 3
1. 2. 3. 4 4. 108 5. 6. 7. 8. 36 9. 128 10.
6 3 3 2 10 2 ln 2

320 Integral
EXERCISES 3 .1
1. Find the area of the region bounded by y = 4 – x2 11. Find the area of the region bounded by y = 2x +
and x-axis. 1, x = 0, y = 1 and y = 3.

2. Find the area of the region bounded by the line 12. Find the area of the region bounded by the graphs
y = 3 + x and both of x and y axes. of y = 3x – 1, y-axis, y = 0 and y = 2 lines.

3. Find the area of the region between y = 2x – 1 13. Find the area of the region bounded by the graphs
and x-axis on the interval [0, 3]. of y = ñx, y-axis, y = 1 and y = 3 lines.

4. Find the area of the region bounded by y = 1 – 3x 14. Find the area between x = y2 – 4 and y-axis.
and x-axis on the interval [2, 5].

15. Find the area of the region bounded by


5. Find the area of the region bounded by y = x2 + 5 x = y2 – 3y + 2 and y-axis.
and x-axis on the interval [0, 3].

16. Find the area of the region bounded by y = x2 and


6. Find the area of the region bounded by ›
y = 1 on the interval [–2, 1].
2
y = x – 3x – 4 and x-axis on the interval [-1, 7].

17. Find the area of the region bounded by the graphs


7. Find the area of the region between the graphs of y = 2x2 – 3x + 1 and y = 3 on the interval
y = x – 1, y = 0, x = 1 and x = 3. [2, 3].

8. Find the area of the region bounded by the graphs 18. Find the area of the region between y = x3 + 1
3
of y = x – 1, y = 0, x = 1 and x = 2. ›
and y = 5 on the interval [0, 2].

9. Find the area of the region bounded by the graphs 19. Find the area of the region bounded by the graphs
of y = 3 – x2, y = 0, x = 0 and x = 2. ›
of y = 1 + 3x, y = 8, x = 2 and x = 3.

10. Find the area of the region between the graphs of 20. Find the area of the region bounded by y = 2x – 5,
› 2
y = –2, x = 1 and x = 3.
y = x + 3x – 1, y = 0, x = –3 and x = 0.

Applications of the Definite Integral 321


21. Find the area of the region bounded by the curves 31. Find the area of the region bounded by the curves
2 2
y = x – 1 and y = 1 – x . f(x) = 3 – x2 and g(x) = 2.

22. Find the area of the region bounded by the curves


›
y = 2x2 – 3x + 5 and y = 10 – x – x2. 32. Find the area of the region bounded by the curves
y = x2 and x = y2.

23. Find the area of the region bounded by the curves


f(x) = x3 and g(x) = ñx.
33. Find the area of the region bounded by the curves
›
x = y2 and y = x – 3.

24. Find the area of the region bounded by the curves


y = 4 – x2 and y = x2 + 2.

34. Find the area of the region bounded by y = cos x


and y = sin x on the interval [0, π]
25. Find the area of the region bounded by the curves
y = 2x2 – 3x + 1 and y = –8x + 4.

35. Find the area of the region bounded by the graphs


26. Find the area of the region bounded by the curves › π
of y = sin3x, y = 2cos x, x = 0 and x = .
y = x2 – 1 and y = 3x +3. 2

27. Find the area of the region bounded by the curves


36. Find the area of the region bounded by the graphs
y = 4 – x2 and y = 2x + 1. π
of y = sin x and y = 3cos 2x on [0, ].
3

28. Find the area of the region bounded by the curves


y = x3 – 2x2 and y = 3x.
37. Find the area of the region bounded by the graphs
›
5π 7 π
of y = cos 2x and y = 2sin x on [ , ].
3 4
29. Find the area of the region bounded by the curves
›
y = x3 + x2 + 2x and y = 7x2 – 9x + 6.

38. Find the area of the region bounded by the graphs


30. Find the area of the region bounded by the curves cos x sin x π π
of y = and y = on [ , ].
f(x) = x2 and g(x) = 4. 3 3 3 2

322 Integral
39. Find the area of the region bounded by the graphs 46. If the area of the region bounded by y = ax2 (a > 0),
π
of y = 2sin x and y = 3cos x on [0, ]. x-axis and the line x = 3 is 18 cm2. What is the
6
value of a.

40. Find the area of the region bounded by y = sin x


and x-axis on [0, 2π].
1
47. Find the area of the region bounded by y = ,
x
x-axis, x = 1 and x = e3.

41. Find the area of the region between y = 5 ⋅ cos4x


π
and x-axis on [0, ].
4

48. Find the area of the region bounded by y = lnx,


x-axis, x = e and x = e2.

42. Find the area of the region bounded by y = 3 ⋅ sin


π
x, x-axis, x = 0 and x = .
3

49. In the figure the graph of the function f(x) is


1 5

43. What is the area of the region between the


given. ∫
−2
f ( x) dx = –5 and ∫ f ( x) dx = 5. Find the
−3
›
tangent line of the f(x) = ln x at the point x = e, total area of the shaded region.
y
the function f(x) = lnx and x-axis.

–3 –2 1 5
x

44. What is the area of the region bounded by the


graphs of y = x2, y = 3x2 and y = 4x.

50. In the figure the graph of y


45. In the figure the shaded y f(x) is given.
area is 12 cm2 and 7
1 2 7
2 If ∫ f ( x) dx = –3 and x
∫ f ( x) dx = 0. 2
x
1

the area B is 8 cm2 then


B
−2 –2
2 7
What is ∫ f ( x) dx.
0
f(x) what is ∫| f ( x)| dx?
1

Applications of the Definite Integral 323


2 OTHER APPLICATIONS OF THE DEFINITE INTEGRAL
A. VOLUME OF REVOLUTION
When a region is revolved about an axis we get a solid figure. This figure is called as a solid
revolution. To find the volume of this solid figure we can use integration.
Let f(x) be a continuous function defined on [a; b] the volume of the solid that we get by
revolving the area between the graph of f(x) and x-axis on [a, b], about x-axis is;
b
V = π ⋅ ∫ f 2 ( x) dx.
a

EXAMPLE 14 What is the volume of the solid figure that we generate by revolving the area between y = 3x
and x-axis around x-axis on the interval [0, 2].
2 y
y = 3x
Solution V = π ⋅ ∫ (3x)2 dx
0

2
= π ⋅ ∫ 9 x2 dx
0

2
= 3πx3 | x
0 2

= 24 π – 0
= 24 π.

EXAMPLE 15 What is the volume of the figure which can be obtained by revolving the region, between
y = x2 + 2 and x-axis, around x-axis on the interval [1, 3].
3 y
Solution V = π ⋅ ∫ ( x2 + 2)2 dx
1

3
= π ⋅ ∫ ( x4 + 4 x2 + 4) dx 3
1

x5 4x3 3
x
= π ⋅( + + 4 x) | 1 3
5 3 1

⎡ 243 1 4 ⎤
= π ⋅ ⎢( + 36+12) − ( + + 4) ⎥
⎣ 5 5 3 ⎦
1366 π
= .
15

324 Integral
Note y
y = g(x)
If we turn the figure around y-axis we should use d
x = f(y) and integrate it with respect to dy.
d c
V = π ⋅ ∫ f 2 ( y) dy.
c

EXAMPLE 16 Find the volume of the solid figure that we generate by rotating the region between
f(x) = 3x – 1, y-axis, y = 2 and y = 5 lines around y-axis.
y
y +1
Solution y = 3x – 1 ⇒ x =
3
5 5 2 5
y +1 2 y + 2y + 1
V = π ⋅ ∫( ) dy = π ⋅ ∫ dy
2 3 2 9
2
y3 y2 y 5
= π ⋅( + + ) |
27 9 9 2 x
1
125 25 5 8 4 2 –1
3
= π ⋅( + + − − − )
27 9 9 27 9 9
= 7π.

Note y

If we revolve the area between two curves f(x) and y = f(x)


g(x) on [a, b] interval then the volume of the solid
y = g(x)
figure will be:
b
V = π ⋅ ∫ (f 2 ( x) − g 2 ( x ) ) dx. a b
x
a

Applications of the Definite Integral 325


EXAMPLE 17 Find the volume of the solid figure which can be generated by revolving the area of the region
between y = 2x2 + 2 and y = 3 – 2x2 around x-axis.

1 y
Solution y1 = y2 ⇒ 2 x2 + 2 = 3 – 2 x2 ⇒ x = ±
2
1 3
2


2 2 2 2
V = π⋅ ⎡⎣(3 − 2 x ) − (2 x + 2) ⎤⎦ dx
−1
2 2
1 1
2
20 x3 2

∫ ( −20 x
2
= π⋅ +5) dx = π ⋅( − +5 x) |
−1
3 −1
2
2 x

⎡ 20 5 20 5 ⎤
= π ⋅ ⎢( − + ) − ( − )
⎣ 24 2 24 2 ⎥⎦
10 π
= .
3

EXAMPLE 18 What is the volume of the solid figure generated by revolving the area bounded by y = ex,
x = 2, x = 4 and y = 1 about the x-axis.
4 y y = ex
Solution V = π ⋅ ∫ ( e 2 x − 1) dx
2

e2 x 4
= π ⋅( − x) |
2 2
1
⎛ e8 y=1
e4 ⎞
= π ⋅ ⎜ ( − 4) − ( − 2) ⎟
⎝ 2 2 ⎠ x
2 4
π
= ( e8 − e 4 − 4).
2

EXAMPLE 19 Find the volume of the solid figure generated by revolving the area bounded by y = x3 and
y = ñx about x-axis for 60°.

326 Integral
Solution Solve the equations together to find the intersection points.
y
x3 = x; x1 = 0 and x2 = 1 y = x2
y = ñx
60
V= (Total Volume)
360
1⎛ 1 ⎞
= ⎜ π ⋅ ∫ (( x )2 − ( x3 )2 ) dx ⎟
6⎝ 0 ⎠ x
1
1 ⎛ ⎞
= ⋅ ⎜ π ⋅ ∫ ( x − x6 ) dx ⎟
6 ⎝ 0 ⎠
π x2 x7 1 π 1 1 5π
= ⋅ ( − ) | = ⋅ ( − − 0) = .
6 2 7 0 6 2 7 84

Check Yourself 2
1. Find the volume of the solid figure generated by revolving the area between y = x2 – 4 and
x-axis about x-axis.
2. Find the volume of the solid figure generated by revolving the area bounded by
y = 4x – 1, x-axis, x = 1 and x = 3 lines about x-axis.
3. Find the volume of the solid figure generated by revolving the area bounded by
y = 3x2 + 2x + 1, x-axis, x = 0 and x = 2 lines about x-axis.
4. Find the volume of the solid figure generated by revolving the area bounded by
y = 2x + 1, y-axis, y = 2 and y = 5 lines about y-axis.
5. Find the volume of the solid figure generated by revolving the area bounded by
y = 2x2 + x + 2, y = 1, x = 1 and x = 2 lines about x-axis.
4
6. Show by using integral that the volume of a sphere with radius R is π R3 .
3
Answers
512 π 326 π 2134 π 21π 349π 4 3
1. 2. 3. 4. 5. 6. πR
15 3 15 4 5 3

B. ARCLENGTH (OPTIONAL)
The length of any curve ( or line ) between the points a and b for a continuous and differ-
entiable function f is:
b
L = ∫ 1+ ( f ′( x) ) dx.
2

Applications of the Definite Integral 327


EXAMPLE 20 Find the length of the graph of f(x) = 4(x – 1)3/2 between x = 1 and x = 2.

Solution By using the formula we can get the answer.

f ′( x) = 6 ⋅ ( x – 1 )1/ 2

( ) dx = ∫
2 2 2 2
1
L = ∫ 1 + 6( x − 1) 2
1 + 36( x −1) dx = ∫ 36 x −35 dx
1 1 1

du
u = 36 x – 35 ⇒ du = 36 or dx =
36
3 3
1 1 u 2 (36 x − 35) 2 2
dx =
36 ∫ u 2 du =
54
=
54
|
1

37 37 − 1
= .
54

EXAMPLE 21 Find the length (circumference) of a circle with radius 2.

Solution Lets take the circle x2 + y2 = 4.


2
So y = ± 4 − x .
Lets divide the circle into 4 parts and find the length of just one part.
2
So we get L = 1+(( 4 − x2 )) 2 dx.
∫ 0
y

2
x
The circumference of circle = 4 ⋅ ∫ 1+( − ) 2 dx y = – 4 – x2
0 4 − x2
2 x
x2
=4⋅∫ 1+ dx
0 4 − x2 x2 + y2 = 9

2
4
=4⋅∫ dx
0 4 − x2
2
dx
=8⋅∫
0 4 − x2
x 2
= 8 ⋅ (arcsin ) |
2 0
= 8 ⋅ (arcsin1 – arcsin0)

= 4π.

328 Integral
C. AREAS OF SURFACE OF REVOLUTION (OPTIONAL)
If a function f(x) has a continuous first derivative on [a, b] then the area of the surface
generated by revolving the curve about x-axis is
b
SAREA = 2π ⋅ ∫ f ( x) ⋅ 1+ ( f ′( x) ) dx.
2

EXAMPLE 22 Find the surface area of the sphere with radius r = 3 cm.

Solution Lets take the circle x2 + y2 = 9 ⇒ y = ± 9 − x2 .


Lets use the circle between 0 and 3 and rotate it. It will give us half of the surface area of the
sphere so we will multiply the result by 2 to get all surface area.

y
( ) dx
3 2
Surface Area = 2 ⋅ 2 π ⋅ ∫ 9 − x2 ⋅ 1+ ( 9 − x2 )'
0
3 x2 + y2 = 9
3 2
x
= 4π ⋅ ∫ 9 − x2 ⋅ 1+ dx
0 9 − x2
x
3
–3 3
3
= 4 π ⋅ ∫ 9 − x2 ⋅ dx
0 9 − x2
–3
3
= 4π ⋅ (3x) |
0

= 36 π cm 2 .

Check Yourself 3
1. Find the length of the curve y = 2(x + 3 )3/2 between x = 1 to x = 3.
2. Find the length of the curve y = 4x2 – x + 1 on [0, 1]
3. Find the length of the curve y = x3/2 on the interval [0, 2].
4. Find the area of the surface of revolution which is generated by rotating y = 2x + 1 about
x-axis on the interval [1, 3].
5. What is the surface area of a parabolic reflector whose shape is obtained by rotating the
parabola y = ñx on [1, 2] about x-axis.
Answers
110 55 − 74 37 9 2 − ln( 2 − 1) 22 22 − 8 9π 5 π 5
1. 2. 3. 4. 20πñ5 5. −
27 4 27 2 6

Applications of the Definite Integral 329


EXERCISES 3 .2
A. Volume of Revolution 11. Find the volume of solid figure generated by
1. Find the volume of solid figure generated by revolving the area of the region bounded by
revolving the area of the region bounded by y = 4x – 1, x-axis on [0, 3] about y-axis.
y = 2x + 5, x = 2 and x = 3 around x-axis.
12. Find the volume of solid figure generated by
2. Find the volume of solid figure generated by revolving the area of the region bounded by
revolving the area of the region bounded by y = ex, x = 2 and xy-axes about x-axis.
y = x2 + 1 and x-axis on [0, 1] about x-axis.
13. Find the volume of solid figure generated by
3. Find the volume of solid figure generated by revolving the area of the region bounded by
revolving the area of the region bounded by y = sin x, x = 0, x = π and x-axis about x-axis.
y = x3 – 1 and x-axis on [1, 2] around x-axis.
14. Find the volume of solid figure generated by
4. Find the volume of solid figure generated by
revolving the area of the region bounded by
revolving the area of the region bounded by π
y = 3x + 1, x-axis, x = 1 and x = 3 lines about y = cos 2x, x = , x and y axes about x-axis.
2
x-axis.
15. Find the volume of solid figure generated by
5. Find the volume of solid figure generated by
revolving the area of the region bounded by y = x2
revolving the area of the region bounded by
and y = x, about x-axis.
y = 1 – x2 and x-axis, around x-axis.

16. Find the volume of solid figure generated by


6. Find the volume of solid figure generated by
revolving the area of the region bounded by y = x2
revolving the area of the region bounded by
and y = x, about y-axis.
y = x2 – 4 and x-axis about x-axis.

7. Find the volume of solid figure generated by 17. Find the volume of solid figure generated by
revolving the area of the region bounded by revolving the area of the region bounded by
y = 2x – 1, y-axis, y =1, y = 2 lines, about x-axis. y2 = x + 4 , x = 2, y = 2, about x-axis.

8. Find the volume of solid figure generated by 18. Find the volume of solid figure generated by
revolving the area of the region bounded by ›
revolving the area of the region bounded by
y = 2x2 – 1, y-axis, y = 0 and y = 3 about y-axis.
y = 6 – x2, y = 3, x = 1, x = 2 about y = 1.
9. Find the volume of solid figure generated by
› revolving the area of the region bounded by 19. Find the volume of a cone with radius r = 3 cm
3
y = x , x-axis, x = 0 and x = 2 about y-axis. and altitude 4 cm by using integral.

10. Find the volume of solid figure generated by 20. Find the volume of solid figure generated by
› revolving the area of the region bounded by revolving the area of the region bounded by
y = 1 – x2, x-axis, x = 1 and x = 3 about y-axis. y = x2 + 1 and y = 3x – 1, about x-axis.

Applications of the Definite Integral 330


21. Find the volume of solid figure generated by 30. Find the volume of solid figure generated by
› revolving the area of the region bounded by revolving the area of the region bounded by
y = 3x2 – 5x + 1 and y = x2 + 4x – 1, about y = x2 – 4 and y = 3x + 6 about x-axis.
x-axis.
31. Find the volume of solid figure generated by
22. Find the volume of solid figure generated by revolving the area of the region bounded by
revolving the area of the region bounded by y = x2 + 4 and y = 2 on the interval [1, 3] about
y = 5 – x2 and y = x2 + 3, about x-axis. x-axis.

B. Arclength
23. Find the volume of solid figure generated by
revolving the area of the region bounded by 32. Find the arclength of the curve y = x2 – 1 between
f(x) = –x2, g(x) = x2 – 3, about x-axis. ›
x = 0 and x = 1.

24. Find the volume of solid figure generated by 33. Find the arclength of the curve y = 2 ⋅ (x – 1)3/2
›
revolving the area of the region bounded by on the interval [1, 2].
1
f(x) = , x-axis, x = 1, x = 3 about y-axis. 34. Find the circumference of circle (by using
x
integral) whose radius is 5 cm.
25. Find the volume of solid figure generated by
35. Find the arclength of the parabola y2 = x on the
revolving the area of the region bounded by interval [0, 1].
1
f(x) = , x-axis, x = 1, x = 3 about x-axis.
x 36. Find the arclength of the graph y = x3/2 over [0, 1].

26. Find the volume of solid figure generated by 37. Find the arclength of the graph y = 3x + 1
revolving the area of the region bounded by between x = 0 and x = 4.
y = x2 + x + 1, x = 1, x = 2, x-axis about x-axis
for 90°. C. Areas of Surface of Revolution
38. Find the surface area of a sphere with radius
27. Find the volume of solid figure generated by 2 cm. (by using integration)
revolving the area of the region bounded by
y = ex, x = 1, x = 2, x-axis about x-axis for 120°. 39. Find the surface area of a solid figure which is
generated by revolving the parabola y = x2 on the
28. Find the volume of solid figure generated by interval [0, 1].

revolving the area of the region between y = tan


1 40. Find the surface area of the solid generated by
x and x-axis on the interval [0, ] about x-axis for 2 x3 / 2
x revolving the curve y = on [1, 2] about
180°. 3
x-axis.
29. Find the volume of solid figure generated by
41. Compute the surface area of the solid obtained by
revolving the area of the region bounded by
rotating the graph of f(x) = ñx on the interval
y = 2x2 + 3x – 1 and y = x2 + x – 2 about x-axis,
[0, 1], about y-axis.
on [1; 3].
331 Integral
CHAPTER SUMMARY
1. Integral as the Area B. Arclength
 Let f : [a, b] → \ be a continuous, positively defined The length of any curve (or line) between the points a
function (f(x) ≥ 0) then the area of the region and b for a continuous and differentiable function f is:
b
between y = f(x) and x-axis on the interval [a, b] is
L = ∫ 1+ ( f '( x) ) dx.
2

b a
A = ∫ f ( x) dx
a C. Areas of Surface of Revolution
If a function f(x) has a continuous first derivative on
 Let y = f(x) be any continuous function in the [a, b] then the area of the surface generated by revolving
interval [a; b] then the area between y = f(x) and the curve about x-axis is
x - axis is b
SAREA = 2 π ⋅ ∫ f ( x) ⋅ 1+ ( f ′( x) ) dx.
2
b
a
∫a
f ( x) dx

 The area between y = f(x), y-axis, y = m and y = n


lines is
n Concept Check
A = ∫ f ( y) dy
m • How can we find the area by using the integral?

 Let f(x) and g(x) be two curves then the area between • How can we find the area between a function and x-axis
f(x) and g(x) on the interval [a, b] is if the function is under x-axis?
b • How can we find the area between two curves if they
A = ∫ [ f ( x) − g( x )] dx
a
intersect at countable number of points?

2. Other Applications of the Definite Integral • How can we find the area between a function and y-axis?
A. Volume of Revolution • How can we find the volume of a function by using
Let f(x) be a continuous function defined on [a; b] the integration ?
volume of the solid that we get by revolving the area • How can we find the length of a curve on a closed
between f(x) and x-axis on [a, b] is interval [a, b]?
b
V = π ⋅ ∫ f 2 ( x) dx • How can we find the area of revolution of a surface by
a using integral?
Note 1
If we turn the figure around y-axis we should use
x = f(y) and integrate it with respect to dy.
d
V = π ⋅ ∫ f 2 ( y) dy.
c

Note 2
If we revolve the area between two curves f(x) and g(x)
on [a, b] interval then the volume of the solid
b
Figure will be: V = π ⋅ ∫ f 2 ( x) − g 2( x) dx.
a

Chapter Summary 332


CHAPTER REVIEW TEST 3A
1. y 5. What is the area of the region bounded by the
graphs of y = x2 – 3x + 2, x-axis, x = 1 and
x = 3 lines?
–3 –1 3 6 7
1 5
O
x A) B) C) 1 D) E) 21
6 6 12 5

3
In the figure, ∫ f ( x)
−1
dx =12 cm2 and
6

∫ f ( x) dx = 5 cm
2

−3
6. What is the area of the region bounded by the
What is the total area of the shaded region ?
graphs of y = x3, y-axis and y = 8 line?
A) 7 B) 12 C) 17 D) 19 E) 21
8 16
A) 12 B) C) D) 8 E) 17
3 3

2. What is the area of the region bounded by the


graphs of y = 2x – 3, x – axis, x = 1 and x = 4
lines?
A) 6 B) 9 C) 10 D) 16 E) 21
7. What is the area of the region between the graphs
of y = x2 – 7x + 10 and y = x + 3 lines?

A) 33 B) 27 C) 18 D) 36 E) 42
3. What is the area of the region bounded by the
graphs of y = –x + 5, x-axis, x = 3 and x = 5
lines?

A) 27 B) 13 C) 7 D) 3 E) 2

8. What is the area of the region between the graphs


4. What is the area of the region bounded by the of y = x2 – x + 1 and y = –x2 + 2x + 3
graph of y = x2 – 9 and x-axis?
146 68 125 17
A) B) C) D) E) 49
A) 27 B) 36 C) 28 D) 40 E) 49 7 3 4 2

333 Integral
9. What is the area of the region bounded by the 13. What is the volume of the solid figure that we
graphs of y = sin 2x and x-axis on the interval generate by turning the area between the graphs
[0, π]? of y = x2 + 2 and y = 3 around x-axis?
27π 104 π 48π
A) 2 B) 4 C) 5 D) 3π E) 4π A) B) C)
5 15 5
163π 32π
D) E)
15 3

14. What is the volume of the solid figure that we


10. What is the area of the region bounded by the
generate by turning the area between the graphs
graphs of y = 3x – 4, y-axis, y = 1 and y = 4
of y = 3x + 2 and y = x + 8 around x-axis on the
lines?
interval [0, 1]?
7 13 8 43
A) B) C) D) E) 10 178 π
3 2 5 7 A) 65π B) 27π C)
3
147 π 63π
D) E)
3 3

15. What is the volume of the solid figure that we


11. What is the volume of the solid figure that we generate by revolving the area of the region
generate by revolving the area between y = x and between the graphs of y = 3x – 1, y-axis, y = 1
x-axis around x-axis on the interval [0, 3]? and y = 2 lines around y-axis ?
π 27π 19π π 47π
A) B) 27π C) D) 27 E) 9π A) B) C)
3 2 27 3 9
17 π 2π
D) E)
5 3

16. What is the volume of the solid figure that we


generate by revolving the area of the region
12.What is the volume of the solid figure that we gen-
between the graphs of y = 2x – 1 and x-axis for
erate by revolving the area between
180° on the interval [0, 2]?
y = x2 + 1, x-axis, x = 0 and x = 1 lines around
x-axis? 21π 21π 14π
A) B) C)
3 4 3
19π 28π 17 π 7π
A) π B) C) 4π D) E) D) E) 5π
2 15 3 3
Chapter Review Test 1 334
CHAPTER REVIEW TEST 3B
1. What is the area of the region bounded by the 5. What is the area of the region bounded by the
graphs of y = 3x – 1, x = 0 and y = 2 lines? graphs of y = 2x + 1, y = 5 – 2x and x - axis ?
3 7 6 7
A) B) C) D) 1 E) 1 5 7
7 6 7 3 A) B) C) 1 D) E) 21
6 6 12 5

2. What is the area of the region bounded by the


graphs of y = 3x2 + 4, x - axis, x = 1 and x = 3
lines? 6. What is the area of the region bounded by the
graphs of y = x3 – 4x and x – axis?
A) 34 B) 29 C) 21 D) 16 E) 9
8 16
A) 12 B) C) D) 8 E) 16
3 3

y
3. In the figure we have
the graph of f(x) = x2.
If 2S1 = S2 then find a.
7. What is the area of the region between the graphs
S2 of y = x2 – 6x, y = –9 and y - axis?
S1
x
a 3 A) 32 B) 27 C) 18 D) 9 E) 4

3 3
A) 2 B) 2 3 2 C) 9 D) 8 E) 13

8. What is the area of the region between the graphs


4. What is the area of the region bounded by the of y = x2 – x + 1 and y = –x2 – 4x + 3
graph of y = 5x + 1 and xy - axes?
145 63 127 17
A) B) C) D) E) 49
A) 1 B) 3 C) 0.3 D) 0.2 E) 0.1 8 8 25 2

335 Integral
9. What is the area of the region bounded by the 13. What is the volume of the solid figure that we gen-
graphs of y = cos 2x, x = π and x - axis? erate by revolving the area between y = x2 – 2x
and x - axis around x - axis ?
A) 2 B) 4 C) 5 D) 3π E) 4π
19π 28π 16 π
A) 2π B) C) 7π D) E)
3 15 15

10. What is the area of the region bounded by the 14. What is the volume of the solid figure that we
graphs of y = cos 2x and y = sin x on the interval generate by turning the area between the graphs
[0, π]? of y = 5x – 1, x = 1, x = 2 and x - axis around
y - axis.
A) 3ñ3 – 2 B) ñ3 – 2 C) 12π – 2
35π 178 π 147 π 63π
D) 6ñ3 + 2 E) 6ñ3 – 2 A) B) 27π C) D) E)
3 3 3 2

15. What is the volume of the solid figure that we


generate by revolving the area of the region
11. What is the volume of the solid figure that we
between the graphs of y = x2 – 3x – 4, x - axis,
generate by turning the area between the graphs
x = 2 and x = 5 around x - axis?
of y = 3x – 1, x = 2 and x = 3 around x - axis ?
197 π 28π 441π 17 π 2π
17 32 28 17 A) B) C) D) E)
A) B) C) 9 D) E) 27 3 10 3 3
3 3 3 6

16. What is the volume of the solid figure that we


generate by revolving the area of the region
12. What is the area of the region bounded by the between the graphs of y = 2x – 1 and x - axis for
graphs of x = y2 + 1, y = 2 and y = 5. 180° on the interval [0, 2] ?

27π 64π 64π 14π 96 π


A) B) 16π C) 22π D) 42π E) 43π A) B) C) D) E) 5π
5 3 5 5 5

Chapter Review Test 1 336

Das könnte Ihnen auch gefallen